You are on page 1of 210

THIS PDF/MATERIAL IS FIRST

UPLOADED BY A TELEGRAM
CHANNEL –

CBSE / ISC Class 11


To join this Channel on Telegram, click on the link
below:
https://t.me/std_XI

On this channel, we regularly upload high-


quality & premium materials for ISC and CBSE
boards, free of cost, which are easily
downloadable in PDF format.

Note: We have exercised due care and caution in collecting


all the data before posting this PDF.
If any omission, inaccuracy or printing error occurs with
regard to the data contained in this book, we will not be held
responsible or liable.
We will be grateful if you could point out such error or offer
your suggestion which will be of great help for other readers.
Join our channel for more.

CLICK HERE TO JOIN THE CHANNEL


@ebooks_for_freee on Telegram
Corporate

DISHA PUBLICATION
Office
45, 2nd Floor, Maharishi Dayanand Marg,
Corner Market, Malviya Nagar, New Delhi - 110017
Tel : 49842349 / 49842350

© Copyright
No part of this publication may be reproduced in
any form without prior permission of the publisher.

Disha
The author and the publisher do not take any legal
responsibility for any errors or misrepresentations
that might have crept in. We have tried and made
our best efforts to provide accurate up-to-date
information in this book.
All Right Reserved

Typeset by Disha DTP Team

www.dishapublication.com www.mylearninggraph.com
Books & ebooks Etests
for School &
Competitive for
Exams Competitive
Exams

Write to us at feedback_disha@aiets.co.in

@ebooks_for_freee on Telegram
Contents
S.no. Chapter Name Page no.
Questions Solutions
1. Physical world, Units and Measurements 1-3 101-103

2. Motion in a Straight Line 4-7 104-107

3. Motion in a Plane 8-11 108-111

4. Laws of Motion 12-15 112-115

5. Work, Energy and Power 16-19 116-120

6. System of Particles and Rotational Motion 20-23 121-124

7. Gravitation 24-27 125-127

8. Mechanical Properties of Solids 28-30 128-130

9. Mechanical Properties of Fluids 31-34 131-134

10. Thermal Properties of Matter 35-38 135-138

11. Thermodynamics 39-41 139-142

12. Kinetic Theory 42-44 143-145

13. Oscillations 45-48 146-149

14. Waves 49-51 150-153

15. Electric Charges and Fields 52-55 154-158

16. Electrostatic Potential and Capacitance 56-58 159-161

17. Current Electricity 59-62 162-165

@ebooks_for_freee on Telegram
18. Moving Charges and Magnetism 63-66 166-168

19. Magnetism and Matter 67-69 169-171

20. Electromagnetic Induction 70-72 172-174

21. Alternating Current 73-75 175-178

22. Electromagnetic Waves 76-78 179-180

23. Ray Optics and Optical Instruments 79-81 181-185

24. Wave Optics 82-84 186-188

25. Dual Nature of Radiation and Matter 85-88 189-192

26. Atoms 89-91 193-196

27. Nuclei 92-94 197-199

28. Semiconductor Electronics : Materials,


Devices and Simple Circuits 95-97 200-202

29. Communication Systems 98-100 203-204

@ebooks_for_freee on Telegram
PHYSICAL WORLD, 1
UNITS AND
MEASUREMENTS

MCQs with One Correct Answer az


a -
5. In the relation P = e k q where P is pressure,
1. If x = at + bt2, where x is the distance travelled b
by the body in kilometers while t is the time in Z is distance, k is Boltzmann constants and q is
seconds, then the unit of b is the temperature. The dimensional formula of b
will be
(a) km/s (b) kms (a) [M0L2T0] (b) [M1L2T1]
(c) km/s2 (d) kms2 1
(c) [M L T ]0 –1 (d) [M0L2T–1]
2. A metal sample carrying a current along X-axis 6. In a new system of units, the fundamental
with density Jx is subjected to a magnetic field Bz quantities mass, length and time are replaced by
(along z-axis). The electric field Ey developed along acceleration ‘a’, density ‘r’ and frequency ‘f’.
Y-axis is directly proportional to Jx as well as Bz. The dimensional formula for force in this system
The constant of proportionality has SI unit. is
(a) [ra4 f ] (b) [ra4 f –6]
m2 m3 (c) [r–1a–4f 6] (d) [r–1a–4 f –1]
(a) (b)
A As
b k .q.t 3
m2 As 7. A formula is given as P = 1+
(c) (d) a m.a
As m3
where P = pressure; k = Boltzmann’s constant;
3. The refractive index of water measured by the
real depth
q = temperature; t = time; ‘a’ and ‘b’ are
relation m = is found to have constants.
apparent depth
values of 1.34, 1.38, 1.32 and 1.36; the mean Dimensional formula of ‘b’ is same as
value of refractive index with percentage error is (a) Force
(a) 1.35 ± 1.48 % (b) 1.35 ± 0 % (b) Linear momentum
(c) 1.36 ± 6 % (d) 1.36 ± 0 % (c) Angular momentum
4. Write the dimensions of a × b in the relation (d) Torque
b - x2 8. The pair of physical quantities that has the
E= , where E is the energy, x is the
at different dimensions is :
displacement and t is time (a) Reynolds number and coefficient of friction
(a) ML2T (b) M–1 L2T 1 (b) Curie and frequency of a light wave
(c) ML T 2 –2 (d) MLT–2 (c) Latent heat and gravitational potential
(d) Planck’s constant and torque

@ebooks_for_freee on Telegram
2 PHYSICS

9. Force F is given in terms of time t and distance x divisions of the vernier scale. If the smallest
by F = A sin (Ct) + B cos (Dx). Then, dimensions division of the main scale is half- a degree
A C (= 0.5°), then the least count of the instrument is:
of and are (a) half minute (b) one degree
B D
(a) [M0 L0 T0], [M0 L0 T–1] (c) half degree (d) one minute
(b) [M L T–2], [M0 L–1 T0] Î0
(c) [M0 L0 T0], [M0 L T–1] 16. In SI units, the dimensions of is:
m0
(d) [M0 L1 T–1], [M0 L0 T0] (a) A–1TML3 (b) AT2 M–1L–1
10. The respective number of significant figures for (c) AT –3 ML3/2 (d) A2T3 M–1L–2
the numbers 23.023, 0.0003 and 2.1 × 10–3 are 17. From the following combinations of physical
(a) 5, 1, 2 (b) 5, 1, 5 constants (expressed through their usual
(c) 5, 5, 2 (d) 4, 4, 2 symbols) the only combination, that would have
11. N divisions on the main scale of a vernier calliper the same value in different systems of units, is:
coincide with (N + 1) divisions of the vernier scale. ch
(a)
If each division of main scale is ‘a’ units, then the 2pe o2
least count of the instrument is
e2
a (b) (me = mass of electron)
(a) a (b) 2pe o Gme2
N
m o eo G
N a (c)
(c) ´a (d) c2 he 2
N +1 N +1
12. Given that K = energy, V = velocity, T = time. If 2p m o eo h
they are chosen as the fundamental units, then (d)
ce2 G
what is dimensional formula for surface tension?
18. A student measuring the diameter of a pencil of
(a) [KV–2T –2 ] (b) [K2 V2T–2 ] circular cross-section with the help of a vernier
(c) [K2V–2 T–2 ] (d) [KV2T2] scale records the following four readings 5.50
13. In the formula X = 5YZ 2 , X and Z have mm, 5.55 mm, 5.45 mm, 5.65 mm, The average of
these four reading is 5.5375 mm and the stan-
dimensions of capacitance and magnetic field,
dard deviation of the data is 0.07395 mm. The
respectively. What are the dimensions of Y in average diameter of the pencil should therefore
SI units ? be recorded as :
(a) [M–3 L–2 T8 A4] (b) [M–1L–2T4 A2] (a) (5.5375 ± 0.0739) mm
(c) [M–2 L0 T–4 A–2] (d) [M–2L–2T6 A3] (b) (5.5375 ± 0.0740) mm
14. The relative error in the determination of the (c) (5.538 ± 0.074) mm
surface area of a sphere is a. Then the relative (d) (5.54 ± 0.07) mm
error in the determination of its volume is 19. A quantity x is given by (IFv2/WL4) in terms of
2 2 moment of inertia I, force F, velocity v, work W
(a) a (b) a
3 3 and Length L. The dimensional formula for x is
3 same as that of :
(c) a (d) a (a) planck’s constant
2
(b) force constant
15. In an experiment the angles are required to be
measured using an instrument, 29 divisions of (c) energy density
the main scale exactly coincide with the 30 (d) coefficient of viscosity

@ebooks_for_freee on Telegram
Physical World, Units and Measurements 3
20. The period of revolution (T) of a planet moving in the measurement of the current and the voltage
round the sun in a circular orbit depends upon difference are 3% each, then percentage error in
the radius (r) of the orbit, mass (M) of the sun the value of resistance of the wire is
and the gravitation constant (G). Then T is 26. If 3.8 × 10–6 is added to 4.2 × 10–5 giving the
proportional to regard to significant figures then the result will
(a) r1/2 (b) r be x × 10–5. Find the value of x.
(c) r 3/2 (d) r2 27. The mass of a liquid flowing per second per unit
area of cross section of a tube is proportional to
Numeric Value Answer Px and vy, where P is the pressure difference
and v is the velocity. Then x ÷ y is
21. The current voltage relation of a diode is given
by I = (e1000 V/T – 1) mA, where the applied 28. The specific resistance r of a circular wire of
voltage V is in volts and the temperature T is in radius r, resistance R and length l is given by
degree kelvin. If a student makes an error pr 2 R
r= . Given, r = 0.24 ± 0.02 cm, R = 30 ± 1W
measuring ±0.01 V while measuring the current l
of 5 mA at 300 K, what will be the error in the and l = 4.80 ± 0.01 cm. The percentage error in r
value of current in mA? is nearly
22. A physical quantity P is described by the relation 29. To determine the Young’s modulus of a wire, the
P = a1/2 b2 c3 d–4 formula is
If the relative errors in the measurement of a, b, F L
Y= ´ : where L = length, A = area of
c and d respectively, are 2%, 1%, 3% and 5%, A DL
then the percentage error in P will be : cross-section of the wire, DL = change in length
23. The density of a material in SI unit is 128 kg of the wire when stretched with a force F. The
m–3. In certain units in which the unit of conversion factor to change it from CGS to MKS
length is 25 cm and the unit of mass is 50 g, system is
the numerical value of density of the material
30. T h e p er i od of osci l l at i on of a si mp l e
is:
24. If the screw on a screw-gauge is given six rotations, L
pendulum is T = 2p . Measured value of L
it moves by 3 mm on the main scale. If there are 50 g
divisions on the circular scale the least count is 20.0 cm known to 1 mm accuracy and time for
(in cm) of the screw gauge is: 100 oscillations of the pendulum is found to be
25. Resistance of a given wire is obtained by 90 s using a wrist watch of 1s resolution. The
measuring the current flowing in it and the voltage percentage accuracy in the determination of
difference applied across it. If the percentage errors g is:

ANSWER KEY
1 (c) 4 (b) 7 (b) 10 (a) 13 (a) 16 (d) 19 (c) 22 (32) 25 (6) 28 (20)
2 (b) 5 (a) 8 (d) 11 (d) 14 (c) 17 (b) 20 (c) 23 (40) 26 (4.6) 29 (0.1)
3 (a) 6 (b) 9 (c) 12 (a) 15 (d) 18 (d) 21 (0.2) 24 (0.001) 27 (–1) 30 (3)

@ebooks_for_freee on Telegram
4 PHYSICS

MOTION IN A 2
STRAIGHT LINE

MCQs with One Correct Answer 5. For the velocity time graph shown in the figure
below the distance covered by the body in the
1. The displacement x of a particle varies with last two seconds of its motion is what fraction
time t as x = ae-at + bebt, where a, b, a and b of the total distance travelled by it in all the seven
are positive constants. The velocity of the
seconds?
particle will
(a) be independent of a and b 1
(a) 10
(b) drop to zero when a = b 2 B C
8
(c) go on decreasing with time 1 ­
(b) 6
(d) go on increasing with time 4
velocity
(ms ) 4
–1
2. Which of the following graph cannot possibly 2 2 A D
represent one dimensional motion of a particle? (c)
x v
3 0 1 2 3 4 5 6 7 8
1 time (s)
(d)
(a) t (b) 3
t
6. A particle moves for 20 seconds with velocity 3
v m/s and then with velocity 4 m/s for another 20
seconds and finally moves with velocity 5 m/s
for next 20 seconds. What is the average velocity
(c) t (d) All of the above of the particle ?
(a) 3 m/s (b) 4 m/s
3. A body moving with a uniform acceleration (c) 5 m/s (d) Zero
crosses a distance of 65 m in the 5 th second and 7. The distance travelled by a body moving along
105 m in 9th second. How far will it go in 20 s? a line in time t is proportional to t3.
(a) 2040 m (b) 240 m The acceleration-time (a, t) graph for the motion
(c) 2400 m (d) 2004 m of the body will be
4. When two bodies move uniformly towards each
other, the distance decreases by 6 ms–1. If both a
bodies move in the same directions with the a
same speed (as above), the distance between (a) (b)
them increases by 4 ms–1. Then the speed of the t t
two bodies are
(a) 3 ms–1 and 3 ms–1
a a
(b) 4 ms–1 and 2 ms–1 (c) (d)
(c) 5 ms–1 and 1 ms–1
(d) 7 ms–1 and 3 ms–1 t t

@ebooks_for_freee on Telegram
Motion in a Straight Line 5
8. A goods train accelerating uniformly on a 14. If a body looses half of its velocity on penetrating
straight railway track, approaches an electric pole 3 cm in a wooden block, then how much will it
standing on the side of track. Its engine passes penetrate more before coming to rest?
the pole with velocity u and the guard’s room (a) 1 cm (b) 2 cm
passes with velocity v. The middle wagon of the (c) 3 cm (d) 4 cm.
train passes the pole with a velocity. 15. Consider a rubber ball freely falling from a height
u+v 1 2 h = 4.9 m onto a horizontal elastic plate. Assume
(a) (b) u + v2 that the duration of collision is negligible and
2 2
the collision with the plate is totally elastic.
æ u 2 + v2 ö
Then the velocity as a function of time and the
(c) uv (d) ç ÷
è 2 ø height as a function of time will be :
v y
9. A juggler keeps on moving four balls in the air
+v1
h
throwing the balls after intervals. When one ball
(a) O t
leaves his hand (speed = 20 ms–1) the position –v1
t
of other balls (height in m) will be v y
(Take g = 10 ms–2) +v1
h
(a) 10, 20, 10 (b) 15, 20, 15
(b) O t1 2t1 4t1 t
(c) 5, 15, 20 (d) 5, 10, 20 –v1 t
10. A car, starting from rest, accelerates at the rate f y
through a distance S, then continues at constant
h
speed for time t and then decelerates at the rate
(c) O t1 2t1
t
f
to come to rest. If the total distance traversed t
2 v y
is 15 S, then v1 h

1
(a) S = ft 2 (b) S = f t (d) O t t
6
1 2 1 2
(c) S = ft (d) S = ft 16. The position of a particle as a function of time t,
4 72
is given by
11. A particle located at x = 0 at time t = 0, starts
x(t) = at + bt2 – ct3
moving along with the positive x-direction with
where, a, b and c are constants. When the particle
a velocity 'v' that varies as v = a x . The attains zero acceleration, then its velocity will
displacement of the particle varies with time as be:
(a) t 2 (b) t (c) t 1/2 (d) t 3
b2 b2
12. A person climbs up a stalled escalator in 60 s. If (a) a+ (b) a+
standing on the same but escalator running with 4c 3c
constant velocity he takes 40 s. How much time b2 b2
(c) a+ (d) a +
is taken by the person to walk up the moving c 2c
escalator? 17. A car is standing 200 m behind a bus, which is
(a) 37 s (b) 27 s (c) 24 s (d) 45 s also at rest. The two start moving at the same
13. From a tower of height H, a particle is thrown instant but with different forward accelerations.
vertically upwards with a speed u. The time taken The bus has acceleration 2 m/s2 and the car has
by the particle, to hit the ground, is n times that acceleration 4 m/s2. The car will catch up with
taken by it to reach the highest point of its path. the bus after a time of :
The relation between H, u and n is: (a) (b)
110 s 120 s
(a) 2gH = n2u2 (b) gH = (n –2)2 u2d
2
(c) 2gH = nu (n – 2) (d) gH = (n – 2)u2 (c) 10 2 s (d) 15 s

@ebooks_for_freee on Telegram
6 PHYSICS
18. A person standing on an open ground hears the Numeric Value Answer
sound of a jet aeroplane, coming from north at
an angle 60º with ground level. But he finds the 21. A parachutist after bailing out falls 50 m without friction.
aeroplane right vertically above his position. If When parachute opens, it decelerates at 2 m/s2 . He
v is the speed of sound, speed of the plane is: reaches the ground with a speed of 3 m/s. At what
3 2v height (in m), did he bail out ?
(a) v (b)
2 3 22. An automobile travelling with a speed of 60 km/
v h, can brake to stop within a distance of 20m. If
(c) v (d) the car is going twice as fast i.e., 120 km/h, the
2
19. A passenger train of length 60 m travels at a speed stopping distance (in m) will be
of 80 km/hr. Another freight train of length 120 m 23. The speed verses time graph for a particle is
travels at a speed of 30 km/h. The ratio of times shown in the figure. The distance travelled (in
taken by the passenger train to completely cross m) by the particle during the time interval t = 0 to
the freight train when: (i) they are moving in same t = 5 s will be __________.
direction, and (ii) in the opposite directions is:
11 5 3 25 10
(a) (b) (c) (d) 8
5 2 2 11 u
20. The graph shown in figure shows the velocity v –1 6
(ms )
versus time t for a body. 4
Which of the graphs 2
represents the
1 2 3 4 5
corresponding time
acceleration versus time
(s)
graphs?
24. The distance x covered by a particle in one
dimensional motion varies with time t as x2 = at2
a + 2bt + c. If the acceleration of the particle
(a) t depends on x as x–n, where n is an integer, the
value of n is ______.
25. A ball is dropped from the top of a 100 m high
1
a
tower on a planet. In the last s before hitting
2
(b) t the ground, it covers a distance of 19 m.
Acceleration due to gravity (in ms–2) near the
surface on that planet is _______.
26. An object, moving with a speed of 6.25 m/s, is
a decelerated at a rate given by
(c) t
dv
= -2.5 v where v is the instantaneous
dt
speed. The time (in second) taken by the object,
to come to rest, would be:
a
27. A cat, on seeing a rat at a distant of d = 5 m,
(d) t
starts with velocity u = 5 ms–1 and moves with
acceleration a = 2.5 ms–2 in order to catch it, while

@ebooks_for_freee on Telegram
Motion in a Straight Line 7
the rate with acceleration b starts from rest. For 29. A body is thrown vertically upwards with
what value of b will be the cat overtake the rat ? velocity u. The distance travelled by it in the
(in ms–2) fifth and the sixth seconds are equal. The
28. A particle is moving in a straight line with initial velocity u (in m/s) is given by (g = 9.8 m/s2)
velocity and uniform acceleration a. If the sum 30. If you throw a ball vertically upward with an
initial velocity of 50 m/s, approximately how long
of the distance travelled in t th and (t + 1)th
(in second) would it take for the ball to return to
seconds is 100 cm, then its velocity after t
your hand? Assume air resistance is negligible.
seconds, in cm/s, is

ANSWER KEY
1 (d) 4 (c) 7 (b) 10 (d) 13 (c) 16 (b) 19 (a) 22 (80) 25 (8) 28 (50)
2 (d) 5 (b) 8 (d) 11 (a) 14 (a) 17 (c) 20 (b) 23 (20) 26 (2) 29 (49)
3 (c) 6 (b) 9 (b) 12 (c) 15 (b) 18 (d) 21 (293) 24 (3) 27 (5) 30 (10)

@ebooks_for_freee on Telegram
8 PHYSICS

3
MOTION IN A PLANE

MCQs with One Correct Answer æ 20 ö


5. A particle moves along a circle of radius ç ÷m
r Ù Ù r Ù Ù è pø
1. If A = 3 i + 4 j and B = 7 i + 24 j , the vector with constant tangential acceleration. It the
having the same magnitude as B and parallel to velocity of particle is 80 m/sec at end of second
A is revolution after motion has begun, the tangential
Ù Ù Ù Ù
(a) 5 i + 20 j (b) 15 i + 10 j acceleration is
Ù Ù Ù Ù (a) 40 p m/sec2 (b) 40 m/sec2
(c) 20 i + 15 j (d) 15 i + 20 j
(c) 640 p m/sec2 (d) 160 p m/sec2
2. Two balls are projected at an angle q and
(90º – q) to the horizontal with the same speed. 6. A point P moves in
The ratio of their maximum vertical heights is counter-clockwise
(a) 1 : 1 (b) tanq : 1 direction on a circular
path as shown in the y
(c) 1 : tanq (d) tan2q : 1
figure. The movement
3. A stone projected with a velocity u at an angle q
with the horizontal reaches maximum height H1. of 'P' is such that it B
P(x,y)
When it is projected with velocity u at an angle sweeps out a length s
æp ö = t3 + 5, where s is in 20
m
çè - q÷ø with the horizontal, it reaches maximum metres and t is in
2 x
height H2. The relation between the horizontal seconds. The radius O A
range R of the projectile, heights H1 and H2 is
of the path is 20 m. The acceleration of 'P' when
(a) R = 4 H1H2 (b) R = 4(H1 – H2) t = 2 s is nearly.
H12 (a) 13m/s2 (b) 12 m/s2
(c) R = 4 (H1 + H2) (d) R=
H 22 (c) 7.2 ms2 (d) 14m/s2
gx 2 7. The vectors A and B are such that
4. The equation of a projectile is y = 3x -
2
The angle of projection is given by | A + B |=| A - B |

1 The angle between the two vectors is


(a) tan q = (b) tan q = 3
3 (a) 60° (b) 75°
p (c) 45° (d) 90°
(c) (d) zero.
2

@ebooks_for_freee on Telegram
Motion in a Plane 9
8. Two balls are projected simultaneously in the ur ur
13. Two vectors A and B have equal magnitudes.
same vertical plane from the same point with ur ur
velocities v 1 and v 2 with angle q1 and q 2 ( )
The magnitude of A + B is ‘n’ times the
respectively with the horizontal. If v1 cos q1 = v2 ur ur ur
cos q2, the path of one ball as seen from the ( )
magnitude of A - B . The angle between A and
position of other ball is : ur
B is:
(a) parabola é n 2 - 1ù é n - 1ù
(a) cos -1 ê 2 ú (b) cos -1 ê
(b) horizontal straight line
ë n + 1û ë n + 1úû
(c) vertical straight line é n2 -1ù é n -1ù
(d) straight line making 45° with the vertical (c) sin -1 ê 2 ú (d) sin -1 ê
ë n + 1û ë n + 1úû
9. A projectile with same projection velocity can
14. Ship A is sailing towards north-east with velocity
have the same range ‘R’ for two angles of
km/hr where points east and , north. Ship B is at
projection. If ‘T1’ and ‘T2’ be time of flights in
a distance of 80 km east and 150 km north of
the two cases, then the product of the two time
Ship A and is sailing towards west at 10 km/hr. A
of flights is directly proportional to
will be at minimum distance from B in:
1 (a) 4.2 hrs. (b) 2.6 hrs.
(a) R (b)
R (c) 3.2 hrs. (d) 2.2 hrs.
1 15. Two particles A, B are moving on two
(c) (d) R2 concentric circles of radii R1 and R2 with equal
R2
angular speed w. At t = 0, their positions and
10. A bomber plane moves horizontally with a speed direction of motion are shown in the figure :
of 500 m/s and a bomb released from it, strikes Y
the ground in 10 sec. Angle with the ground at
which it strikes the ground will be (g = 10 m/s2)
A
-1 æ 1 ö æ 1ö
(a) tan ç ÷ (b) tan ç ÷
è 5ø è 5ø R1 X

(c) tan–1 (1) (d) tan–1 (5)


R2 B
11. Starting from the origin at time t = 0, with initial
velocity 5 ˆj ms–1, a particle moves in the x–y
p
plane with a constant acceleration of (10iˆ + 4 ˆj) ®
The relative velocity vA ® and t =
- vB is
2w
ms–2. At time t, its coordiantes are (20 m, y0 m). given by:
The values of t and y0 are, respectively :
(a) w(R1 + R2) iˆ (b) –w(R1 + R2) iˆ
(a) 2 s and 18 m (b) 4 s and 52 m
(c) 2 s and 24 m (d) 5 s and 25 m (c) w(R2 – R1) iˆ (d) w(R1 – R2) iˆ
12. The position vector of a particle changes with r
16. A particle is moving with velocity n = k ( yiˆ + xjˆ) ,
time according to the relation where k is a constant. The general equation for
r
r (t)= 15 t 2 $i + (4 - 20 t 2 ) $j. What is the its path is
(a) y = x2 + constant
magnitude of the acceleration at t = 1?
(b) y2 = x + constant
(a) 40 (b) 25 (c) xy = constant
(c) 100 (d) 50 (d) y2 = x2 + constant

@ebooks_for_freee on Telegram
10 PHYSICS
17. A particle moves such that its position vector H H
r (a) (b)
r (t) = cos wt iˆ + sin wt ĵ where w is a constant 4 2
and t is time. Then which of the following 3H 7H
r (c) (d)
statements is true for the velocity v (t) and 4 8
r
acceleration a (t) of the particle:
r r r Numeric Value Answer
(a) v is perpendicular to r and a is directed ® ®
away from the origin 21. The resultant of two vectors A and B is
r r r ®
(b) v and a both are perpendicular to r perpendicular to the vector A and its magnitude
r r r ®
(c) v and a both are parallel to r is equal to half the magnitude of vector B . The
r r r ®
angle (in degree) between A and B is
®
(d) v is perpendicular to r and a is directed
towards the origin 22. A body is thrown horizontally from the top of a
18. The position of a projectile launched from the tower of height 5 m. It touches the ground at a
r
( )
origin at t = 0 is given by r = 40iˆ + 50 ˆj m at t distance of 10 m from the foot of the tower. The
initial velocity (in ms–1) of the body is (g =
= 2s. If the projectile was launched at an angle q
10 ms–2)
from the horizontal, then q is
23. A particle describes uniform circular motion in a
(take g = 10 ms–2) circle of radius 2 m, with the angular speed of 2
2 3 rad s–1 . The magnitude of the change in its
(a) tan -1 (b) tan
-1
p
3 2 velocity in s is _____m s–1.
2
7 -1 4
(c) tan -1 (d) tan 24. A particle has an initial velocity of 3iˆ + 4 ˆj and
4 5
an acceleration of 0.4 ˆi + 0.3jˆ . Its speed after
19. Two particles are projected simultaneously from
the level ground as shown in figure. They may 10 s is :
collide after a time : Ù Ù Ù
25. If a vector 2 i + 3 j + 8 k is perpendicular to the

vector 4 ˆj - 4iˆ + akˆ , then the value of a is

26. (
A particle moves from the point 2.0iˆ + 4.0 ˆj m , )
(a)
x sin q 2
(b)
x cos q2 ( )
at t = 0, with an initial velocity 5.0iˆ + 4.0 ˆj ms -1.
u1 u2 It is acted upon by a constant force which
x sin q2 2 x sin q1 produces a constant acceleration
(c) u1 sin ( q 2 - q1 ) (d) u 2 sin (q 2 - q1 )
( 4.0iˆ + 4.0 ˆj ) ms -2
. What is the distance (in m)
20. A stone is projected from a horizontal plane. It
of the particle from the origin at time 2s?
attains maximum height H and strikes a
stationary smooth wall and falls on the ground 27. A particle starts from the origin at t = 0 with an
vertically below the maximum height. Assuming initial velocity of 3.0 iˆ m/s and moves in the x-
the collision to be elastic, the height of the point
on the wall where ball will strike is: y plane with a constant acceleration
(6.0 iˆ + 4.0 ˆj) m/s2. The x-coordinate of the
particle at the instant when its y-coordinate is
32 m is D meters. The value of D is:

@ebooks_for_freee on Telegram
Motion in a Plane 11
28. A particle is moving along the x-axis with its about the point ($i + 2 $j + k$ ) m will be x N-m.
coordinate with time ‘t’ given by x(t) = 10 + 8t –
The value of x is ______.
3t2. Another particle is moving along the y-axis
with its coordinate as a function of time given by r r r
30. The sum of two forces P and Q is R such
y(t) = 5 – 8t3. At t = 1 s, the speed of the second
r r
particle as measured in the frame of the first that | R | = | P | . The angle q (in degrees) that
particle is given as v . Then v (in m/s) is____ r r r
® the resultant of 2 P and Q will make with Q
29. A force F = (i$ + 2 $j + 3k$ ) N acts at a point is _______.
(4$i + 3 $j - k$ ) m. Then the magnitude of torque

ANSWER KEY
1 (d) 4 (b) 7 (d) 10 (a) 13 (a) 16 (d) 19 (c) 22 (10) 25 (–0.5) 28 (580)
2 (d) 5 (b) 8 (c) 11 (a) 14 (b) 17 (d) 20 (c) 23 (8) 26 (20Ö2) 29 (195)
3 (a) 6 (d) 9 (a) 12 (d) 15 (c) 18 (c) 21 (150) 24 (7Ö2) 27 (60) 30 (90)

@ebooks_for_freee on Telegram
12 PHYSICS

LAWS OF MOTION
4

MCQs with One Correct Answer 4. A horizontal force of 10 N is necessary to just


hold a block stationary against a wall. The
1. A particle of mass m is moving in a straight line coefficient of friction between the block and the
with momentum p. Starting at time t = 0, a force F wall is 0.2. The weight of the block is
= kt acts in the same direction on the moving (a) 20 N
particle during time interval T so that its (b) 50 N
10N
momentum changes from p to 3p. Here k is a (c) 100 N
constant. The value of T is : (d) 2 N
k 2k 5. A body of mass 2kg slides down with an
p 2p
(a) 2 (b) 2 (c) (d) acceleration of 3m/s2 on a rough inclined plane
p k p k having a slope of 30°. The external force required
2. A rocket with a lift-off mass 3.5 × 104 kg is to take the same body up the plane with the
blasted upwards with an initial acceleration of same acceleration will be: (g = 10m/s2)
10m/s2. Then the initial thrust of the blast is (a) 4N (b) 14N
(c) 6N (d) 20N
(a) 3.5 ´ 10 5 N (b) 7.0 ´ 10 5 N
6. A block of mass m = 10 kg rests on a horizontal
(c) 14.0 ´ 10 5 N (d) 1.75 ´ 10 5 N table. The coefficient of friction between the
3. A mass ‘m’ is supported by a massless string block and the table is 0.05. When hit by a bullet
wound around a uniform hollow cylinder of mass of mass 50 g moving with speed n, that gets
m and radius R. If the string does not slip on the embedded in it, the block moves and comes to
cylinder, with what acceleration will the mass stop after moving a distance of 2 m on the table.
fall or release? If a freely falling object were to acquire speed
2g n
(a) after being dropped from height H, then
3 10
R neglecting energy losses and taking g = 10 ms–2,
g m
(b) the value of H is close to:
2
(a) 0.05 km (b) 0.02 km
5g (c) 0.03 km (d) 0.04 km
(c) 7. A mass of 10 kg is suspended vertically by a
6
m
(d) g rope from the roof. When a horizontal force is
applied on the rope at some point, the rope

@ebooks_for_freee on Telegram
Laws of Motion 13

deviated at an angle of 45°at the roof point. If 13. A block of mass m is connected to another block
the suspended mass is at equilibrium, the of mass M by a spring (massless) of spring
magnitude of the force applied is (g = 10 ms–2) constant k. The block are kept on a smooth
horizontal plane. Initially the blocks are at rest
(a) 200 N (b) 140 N
and the spring is unstretched. Then a constant
(c) 70 N (d) 100 N force F starts acting on the block of mass M to
8. A conical pendulum of length 1 m makes an angle pull it. Find the force on the block of mass m.
q = 45° w.r.t. Z-axis and moves in a circle in the
MF mF
XY plane.The radius of the circle is 0.4 m and its (a) (b)
centre is vertically below O. The speed of the (m + M ) M
pendulum, in its circular path, will be : mF
(Take g = 10 ms–2) Z (c) ( M + m ) F (d)
O m ( m + M)
(a) 0.4 m/s
q 14. A string of negligible mass going over a clamped
(b) 4 m/s
pulley of mass m supports a block of mass M as
(c) 0.2 m/s shown in the figure. The force on the pulley by
C
(d) 2 m/s the clamp is given by
9. A particle of mass 0.3 kg subject to a force
(a) 2 Mg
F = – kx with k = 15 N/m . What will be its initial m
acceleration if it is released from a point 20 cm (b) 2 mg
away from the origin ?
(a) 15 m/s2 (b) 3 m/s2 (c) ( M + m)2 + m 2 g
(c) 10 m/s 2 (d) 5 m/s2 M
10. When forces F1, F2, F3 are acting on a particle
(d) (M + m)2 + M2 g
of mass m such that F2 and F3 are mutually 15. A car is moving along a straight horizontal road
perpendicular, then the particle remains with a speed v0. If the coefficient of friction
stationary. If the force F1 is now removed then between the tyres and the road is m, The shortest
the acceleration of the particle is distance in which the car can be stopped is
(a) F1/m (b) F2F3 /mF1
(c) (F2 - F3)/m (d) F2 /m. v02 v0
(a) (b)
11. A lift is moving down with acceleration a. A man 2 mg mg
in the lift drops a ball inside the lift. The
2
acceleration of the ball as observed by the man æ v0 ö v0
in the lift and a man standing stationary on the (c) ç ÷ (d)
m
è mg ø
ground are respectively
(a) g, g (b) g – a, g – a 16. A uniform metal chain is placed on a rough table
(c) g – a, g (d) a, g such that one end of chain hangs down over the
edge of the table. When one-third of its length
12. Two blocks m1 = 5 gm and m2 = 10 gm are hung
vertically over a light frictionless pulley as hangs down over the edge, the chain starts
shown here. What is the velocity of separation sliding. Then the value of coefficient of static
of the masses after 1 second when they are left friction is
free? [take g = 10 m/s2] 3 1
(a) (b)
(a) 20/3 m/s 4 4
(b) 10/3 m/s 2 1
(c) 5/3 m/s m1 (c) (d)
3 2
(d) 2/3 m/s m2

@ebooks_for_freee on Telegram
14 PHYSICS

17. An insect crawls up a hemispherical surface very is connected to each mass and wraps halfway
slowly (see fig.). The coefficient of friction around a massless and frictionless pulley, as
between the insect and the surface is 1/3. If the shown. The pulley is pulled by horizontal force
line joining the center of the hemispherical of magnitude F = 6mg towards right as shown. If
surface to the insect makes an angle a with the X
vertical, the maximum possible value of a is the magnitude of acceleration of pulley is
given by 2

(a) cot a = 3 m/s2, find the value of 2


X . (Take g = 10 m/s )
a
(b) tan a = 3 F = 6mg
m
(c) sec a = 3
(d) cosec a = 3 2m
18. A ball of mass 0.2 kg is thrown vertically upwards /////////////////////////////////////////////////////////////
by applying a force by hand. If the hand moves 22. A 40 kg slab rests on a frictionless floor as shown
0.2 m while applying the force and the ball goes in the figure. A 10 kg block rests on the top of
upto 2 m height further, find the magnitude of the slab. The static coefficient of friction between
the force. (Consider g = 10 m/s2). the block and slab is 0.60 while the coefficient of
(a) 4 N (b) 16 N kinetic friction is 0.40. The 10 kg block is acted
upon by a horizontal force 100 N. If
(c) 20 N (d) 22 N g = 9.8 m/s2, the resulting acceleration (in m/s2)
19. A person with his hands in his pockets is skating of the slab will be
on ice at the velocity of 10 m/s and describes a 100 N A
10 kg
circle of radius 50 m. What is his inclination with 40 kg B
vertical?
23. Two blocks of masses 5 kg and 3 kg are placed in
-1 æ 1 ö -1 æ 3 ö contact on a horizontal frictionless surface as
(a) tan ç ÷ (b) tan ç ÷
è 10 ø è5ø shown in the figure. A force of 4N is applied on
mass 5 kg. The acceleration (in m/s2) of the mass
æ 1ö
(c) tan -1 (1) (d) tan -1 ç ÷ 3 kg will be
4N
è 5ø 5 kg
3 kg
20. A monkey is decending from the branch of a tree
with constant acceleration. If the breaking 24. The coefficient of friction between a body and
strength is 75% of the weight of the monkey, the the surface of an inclined plane at 45° is 0.5. If g
minimum acceleration with which monkey can = 9.8 m/s2, the acceleration of the body in
slide down without breaking the branch is downwards in m/s2 is
25. A body of mass 0.4 kg is whirled in a vertical
3g
(a) g (b) circle making 2 rev/sec. If the radius of the circle
4 is 1.2 m, then tension (in N) in the string when
g g the body is at the top of the circle, is
(c) (d)
4 2 26. A block starts moving up an inclined plane of
inclination 30° with an initial velocity of v0. It
Numeric Value Answer comes back to its initial position with velocity
21. A block of mass m is placed on top of a block of v0
. The value of the coefficient of kinetic friction
mass 2m which in turn is placed on fixed 2
horizontal surface. The coefficient of friction between the block and the inclined plane is close
between all surfaces is µ = 1. A massless string I
to . The nearest integer to I is _________.
1000

@ebooks_for_freee on Telegram
Laws of Motion 15
27. The minimum velocity (in ms-1) with which a car 29. Two blocks of mass M1 = 20 kg 480 N
driver must traverse a flat curve of radius 150 m and M2 = 12 kg are connected
M1
and coefficient of friction 0.6 to avoid skidding is by a metal rod of mass 8 kg. The
28. The minimum force required to start pushing a system is pulled vertically up by
body up rough (frictional coefficient m) inclined applying a force of 480 N as
plane is F1 while the minimum force needed to shown. The tension (in N) at the
prevent it from sliding down is F2. If the inclined mid-point of the rod is :
plane makes an angle q from the horizontal such
M2
F1
that tan q = 2m then the ratio is
F2 30. A spring balance is attached to the ceiling of a
lift. A man hangs his bag on the spring and the
spring reads 49 N, when the lift is stationary. If
the lift moves downward with an acceleration of
5 m/s2, the reading (in N) of the spring balance
will be

ANSWER KEY
1 (b) 4 (d) 7 (d) 10 (a) 13 (d) 16 (d) 19 (d) 22 (0.98) 25 (71.8) 28 (3)
2 (b) 5 (d) 8 (d) 11 (c) 14 (d) 17 (a) 20 (c) 23 (0.5) 26 (346) 29 (192)
3 (b) 6 (d) 9 (c) 12 (a) 15 (a) 18 (d) 21 (5) 24 (3.47) 27 (30) 30 (24.5)

@ebooks_for_freee on Telegram
16 PHYSICS

WORK, ENERGY AND


5
POWER

MCQs with One Correct Answer 5. A uniform chain of length 2 m is kept on a table
such that a length of 60 cm hangs freely from the
1. A particle moves in a straight line with retardation
edge of the table. The total mass of the chain is
proportional to its displacement. Its loss of
4 kg. What is the work done in pulling the entire
kinetic energy for any displacement x is
proportional to chain (hanging portion) on the table ?
(a) x (b) ex (a) 12 J (b) 3.6 J
(c) x 2 (d) loge x (c) 7.2 J (d) 1200 J
2. A body is moved along a straight line by a 6. The potential energy function for the force
machine delivering constant power. The between two atoms in a diatomic molecule is
distance moved by the body in time ‘t’ is a b
approximately given by U(x) = 12 – 6 where
proportional to x x
(a) t 3/4 (b) t 3/2 (c) t 1/4 (d) t 1/2 a and b are constants and x is the distance between
3. A ball is let to fall from a height h0. There are n the atoms. If the dissociation energy of the
collisions with the earth. If the velocity of molecule is D = [U(x = ¥) – Uat equilibrium], D is
rebound after n collisions is vn and the ball rises b2 b2 b2 b2
to a height hn then coefficient of restitution e is (a) (b) (c) (d)
4a 2a 12 a 6a
given by 7. In the figure shown, a particle of mass m is
hn h0 released from the position A on a smooth track.
(a) e = h (b) e = h
n n
0 n When the particle reaches at B, then normal
reaction on it by the track is
hn h
(c) ne = (d) ne = n (a) mg A
h0 h0 (b) 2mg B
4. Velocity–time graph for a body of mass 10 kg is 2
mg 3h h
shown in figure. Work–done on the body in first (c)
3
two seconds of the motion is :
v (m/s) m 2g
-1 (d)
(a) – 9300 J 50 ms h
8. A 10 H.P. motor pumps out water from a well of
(b) 12000 J depth 20 m and fills a water tank of volume 22380
litres at a height of 10 m from the ground. The
running time of the motor to fill the empty water
(c) –4500 J
tank is (g = 10ms–2)
(a) 5 minutes (b) 10 minutes
(d) –12000 J (0,0) 10s t(s) (c) 15 minutes (d) 20 minutes

@ebooks_for_freee on Telegram
Work, Energy and Power 17
9. Two small particles of equal masses start moving 14. A wedge of mass M = 4m lies on a frictionless
in opposite directions from a point A in a plane. A particle of mass m approaches the
horizontal circular orbit. Their tangential wedge with speed v. There is no friction between
velocities are v and 2v, respectively, as shown the particle and the plane or between the particle
in the figure. Between collisions, the particles and the wedge. The maximum height climbed
move with constant speeds. After making how by the particle on the wedge is given by:
many elastic collisions, other than that at A, these
v2 2v 2
two particles will again reach the point A? (a) (b)
v A g 7g
(a) 4 2v
2v 2 v2
(c) (d)
(b) 3 5g 2g
15. The block of mass M moving on the frictionless
(c) 2
horizontal surface collides with the spring of
(d) 1 spring constant k and compresses it by length
L. The maximum momentum of the block after
10. A spring of spring constant 5 × 103 N/m is
collision is
stretched initially by 5cm from the unstretched
position. Then the work required to stretch it
M
further by another 5 cm is
(a) 12.50 N-m (b) 18.75 N-m
(c) 25.00 N-m (d) 6.25 N-m
kL2 ML2
11. A particle moves in one dimension from rest (a) (b) Mk L (c) (d) zero
under the influence of a force that varies with 2M k
the distance travelled by the particle as shown 16. A mass ‘m’ moves with a velocity ‘v’ and collides
in the figure. The kinetic energy of the particle inelastically with another identical mass. After
after it has travelled 3 m is :
collision the lst mass moves with velocity
v
(a) 4 J 3
in a direction perpendicular to the initial direction
(b) 2.5 J
of motion. Find the speed of the 2 nd mass after
(c) 6.5 J collision.
(d) 5 J m m v
3
th A before Aafter
æ1ö collision
12. A bullet looses ç ÷ of its velocity passing collision
ènø
through one plank. The number of such planks (a) 3v (b) v
that are required to stop the bullet can be: v 2
(c) (d) v
2 2 3 3
n 2n
(a) (b) 17. A particle is moving in a circle of radius r under
2n - 1 n -1
(c) infinite (d) n the action of a force F = ar2 which is directed
towards centre of the circle. Total mechanical
13. At time t = 0 a particle starts moving along the x-
energy (kinetic energy + potential energy) of the
axis. If its kinetic energy increases uniformly with
particle is (take potential energy = 0 for r = 0) :
time ‘t’, the net force acting on it must be
proportional to 1 3 5 3
(a) ar (b) ar
(a) constant (b) t 2 6
1 4 3
(c) (d) t (c) αr (d) ar3
t 3

@ebooks_for_freee on Telegram
18 PHYSICS

18. In a collinear collision, a particle with an initial one of the balls was at rest. After the collision,
speed n 0 strikes a stationary particle of the both the balls move with same speed. What will
same mass. If the final total kinetic energy is be the angle (in degree) between the velocities
50% greater than the original kinetic energy, the of the balls after the collision ?
magnitude of the relative velocity between the
two particles, after collision, is: 24. A car of weight W is on an inclined road that
n0 rises by 100 m over a distance of 1 km and applies
(a) (b) 2n0 W
4 a constant frictional force on the car. While
20
n0 n0 moving uphill on the road at a speed of 10 ms–1,
(c) (d)
2 2 P
the car needs power P. If it needs power while
19. A body of mass 3 kg is under a constant force 2
which causes a displacement s in metre in it, given moving downhill at speed v then value of v
(in ms–1) is:
1
by the relation s = t 3 , where t is in second. Work 25. The potential energy (in joule) of a body of mass
3
2 kg moving in the x – y plane is given by
done by the force in 2 second is
3 U = 6x + 8y, where x and y are in metre.
(a) J (b) 24 J
8 If the body is at rest at point (6m, 4m) at time t = 0,
19 5 it will cross y-axis at time t (in second) equal to
(c) J (d) J
5 19 26. B
20. A running man has half the kinetic energy of C
that of a boy of half of his mass. The man speeds q
up by 1m/s so as to have same K.E. as that of the A
boy. The original speed of the man will be A small block starts slipping down from a point
B on an inclined plane AB, which is making an
(a) 2 m/ s (b) ( )
2 -1 m / s angle q with the horizontal section BC is smooth
and the remaining section CA is rough with a
1 1
m/s coefficient of friction m. It is found that the block
(c)
( 2 -1 ) (d)
2
m/ s
comes to rest as it reaches the bottom (point A)
of the inclined plane. If BC = 2AC, the coefficient
Numeric Value Answer of friction is given by m = k tanq. The value of k
is ______.
21. A particle of mass m moving in the x direction
with speed 2v is hit by another particle of mass 27. A cricket ball of mass 0.15 kg is thrown vertically
2m moving in the y direction with speed v. If the up by a bowling machine so that it rises to a
collision is perfectly inelastic, the percentage maximum height of 20 m after leaving the
loss in the energy during the collision is close machine. If the part pushing the ball applies a
to : constant force F on the ball and moves
horizontally a distance of 0.2 m while launching
22. Four smooth steel balls of equal mass at rest are the ball, the value of F (in N) is (g = 10 ms–2)
free to move along a straight line without friction.
______.
The first ball is given a velocity of 0.4 m/s. It
collides head on with the second elastically, the 28. A particle (m = l kg) slides down a frictionless
second one similarly with the third and so on. track (AOC) starting from rest at a point A (height
The velocity (in m/s) of the last ball is 2 m). After reaching C, the particle continues to
23. A ball collides elastically with another ball of the move freely in air as a projectile. When it
same mass. The collision is oblique and initially reaching its highest point P (height 1 m), the

@ebooks_for_freee on Telegram
Work, Energy and Power 19
kinetic energy of the particle (in J) is: (Figure 29. A body of mass 2 kg is driven by an engine
drawn is schematic and not to scale; take delivering a constant power of 1 J/s. The body
g = 10 ms–2) ______. starts from rest and moves in a straight line. After
9 seconds, the body has moved a distance
Height (in m) ______.
A
P 30. Two bodies of the same mass are moving with
the same speed, but in different directions in a
2m C plane. They have a completely inelastic collision
and move together thereafter with a final speed
which is half of their initial speed. The angle
O between the initial velocities of the two bodies
(in degree) is ______.

ANSWER KEY
1 (c) 4 (c) 7 (a) 10 (b) 13 (c) 16 (d) 19 (b) 22 (0.4) 25 (2) 28 (10.00)
2 (b) 5 (b) 8 (c) 11 (c) 14 (c) 17 (b) 20 (c) 23 (45) 26 (3) 29 (18)
3 (a) 6 (a) 9 (c) 12 (a) 15 (b) 18 (b) 21 (56) 24 (15) 27 (150.00) 30 (120)

@ebooks_for_freee on Telegram
20 PHYSICS

SYSTEM OF PARTICLES 6
AND ROTATIONAL
MOTION

MCQs with One Correct Answer 4. A particle of mass 2 kg is on a smooth


horizontal table and moves in a circular path of
1. The centre of mass of three particles of masses 1
radius 0.6 m. The height of the table from the
kg, 2 kg and 3 kg is at (3, 3, 3) with reference to a
ground is 0.8 m. If the angular speed of the
fixed coordinate system. Where should a fourth
particle is 12 rad s–1, the magnitude of its angular
particle of mass 4 kg be placed so that the centre
momentum about a point on the ground right
of mass of the system of all particles shifts to a
under the centre of the circle is :
point (1, 1, 1) ?
(a) 14.4 kg m2s–1 (b) 8.64 kg m2s–1
(a) (– 1, – 1, – 1) (b) (– 2, – 2, – 2)
(c) 20.16 kg m2s–1 (d) 11.52 kg m2s–1
(c) (2, 2, 2) (d) (1, 1, 1)
5. The moment of inertia of a rod about an axis
2. A loop of radius r and mass m rotating with an
through its centre and perpendicular to it is
angular velocity w0 is placed on a rough
1
horizontal surface. The initial velocity of the ML2 (where, M is the mass and L is the length
12
centre of the hoop is zero.What will be the
of the rod). The rod is bent in the middle so that the
velocity of the centre of the hoop when it ceases
two halves make an angle of 60°. The moment of
to slip ?
inertia of the bent rod about the same axis would be
rw0 rw0
(a) (b) 1 1
4 3 (a) ML2 (b) ML2
48 12
rw0
(c) (d) rw0 1 ML2
2 (c) ML2 (d)
24 8 3
3. Three bricks each of length L and mass M are
arranged as shown from the wall. The distance 6. A solid cylinder rolls up an inclined plane of
of the centre of mass of the system from the wall angle of inclination 30°. At the bottom of the
is inclined plane, the C.M. of the cylinder has a
Wall
(a) L/4 speed of 5 m/s. How long will it take to return to
the bottom?
(b) L/2
L/4 (a) 1.53 sec (b) 9.23 sec
(c) (3/2)L L/2
L (c) 11.11 sec (d) 15.55 sec
(d) (11/12)L

@ebooks_for_freee on Telegram
System of Particles and Rotational Motion 21
7. A ‘T’ shaped object with dimensions shown in (b) it will go downwards just after it is lowered
the figure, is lying on a smooth floor. A force (c) it will go downwards first and then climb
‘ F ’ is applied at the point P parallel to AB, such up
that the object has only the translational motion (d) it will climb upwards and then move
downwards
without rotation. Find the location of P with
respect to C. 11. The moment of inertia of a body about a given axis
l
is 1.2 kg m2. Initially, the body is at rest. In order
3 A B
(a) l m to produce a rotational kinetic energy of 1500
2 joule, an angular acceleration of 25 radian/sec2
2 P
(b) l 2l must be applied about that axis for a duration of
3 F 2m
(a) 4 seconds (b) 2 seconds
(c) l
(c) 8 seconds (d) 10 seconds
4
(d) l 12. A solid sphere of mass M and radius R is placed
3 C
R on a rough horizontal surface. It is struck by a
8. A circular hole of radius is made in a thin horizontal cuestick at a height h above the
4
uniform disc having mass M and radius R, as surface.
shown in figure. The moment of inertia of the The value of h so that the sphere performs pure
remaining portion of the disc about an axis pass-
ing through the point O and perpendicular to rolling motion immediately after it has been struck
the plane of the disc is : is
219 MR 2 J
(a)
256 h
237 MR 2 R
(b) R
512 R/4
O o'
19 MR 2 2R 5R
(c) 3R/4 (a) (b)
512 5 2
2
197 MR
(d) 7R 9R
256 (c) (d)
9. A uniform solid cylindrical roller of mass ‘m’ is 5 5
being pulled on a horizontal surface with force F 13. A hot solid sphere is rotating about a diameter
parallel to the surface and applied at its centre. If at an angular velocity w. If it cools so that its
the acceleration of the cylinder is ‘a’ and it is 1
rolling without slipping then the value of ‘F’ is : radius reduces to of its original value, its
n
5 angular velocity becomes
(a) ma (b) ma
3 w w
3 (a) (b)
(c) ma (d) 2 ma n n2
2
10. A disc is rotated about its axis with a certain (c) wn (d) n2 w
angular velocity and lowered gently on a rough 14. Three rings each of mass M and radius R are
arranged as shown in the figure. The moment of
inclined plane as shown in fig., then
inertia of the system about YY¢ will be
1
µ= Y
3

30º

(a) it will rotate at the position where it was


placed and then will move downwards Y¢

@ebooks_for_freee on Telegram
22 PHYSICS

9 3 pulled with a horizontal force of 40 N, and the


(a) MR 2 (b) MR 2 cylinder is rolling without slipping on a
2 2
horizontal surface (see figure), then the angular
7
(c) 5MR 2 (d) MR 2 acceleration of the cylinder will be (Neglect the
2 mass and thickness of the string)
15. A torque of 30 N-m is applied on a 5 kg wheel 40 N
whose moment of inertia is 2 kg–m2 for 10 sec.
The angle covered by the wheel in 10 sec will be
(a) 750 rad (b) 1500 rad
(a) 20 rad/s2 (b) 16 rad/s2
(c) 3000 rad (d) 6000 rad 2
(c) 12 rad/s (d) 10 rad/s2
16. A mass m hangs with the help of a string wrapped
20. A circular disc of radius R is removed from a
around a pulley on a frictionless bearing. The
bigger circular disc of radius 2R such that the
pulley has mass m and radius R. Assuming pulley circumferences of the discs coincide. The centre
to be a perfect uniform circular disc, the of mass of the new disc is a/R form the centre of
acceleration of the mass m, if the string does not the bigger disc. The value of a is
slip on the pulley, is: (a) 1/4 (b) 1/3
2 (c) 1/2 (d) 1/6
(a) g (b) g
3
g
Numeric Value Answer
3
(c) (d) g
3 2 21. A wheel rotates with a constant acceleration of
17. A solid sphere and solid cylinder of identical 2.0 radian/sec2. If the wheel starts from rest, the
radii approach an incline with the same linear number of revolutions it makes in the first ten
velocity (see figure). Both roll without slipping seconds will be approximately
all throughout. The two climb maximum heights 22. A circular thin disc of mass 2 kg has a diameter
0.2 m. Calculate its moment of inertia about an
hsph
hsph and hcyl on the incline. The ratio is axis passing through the edge tangential to its
hcyl axis and perpendicular to the plane of the disc
given by : (in kg-m2)
23. A stone of mass m, tied to the end of a string, is
whirled around in a horizontal circle (neglect the
force due to gravity). The length of the string is
2 reduced gradually keepin g the an gular
(a) (b) 1 momentum of the stone about the centre of the
5
circle constant. Then, the tension in the string is
14 4
(c) (d) given by T = Arn, where A is a constant, r is the
15 5
instantaneous radius of the circle. The value of
18. A homogeneous solid cylindrical roller of
n is equal to
radius R and mass M is pulled on a cricket
24. A system of uniform cylinders and plates is
pitch by a horizontal force. Assuming rolling
shown in fig. All the cylinders are identical and
without slipping, angular acceleration of the there is no slipping at any contact. The velocity
cylinder is: of lower and upper plates is V and 2V, respectively
3F F as shown in fig. Then the ratio of angular speeds
(a) (b) of the upper cylinders to lower cylinders is
2mR 3m R
2v
F 2F
(c) 2mR
(d) 3m R v
19. A string is wound around a hollow cylinder of
mass 5 kg and radius 0.5 m. If the string is now

@ebooks_for_freee on Telegram
System of Particles and Rotational Motion 23
25. A tennis ball (treated as hollow spherical shell) 27. A long cylindrical vessel is half filled with a liquid.
starting from O rolls down a hill. At point A the When the vessel is rotated about its own vertical
ball becomes air borne leaving at an angle of 30° axis, the liquid rises up near the wall. If the radius
with the horizontal. The ball strikes the ground of vessel is 5 cm and its rotational speed is 2
at B. What is the value of the distance AB
rotations per second, then the difference in the
(in m)? (Moment of inertia of a spherical shell of
heights between the centre and the sides, in cm,
2
mass m and radius R about its diameter = mR 2 ) will be :
3 28. A thin rod of mass 0.9 kg and length 1 m is
O suspended, at rest, from one end so that it can
freely oscillate in the vertical plane. A particle of
move 0.1 kg moving in a straight line with velocity
2.0 m 80 m/s hits the rod at its bottom most point and
sticks to it (see figure). The angular speed
30°
0.2 m A B
(in rad/s) of the rod immediately after the collision
will be ______________.
a 29. A person of 80 kg mass is standing on the rim of
26. A square shaped hole of side l = is carved a circular platform of mass 200 kg rotating about
2
its axis at 5 revolutions per minute (rpm). The
a
out at a distance d = from the centre ‘O’ of a person now starts moving towards the centre of
2 the platform. What will be the rotational speed
uniform circular disk of radius a. If the distance
of the centre of mass of the remaining portion (in rpm) of the platform when the person reaches
its centre __________.
a
from O is - , value of X (to the nearest integer) 30. An massless equilateral triangle EFG of side 'a'
X (As shown in figure) has three particles of mass
is ___________. m situated at its vertices. The moment of inertia
of the system about the line EX perpendicular to
N
EG in the plane of EFG is ma 2 where N is an
20
integer. The value of N is _________.
a X
F
O
d
l = a/2

E a G

ANSWER KEY
1 (b) 4 (a) 7 (d) 10 (a) 13 (d) 16 (b) 19 (b) 22 (0.03) 25 (2.08) 28 (20)
2 (c) 5 (b) 8 (b) 11 (b) 14 (d) 17 (c) 20 (b) 23 (–3) 26 (23.00) 29 (9.00)
3 (d) 6 (a) 9 (c) 12 (c) 15 (a) 18 (d) 21 (16) 24 (3) 27 (2) 30 (25)

@ebooks_for_freee on Telegram
24 PHYSICS

7
GRAVITATION

MCQs with One Correct Answer 4. The mass density of a spherical body is given
k
1. Suppose the gravitational force varies inversely by r (r) = for r < R and r (r) = 0 for r > R,
as the nth power of distance. Then the time r
period of a planet in circular orbit of radius ‘R’ where r is the distance from the centre.
around the sun will be proportional to The correct graph that describes qualitatively
æ n -1 ö the acceleration due to gravity, g of a test
ç ÷
(a) Rn (b) Rè 2 ø particle as a function of r is :
æ n +1 ö æ n -2 ö
ç ÷ ç ÷
(c) Rè 2 ø (d) Rè 2 ø g g
2. A straight rod of length L extends from x = a to x (a) (b)
= L + a. Find the gravitational force it exerts on a
point mass m at x = 0 if the linear density of rod R r R r
µ =A + Bx2 . g
g
éA ù
(a) G m ê + BL ú (c) (d)
ë a û
é æ1 1 ö ù
(b) Gm ê A ç - ÷ + BL ú R r R r
ë è a a + Lø û
5. The change in potential energy, when a body of
é A ù
(c) Gm ê BL + mass m is raised to a height nR from the earth’s
ë a + L úû
é Aù surface is (R = radius of earth)
(d) G m ê BL - ú
ë aû æ n ö
3. Two concentric uniform shells of mass M1 and (a) mgRç ÷ (b) nmgR
ç n – 1÷
M2 are as shown in the figure. A particle of mass è ø
m is located just within the shell M2 on its inner
surface. Gravitational force on ‘m’ due to M1 and æ n2 ö æ n ö
(c) mgR ç ÷ (d) mgR ç ÷
M2 will be ç n 2 +1 ÷ è n +1 ø
è ø
(a) zero M2
GM 1 m 6. Which of the following most closely depicts
(b) a M1 the correct variation of the gravitational
b2
G (M1 + M 2 ) m potential V(r) due to a large planet of radius R
(c) b and uniform mass density ? (figures are not
b2 m
(d) None of these drawn to scale)

@ebooks_for_freee on Telegram
Gravitation 25

Gm( M 2 - M1 )
V(r) (a)
a
V(r)
Gm( M 2 - M1 )
(a) (b) r (b) ( 2 + 1)
O r O a 2

(c) Gm( M 2 - M1 )
V(r) ( 2 - 1)
r V(r) r a 2
(c) O (d)
O Gm( M 2 - M1 )
(d) a
2
7. A satellite of mass m revolves around the earth 10. The depth d at which the value of acceleration
of radius R at a height ‘x’ from its surface. If g is 1
the acceleration due to gravity on the surface of due to gravity becomes times the value at the
n
the earth, the orbital speed of the satellite is surface of the earth, is [R = radius of the earth]
gR 2 gR
æ n -1 ö
(a) (b) R
R+x R-x (a) (b) Rç ÷
n è n ø
æ gR 2 ö 1/ 2
(c) gx (d) çç ÷
÷ R æ n ö
è R + x ø Rç
(c) (d) ÷
8. Three equal masses (each m) are placed at the n 2 è n +1ø
corners of an equilateral triangle of side ‘a’. Then 11. A particle of mass M is situated at the centre of
the escape velocity of an object from the a spherical shell of same mass and radius a. The
circumcentre P of triangle is : a
gravitational potential at a point situated at
4
distance from the centre, will be:
5GM 2 GM
(a) - (b) -
a a
GM 4 GM
(c) - (d) -
a a
12. If the gravitational force between two objects
2 3 Gm 3 Gm were proportional to 1/R (and not as 1/R2) where
(a) (b)
a a R is separation between them, then a particle in
circular orbit under such a force would have its
6 3 Gm 3 3 Gm orbital speed v proportional to
(c) (d)
a a (a) 1/R2 (b) R0
9. Two rings each of radius ‘a’ are coaxial and the (c) R1 (d) 1/R
distance between their centres is a. The masses
of the rings are M1 and M2. The work done in 13. A satellite is moving with a constant speed ‘V’
transporting a particle of a small mass m from in a circular orbit about the earth. An object of
centre C1 to C2 is : mass ‘m’ is ejected from the satellite such that it
M1 M2 just escapes from the gravitational pull of the
earth. At the time of its ejection, the kinetic
a a
energy of the object is
a 1
C1 C2 (a) mV 2 (b) mV 2
2
3
(c) mV 2 (d) 2mV 2
2

@ebooks_for_freee on Telegram
26 PHYSICS

14. From a sphere of mass M and radius R, a smaller The difference between the final and initial total
R energies is:
sphere of radius is carved out such that the
2 GMm GMm
cavity made in the original sphere is between its (a) - (b) +
centre and the periphery (See figure). For the 2R 6R
configuration in the figure where the distance GMm GMm
between the centre of the original sphere and (c) - (d)
6R 2R
the removed sphere is 3R, the gravitational force
18. From a solid sphere of mass M and radius R, a
between the two sphere is:
spherical portion of radius R/2 is removed, as
shown in the figure. Taking gravitational
potential V = 0 at r = ¥, the potential at the centre
of the cavity thus formed is :
(G = gravitational constant)

3R
2
41 GM 41 GM 2
(a) 2 (b)
3600 R 450 R 2
-2GM -2GM
59 GM 2 GM 2 (a) (b)
(c) (d) 3R R
450 R 2 225 R 2
-GM -GM
15. Two particles of equal mass ‘m’ go around a circle (c) (d)
2R R
of radius R under the action of their mutual
19. Two bodies of masses m and 4 m are placed at a
gravitational attraction. The speed of each
distance r. The gravitational potential at a point
particle with respect to their centre of mass is
on the line joining them where the gravitational
Gm Gm field is zero is:
(a) (b)
4R 3R 4Gm 6Gm
Gm Gm (a) - (b) -
(c) (d) r r
2R R
16. The change in the value of ‘g’ at a height ‘h’ 9Gm
(c) - (d) zero
r
above the surface of the earth is the same as at a
depth ‘d’ below the surface of earth. When both 20. Figure shows elliptical path abcd of a planet
around the sun S such that the area of triangle
‘d’ and ‘h’ are much smaller than the radius of
earth, then which one of the following is correct? 1
csa is the area of the ellipse. (See figure) With
3h h 4
(a) d = (b) d = db as the semimajor axis, and ca as the semiminor
2 2
(c) d = h (d) d =2 h axis. If t1 is the time taken for planet to go over
17. A body of mass m is moving in a circular orbit of path abc and t2 for path taken over cda then:
c
radius R about a planet of mass M. At some
instant, it splits into two equal masses. The first d b
R S
mass moves in a circular orbit of radius , and
2
a
3R (a) t1 = 4t2 (b) t1 = 2t2
the other mass, in a circular orbit of radius .
2 (c) t1 = 3t2 (d) t1 = t2

@ebooks_for_freee on Telegram
Gravitation 27

Numeric Value Answer 28. Two satellites of masses m and 2m are revolving
around a planet of mass M with different speeds
21. If the distance of earth is halved from the sun, in orbits of radii r and 2r respectively. The ratio
then the no. of days in a year will be of minimum and maximum forces on the planet
22. Mass M is divided into two parts xM and due to satellites is
(1 – x )M. For a given separation, the value of x
for which the gravitational attraction between
the two pieces becomes maximum is r
23. The mass of the earth is 81 times that of the 2r M
moon and the radius of the earth is 3.5 times that
of the moon. The ratio of the acceleration due to
gravity at the surface of the moon to that at the 29. Take the mean distance of the moon and the sun
surface of the earth is from the earth to be 0.4 × 106 km and 150 × 106
24. The escape velocity for a rocket from earth is km respectively. Their masses are 8 × 1022 kg
11.2 km/sec. Its value on a planet where and 2 × 1030 kg respectively. The radius of the
acceleration due to gravity is double that on the earth is 6400 km. Let DF1 be the difference in the
earth and diameter of the planet is twice that of forces exerted by the moon at the nearest and
earth will be in km/sec farthest points on the earth and DF2 be the
25. The earth is assumed to be a sphere of radius R. difference in the force exerted by the sun at the
A platform is arranged at a height R from the nearest and farthest points on the earth. Then,
surface of the earth. The escape velocity of a DF1
body from this platform is fv, v is its escape the number closest to is:
DF2
velocity from the surface of the earth. The value
of f is 30. The value of acceleration due to gravity is g1
26. The gravitational potential difference between R
at a height h = (R = radius of the earth) from
the surface of a planet and a point 10 m above is 2
4.0 J/kg. The gravitational field (in N/kg) in this the surface of the earth. It is again equal to g1
region, assumed uniform is: and a depth d below the surface of the earth.
27. A geo-stationary satellite orbits around the earth ædö
in a circular orbit of radius 36,000km. Then, the The ratio ç ÷ equals :
è Rø
time period (in hr) of a spy satellite orbiting a
few hundred km above the earth's surface (Rearth
= 6,400km) will approximately be

ANSWER KEY
1 (c) 4 (b) 7 (d) 10 (b) 13 (b) 16 (d) 19 (c) 22 (0.5) 25 (0.70) 28 (0.33)
2 (b) 5 (d) 8 (c) 11 (a) 14 (a) 17 (c) 20 (c) 23 (0.15) 26 (0.40) 29 (2)
3 (b) 6 (c) 9 (c) 12 (b) 15 (a) 18 (d) 21 (129) 24 (22.4) 27 (2) 30 (0.56)

@ebooks_for_freee on Telegram
28 PHYSICS

MECHANICAL 8
PROPERTIES OF SOLIDS

MCQs with One Correct Answer 6. The adjacent graph shows the extension (Dl) of
a wire of length 1m suspended from the top of a
1. The upper end of a wire of diameter 12mm and roof at one end with a load W connected to the
length 1m is clamped and its other end is twisted other end. If the cross-sectional area of the wire
through an angle of 30°. The angle of shear is is 10–6m2, calculate the Young’s modulus of the
(a) 18° (b) 0.18° material of the wire.
(c) 36° (d) 0.36° 4
(a) 2 × 1011 N/m2

(l ×10 )m
3
2. What will be the density of ocean water at a

–4
2
depth where the pressure is 80 atm ? [Given that (b) 2 × 10–11 N/m2
1
its density at the surface is 1.03 × 103 kg/m3, (c) 2 × 10–12 N/m2
compressibility of water = 45.8 × 10–11/Pa.
Given: 1 atm = 1.013 × 105 Pa.] (d) 2 × 10–13 N/m2 20 40 60 80 W(N)

(a) 1.03 × 103 kg/m3 (b) 5.03 × 103 kg/m3 7. The force required to stretch a steel wire of
(c) 8.03 × 103 kg/m3 (d) 9.03 × 103 kg/m3 1 cm2 cross-section to 1.1 times its length would
3. The value of tan (90° – q) in the graph gives be [Y = 2 × 1011 Nm–2]
(a) 2 × 106 N (b) 2 × 103 N
(a) Young's modulus (c) 2 × 10 N –6 (d) 2 × 10–7 N
of elasticity 8. A steel ring of radius r and cross-section area 'A'
(b) compressibility is fitted on to a wooden disc of radius R(R > r). If
Strain

Young's modulus be E, then the force with which


(c) shear strain the steel ring is expanded is
q
(d) tensile strength R æ R-r ö
Stress (a) AE (b) AE ç ÷
4. Two wires are made of the same material and r è r ø
have the same volume. However wire 1 has E æ R-r ö Er
A çè A ÷ø
(c) (d)
cross-sectional area A and wire 2 has cross- AR
sectional area 3A. If the length of wire 1 increases 9. One end of a uniform wire of length L and of
by Dx on applying force F, how much force is weight W is attached rigidly to a point in the
needed to stretch wire 2 by the same amount? roof and W1 weight is suspended from lower
(a) 4 F (b) 6 F end. If A is area of cross-section of the wire, the
(c) 9 F (d) F L
stress in the wire at a height from the upper
5. A beam of metal supported at the two edges is 4
end is
loaded at the centre. The depression at the
centre is proportional to (a) W1 + W (b) W1 + 3W / 4
A A
(a) Y 2 (b) Y W1 + W / 4
(c) 1/Y (d) 1/Y 2 (c) (d) 4W1 + 3W
A A

@ebooks_for_freee on Telegram
Mechanical Properties of Solids 29
10. The length of a metal wire is l1 when the tension 15. Four identical hollow cylindrical columns of mild
in it is T1 and is l2 when the tension is T2. The steel support a big structure of mass 50,000 kg.
original length of the wire is The inner and outer radii of each column are
l1 + l 2 l1T2 + l 2T1 30 cm and 60 cm respectively. Assuming the
(a) (b) load distribution to be uniform, the
2 T1 + T2
compressional strain in each column is [The
l1T2 - l 2T1
(c) (d) T1T2 l1l 2 Young's modulus of steel is 2 × 1011 Pa.]
T2 - T1 (a) 2.31 × 10–7 (b) 4.12 × 10–7
11. If the ratio of diameters, lengths and Young’s (c) 7.21 × 10 –7 (d) 9.93 × 10–7
modulus of steel and copper wires shown in the 16. Two blocks of masses m and M are connected
figure are p, q and s respectively, by means of a metal wire of cross-sectional area
then the corresponding ratio of Steel A passing over a frictionless fixed pulley as
increase in their lengths would be shown in the figure. The system is then released.
2m If M = 2 m, then the stress produced in the wire
7q 5q
(a) (5 sp ) (b) is :
(7 sp 2 ) Copper
7q 2q 2mg
(c) (d) (5 sp )
5m (a)
(5sp 2 ) 3A
12. A 14.5 kg mass, fastened to the end of a steel wire 4mg T
of unstretched length 1m, is whirled in a vertical (b)
3A m
circle with an angular velocity of 2 rev/s at the
mg T
bottom of the circle. The cross-sectional area (c)
of the wire is 0.065 cm2. The elongation of the A
wire when the mass is at the lowest point of 3mg
its path is [Ysteel = 2 × 1011 N/m2] (d) M
4A
(a) 9.67 mm (b) 6.67 mm 17. Two strips of metal are riveted together at their
(c) 1.87 mm (d) 0.12 mm ends by four rivets, each of diameter 6 mm.
13. The elongation of the steel and brass wire in What is the maximum tension that can be
the adjacent figure are respectively. [Unloaded exerted by the riveted strip if the shearing stress
length of steel wire is 1.5 m and of brass wire on the rivet is not to exceed 6.9 × 107 Pa? Assume
is 1m, diameter of each wire that each rivet is to carry one quarter of the
= 0.25 cm. Young's modulus of steel is 2 × 1011 load.
Pa and that of brass is 0.91 × 1011 Pa.] (a) 7800 N (b) 7000 N
(c) 9000 N (d) 1000 N
(a) 1.49 × 10–4 m, 1.3 × 10–4 m 1.5 m Steel 18. Consider a long steel bar under a tensile stress
due to forces F acting at the edges along the
4 kg
(b) 2.94 × 10–4 m, 2.3 × 10–4 m length of the bar (figure). Consider a plane
making an angle q with the length. The tensile
(c) 5.12 × 10–4 m, 3.2 × 10–4 m 1 m Brass
and shearing stresses on this plane are
respectively a
6 kg q
(d) 1.12 × 10–4 m, 6.2 × 10–4 m F
14. The edge of an aluminium cube is 10 cm long. F
One face of the cube is firmly fixed to a vertical
a
wall. A mass of 100 kg is then attached to the F F
opposite face of the cube. The shear modulus (a) cos 2 q, sin 2q
A 2A
of aluminium is 25 G Pa. What is the vertical F F
deflection of this face? (b) sin 2 q, sin 4q
5A 2A
(1Pa = 1N/m2) (g = 10 m/s2). F F
(a) 4 × 10–7 m (b) 3 × 10–7 m (c) sin 3 q, sin 5q
–7 9A 3A
(c) 2 × 10 m (d) 1 × 10–7 m (d) None of these

@ebooks_for_freee on Telegram
30 PHYSICS

19. A bottle has an opening of radius a and length 24. What is the bulk modulus (in Pa) of water for
b. A cork of length b and radius (a + Da) where the given data : Initial volume = 100 litre,
(Da < < a) is compressed to fit into the opening pressure increase = 100 atmosphere, final volume =
completely (see figure). If the bulk modulus of 100.5 litre (1 atmosphere = 1.013 × 105 Pa)
cork is B and frictional coefficient between the 25. The breaking stress of the material of a wire is
bottle and cork is m then the force needed to 6 × 106 Nm–2. Then density r of the material is
push the cork into the bottle is : a 3 × 103 kg m–3. If the wire is to break under its
(a) (pmB b) a b
own weight, the length of the wire (in m) made of
that material should be (take g = 10 ms–2)
(b) (2pmBb) Da 26. A body of mass m = 10 kg is attached to one end of
(c) (pmB b) Da a wire of length 0.3 m. The maximum angular speed
(in rad s–1) with which it can be rotated about its
(d) (4 pmB b) Da other end in space station is (Breaking stress of
20. A copper wire of length 1.0 m and a steel wire of wire = 4.8 × 107 Nm–2 and area of cross-section of
length 0.5 m having equal cross-sectional areas the wire = 10–2 cm2) is _______ .
are joined end to end. The composite wire is 27. A steel wire can sustain 100 kg weight without
stretched by a certain load which stretches the breaking. If the wire is cut into two equal parts,
copper wire by 1 mm. If the Young’s modulii of
each part can sustain a weight (in kg) of
copper and steel are respectively 1.0 × 1011 Nm–2
and 2.0 × 1011 Nm–2, the total extension of the 28. Two steel wires having same length are
composite wire is :
suspended from a ceiling under the same load.
(a) 1.75 mm (b) 2.0 mm
If the ratio of their energy stored per unit volume
(c) 1.50 mm (d) 1.25 mm is 1 : 4, the ratio of their diameters is:
Numeric Value Answer 29. Young’s moduli of two wires A and B are in the
21. A 2 m long rod of radius 1 cm which is fixed from ratio 7 : 4. Wire A is 2 m long and has radius R.
one end is given a twist of 0.8 radians. The shear Wire B is 1.5 m long and has radius 2 mm. If the
strain developed will be two wires stretch by the same length for a given
22. If a rubber ball is taken at the depth of 200 m in a load, then the value of R (in mm) is close to :
pool, its volume decreases by 0.1%. If the
30. The elastic limit of brass is 379 MPa. What should
density of the water is 1 × 103 kg/m3 and g =
10m/s 2 , then the volume elasticity in be the minimum diameter (in mm) of a brass rod if
N/m2 will be it is to support a 400 N load without exceeding its
23. A uniform cube is subjected to volume elastic limit?
compression. If each side is decreased by 1%,
then bulk strain is

ANSWER KEY
9
1 (b) 4 (c) 7 (a) 10 (c) 13 (a) 16 (b) 19 (d) 22 (2×10 ) 25 (200) 28 (1.41)
2 (a) 5 (c) 8 (b) 11 (c) 14 (a) 17 (a) 20 (d) 23 (0.03) 26 (4) 29 (1.75)
9
3 (a) 6 (a) 9 (b) 12 (c) 15 (c) 18 (a) 21 (0.04) 24 (2.026×10 ) 27 (100) 30 (1.15)

@ebooks_for_freee on Telegram
MECHANICAL 9
PROPERTIES OF
FLUIDS

MCQs with One Correct Answer 4. Air of density 1.2 kg m–3 is blowing across the
horizontal wings of an aeroplane in such a way
1. The total weight of a piece of wood is 6 kg. In that its speeds above and below the wings are
1 150 ms–1 and 100 ms–1, respectively. The
the floating state in water its part remains pressure difference between the two sides of
3
inside the water. On this floating piece of wood the wings, is :
what maximum weight is to be put such that the (a) 60 Nm–2 (b) 180 Nm–2
whole of the piece of wood is to be drowned in (c) 7500 Nm–2 (d) 12500 Nm–2
the water? 5. If it takes 5 minutes to fill a 15 litre bucket from a
(a) 15 kg (b) 14 kg
(c) 10 kg (d) 12 kg 2
water tap of diameter cm then the Reynolds
2. A hydraulic automobile lift is designed to lift p
number for the flow is (density of water = 103
cars with a maximum mass of 3000 kg. The area
kg/m3) and viscosity of water = 10–3 Pa s) close
of cross - section of the piston carrying the load to :
is 425 cm2. What maximum pressure would the (a) 1100 (b) 11000
smaller piston have to bear? (c) 550 (d) 5500
(a) 15.82 × 105 Pa (b) 6.92 × 105 Pa 6. Water flows into a large tank with flat bottom
(c) 2.63 × 105 Pa (d) 1.12 × 105 Pa at the rate of 10–4 m3 s–(1) Water is also leaking
3. A U-shaped tube contains a liquid of density r out of a hole of area 1 cm2 at its bottom. If the
and it is rotated about the left dotted line as height of the water in the tank remains steady,
shown in the figure. Find the difference in the then this height is:
levels of liquid column. (a) 5.1 cm (b) 7 cm
w2 L2 (c) 4 cm (d) 9 cm
(a)
2g w 7. A spherical drop of radius R is divided into eight
equal droplets. If surface tension is T, then the
w2 L2 work done in this process is
(b) H
2 2g (a) 2pR2T (b) 3pR2T
2 w2 L2 (c) 4pR2T (d) 2pRT2
(c)
g 8. A U-shaped wire is dipped in a soap solution
L
2 2w2 L2 and removed. The thin soap film formed between
(d)
g the wire and the light slider supports a weight of

@ebooks_for_freee on Telegram
32 PHYSICS

1.5 × 10–2 N (which includes the small weight of l


the slider). The length of the slider is 30 cm. What (a) h
2p
is the surface tension of the film? l 4h
A v1
(a) 2.5 × 10–2 Nm–1 (b) 5.5 × 10–2 Nm–1 (b)
3p
(c) 9.5 × 10–2 Nm–1 (d) 11.5 × 10–2 Nm–1 l v2
(c) B
9. Water rises in a capillary tube to a certain height 3p
such that the upward force due to surface tension l
is balanced by 7.5 × 10–4N force due to the (d)
2p
weight of the liquid. If the surface tension of 13. If the terminal speed of a sphere of gold (density
water is 6 × 10–2Nm–1, the inner circumference = 19.5 kg/m3) is 0.2 m/s in a viscous liquid (den-
of the capillary must be sity = 1.5 kg/m3), find the terminal speed of a
(a) 1.25 × 10–2m (b) 0.50 × 10–2m sphere of silver (density = 10.5 kg/m3) of the
(c) 6.5 × 10–2m (d) 12.5 × 10–2m same size in the same liquid
(a) 0.4 m/s (b) 0.133 m/s
10. On heating water, bubbles being formed at the
(c) 0.1 m/s (d) 0.2 m/s
bottom of the vessel detach and rise. Take the 14. Two tubes of radii r1 and r2, and lengths l1 and
bubbles to be sphere of radius R and making a l2, respectively, are connected in series and a
circular contact of radius r with the bottom of liquid flows through each of them in streamline
the vessel. If r << R and the surface tension of conditions. P1 and P2 are pressure differences
water is T, value of r just before bubbles detach
is: across the two tubes. If P2 is 4P1 and l2 is l1 , then
4
(density of water is rw) the radius r2 will be equal to :
(a) r1 (b) 2r1
r1
R (c) 4r1 (d)
2
15. There is a circular tube in a vertical plane. Two
2r liquids which do not mix and of densities d1 and
rw g 2rw g d2 are filled in the tube. Each liquid subtends 90º
(a) R2 (b) R2 angle at centre. Radius joining their interface
3T 3T
d1
rw g 3rw g makes an angle a with vertical. Ratio is:
(c) R 2
(d) R 2 d2
T T
11. A U tube contains water and methylated spirit
separated by mercury. The mercury columns in
the two arms are in level with 10.0 cm of water in
one arm and 12.5 cm of spirit in the other, the
relative density of spirit is a d2
(a) 0.8 (b) 1.32
(c) 2.38 (d) 3.52
12. A square hole of side length l is made at a depth
of h and a circular hole of radius r is made at a d1
depth of 4h from the surface of water in a water 1 + sin a 1 + cos a
(a) (b)
tank kept on a horizontal surface (See figure). If 1 - sin a 1 - cos a
l << h, r << h and the rate of water flow from the 1 + tan a 1 + sin a
two holes is the same, then r is equal to (c) (d)
1 - tan a 1 - cos a

@ebooks_for_freee on Telegram
Mechanical Properties of Fluids 33
16. A spherical solid ball of volume V is made of a h hr'
(a) (b)
material of density r1. It is falling through a liquid r - r' r
of density r1 (r2< r1). Assume that the liquid hr' hr
applies a viscous force on the ball that is (c) (d)
r - r' r - r'
proportional to the square of its speed 20. A homogeneous solid cylinder of length L (L <
v, i.e., Fviscous = –kv2 (k > 0). The terminal speed of
H/2) cross-sectional area A/5 is immersed such
the ball is
that it floats with its axis vertical at the liquid-
Vg (r1 – r2 ) Vgr1 liquid interface with length L/4 in the denser
(a) (b)
k k liquid as shown in the fig. The lower density
Vg r1 Vg (r1 – r2 ) liquid is open to atmosphere having pressure
(c) (d) P0. Then density of solid (material of cylinder) D
k k
is given by
17. A jar is filled with two non-mixing liquids 1 and 2
having densities r1 and, r2 respectively. A solid 5
(a) d
ball, made of a material of density r3, is dropped 4
in the jar. It comes to equilibrium in the position 4 H/2 d
(b) d 3L/ 4
shown in the figure.Which of the following is 5 L
true for r1, r2and r3? (c) d H/2 2d
d
(d)
r1
5

r3 Numeric Value Answer


21. A cylindrical vessel of height 500 mm has an
orifice (small hole) at its bottom. The orifice is
initially closed and water is filled in it up to
(a) r3 < r1 < r2 (b) r1 > r3 > r2 height H. Now the top is completely sealed with
(c) r1 < r2 < r3 (d) r1 < r3 < r2 a cap and the orifice at the bottom is opened.
Some water comes out from the orifice and the
18. A thin uniform tube is bent into a circle of radius water level in the vessel becomes steady with
r in the vertical plane. Equal volumes of two height of water column being 200 mm. Find the
immiscible liquids, whose densities are r1 and r1 fall in height (in mm) of water level due to opening
(r1 > r2) fill half the circle. The angle q between of the orifice.
the radius vector passing through the common [Take atmospheric pressure = 1.0 × 105 N/m2,
interface and the vertical is density of water = 1000 kg/m3 and g = 10 m/s2.
é p æ r - r öù Neglect any effect of surface tension.]
(a) q = tan -1 ê ç 1 2 ÷ú 22. When a ball is released from rest in a very long
ë 2 è r1 + r 2 øû column of viscous liquid, its downward
acceleration is ‘a’ (just after release). Its
pæ r -r ö
(b) q = tan -1 ç 1 2 ÷ acceleration when it has acquired two third of
2 è r1 + r2 ø the maximum velocity is a/X. Find the value of
X.
ær ö
(c)q = tan -1 p ç 1 ÷ 23. An isolated and charged spherical soap bubble
è r2 ø has a radius r and the pressure inside is
(d) None of above atmospheric. If T is the surface tension of soap
19. A body of density r' is dropped from rest at a
height h into a lake of density r where r > r' solution, then charge on drop is X pr 2rTe 0
neglecting all dissipative forces, calculate the
maximum depth to which the body sinks. find the value of X.

@ebooks_for_freee on Telegram
34 PHYSICS

24. A 20 cm long capillary tube is dipped in water. = 900 kgm–3, g = 10 ms–2] (Give answer in closest
The water rises up to 8 cm. If the entire integer) __________.
arrangement is put in a freely falling elevator the 28. An air bubble of radius 0.1 cm is in a liquid having
length (in m) of water column in the capillary surface tension 0.06 N/m and density 10 3 kg/m3.
tube will be The pressure inside the bubble is 1100 Nm–2
25. A cylinder of height 20 m is completely filled greater than the atmospheric pressure. At what
with water. The velocity of efflux of water (in depth (in m) is the bubble below the surface of
ms–1) through a small hole on the side wall of the the liquid? (g = 9.8 ms–2)
cylinder near its bottom is 29. An open glass tube is immersed in mercury in
26. Two identical charged spheres are suspended such a way that a length of 8 cm extends above
by strings of equal lengths. The strings make an the mercury level. The open end of the tube is
angle of 30° with each other. When suspended then closed and sealed and the tube is raised
in a liquid of density 0.8g cm–3, the angle remains vertically up by additional 46 cm. What will be
the same. If density of the material of the sphere length (in cm) of the air column above mercury in
is 1.6 g cm–3, the dielectric constant of the liquid the tube now?
is (Atmospheric pressure = 76 cm of Hg)
30. The velocity of water in a river is 18 km/h near
27. When a long glass capillary tube of radius 0.015 the surface. If the river is 5 m deep, find the
cm is dipped in a liquid, the liquid rises to a height shearing stress (in N/m2) between the horizontal
of 15 cm within it. If the contact angle between layers of water. The co-efficient of viscosity of
the liquid and glass to close to 0°, the surface water = 10–2 poise.
tension of the liquid, in milliNewton m–1, is [r(liquid)

ANSWER KEY
1 (d) 4 (c) 7 (c) 10 (b) 13 (c) 16 (a) 19 (c) 22 (3) 25 (20) 28 (0.1)
2 (b) 5 (d) 8 (a) 11 (a) 14 (d) 17 (d) 20 (a) 23 (8) 26 (2) 29 (16)
–2
3 (a) 6 (a) 9 (a) 12 (a) 15 (c) 18 (d) 21 (6) 24 (20) 27 (101) 30 (10 )

@ebooks_for_freee on Telegram
THERMAL PROPERTIES 10
OF MATTER

MCQs with One Correct Answer expansion of mercury is 1.82 × 10–4/ºC and linear
expansion of glass is 0.1 × 10–4 /ºC respectively?
1. Two rods, one of aluminum and the other made (a) 21.2 cc (b) 15.2 cc
of steel, having initial length l1 and l2 are (c) 1.52 cc (d) 2.12 cc
connected together to form a single rod of length 5. The coefficient of linear expansion of crystal in
l1 + l2. The coefficients of linear expansion for one direction is a1 and that in every direction
aluminum and steel are aa and as respectively. perpendicular to it is a2 . The coefficient of
If the length of each rod increases by the same cubical expansion is
amount when their temperature are raised by t°C,
(a) a1 + a2 (b) a1 + 2a2
then find the ratio l1/(l1 + l2).
(c) 2a1 + a2 (d) a1 + a2/2
(a) a s/ a a (b) a a/ a s
6. In a vertical U-tube containing a liquid, the two
(c) a s/( a a + a s) (d) a a/( a a + a s) arms are maintained at different temperatures t1
2. If a graph is plotted taking the temperature in and t2. The liquid columns in the two arms have
Fahrenheit along Y-axis and the corresponding heights l1 and l2 respectively. The coefficient of
temperature in Celsius along the X-axis, it will be volume expansion of the liquid is equal to
a straight line l1 – l2
(a) having a + ve intercept on Y-axis (a)
l2 t1 – l1t2
(b) having a + ve intercept on X-axis
l1 – l2
(c) passing through the origin (b) t1
l1t1 – l2 t2
(d) having a – ve intercepts on both the axis t2
l1 + l2 l1
3. A steel rail of length 5 m and area of cross-section (c)
l t +l t l
40 cm2 is prevented from expanding along its 21 12 2

length while the temperature rises by 10°C. If l1 + l2


coefficient of linear expansion and Young’s (d) l t l t
11+ 2 2
modulus of steel are 1.2 × 10–5 K–1 and 2 × 1011 7. In an experiment a sphere of aluminium of mass
Nm–2 respectively, the force developed in the 0.20 kg is heated upto 150°C. Immediately, it is
rail is approximately: put into water of volume 150 cc at 27°C kept in a
(a) 2 × 107 N (b) 1 × 105 N calorimeter of water equivalent to 0.025 kg. Final
(c) 2 × 10 N 9 (d) 3 × 10–5 N temperature of the system is 40°C. The specific
4. A glass flask of volume one litre at 0°C is filled heat of aluminium is :
full with mercury at this temperature. The flask (take 4.2 joule=1 calorie)
and mercury are now heated to 100°C. How much (a) 378 J/kg – °C (b) 315 J/kg – °C
mercury will spill out, if coefficient of volume (c) 476 J/kg – °C (d) 434 J/kg – °C

@ebooks_for_freee on Telegram
36 PHYSICS

8. A black body at 1227°C emits radiations with 12. Three rods of same dimensions are arranged as
maximum intensity at a wavelength of 5000Å. If shown in figure. They have thermal
the temperature of the body is increased by conductivities K1, K2 and K3 . The points P and
1000°C, the maximum intensity will be observed Q are maintained at different temperatures for
at the heat to flow at the same rate along PRQ and
PQ then which of the following option is correct?
(a) 5000Å (b) 6000Å
R
(c) 3000Å (d) 4000Å 1
(a) K3 = ( K1 + K 2 )
9. Two rods of same length transfer a given amount 2
of heat in 12 second, when they are joined as K1 K2
shown in figure (i). But when they are joined as (b) K3 = K1 + K 2
shown in figure (ii), then they will transfer same
heat in same conditions in K1K 2 P Q
(c) K3 = K3
K1 + K2

l
(d) K3 = 2( K1 + K 2 )
l
l
Fig. (i) Fig. (ii) 13. A copper sphere cools from 62°C to 50°C in 10
minutes and to 42°C in the next 10 minutes.
(a) 24 s (b) 13 s Calculate the temperature of the surroundings.
(c) 15 s (d) 48 s (a) 28°C (b) 26°C
10. Two rigid boxes containing different ideal gases (c) 32°C (d) 62°C
are placed on a table. Box A contains one mole
14. The figure shows a system of two concentric
of nitrogen at temperature T0, while Box B
spheres of radii r1 and r2 kept at temperatures
contains one mole of helium at temperature
T1 and T2, respectively. The radial rate of flow of
æ 7ö heat in a substance between the two concentric
çè ÷ø T0 . The boxes are then put into thermal
3 spheres is proportional to
contact with each other, and heat flows between
them until the gases reach a common final ær ö
temperature (ignore the heat capacity of boxes). (a) ln ç 2 ÷
Then, the final temperature of the gases, Tf in è r1 ø r1
T1
terms of T0 is
(r2 - r1 ) r2 T2
(b)
3 7 (r1 r2 )
(a) T f = T0 (b) T f = T0
7 3
(c) (r2 – r1)
3 5
(c) T f = T0 (d) T f = T0 r1 r2
2 2 (d)
(r2 - r1 )
11. A bullet of mass 10gm moving with a speed of 20
m/s hits an ice block of mass 990gm kept on a 15. A thermometer graduated according to a linear
frictionless floor and gets stuck in it. How much scale reads a value x0 when in contact with
ice will melt if 50% of the lost KE goes to ice ? boiling water, and x0/3 when in contact with ice.
(Initial temperature of the ice block = 0°C, J = 4.2 What is the temperature of an object in °C, if this
J/cal and latent heat of ice = 80 cal/g) thermometer in the contact with the object reads
x0/2?
(a) 0.001 gm (b) 0.002 gm
(a) 25 (b) 60
(c) 0.003 gm (d) None of these
(c) 40 (d) 35

@ebooks_for_freee on Telegram
Thermal Properties of Matter 37
16. A long metallic bar is carrying heat from one of

loge (q – q0)
its ends to the other end under steady–state.

loge (q – q0)
The variation of temperature q along the length
x of the bar from its hot end is best described by (a) (b)
which of the following figures?
0 0
q q t t

loge (q – q0)
loge (q – q0)
(a) (b)
x x (c) (d)
q q
0 0
t t

(c) (d) Numeric Value Answer


x x 21. The coefficient of apparent expansion of mercury in
17. 500 g of water and 100 g of ice at 0°C are in a a glass vessel is 153 × 10–6/ºC and in a steel vessel
calorimeter whose water equivalent is 40 g. 10 g is 144 × 10–6/ºC. If a for steel is 12 × 10–6/ºC, then
that of glass (in /°C) is
of steam at 100°C is added to it. Then water in
22. A pendulum clock loses 12 s a day if the
the calorimeter is : (Latent heat of ice = 80 cal/g,
temperature is 40°C and gains 4 s a day if the
Latent heat of steam = 540 cal/ g) temperature is 20° C. The temperature (in °C) at
(a) 580 g (b) 590 g which the clock will show correct time is
(c) 600 g (d) 610 g 23. The temperature of the two outer surfaces of a
composite slab, consisting of two materials
18. Two rods A and B of identical dimensions are at
having coefficient of thermal conductivity K and
temperature 30°C. If A is heated upto 180°C and
2K and thickness x and 4x respectively are T2
B upto T°C, then the new lengths are the same.
and T1 (T2 > T1 ). The rate of heat transfer
If the ratio of the coefficients of linear expansion
through the slab, in a steady state is
of A and B is 4 : 3, then the value of T is :
(a) 230°C (b) 270°C æ A(T2 - T1 )K ö
çè x ÷ø f with f equal to
(c) 200°C (d) 250°C
x 4x
19. A large cylindrical rod of length L is made by
joining two identical rods of copper and steel of T2 K 2K T1
length æç ö÷ each. The rods are completely
L
è 2ø 24. A body cools from 50.0°C to 48°C in 5s. How
insulated from the surroundings. If the free end long (in s) will it take to cool from 40.0°C to 39°C?
of copper rod is maintained at 100°C and that of Assume the temperature of surroundings to be
steel at 0°C then the temperature of junction is 30.0°C and Newton's law of cooling to be valid.
(Thermal conductivity of copper is 9 times that 25. A bakelite beaker has volume capacity of 500 cc
of steel) at 30°C. When it is partially filled with Vm volume
(a) 90°C (b) 50°C (at 30°C) of mercury, it is found that the unfilled
(c) 10°C (d) 67°C volume of the beaker remains constant as
20. A liquid in a beaker has temperature q(t) at time temperature is varied. If g(beaker) = 6 × 10–6 °C–1
t and q0 is temperature of surroundings, then and g(mercury) = 1.5 × 10–4 °C–1, where g is the
according to Newton's law of cooling the correct coefficient of volume expansion, then Vm (in cc)
graph between loge(q – q0) and t is : is close to __________.

@ebooks_for_freee on Telegram
38 PHYSICS

26. M grams of steam at 100°C is mixed with 200 g 29. Two spheres of the same material have radii 1 m
of ice at its melting point in a thermally insulated and 4 m and temperatures 4000 K and 2000 K
container. If it produces liquid water at 40°C [heat respectively. The ratio of the energy radiated
of vaporization of water is 540 cal/ g and heat of per second by the first sphere to that by the
fusion of ice is 80 cal/g], the value of M (in g) is second is
________. 30. At 40oC, a brass wire of 1 mm radius is hung
27. According to Newton’s law of cooling, the rate of from the ceiling. A small mass, M is hung from
cooling of a body is proportional to (Dq)n, where the free end of the wire. When the wire is cooled
Dq is the difference of the temperature of the body down from 40oC to 20oC it regains its original
and the surroundings, and n is equal to length of 0.2 m. The value of M (in kg) is close
28. If the temperature of the sun were to increase to: (Coefficient of linear expansion and Young’s
from T to 2T and its radius from R to 2R, then the modulus of brass are 10–5/oC and 1011 N/m2,
ratio of the radiant energy received on earth to respectively; g = 10 ms–2)
what it was previously will be

ANSW ER KEY
1 (c) 4 (b) 7 (d) 10 (c) 13 (b) 16 (a) 19 (a) 22 (25) 25 (20) 28 (64)
2 (a) 5 (b) 8 (c) 11 (c) 14 (d) 17 (b) 20 (a) 23 (0.33) 26 (40) 29 (1)
–6
3 (b) 6 (a) 9 (d) 12 (c) 15 (a) 18 (a) 21 (9×10 ) 24 (10) 27 (1) 30 (6.28)

@ebooks_for_freee on Telegram
11
THERMODYNAMICS

MCQs with One Correct Answer (a) R (b) 2 R


R R
1. Which of the following is not a thermodynamical (c) (d)
3 4
function 5. Four curves A, B, C and D are drawn in the figure
(a) Enthalpy (b) Work done for a given amount of a gas. The curves which
(c) Gibb’s energy (d) Internal energy represent adiabatic and isothermal changes are
2. A gas can be taken from A to B via two different (a) C and D respectively
processes ACB and ADB. B
(b) D and C respectively P C
P A
D
C B (c) A and B respectively
(d) B and A respectively V
6. One mole of an ideal gas at an initial temperature
of T K does 6R joules of work adiabatically. If
A D the ratio of specific heats of this gas at constant
V pressure and at constant volume is 5/3, the final
When path ACB is used 60 J of heat flows into temperature of gas will be
the system and 30J of work is done by the (a) (T – 4) K (b) (T + 2.4) K
(c) (T – 2.4) K (d) (T + 4) K
system. If path ADB is used work done by the
7. A thermally insulted vessel contains 150 g of
system is 10 J. The heat Flow into the system in
water at 0°C. Then the air from the vessel is
path ADB is :
pumped out adiabatically. A fraction of water
(a) 40 J (b) 80 J turns into ice and the rest evaporates at 0°C
(c) 100 J (d) 20 J itself. The mass of evaporated water will be
3. Unit mass of a liquid with volume V 1 is closed to :
completely changed into a gas of volume V2 at a (Latent heat of vaporization of water = 2.10 ×
constant external pressure P and temperature T. 106 J kg–1 and Latent heat of Fusion of water
If the latent heat of evaporation for the given
= 3.36 × 105 J kg–1)
mass is L, then the increase in the internal energy
of the system is (a) 150 g (b) 20 g
(a) Zero (b) P(V2 – V1) (c) 130 g (d) 35 g
(c) L – P(V2 – V1) (d) L 8. Two Carnot engines A and B are operated in
4. The specific heat capacity of a monoatomic gas series. The engine A receives heat from the
for the process TV2 = constant is (where R is gas source at temperature T1 and rejects the heat to
constant) the sink at temperature T. The second engine B

@ebooks_for_freee on Telegram
40 PHYSICS

receives the heat at temperature T and rejects to undergoes and adiabatic change until the
its sink at temperature T2. For what value of T temperature returns to the initial value. What is
the efficiencies of the two engines are equal? the final volume of the gas?
T1 + T2 T -T
(a) (b) 1 2 (a) 112.4 lit. (b) 115.2 lit
2 2
(c) T 1T 2 (d) T1T2 (c) 120 lit (d) 125 lit
15. The relation between U, P and V for an ideal gas
9. An ideal heat engine works between in an adiabatic process is given by relation U = a
temperatures T1 = 500 K and T2 = 375 K. If the + bP V. Find the value of adiabatic exponent (g)
engine absorbs 600J of heat from the source, of this gas.
then the amount of heat released to the sink is: b +1 b +1
(a) 450 J (b) 600 J (a) (b)
b a
(c) 45 J (d) 500 J a +1 a
10. In a Carnot engine, the temperature of reservoir (c) (d)
b a+b
is 927°C and that of sink is 27°C. If the work 16. A thermodynamic system undergoes cyclic
done by the engine when it transfers heat from process ABCDA as shown in fig. The work
reservoir to sink is 12.6 × 106J, the quantity of done by the system in the cycle is :
heat absorbed by the engine from the reservoir (a) P0V0
is P C B
(b) 2P0V0 3P0
(a) 16.8 × 106 J (b) 4 × 106 J 2P0
(c) 7.6 × 106 J (d) 4.2 × 106 J P0 V0
(c) P0 D
11. A reversible engine converts one-sixth of the 2 A
heat input into work. When the temperature of V0 2V0 V
(d) Zero
the sink is reduced by 62ºC, the efficiency of the
17. An ideal gas goes through
engine is doubled. The temperatures of the
a r eversible cycle V
source and sink are c
a®b®c®d has the V - T b
(a) 99ºC, 37ºC (b) 80ºC, 37ºC diagram shown below.
(c) 95ºC, 37ºC (d) 90ºC, 37ºC Process d®a and b®c are d
a
12. Adiabatic modulus of elasticity of gas is 2.1 × adiabatic. T
105 N/m2. What will be its iosthermal modulus The corresponding P - V diagram for the process
æC ö
is (all figures are schematic and not drawn to
p
of elasticity ? çç C = 1.4 ÷÷ scale) :
è v ø P P
(a) 1.8 × 105 N/m2 (b) 1.5 × 105 N/m2 d c a b
5
(c) 1.4 × 10 N/m 2 (d) 1.2 × 105 N/m2. (a) (b) c
a b d
13. In an adiabatic process, the pressure is increased
2 3 V V
by % . If g = , then the volume decreases by P P
a
3 2 d c b
nearly
(c) a b (d) d c
4 2
(a) % (b) % V V
9 3 18. An ideal monatomic gas with pressure P, volume
9 V and temperature T is expanded isothermally to
(c) 1% (d) % a volume 2V and a final pressure Pi. If the same
4 gas is expanded adiabatically to a volume 2V,
14. Two moles of helium gas (g = 5/3) are initially at P
temperature 27°C and occupy a volume of 20 the final pressure is Pa. The ratio a is
Pi
litres. The gas is first expanded at constant –1/3 1/3
(a) 2 (b) 2
pressure until the volume is doubled. Then, it
(c) 22/3 (d) 2–2/3

@ebooks_for_freee on Telegram
Thermodynamics 41
19. Three samples of the same gas A, B and C 25. A Carnot engine whose efficiency is 50% has an
exhaust temperature of 500 K. If the efficiency is to
æ 3ö
ç g = ÷ have initially equal volume. Now the be 60% with the same intake temperature, the exhaust
è 2ø temperature must be (in K)
volume of each sample is double. The process is 26. An engine takes in 5 mole of air at 20°C and 1
adiabatic for A, Isobaric for B and isothermal for atm, and compresses it adiabaticaly to 1/10th of
C. If the finanl pressures are equal for all the
the original volume. Assuming air to be a
three samples, the ratio of their initial pressure is
diatomic ideal gas made up of rigid molecules,
(a) 2 2 : 2 :1 (b) 2 2 :1: 2 the change in its internal energy during this
(c) 2 :1: 2 (d) 2 :1: 2 process comes out to be X kJ. The value of X to
20. A Carnot engine whose low temperature reservoir the nearest integer is ________.
is at 7°C has an efficiency of 50%. It is desired to 27. Starting at temperature 300 K, one mole of an
increase the efficiency to 70%. By how many ideal diatomic gas (g = 1.4) is first compressed
degrees should the temperature of the high
temperature reservoir be increased? V1
adiabatically from volume V1 to V2 = . It is
(a) 840 K (b) 280 K 16
(c) 560 K (d) 373 K then allowed to expand isobarically to volume
2V2. If all the processes are the quasi-static then
Numeric Value Answer the final temperature of the gas (in °K) is (to the
21. An ideal gas at 27ºC is compressed adiabatically nearest integer) ______.
8 28. A Carnot engine operates between two
to of its original volume. The rise in
27 reservoirs of temperatures 900 K and 300 K. The
æ 5ö engine performs 1200 J of work per cycle. The
temperature (in °C) is ç g = ÷
è 3ø heat energy (in J) delivered by the engine to the
22. During an adiabatic process of an ideal gas, if P low temperature reser voir, in a cycle, is
1 _______.
is proportional to 1.5 , then the ratio of specific
V 29. Two Carnot engines A and B are operated in
heat capacities at constant pressure to that at series. The first one, A receives heat at T1
constant volume for the gas is (= 600 K) and rejects to a reservoir at
23. During an adiabatic process, the pressure of a temperature T2. The second engine B receives
gas is found to be proportional to the cube of its
heat rejected by the first engine and in turn,
absolute temperature. The ratio CP/CV for the rejects to a heat reservoir at T3 (= 400 K).
gas is Calculate the temperature T2 (in K) if the work
24. A Carnot freezer takes heat from water at 0°C outputs of the two engines are equal.
inside it and rejects it to the room at a temperature 30. A heat engine is involved with exchange of heat
of 27°C. The latent heat of ice is 336 × 103 J kg– of 1915 J, – 40 J, +125 J and – Q J, during one
1. If 5 kg of water at 0°C is converted into ice at cycle achieving an efficiency of 50.0%. The
0°C by the freezer, then the energy consumed value of Q (in J) is :
(in J) by the freezer is close to :

ANSWER KEY
1 (b) 4 (a) 7 (b) 10 (a) 13 (a) 16 (d) 19 (b) 22 (1.5) 25 (400) 28 (600)
2 (a) 5 (c) 8 (d) 11 (a) 14 (a) 17 (b) 20 (d) 23 (1.5) 26 (46) 29 (500)
5
3 (c) 6 (a) 9 (a) 12 (b) 15 (a) 18 (d) 21 (402) 24 (1.67 × 10 ) 27 (1818) 30 (980)

@ebooks_for_freee on Telegram
42 PHYSICS

12
KINETIC THEORY

MCQs with One Correct Answer æ an 2 ö


(V - bn) ç P + 2 ÷ = nRT is
ç V ÷ø
1. The pressure is P, volume V and temperature T è
of a gas in the jar A and the other gas in the jar B æ V - nb ö 2 æ V1 - V2 ö
is at pressure 2P, volume V/4 and temperature (a) nRT log e ç 2 ÷ + an ç ÷
2T, then the ratio of the number of molecules in è V1 - nb ø è V1V2 ø
the jar A and B will be æ V - nb ö 2 æ V1 - V2 ö
(a) 1 : 1 (b) 1 : 2 (b) nRT log10 ç 2 ÷ – an ç ÷
(c) 2 : 1 (d) 4 : 1 è V1 - nb ø è V1V2 ø
2. Which one the following graphs represents the æ V - nb ö 2 æ V1 - V2 ö
(c) nRT log e ç 2 ÷ + bn ç ÷
behaviour of an ideal gas at constant è V1 - nb ø è V1V2 ø
temperature?
æ V - nb ö 2 æ V1V2 ö
PV PV (d) nRT log e ç 1 ÷ + an ç ÷
è V2 - nb ø è V1 – V2 ø
(a) (b) 5. A horizontal uniform glass tube of 100 cm, length
V V sealed at both ends contain 10 cm mercury
PV PV
column in the middle. The temperature and
pressure of air on either side of mercury column
(c) (d) are respectively 31°C and 76 cm of mercury. If
V V the air column at one end is kept at 0°C and the
3. One mole of an ideal gas undergoes a process
other end at 273°C, find the pressure of mercury
P0 of air which is at 0°C. (in cm of Hg)
P= 2 Hg
æV ö (a) 194.32 cm
1+ ç 0 ÷
è Vø
(b) 181.5 cm
Here P 0 and V 0 are constant. Change in
temperature of the gas when volume is changed (c) 173.2 cm
10 cm
from V = V0 to V = 2V0 100 cm
(d) 102.4 cm
-2P0 V0 11P0 V0
(a) (b) 6. A gaseous mixture consists of 16 g of helium
5R 10R C
-5P0 V0 and 16 g of oxygen. The ratio p of the mixture
(c) (d) P0V0 Cv
4R is
4. Work done by a system under isothermal change (a) 1.62 (b) 1.59
from a volume V1 to V2 for a gas which obeys
(c) 1.54 (d) 1.4
Vander Waal's equation

@ebooks_for_freee on Telegram
Kinetic Theory 43
7. Two gases-argon (atomic radius 0.07 nm, atomic 273k B
weight 40) and xenon (atomic radius 0.1 nm, (a) 0 (b)
2Mg
atomic weight 140) have the same number 546k B 819k B
density and are at the same temperature. The (c) (d)
ratio of their respective mean free times is closest 3Mg 2Mg
to: 14. Four mole of hydrogen, two mole of helium and
(a) 3.67 (b) 1.83 one mole of water vapour form an ideal gas
(c) 1.09 (d) 4.67 mixture. What is the molar specific heat at
8. The adjoining figure shows graph of pressure constant pressure of mixture?
and volume of a gas at two tempertures T1 and 16 7
T2. Which of the following inferences is correct? (a) R (b) R
P
7 16
(a) T1 > T2 23
(c) R (d) R
(b) T1 = T2 7
(c) T1 < T2 T2 15. One mole of an ideal monatomic gas undergoes
(d) None of these
T1 a process described by the equaton PV3 =
V
9. A jar has a mixture of hydrogen and oxygen constant. The heat capacity of the gas during
gases in the ratio of 1 : 5. The ratio of mean this process is
kinetic energies of hydrogen and oxygen 3
(a) R (b) R
molecules is 2
(a) 1 : 16 (b) 1 : 4 5
(c) 1 : 5 (d) 1 : 1 (c) R (d) 2 R
2
10. In the isothermal expansion of 10g of gas from 16. The temperature, at which the root mean square
volume V to 2V the work done by the gas is 575J. velocity of hydrogen molecules equals their
What is the change in root mean square speed
escape velocity from the earth, is closest to :
of the molecules of the gas at the end of the
process? [Boltzmann Constant kB = 1.38 × 10–23 J/K
(a) 398m/s (b) 520m/s Avogadro Number NA = 6.02 × 1026 /kg
(c) 0 (d) 532m/s Radius of Earth : 6.4 × 106 m
11. The kinetic theory of gases states that the Gravitational acceleration on Earth = 10 ms–2]
average squared velocity of molecules varies (a) 800 K (b) 3 × 105 K
linearly with the mean molecular weight of the (c) 10 K 4 (d) 650 K
gas. If the root mean square (rms) velocity of 17. A gas molecule of mass M at the surface of the
oxygen molecules at a certain temperature is 0.5 Earth has kinetic energy equivalent to 0°C. If it
km/sec. The rms velocity for hydrogen molecules were to go up straight without colliding with
at the same temperature will be : any other molecules, how high it would rise?
(a) 2 km/sec (b) 4 km/sec
Assume that the height attained is much less
(c) 8 km/sec (d) 16 km/sec
12. The root mean square speed of hydrogen than radius of the earth. (kB is Boltzmann
molecules at 300 K is 1930 m/s. Then the root constant).
273k B
mean square speed of oxygen molecules at 900 (a) 0 (b)
K will be. 2Mg
(a) 1930 3 m/s (b) 836 m/s 546k B 819k B
(c) (d)
1930 3Mg 2Mg
(c) 643 m/s (d) m /s 18. For the P-V diagram given for an ideal gas,
3
13. A gas molecule of mass M at the surface of the 1
Earth has kinetic energy equivalent to 0°C. If it P
Constant
were to go up straight without colliding with P=
V
any other molecules, how high it would rise?
Assume that the height attained is much less
than radius of the earth. (kB is Boltzmann 2
constant). V

@ebooks_for_freee on Telegram
44 PHYSICS

Which one correctly represents the T-P diagram? 24. Using equipartition of energy, the specific heat
2
2 (in J kg–1 K–1) of aluminium at room
T temperature can be estimated to be (atomic
(a) T (b) weight of aluminium = 27)
1 1 25. The temperature of an open room of volume 30 m3
P
P increases from 17°C to 27°C due to sunshine. The
T T atmospheric pressure in the room remains 1 × 105
2 1 1 2 Pa. If ni and nf are the number of molecules in the
(c) (d) room before and after heating, then nf – ni will be:
P P
26. Initially a gas of diatomic molecules is contained
19. At room temperature a diatomic gas is found to in a cylinder of volume V1 at a pressure P1 and
have an r.m.s. speed of 1930 ms–1. The gas is: temperature 250 K. Assuming that 25% of the
molecules get dissociated causing a change in
(a) H2 (b) Cl2 (c) O2 (d) F2
number of moles. The pressure of the resulting
20. Three perfect gases at absolute temperatures T1,
gas at temperature 2000 K, when contained in a
T2 and T3 are mixed. The masses of molecules are
volume 2V1 is given by P2. The ratio P2/P1 is
m1, m2 and m3 and the number of molecules are ______.
n1, n2 and n3 respectively. Assuming no loss of 27. The change in the magnitude of the volume of
energy, the final temperature of the mixture is : an ideal gas when a small additional pressure
n1T1 + n2T2 + n3T3 DP is applied at a constant temperature, is the
(a) n1 + n2 + n3 same as the change when the temperature is
reduced by a small quantity DT at constant
n1T12 + n2T22 + n3T32 pressure. The initial temperature and pressure
(b)
n1T1 + n2T2 + n3T3 of the gas were 300 K and 2 atm. respectively. If
n12T12 + n22T22 + n32T32 | DT |= C | DP | , then value of C in (K/atm.) is
(c) __________.
n1T1 + n2T2 + n3T3
28. Nitrogen gas is at 300°C temperature. The
(T1 + T2 + T3 ) temperature (in K) at which the rms speed of a
(d)
3 H2 molecule would be equal to the rms speed of
a nitrogen molecule, is _____________. (Molar
Numeric Value Answer mass of N2 gas 28 g);
21. A cylinder rolls without slipping down an inclined 29. A mixture of 2 moles of helium gas (atomic mass
plane, the number of degrees of freedom it has is = 4u), and 1 mole of argon gas (atomic mass =
22. Internal energy of n 1 mol of hydrogen of 40u) is kept at 300 K in a container. The ratio of
temperature T is equal to the internal energy of their rms speeds
n2 mol of helium at temperature 2T. The ratio
é Vrms ( helium ) ù
n1 ê ú is close to :
n2
is : ë Vrms ( argon ) û
23. One mole of ideal monatomic gas (g = 5/3) is 30. A closed vessel contains 0.1 mole of a monatomic
mixed with one mole of diatomic gas (g = 7/5). ideal gas at 200 K. If 0.05 mole of the same gas at
What is g for the mixture? g denotes the ratio of 400 K is added to it, the final equilibrium
temperature (in K) of the gas in the vessel will be
specific heat at constant pressure, to that at
close to _________.
constant volume
ANSWER KEY
1 (d) 4 (a) 7 (c) 10 (c) 13 (d) 16 (c) 19 (a) 22 (1.2) 25 (–2.5×1025 ) 28 (41)
2 (b) 5 (d) 8 (c) 11 (a) 14 (d) 17 (d) 20 (a) 23 (1.5) 26 (5) 29 (3.16)
3 (b) 6 (a) 9 (d) 12 (b) 15 (a) 18 (c) 21 (2) 24 (925) 27 (150) 30 (266.67)

@ebooks_for_freee on Telegram
Oscillations 45

13
OSCILLATIONS

MCQs with One Correct Answer k k


(a) m2 (b) m1
1. The motion of a particle is given by x = A sinwt +
B coswt. The motion of the particle is
(a) not simple harmonic km1 km 2 m1
(c) (d)
(b) simple harmonic with amplitude (A–B)/2 m 22 m12 m2
(c) simple harmonic with amplitude (A + B)/2
5. In damped oscillations, the amplitude of
(d) simple harmonic with amplitude A + B 2 2 oscillations is reduced to one-third of its inital
value a0 at the end of 100 oscillations. When the
2. Two particles P and Q start from origin and
oscillator completes 200 oscillations, its
execute simple harmonic motion along X-axis
amplitude must be
with same amplitude but with periods 3 seconds
and 6 seconds respectively. The ratio of the (a) a0/2 (b) a0/4
velocities of P and Q when they meet at mean (c) a0/6 (d) a0/9
position is 6. The displacement y(t) = A sin (wt + f) of a particle
(a) 1 : 2 (b) 2 : 1 2p
executing S.H.M. for f = is correctly
(c) 2 : 3 (d) 3 : 2 3
3. A boy is executing Simple Harmonic Motion. At represented by
y y
a displacement x its potential energy is E1 and at
a displacement y its potential energy is E2. If the
potential energy is E at displacement (x + y) then:
(a) (b)
t t
(a) E = E1 - E2
y y
(b) E = E1 + E2
(c) E = E1 + E2
(d) E = E1 - E2 (c) (d)
t t
4. Two masses m1 and m2 are suspended together
by a massless spring of force constant k, as
shown in figure. When the masses are in 7. A particle of mass 1 kg is placed on a platform
equilibrium, mass m 1 is removed without and the platform executes S.H.M. along vertical
disturbing the system. The angular frequency line, along with particle. The amplitude of
of oscillation of mass m2 is oscillation is 5 cm and at topmost position

@ebooks_for_freee on Telegram
46 PHYSICS

particle is just weightless. Maximum speed of on the figure. The simple harmonic motion of
particle is [in m/s] the x-projection of the radius vector of the
3 1 rotating particle P is [radius = B]
(a) (b) y
2 5 2 æ 2pt ö p(t = 0)
(a) x(t ) = B sin ç ÷ T = 30s
p 1 è 30 ø
(c) (d) x
2 7 2 7 æ pt ö B O
8. Two particles are performing simple harmonic (b) x(t ) = B cos ç ÷
è 15 ø
motion in a straight line about the same
equilibrium point. The amplitude and time period æ pt p ö
(c) x(t ) = B sin ç + ÷
for both particles are same and equal to A and T, è 15 3 ø
respectively. At time t=0 one particle has
displacement A while the other one has æ pt p ö
(d) x(t ) = B cos ç + ÷
è 15 3 ø
-A
displacement and they are moving 11. A point particle of mass 0.1 kg is executing S.H.M.
2
of amplitude of 0.1 m. When the particle passes
towards each other. If they cross each other at
through the mean position, its kinetic energy is
time t, then t is:
8 × 10–3 joule. Obtain the equation of motion of
5T T
(a) (b) this particle if this initial phase of oscillation is
6 3 45º.
T T
(c) (d) æ pö
4 6 (a) y = 0.1sin ç 4t + ÷
9. A rod of length l is in motion such that its ends è 4ø
A and B are moving along x-axis and y-axis
æ pö
(b) y = 0.2sin ç 4t + ÷
dq è 4ø
respectively. It is given that = 2 rad/sec
dt
æ pö
always. P is a fixed point on the rod. (c) y = 0.1sin ç 2t + ÷
è 4ø
y
æ pö
(d) y = 0.2sin ç 2t + ÷
B
P
è 4ø
q 12. What do you conclude from the graph about
the frequency of KE, PE and SHM ?
Energy
Total energy
A
x
M A B
KE
Let M be the projection of P on PE
x-axis. For the time interval in which q changes
0 t
p T/4 2T/4 3T/4 4T/4
from 0 to , choose the correct statement.
2
(a) Frequency of KE and PE is double the
(a) The speed of M is always directed towards
frequency of SHM
right
(b) Frequency of KE and PE is four times the
(b) M executes S.H.M.
frequency SHM.
(c) M moves with constant speed
(c) Frequency of PE is double the frequency
(d) M moves with constant velocity
of K.E.
10. Figure shows the circular motion of a particle.
(d) Frequency of KE and PE is equal to the
The radius of the circle, the period, sense of
revolution and the initial position are indicated frequency of SHM.

@ebooks_for_freee on Telegram
Oscillations 47
13. A particle which is simultaneously subjected to after completing 15 oscillations, its energy has
two perpendicular simple harmonic motions become 15 J, its damping constant (in s–1) is:
represented by; x = a1 cos wt and y = a2 cos 2 wt 1 1
traces a curve given by: (a) (b) ln3
2 30
y y
1
(c) 2 (d) ln3
15
a2 a2
18. What is the ratio of the frequencies in the
a1
(a) O
x (b) O a1
x following arrangement of springs?

y y k1

k1 k2
a2 a2
a1 a1 (i) (ii) k2
(c) O
x (d) O
x

m m
14. A flat horizontal board moves up and down k1 + k2 k1 k2
(vertically) in SHM with amplitude A. Then the (a) (b)
k1k2 k1 – k2
shortest permissible time period of the vibration
such that an object placed on the board may not k1k2 – k1 ( k2 – k1 ) k2
loose contact with the board is (c) k1 + k2 (d) k1 + k 2
g A 19. A particle of mass m oscillates with a potential
(a) 2p (b) 2p
A g energy U = U0 + a x2, where U0 and a are
p A constants and x is the displacement of particle
2A
(c) 2p (d) from equilibrium position. The time period of
g 2 g
oscillation is
15. The bob of a simple pendulum executes simple
harmonic motion in water with a period t, while m m
(a) 2p (b) 2p
the period of oscillation of the bob is t0 in air. a 2a
Nglecting frictional force of water and given that 2m m
the density of the bob is (4/3) × 1000 kg/m3. What (c) p (d) 2p
a a2
relationship between t and t0 is true
(a) t = 2t0 (b) t = t0/2 20. A pendulum made of a uniform wire of cross
(c) t = t0 (d) t = 4t0 sectional area A has time period T. When an
16. A particle of mass m is attached to a spring (of additional mass M is added to its bob, the time
spring constant k) and has a natural angular period changes to TM. If the Young's modulus
frequency w0. An external force F(t) proportional 1
to cos wt(w ¹ w0) is applied to the oscillator. The of the material of the wire is Y then is equal
Y
displacement of the oscillator will be proportional
to to: (g = gravitational acceleration)
1 1
(a) 2 2
(b) 2 2 é æ T ö2 ù 2A
m (w 0 + w ) m (w 0 - w ) (a) ê1 - ç M ÷ ú
ë è T ø û Mg
m m
(c) 2 2
(d) 2 2 é æ T ö2 ù 2A
w0 -w (w 0 +w ) ê1 - ç
(b) ÷ ú
17. A pendulum with time period of 1s is losing ëê è TM ø ûú Mg
energy. At certain time its energy is 45 J. If

@ebooks_for_freee on Telegram
48 PHYSICS

éæ T ö2 ù A 25. A block of mass 0.1 kg is connected to an elastic


(c) êç M ÷ - 1ú spring of spring constant 640 Nm –1 and
ëè T ø û Mg oscillates in a medium of constant 10–2 kg s–1.
éæ T ö2 ù Mg The system dissipates its energy gradually. The
(d) êç M ÷ - 1ú time taken for its mechanical energy of vibration
ëè T ø û 2A to drop to half of its initial value, is closest to :

Numeric Value Answer 26. The displacement of a damped harmonic


oscillator is given by x(t) = e–0.1t. cos(10pt + j).
21. A body of mass 0.01 kg executes simple harmonic
Here t is in seconds.
motion about x = 0 under the influence of a force
as shown in figure. The time period (in s) of The time taken for its amplitude of vibration to
S.H.M. is drop to half of its initial value is close to :
F(N) 27. The amplitude of a damped oscillator decreases
to 0.9 times its original magnitude in 5s. In
80
another 10s it will decrease to a times its original
0.2
–0.2
x(m)
magnitude, where a equals
–80 28. A rod of mass 'M' and length '2L' is suspended
at its middle by a wire. It exhibits torsional
22. In an engine the piston undergoes vertical oscillations; If two masses each of 'm' are
simple harmonic motion with amplitude 7 cm. A attached at distance 'L/2' from its centre on both
washer rests on top of the piston and moves sides, it reduces the oscillation frequency by
with it. The motor speed is slowly increased. 20%. The value of ratio m/M is close to :
The frequency (in Hz) of the piston at which the
29. In an experiment to determine the period of a
washer no longer stays in contact with the
simple pendulum of length 1 m, it is attached to
piston, is close to :
different spherical bobs of radii r1 and r2. The
23. A forced oscillation is acted upon by a force
two spherical bobs have uniform mass distribu-
F = F0 sin wt. The amplitude of oscillation is given tion. If the relative difference in the periods, is
by 55 2w2 - 36w + 9. The resonant angular found to be 5 × 10–4 s, the difference in radii,
|r1– r2| is best given by:
frequency is
30. The displacement of a particle varies according
24. A particle undergoing simple harmonic motion
to the relation x = 4(cos pt + sin pt). The amplitude
has time dependent displacement given by of the particle is
pt
x ( t ) = A sin . The ratio of kinetic to potential
90
energy of this particle at t = 210s will be :

ANSWER KEY
1 (d) 4 (a) 7 (b) 10 (a) 13 (a) 16 (b) 19 (b) 22 (1.9) 25 (3.5) 28 (0.37)
2 (b) 5 (d) 8 (d) 11 (a) 14 (b) 17 (d) 20 (c) 23 (9) 26 (7) 29 (0.1)
3 (b) 6 (a) 9 (b) 12 (a) 15 (a) 18 (a) 21 (0.0314) 24 (0.33) 27 (0.729) 30 (4Ö2)

@ebooks_for_freee on Telegram
Waves 49

14
WAVES

MCQs with One Correct Answer (a) 76 Hz (b) 75 Hz


(c) 50 Hz (d) 25 Hz
1. A wave travelling along the x-axis is described 5. Two trains are moving towards each other with
by the equation y(x, t) = 0.005 cos (a x – bt). If speeds of 20m/s and 15 m/s relative to the
the wavelength and the time period of the wave ground. The first train sounds a whistle of
are 0.08 m and 2.0s, respectively, then a and b in frequency 600 Hz. The frequency of the whistle
appropriate units are heard by a passenger in the second train before
(a) a = 25.00 p , b = p the train meets, is (the speed of sound in air is
0.08 2.0 340 m/s)
(b) a = ,b =
p p (a) 600 Hz (b) 585 Hz
0.04 1.0 (c) 645 Hz (d) 666 Hz
(c) a = ,b =
p p 6. A progressive sound wave of frequency 500 Hz
p is travelling through air with a speed of 350 ms–1.
(d) a = 12.50 p , b =
2.0 A compression maximum appears at a place at a
2. A tuning fork of unknown frequency makes 3 given instant. The minimum time interval after
beats/sec with a standard fork of frequency 384 which the rarefraction maximum occurs at the
Hz. The beat frequency decreases when a small same point, is
piece of wax is put on the prong of the first. The 1
(a) 200 s (b) s
frequency of the fork is: 250
(a) 387 Hz (b) 381 Hz 1 1
(c) s (d) s
(c) 384 Hz (d) 390 Hz 500 1000
3. A car emitting sound of frequency 500 Hz speeds 7. When a wave travel in a medium, the particle
towards a fixed wall at 4 m/s. An observer in the displacement is given by the equation y = a sin
car hears both the source frequency as well as 2p (bt– cx) where a, b and c are constants. The
the frequency of sound reflected from the wall. maximum particle velocity will be twice the wave
velocity if
If he hears 10 beats per second between the two
1
sounds, the velocity of sound in air will be (a) c = (b) c = pa
(a) 330 m/s (b) 387 m/s pa
(c) 404 m/s (d) 340 m/s 1
(c) b = ac (d) b =
4. 26 tuning forks are placed in a series such that ac
each tuning fork produces 4 beats with its 8. The number of possible natural oscillation of air
previous tuning fork. If the frequency of last column in a pipe closed at one end of length
85 cm whose frequencies lie below 1250 Hz are :
tuning fork be three times the frequency of first (velocity of sound = 340 ms– 1)
tuning fork then the frequency of first tuning fork (a) 4 (b) 5
will be (c) 7 (d) 6

@ebooks_for_freee on Telegram
50 PHYSICS

9. A hollow pipe of length 0.8 m is closed at one 14. A small speaker delivers 2 W of audio output.
end. At its open end a 0.5 m long uniform string At what distance from the speaker will one detect
is vibrating in its second harmonic and it 120 dB intensity sound ? [Given reference
intensity of sound as 10–12W/m2]
resonates with the fundamental frequency of the
(a) 40 cm (b) 20 cm
pipe. If the tension in the wire is 50 N and the
(c) 10 cm (d) 30 cm
speed of sound is 320 ms–1, the mass of the
string is 15. A travelling wave represented by
(a) 5 grams (b) 10 grams y = A sin (wt – kx) is superimposed on another
(c) 20 grams (d) 40 grams wave represented by y = A sin (wt + kx). The
10. A uniform tube of length 60.5 cm is held vertically resultant is
with its lower end dipped in water. A sound source (a) A wave travelling along + x direction
of frequency 500 Hz sends sound waves into the (b) A wave travelling along – x direction
tube. When the length of tube above water is 16 (c) A standing wave having nodes at
cm and again when it is 50 cm, the tube resonates nl
with the source of sound. Two lowest frequencies x= , n = 0,1, 2....
(in Hz), to which tube will resonate when it is 2
(d) A standing wave having nodes at
taken out of water, are (approximately).
(a) 281, 562 (b) 281, 843 æ 1ö l
x = ç n + ÷ ; n = 0,1, 2....
(c) 276, 552 (d) 272, 544 è 2ø 2
11. Two wires W1 and W2 have the same radius r 16. A wire of length L and mass per unit length
and respective densities r1 and r2 such that r2 6.0 × 10–3 kgm–1 is put under tension of 540 N.
= 4r1. They are joined together at the point O, as Two consecutive frequencies that it resonates
shown in the figure. The combination is used as a at are: 420 Hz and 490 Hz. Then L in meters is:
sonometer wire and kept under tension T. The (a) 2.1 m (b) 1.1 m
point O is midway between the two bridges. When (c) 8.1 m (d) 5.1 m
a stationary waves is set up in the composite wire,
17. An air column in a pipe, which is closed at one
the joint is found to be a node. The ratio of the
end, will be in resonance wtih a vibrating tuning
number of antinodes formed in W1 and W2 is :
r1 r2
fork of frequency 264 Hz if the length of the
column in cm is (velocity of sound = 330 m/s)
W1 O W2
(a) 125.00 (b) 93.75
(a) 1 : 1 (b) 1 : 2 (c) 62.50 (d) 187.50
(c) 1 : 3 (d) 4 : 1 18. When two sound waves travel in the same
12. Two engines pass each other moving in opposite direction in a medium, the displacements of a
directions with uniform speed of 30 m/s. One of particle located at 'x' at time ‘t’ is given by :
them is blowing a whistle of frequency 540 Hz. y1 = 0.05 cos (0.50 px – 100 pt)
Calculate the frequency heard by driver of y2 = 0.05 cos (0.46 px – 92 pt)
second engine before they pass each other. where y1, y2 and x are in meters and t in seconds.
Speed of sound is 330 m/sec: The speed of sound in the medium is :
(a) 450 Hz (b) 540 Hz (a) 92 m/s (b) 200 m/s
(c) 270 Hz (d) 648 Hz (c) 100 m/s (d) 332 m/s
13. A motor cycle starts from rest and accelerates
19. A source of sound emits sound waves at
along a straight path at 2m/s2. At the starting
frequency f0. It is moving towards an observer
point of the motor cycle there is a stationary
electric siren. How far has the motor cycle gone with fixed speed vs (vs < v, where v is the
when the driver hears the frequency of the siren speed of sound in air). If the observer were to
at 94% of its value when the motor cycle was at move towards the source with speed v0, one
rest? (Speed of sound = 330 ms–1) of the following two graphs (A and B) will
(a) 98 m (b) 147 m given the correct variation of the frequency f
(c) 196 m (d) 49 m heard by the observer as v0 is changed.

@ebooks_for_freee on Telegram
Waves 51
23. An organ pipe P1 closed at one end vibrating in
(A) (B) its first overtone and another pipe P2 open at
f f both ends vibrating in third overtone are in
resonance with a given tuning fork. The ratio of
v0 1/v0 the length of P1 to that of P2 is
The variation of f with v0 is given correctly by: 24. A string of length 0.4 m and mass 10–2 kg is
f0 clamped at its ends. The tension in the string
(a) graph A with slope = is1.6 N. Identical wave pulses are generated at
(v + vs ) one end at regular intervals of time, Dt. The
f0 minimum value of Dt, so that a constructive
(b) graph B with slope =
(v – vs ) interference takes place between successive
f0 pulses is
(c) graph A with slope = 25. A source of sound of frequency 256Hz is moving
(v – vs )
rapidly towards a wall with a velocity of 5m/s.
f0 The speed of sound is 330 m/s. If the observer is
(d) graph B with slope =
(v + vs ) between the wall and the source, then beats per
20. A string is stretched between fixed points second heard will be
separated by 75.0 cm. It is observed to have 26. A bat moving at 10 ms–1 towards a wall sends a
resonant frequencies of 420 Hz and 315 Hz. There sound signal of 8000 Hz towards it. On reflection
are no other resonant frequencies between these it hears a sound of frequency f. The value of f in
two. Then, the lowest resonant frequency for Hz is close to (speed of sound = 320 ms–1)
this string is 27. Two whistles A and B produce tones of
(a) 105 Hz (b) 1.05 Hz frequencies 660 Hz and 596 Hz respectively.
There is a listener at the mid-point of the line
(c) 1050 Hz (d) 10.5 Hz joining them. Now the whistle B and the listener
both start moving with speed 30 m/s away from
Numeric Value Answer
the whistle A. If the speed of sound be 330 m/s,
21. Equation of a stationary and a travelling waves how many beats will be heard by the listener?
are as follows y1 = a sin kx cos wt and y2 = a sin 28. A wire of density 9 × 10–3 kg cm–3 is stretched
(wt – kx). The phase difference between two between two clamps 1 m apart. The resulting
p 3p strain in the wire is 4.9 × 10–4. The lowest
points x1 = and x2 = is f1 in the
3k 2k frequency (in Hz) of the transverse vibrations in
standing wave (y1) and is f2 in travelling wave the wire is
f (Young’s modulus of wire Y = 9 × 1010 Nm–2),
(y2) then ratio 1 is (to the nearest integer), ___________.
f2
22. The amplitude of a wave disturbance 29. A one metre long (both ends open) organ pipe
propagating in the positive y-direction is given is kept in a gas that has double the density of
1 air at STP. Assuming the speed of sound in air
by y = at time t = 0 and at STP is 300 m/s, the frequency difference
1 + x2 between the fundamental and second harmonic
1
y= at t = 2 s, where x and y are in of this pipe is ______ Hz.
é1 + ( x - 1) 2 ù 30. Two identical strings X and Z made of same
ë û
meters. The shape of the wave disturbance does material have tension TX and TZ in them. If their
not change during the propagation. The velocity fundamental frequencies are 450 Hz and 300 Hz,
of wave in m/s is respectively, then the ratio TX/TZ is:
ANSWER KEY
1 (a) 4 (c) 7 (a) 10 (d) 13 (a) 16 (a) 19 (c) 22 (0.5) 25 (7.7) 28 (35.00)
2 (a) 5 (d) 8 (d) 11 (b) 14 (a) 17 (b) 20 (a) 23 (0.375) 26 (8516) 29 (106)
3 (c) 6 (d) 9 (b) 12 (d) 15 (d) 18 (b) 21 (0.85) 24 (0.1) 27 (4) 30 (2.25)

@ebooks_for_freee on Telegram
52 PHYSICS

ELECTRIC CHARGES 15
AND FIELDS

MCQs with One Correct Answer 4. Five point charges, each of value +q, are placed
1. A solid conducting sphere of on five vertices of a regular hexagon of side L.
radius a has a net positive charge The magnitude of the force on a point charge of
2Q. A conducting spherical shell value –q coulomb placed at the center of the
of inner radius b and outer radius a hexagon is
b 2 2
c is concentric with the solid c
1 æ qö 2 æ qö
sphere and has a net charge – Q. (a) ç ÷ (b) ç ÷
pe 0 è L ø pe 0 è L ø
The surface charge density on the inner and outer
2 2
surfaces of the spherical shell will be 1 æ qö 1 æ qö
(c) ç ÷ (d) ç ÷
2Q Q Q Q 2pe 0 è L ø 4pe 0 è L ø
(a) - 2
, 2 (b) - 2
,
4pb 4pc 4pb 4pc 2 5. Two balls of same mass and carrying equal
Q charge are hung from a fixed support of length l.
(c) 0, (d) None of the above At electrostatic equilibrium, assuming that
4pc 2 angles made by each thread is small, the
2. Two spheres carrying charges +6µC and + 9µC,
separation, x between the balls is proportional
separated by a distance d, experiences a force of
to :
repulsion F. when a charge of – 3µC is given to
both the sphere and kept at the same distance (a) l (b) l 2
as before, the new force of repulsion is (c) l 2/3 (d) l 1/3
(a) 3 F (b) F/9 6. Three charges +Q1, +Q2 and q are placed on a
(c) F (d) F/3
straight line such that q is somewhere in
3. Two equally charged, identical metal spheres A
between +Q1 and +Q2. If this system of charges
and B repel each other with a force ‘F’. The
is in equilibrium, what should be he magnitude
spheres are kept fixed with a distance ‘r’ between
and sign of charge q?
them. A third identical, but uncharged sphere C
is brought in contact with A and then placed at Q1Q2 Q1 + Q2
(a) ,+ (b) ,–
the mid point of the line joining A and B. The 2
magnitude of the net electric force on C is ( Q1 + Q2 ) 2

3F Q1Q 2 Q1 + Q2
(a) F (b) (c) ,– (d) ,–
( )
4 2
Q1 + Q2 2
F F
(c) (d)
2 4

@ebooks_for_freee on Telegram
Electric Charges and Fields 53
7. Three point charges +q, –2q and +q are placed 11. Three positive charges of equal value q are placed
at points (x = 0, y = a, z = 0), (x = 0, y = 0, z = 0) and at vertices of an equilateral triangle. The resulting
(x = a, y = 0, z = 0) respectively. The magnitude lines of force should be sketched as in
and direction of the electric dipole moment
vector of this charge assembly are
(a) 2qa along the line joining points (x = 0,
(a) (b)
y = 0, z = 0) and (x = a, y = a, z = 0)
(b) qa along the line joining points (x = 0, y = 0,
z = 0) and (x = a, y = a, z = 0)
(c) 2qa along +ve x direction
(c) (d)
(d) 2qa along +ve y direction
8. Three identical point charge, each of mass m
and charge q, hang from three strings as shown 12. Two point charges + 8q and –2q are located at x
in Fig. The value of q in terms of m, L and q is = 0 and x = L respectively. The location of a
point on the x-axis at which the net electric field
due to these two point charges is zero
q q (a) 8 L (b) 4 L
L g
L
L
(c) 2 L (d)
+q
m
+q +q
4
m m
13. A thin disc of radius b = 2a has a concentric
hole of radius ‘a’ in it (see figure). It carries
(a) q = (16 / 5 ) pe0 mgL2 sin 2 q tan q uniform surface charge ‘s’ on it. If the electric
field on its axis at height ‘h’ (h << a) from its
(b) q = (16 /15) pe 0 mgL2 sin 2 q tan q
centre is given as ‘Ch’ then value of ‘C’ is :
(c) q = (15 /16 ) pe 0 mgL2 sin 2 q tan q
(d) None of these
3R
9. The electric field at a distance from the
2
s s
centre of a charged conducting spherical shell (a) (b)
of radius R is E. The electric field at a distance 4aÎ0 8aÎ0

R s s
from the centre of the sphere is (c) (d)
2 aÎ0 2aÎ0
E 14. A square surface of side L metres is in the plane
(a) (b) zero
2 of the paper. A uniform electric field E (volt /m),
E also in the plane of the paper, is limited only to
(c) E (d)
2 the lower half of the square surface (see figure).
10. Three concentric metallic spherical shells of radii The electric flux in SI units associated with the
R, 2R, 3R, are given charges Q1 , Q2 , Q3 , surface is
respectively. It is found that the surface charge
(a) EL2/2
densities on the outer surfaces of the shells are
equal. Then, the ratio of the charges given to (b) zero
E
the shells, Q1 : Q2 : Q3, is (c) EL2
(a) 1 : 2 : 3 (b) 1 : 3 : 5 (d) EL2 / (2e0)
(c) 1 : 4 : 9 (d) 1 : 8 : 18

@ebooks_for_freee on Telegram
54 PHYSICS
r
15. Consider an electric field E = E0 xˆ where E0 is a 17. A charged ball B hangs from a
constant. The flux through the shaded area (as silk thread S, which makes an P
angle q with a large charged q
shown in the figure) due to this field is S
z
conducting sheet P, as shown
in the figure. The surface charge
(a,0,a) (a,a,a) den sity s of the sheet is B
proportional to
(a) cot q (b) cos q
y (c) tan q (d) sin q
x
(0,0,0) (0,a,0) ur ur
18. Two point dipoles of dipole moment p1 and p 2
(a) 2E0a2 (b) 2E0 a 2 ur ur
are at a distance x from each other and p1 || p 2 .
E0 a 2 The force between the dipoles is :
(c) E0 a2 (d)
2
1 4 p1 p2 1 3 p1 p2
16. A simple pendulum of length L is placed between (a) (b)
4pe0 x 4 4pe0 x3
the plates of a parallel plate capacitor having
electric field E, as shown in figure. Its bob has
1 6 p1 p2 1 8 p1 p2
mass m and charge q. The time period of the (c) (d)
pendulum is given by : 4pe0 x 4 4pe0 x 4

19. If the electric flux entering and leaving an enclosed


surface respectively is f1 and f2, the electric charge
inside the surface will be
(a) (f2 – f1)eo (b) (f1 – f2)/eo
(c) (f2 – f1)/eo (d) (f1 – f2)eo
20. An electric dipole is placed at an angle of 30° to
a non-uniform electric field. The dipole will
experience
(a) a translational force only in the direction of
the field
L (b) a translational force only in a direction
(a) 2p
æ qE ö normal to the direction of the field
çg+ ÷
è m ø (c) a torque as well as a translational force
(d) a torque only
L
(b) 2p
q2 E2 Numeric Value Answer
g2 -
m2 21. An electric dipole, consisting of two opposite
charges of 2 × 10–6 C each separated by a
L
(c) 2p distance 3 cm is placed in an electric field of
æ qE ö 2 × 105 N/C. Maximum torque (in Nm) acting on
çg- ÷
è m ø the dipole is
22. A pendulum bob of mass 30.7 × 10–6 kg carrying
L a charge 2 × 10–8 C is at rest in a horizontal
(d) 2p
2 uniform electric field of 20000 V/m. The tension
æ qE ö
g2 + ç ÷ (in N) in the thread of the pendulum is (g = 9.8
è m ø
m/s2)

@ebooks_for_freee on Telegram
Electric Charges and Fields 55
23. A liquid drop having 6 excess electrons is kept 29. Figure shows five charged lumps of plastic. The
stationary under a uniform electric field of cross-section of Gaussian surface S is indicated.
25.5 kVm–1. The density of liquid is 1.26 × 103 kg Assuming q1 = q4 = 3.1 nC, q2 = q5 = – 5.9 nC, and
m–3. The radius (in m) of the drop is (neglect q3 = – 3.1 nC, the net electric flux (in Nm2/C)
buoyancy). through the surface is
24. The electric field in a region of space is given S
r
by, E = E ˆi + 2E ˆj where E = 100 N/C. The
o o o
q1 q3

flux (in Nm2/C) of the field through a circular


q5
surface of radius 0.02 m parallel to the Y-Z plane q4
q2
is nearly:
25. A charge Q is placed at each of the opposite
corners of a square. A charge q is placed at each 30. Consider a sphere of radius R which carries a
of the other two corners. If the net electrical force uniform charge density r. If a sphere of radius
on Q is zero, then Q/q equals: ur
26. A sphere of radius R carries charge such that its R EA
volume charge density is proportional to the is carved out of it, as shown, the ratio ur
2 EB
square of the distance from the center. What is ur ur
the ratio of the magnitude of the eletric field at a of magnitude of electric field E A and E B ,
distance 2R from the center to the magnitude of respectively, at points A and B due to the
the electric field at a distance of R/2 from the remaining portion is:
center?
27. The surface charge density of a thin charged
disc of radius R is s. The value of the electric
s
field at the centre of the disc is . With
2 Î0
respect to the field at the centre, the electric field
along the axis at a distance R from the centre of
the disc reduces by %
28. A solid sphere of radius R has a charge Q
distributed in its volume with a charge density r
= kr a, where k and a are constants and r is the
distance from its centre.
R 1
If the electric field at r = is times that at r
2 8
= R, find the value of a.

ANSWER KEY
–4 (–2 2)
1 (a) 4 (d) 7 (a) 10 (b) 13 (a) 16 (d) 19 (a) 22 (5 × 10 ) 25 28 (2)
–7
2 (d) 5 (d) 8 (a) 11 (c) 14 (b) 17 (c) 20 (c) 23 (7.8 × 10 ) 26 (2) 29 (–670)
–3
3 (a) 6 (c) 9 (b) 12 (c) 15 (c) 18 (c) 21 (12 × 10 ) 24 (0.125) 27 (70.7%) 30 (0.53)

@ebooks_for_freee on Telegram
56 PHYSICS

ELECTROSTATIC 16
POTENTIAL AND
CAPACITANCE

MCQs with One Correct Answer The force experienced by a negative charge of
2 × 10–6 C located at x = 0.5 m is
1. Three charges 2 q, – q and – q are located at the
(a) 2 × 10–6 N (b) 4 × 10–6 N
vertices of an equilateral triangle. At the centre
(c) 6 × 10–6 N (d) 8 × 10–6 N
of the triangle
5. The electric potential V at any point O (x, y, z all
(a) the field is zero but potential is non-zero
in metres) in space is given by V = 4x2 volt. The
(b) the field is non-zero, but potential is zero
electric field at the point (1 m, 0, 2 m) in
(c) both field and potential are zero volt/metre is
(d) both field and potential are non-zero (a) 8 along negative X-axis
2. The 1000 small droplets of water each of radius r (b) 8 along positive X-axis
and charge Q, make a big drop of spherical shape.
(c) 16 along negative X-axis
The potential of big drop is how many times the
(d) 16 along positive Z-axis
potential of one small droplet ?
6. Two conducting spheres of radii R1 and R2
(a) 1 (b) 10
having charges Q1 and Q2 respectively are
(c) 100 (d) 1000 connected to each other. There is
3. Three charges Q, + q and + q are placed at the (a) no change in the energy of the system
vertices of a right-angle isosceles triangle as (b) an increase in the energy of the system
shown below. The net electrostatic energy of
(c) always a decrease in the energy of the
the configuration is zero, if the value of Q is : system
Q
(d) a decrease in the energy of the system
unless Q1R2 = Q2R1
7. The potential to which a conductor is raised,
depends on
(a) The amount of charge
(b) Geometry and size of the conductor
+q +q
(c) Both (a) and (b)
- 2q (d) None of these
(a) + q (b)
2 +1 8. Calculate the area of the plates of a one farad
-q parallel plate capacitor if separation between
(c) (d) –2q plates is 1 mm and plates are in vacuum
1+ 2
4. The electric potential V (in Volt) varies with x (in (a) 18 × 108 m2 (b) 0.3 × 108 m2
metres) according to the relation V = (5 + 4x2). (c) 1.3 × 10 m8 2 (d) 1.13 × 108 m2

@ebooks_for_freee on Telegram
Electrostatic Potential and Capacitance 57
9. A parallel plate capacitor is charged and then
isolated. What is the effect of increasing the plate
separation on charge, potential, capacitance, d
respectively? K
(a) Constant, decreases, decreases
a
(b) Increases, decreases, decreases
(c) Constant, decreases, increases K Î0 a 2 K Î0 a 2
(d) Constant, increases, decreases (a) (b) In K
2d (K + 1) d (K – 1)
10. Three capacitors C1, C2 and C3 are connected
to a battery as shown. With symbols having their
K Î0 a 2 1 K Î0 a 2
usual meanings, the correct conditions are (c) In K (d)
d 2 d
V 14. A parallel plate capacitor having capacitance
Q2 2 C2

Q1
V1
C1
12 pF is charged by a battery to a potential
difference of 10 V between its plates. The
Q3 C3
charging battery is now disconnected and a
V3
porcelain slab of dielectric constant 6.5 is
V slipped between the plates. The work done by
the capacitor on the slab is:
(a) Q1 = Q2 = Q3 and V1 = V2 = V (a) 692 pJ (b) 508 pJ
(b) V1 = V2 = V3 = V (c) 560 pJ (d) 600 pJ
(c) Q1 = Q2 + Q3 and V = V1 + V2 15. A capacitor C is fully charged with voltage V0.
(d) Q2 = Q3 and V = V2 + V3 After disconnecting the voltage source, it is
connected in parallel with another uncharged
11. A capacitor with capacitance 5mF is charged to
5 mC. If the plates are pulled apart to reduce the C
capacitor of capacitance . The energy loss in
capacitance to 2 ¼F, how much work is done? 2
the process after the charge is distributed
(a) 6.25 × 10–6 J (b) 3.75 × 10–6 J
–6 between the two capacitors is :
(c) 2.16 × 10 J (d) 2.55 × 10–6 J
1 1
12. Four equal point charges Q each are placed in (a) CV02 (b) CV02
the xy plane at (0, 2), (4, 2), (4, – 2) and 2 3
(0, – 2). The work required to put a fifth 1 1
charge Q at the origin of the coordinate (c) CV02 (d) CV02
4 6
system will be: 16. Concentric metallic hollow spheres of radii R and
Q2 æ 1 ö Q2 æ 1 ö 4R hold charges Q1 and Q2 respectively. Given
(a) ç 1+ ÷ (b) ç1 + ÷ that surface charge densities of the concentric
4 pe 0 è 3ø 4 pe0 è 5ø
spheres are equal, the potential difference V(R)
Q2 Q2 – V(4R) is :
(c) (d)
2 2 pe 0 4pe 0
3Q1 3Q2
13. A parallel plate capacitor is made of two square (a) (b)
16pe 0 R 4pe0 R
plates of side ‘a’, separated by a distance d
(d<<a). The lower triangular portion is filled with Q2 3Q1
a dielectric of dielectric constant K, as shown in (c) (d)
4pe0 R 4pe0 R
the figure. Capacitance of this capacitor is:

@ebooks_for_freee on Telegram
58 PHYSICS

17. A solid conducting sphere, having a charge Q, metres. The values of constants are A = 20 SI unit
is surrounded by an uncharged conducting and B = 10 SI unit. If the potential at x = 1 is V1
hollow spherical shell. Let the potential and that at x = –5 is V2, then V1 – V2 (in volt)
difference between the surface of the solid is :
sphere and that of the outer surface of the 22. Determine the charge (in coulomb) on the
hollow shell be V. If the shell is now given a capacitor in the following circuit:
charge of – 4 Q, the new potential difference
between the same two surfaces is :
(a) – 2V (b) 2 V
(c) 4 V (d) V

Numeric Value Answer


23. The 1000 small droplets of water each of radius r
18. In the figure shown below, the charge on the left and charge Q, make a big drop of spherical shape.
plate of the 10 mF capacitor is –30mC. The charge The potential of big drop is how many times the
(in mC) on the right plate of the 6mF capacitor is: potential of one small droplet ?
24. A hollow metal sphere of radius 5 cm is charged
such that the potential on its surface is 10 V. The
6 mF
potential (in volt) at a distance of 2 cm from the
10 m F 2 mF centre of the sphere is
4 mF 25. A solid conducting sphere of radius a is
surrounded by a thin uncharged concentric
19. Voltage rating of a parallel plate capacitor is
conducting shell of radius 2a. A point charge q
500 V. Its dielectric can withstand a maximum
electric field of 106 V/m. The plate area is is placed at a distance 4a from common centre of
10–4 m2. What is the dielectric constant if the conducting sphere and shell. The inner sphere
capacitance is 15 pF ? is then grounded. The charge on solid sphere is
(given Î0 = 8.86 × 10–12 C2 m2) q
. Find the value of x.
20. A parallel plate capacitor has 1mF capacitance. x
One of its two plates is given +2mC charge
and the other plate, +4mC charge. The potential
difference (in volt) developed across the 2a
capacitor is :
a q
21. The electric field in a region is given by
ur
E = ( Ax + B ) iˆ , where E is in NC–1 and x is in

ANSWER KEY
1 (b) 4 (d) 7 (c) 10 (c) 13 (b) 16 (a) 19 (8.5) 22 (200) 25 (4)
2 (c) 5 (a) 8 (d) 11 (b) 14 (b) 17 (d) 20 (1) 23 (100)
3 (b) 6 (d) 9 (d) 12 (b) 15 (d) 18 (18) 21 (180) 24 (10)

@ebooks_for_freee on Telegram
Current Electricity 59

17
CURRENT ELECTRICITY

MCQs with One Correct Answer 5. In a Wheatstone's bridge, three resistances P, Q


and R connected in the three arms and the fourth
1. At what temperature will the resistance of a arm is formed by two resistances S1 and S2
copper wire becomes three times its value at 0°C? connected in parallel. The condition for the
(Temperature coefficient of resistance of copper bridge to be balanced will be
is 4 × 10–3/°C) P 2R P R ( S1 + S 2 )
(a) 600°C (b) 500°C (a) Q = S + S (b) Q = S1 S2
1 2
(c) 450°C (d) 400°C
P R ( S1 + S2 ) P R
2. The voltage V and current I
V (c) Q = 2S S (d) Q = S + S
graphs for a conductor at two 1 2 1 2
T1
6. Figure shows two squares, X and Y, Cut from a
different temperatures T1 and T2
sheet of metal of uniform thickness t. X and Y
T2 are shown in the figure. The
I have sides of length L and 2 L, respectively.
relation between T1 and T2 is O 2L
(a) T1 > T2 (b) T1 < T2 L t
L t 2L
1 X Y
(c) T1 = T2 (d) T1 =
T2 (a)
(b)
3. A cell of emf E is connected across a resistance The resistance Rx and Ry of the squares are
R. The potential difference between the terminals measured between the opposite faces shaded
of the cell is found to be V volt. Then the internal in Fig. What is the value of Rx/Ry?
resistance of the cell must be (a) 1/4 (b) 1/2
(E - V) (c) 1 (d) 2
(a) (E – V) R (b) R
V 7. Two different conductors have same resistance
2 (E - V ) R 2 (E - V ) V at 0°C. It is found that the resistance of the first
(c) (d)
E R conductor at t1°C is equal to the resistance of
4. Forty electric bulbs are connected in series across the second conductor at t2°C. The ratio of the
a 220 V supply. After one bulb is fused the temperature coefficients of resistance of
remaining 39 are connected again in series a
the conductors, 1 is
across the same supply. The illumination will be a2
t1 t 2 - t1
(a) more with 40 bulbs than with 39 (a) t (b)
2 t 2
(b) more with 39 bulbs than with 40
(c) equal in both the cases t 2 - t1 t2
(c) (d) t1
(d) None of these t1

@ebooks_for_freee on Telegram
60 PHYSICS

8. An electrical cable of copper has just one wire whose internal resistance is 0.5W. If the balance
of radius 9 mm. Its resistance is 5 ohm. This single point is obtained at l = 30 cm from the positive
copper wire of the cable is replaced by 6 different end, the e.m.f. of the battery is
well insulated copper wires of same length in
30 E 30 E
parallel, each of radius 3 mm. The total resistance (a) (b) (100 - 0.5i )
of the cable will now be equal to 100.5
(a) 7.5 ohm (b) 45 ohm 30 ( E - 0.5i ) 30 E
(c) (d)
(c) 90 ohm (d) 270 ohm 100 100
9. A cylindrical solid of length L and radius a is where i is the current in the potentiometer wire.
having varying resistivity given by r = r0x, 13. Two electric bulbs rated P1 watt V volts and P2
where r0 is a positive constant and x is watt V volts are connected in parallel and V volts
measured from left end of solid. The cell shown are applied to it. The total power will be
in the figure is having emf V and negligible
internal resistance. The magnitude of electric field
(a) ( P1 + P2 ) watt (b) ( )
P1 P2 watt

as a function of x is best described by æ P1P2 ö æ P1 + P2 ö


r = r 0x (c) ç ÷ watt (d) ç ÷ watt
è P1 + P2 ø è P1P2 ø
14. In an experiment of potentiometer for measuring
x the internal resistance of primary cell a balancing
length l is obtained on the potentiometer wire
V when the cell is open circuit. Now the cell is
2V 2V short circuited by a resistance R. If R is to be
(a) x (b) x
L 2 r0 L2 equal to the internal resistance of the cell the
V balancing length on the potentiometer wire will
(c) x (d) None of these
L2 be
10. In the network shown, each resistance is equal to R. (a) l (b) 2l
The equivalent resistance between adjacent corners (c) l/2 (d) l/4
A and D is 15. In a conductor, if the number of conduction
(a) R electrons per unit volume is 8.5 × 1028 m–3 and
2 mean free time is 25 fs (femto second), it’s
(b) R
3 approximate resistivity is: (me = 9.1 × 10–31 kg)
3 (a) 10–6 W m (b) 10–7 Wm
(c) R
7 –8
(c) 10 Wm (d) 10–5 Wm
8 16. Drift speed of electrons, when 1.5 A of current
(d) R
15 flows in a copper wire of cross section 5 mm2,
11. A current source drives a current in a coil of is v. If the electron density in copper is 9 ×
resistance R1 for a time t. The same source drives 1028/m3 the value of v in mm/s close to (Take
current in another coil of resistance R2 for same charge of electron to be = 1.6 × 10–19C)
time. If heat generated is same, find internal (a) 0.02 (b) 3
resistance of source. [given R1 > R2] (c) 2 (d) 0.2
R1R 2 17. A cell of internal resistance r drives current
(a) (b) R1 + R 2
R1 + R 2 through an external resistance R. The power
(c) zero (d) R 1R 2 delivered by the cell to the external resistance
will be maximum when :
12. The length of a wire of a potentiometer is 100
(a) R = 0.001 r (b) R = 1000 r
cm, and the e. m.f. of its standard cell is E volt. It (c) R = 2r (d) R = r
is employed to measure the e.m.f. of a battery

@ebooks_for_freee on Telegram
Current Electricity 61
18. In the given circuit the cells have zero internal Numeric Value Answer
resistance. The currents (in Amperes) passing
through resistance R1 and R2 respectively, 21. When the switch S, in the circuit shown, is
are: closed then the value of current i (in ampere)
will be:

20 V i1 C i2 10 V
A 2W 4W B
i

2W

S
(a) 1, 2 (b) 2, 2
(c) 0.5, 0 (d) 0, 1
V=0
19. In a building there are 15 bulbs of 45 W, 15
bulbs of 100 W, 15 small fans of 10 W and 2 22. A 100 watt bulb working on 200 volt has
heaters of 1 kW. The voltage of electric main resistance R and a 200 watt bulb working on 100
is 220 V. The minimum fuse capacity (rated 8
value) of the building will be: volt has resistance S. If the R/S is . Find the
x
value of x.
(a) 10 A (b) 25 A
23. A copper wire is stretched to make it 0.5%
(c) 15 A (d) 20 A longer. The percentage change in its electrical
20. In a meter bridge, the wire of length 1 m has a resistance if its volume remains unchanged is:
non-uniform cross-section such that, the 24. In the given circuit the internal resistance of
dR the 18 V cell is negligible. If R1 = 400W, R3 =
variation of its resistance R with length l is
dl 100 W and R4 = 500 W and the reading of an
dR 1 ideal voltmeter across R4 is 5 V, then the value
µ . Two equal resistances are of R2 (in W) will be:
dl l
connected as shown in the figure. The R3 R4
galvanometer has zero deflection when the
jockey is at point P. What is the length AP?
R1

R2
R' R'
18 V
25. A uniform metallic wire has a resistance of 18
G
W and is bent into an equilateral triangle.
P Then, the resistance (in W) between any two
vertices of the triangle is:
l 1 l
26. A 2 W carbon resistor is color coded with
(a) 0.2 m (b) 0.3 m green, black, red and brown respectively. The
(c) 0.25 m (d) 0.35 m maximum current (in mA) which can be passed
through this resistor is:

@ebooks_for_freee on Telegram
62 PHYSICS

27. A current of 2 mA was passed through an 29. A current of 1 mA flows through a copper
unknown resistor which dissipated a power of wire. How many electrons will pass through
4.4 W. Dissipated power (in watt) when an a given point in each second?
ideal power supply of 11 V is connected 30. In the circuit shown in Fig, the current in 4 W
across it is: resistance is 1.2 A. What is the potential
28. The amount of charge Q passed in time t through difference (in volt) between B and C?
a cross-section of a wire is Q = (5 t 2 + 3 t + 1) i1 4W
coulomb. 2W i
A 8W C
The value of current (in ampere) at time t = 5 s is B
i2

ANSWER KEY
1 (b) 4 (b) 7 (d) 10 (d) 13 (a) 16 (a) 19 (d) 22 (1) 25 (4) 28 (53)
2 (a) 5 (b) 8 (a) 11 (d) 14 (c) 17 (d) 20 (c) 23 (1) 26 (20) 29 (6.25×1015)
3 (b) 6 (c) 9 (a) 12 (d) 15 (c) 18 (c) 21 (5) 24 (300) 27 (11×10–5 ) 30 (3.6)

@ebooks_for_freee on Telegram
Moving Charges and Magnetism 63

MOVING CHARGES AND


18
MAGNETISM

MCQs with One Correct Answer 4. The magnetic induction at the centre O in the
figure shown is
1. A beam of electrons is moving with constant
velocity in a region having simultaneous
R2
perpendicular electric and magnetic fields of
strength 20 Vm–1 and 0.5 T respectively at right R1

angles to the direction of motion of the electrons. O


Then the velocity of electrons must be m 0i æ 1 1 ö m 0i æ 1 1 ö
- +
(a) 8 m/s (b) 20 m/s (a) 4 è R1 R2 ÷ø
ç (b) 4 è R1 R2 ÷ø
ç

1 m 0i m 0i
(c) 40 m/s (d) m/s (c) ( R1 - R2 ) (d) ( R1 + R2 )
40 4 4
2. A ring of radius R, made of an insulating material 5. Two concentric circular coils of ten turns each
carries a charge Q uniformly distributed on it. If are situated in the same plane. Their radii are 20
the ring rotates about the axis passing through and 40 cm and they carry respectively 0.2 and
its centre and normal to plane of the ring with 0.4 ampere current in opposite direction. The
constant angular speed w, then the magnitude magnetic field in weber/m2 at the centre is
of the magnetic moment of the ring is (a) m0/80 (b) 7m0/80
(c) (5/4) m0 (d) zero
1
(a) QwR2 (b) QwR 2 6. Circular loop of a wire and a long straight wire
2 carry currents lc and le, respectively as shown in
1 figure. Assuming that these are placed in the
(c) Qw2R (d) Qw2 R same plane. The magnetic fields will be zero at
2
the centre of the loop when the separation H is
3. If a particle of charge 10–12 coulomb moving Ie R
along the xˆ - direction with a velocity 105 m/s (a) I p Wire
c R
experiences a force of 10 –10 newton in Ic R Ic
yˆ - direction due to magnetic field, then the (b) I p
e H
minimum magnetic field is pI c
(a) 6.25 × 103 Tesla (b) 10–15 Tesla (c) I R Ie
e Straight
(c) 6.25 × 10 –3 Tesla (d) 10–3 Tesla Ie p
(d) I R
c

@ebooks_for_freee on Telegram
64 PHYSICS

7. A solenoid of length 1.5 m and 4 cm diameter 12. A proton (mass m)


accelerated by a potential B
possesses 10 turns per cm. A current of 5A is
flowing through it, the magnetic induction at axis difference V flies through a a
inside the solenoid is uniform transverse magnetic
(m0 = 4p × 10–7 weber amp–1 m–1) field B. The field occupies a d

(a) 4p ×10–5 gauss (b) 2p ×10–5 gauss region of space by width ‘d’.
(c) 4p ×10–5 tesla (d) 2p ×10–3 tesla If a be the angle of deviation of proton from
8. Two long conductors, separated by a distance d initial direction of motion (see figure), the value
of sin a will be :
carry current I1 and I2 in the same direction. They
r Bd B qd
exert a force F on each other. Now the current in (a) qV (b)
one of them is increased to two times and its 2m 2 mV
direction is reversed. The distance is also B q q
(c) (d) Bd
increased to 3d. The new value of the force d 2mV 2mV
between them is 13. Proton, deuteron and alpha particle of same
r r
2F F kinetic energy are moving in circular trajectories
(a) - (b) in a constant magnetic field. The radii of proton,
3 3
r deuteron and alpha particle are respectively rp,
r F
(c) – 2 F (d) - rd and ra. Which one of the following relation is
3 correct?
9. A 5 cm × 12 cm coil with number of turns 600 is (a) ra = rp = rd (b) ra = rp < rd
placed in a magnetic field of strength 0.10 Tesla. (c) ra > rd > rp (d) ra = rd > rp
The maximum magnetic torque acting on it when a 14. Two identical particles having same mass m and
current of 10-5 A is flowing through it will be charges +q and – q separated by a distance d
(a) 3.6 × 10–6 N-m enter in a uniform magnetic field B directed
(b) 3.6 × 10–6 dyne-cm perpendicular to paper inwards with speed v1 and
(c) 3.6 × 106 N-m v2 as shown in figure. The particles definetely will
definetely not collide if
(d) 3.6 × 106 dyne-m
m
10. A galvanometer of 50 ohm resistance has 25 (a) d ³ (v + v )
Bq 1 2
×

×
×
×
×
divisions. A current of 4 × 10–4 ampere gives a – v1
deflection of one division. To convert this m
(b) d £ (v1 + v2)
×

×
×
×
×
galvanometer into a voltmeter having a range of Bq
d
25 volts, it should be connected with a resistance 2m
(c) d > (v1 + v2)
×

×
×
×
of Bq × v2
Å

(a) 2450 W in series (b) 2500 W in series. 2m


×

×
×
×
×

(d) d < (v + v2)


(c) 245 W in parallel (d) 2550 W in parallel Bq 1
11. In a mass spectrometer used for measuring the 15. Two long parallel wires carry currents i1 andn i2
masses of ions, the ions are initially accelerated such that i1 > i2. When the currents are in the
by an electric potential V and then made to same direction, the magnetic field at a point
describe semicircular path of radius R using a midway between the wires is 6 × 10–6 T. If the
magnetic field B. If V and B are kept constant, direction of i2 is reversed, the field becomes
i
æ charge on the ion ö 3 × 10–5 T. The ratio of 1 is
the ratio ç ÷ will be i2
è mass of the ion ø 1
(a) (b) 2
proportional to 2
(a) 1/R2 (b) R2 2 3
(c) (d)
(c) R (d) 1/R 3 2

@ebooks_for_freee on Telegram
Moving Charges and Magnetism 65
16. An electron moving in a circular orbit of radius F
r makes n rotations per second. The magnetic
field produced at the centre has a magnitude of (a)
m0 ne m0 n2 e O I1I2
(a) (b)
2r 2r
m0 ne
(c) (d) Zero F
2pr
17. A solenoid of 0.4 m length with 500 turns carries
a current of 3 A. A small coil of 10 turns and of (b)
O I1I2
radius 0.01 m carries a current of 0.4 A.The torque
required to hold the coil with its axis at right angles
to that of solenoid in the middle part of it, is
(a) 6p2 × 10–7 Nm (b) 3p2 × 10–7 Nm F
2
(c) 9p × 10 Nm –7 (d) 12p2 × 10–7 Nm
18. The region between y = 0 and y = d contains a O
r (c)
magnetic field B = Bzˆ . A particle of mass m and I1I2
r
charge q enters the region with a velocity v = viˆ .
mv
if d = , the acceleration of the charged
2qB F
particle at the point of its emergence at the other
side is :
(d)
qv B æ 1 ˆ 3 ˆö O I1I2
(a) ç i- j÷
m è2 2 ø
qv B æ 3 ˆ 1 ˆ ö
(b) ç i + j÷
m è 2 2 ø Numeric Value Answer
qvB æ - ˆj + iˆ ö
(c) ç ÷ 21. An insulating rod of length l carries a charge q
m è 2 ø distributed uniformly on it. The rod is pivoted at
qvB æ iˆ + ˆj ö its mid point and is rotated at a frequency f about
(d) ç ÷ a fixed axis perpendicular to rod and passing
m è 2 ø
19. A circular coil having N turns and radius r carries through the pivot. The magnetic moment of the
a current I. It is held in the XZ plane in a magnetic 1
rod system is pqfl 2 . Find the value of a.
field B. The torque on the coil due to the 2a
magnetic field is : 22. A uniform magnetic field of y
B=1T
Br 2 I magnitude 1T exists in
(a) (b) Bpr2I N region y ³ 0 is along k̂
pN x
Bpr 2 I direction as shown. A
(c) (d) Zero particle of charge 1C is (– 3, –1)
N
20. Two long straight parallel wires, carrying projected from poin t
(adjustable) current I1 and I2, are kept at a (- 3, -1) towards origin with speed 1 m/sec. If
distance d apart. If the force ‘F’ between the mass of particle is 1 kg, then co-ordinates of
two wires is taken as ‘positive’ when the wires centre of circle in which particle moves are
repel each other and ‘negative’ when the wires
attract each other, the graph showing the æ1 bö
çç , - ÷ then find the value of (a + b).
dependence of ‘F’, on the product I1 I2, would èa 2 ÷ø
be :

@ebooks_for_freee on Telegram
66 PHYSICS

23. An electric current is flowing through a circular (a ´ 10-5 T) ˆj + (b ´ 10-5 T) kˆ then find the value
coil of radius R. The ratio of the magnetic field at of (a + b).
the centre of the coil and that at a distance 2 2R
I1 y
I2
from the centre of the coil and on its axis is :
P A B x
24. Two concentric coils each of radius equal to z
2 p cm are placed at right angles to each other. 3
ampere and 4 ampere are the currents flowing in 27. A particle having the same charge as of
each coil respectively. The magnetic induction electron moves in a circular path of radius 0.5
in Weber/m2 at the centre of the coils will be cm under the influence of a magnetic field of

( m 0 = 4p ´ 10 -7 Wb / A.m ) 0.5T. If an electric field of 100V/m makes it to


move in a straight path then the mass (in kg)
25. A long straight wire, carrying current I, is bent at of the particle is
its midpoint to form an angle of 45°. Magnitude (Given charge of electron = 1.6 × 10–19C)
of magnetic field at point P, distant R from point 28. A beam of protons with speed 4 × 105 ms–1 enters
( a - c)m0 I a uniform magnetic field of 0.3 T at an angle of
of bending is equal to then find 60° to the magnetic field. The pitch (in cm) of the
bpR
the value of (a + b + c) resulting helical path of protons is close to :
(Mass of the proton = 1.67 × 10–27 kg, charge of
P
the proton = 1.69 × 10–19 C)
R 45°
29. Magnitude of magnetic field (in SI units) at the
centre of a hexagonal shape coil of side 10 cm,
50 turns and carrying current I (Ampere) in units
m0 I
26. Two infinitely long linear conductors are of is :
p
arranged perpendicular to each other and are in 30. A galvanometer coil has 500 turns and each turn
mutually perpendicular planes as shown in has an average area of 3 × 10–4 m2. If a torque of
figure. If I1 = 2A along the y-axis and I2 = 3A 1.5 Nm is required to keep this coil parallel to a
along –ve z-axis and AP = AB = 1 cm. The value magnetic field when a current of 0.5 A is flowing
r
of magnetic field strength B at P is through it, the strength of the field (in T) is
__________.

ANSWER KEY
1 (c) 4 (a) 7 (d) 10 (a) 13 (b) 16 (a) 19 (b) 22 (5) 25 (7) 28 (4)
2 (b) 5 (d) 8 (a) 11 (a) 14 (c) 17 (a) 20 (a) 23 (27) 26 (7) 29 (500 3 )
–5 –24
3 (d) 6 (a) 9 (a) 12 (d) 15 (d) 18 (Bonus) 21 (6) 24 (5 × 10 ) 27 (2 × 10 ) 30 (20)

@ebooks_for_freee on Telegram
MAGNETISM AND 19
MATTER

MCQs with One Correct Answer 6. The materials suitable for making electromagnets
should have
1. A small bar magnet placed with its axis at 30°
(a) high retentivity and low coercivity
with an external field of 0.06 T experiences a
torque of 0.018 Nm. The minimum work required (b) low retentivity and low coercivity
to rotate it from its stable to unstable equilibrium (c) high retentivity and high coercivity
position is : (d) low retentivity and high coercivity
(a) 6.4 × 10–2 J (b) 9.2 × 10–3 J 7.
(c) 7.2 × 10 J –2 (d) 11.7 × 10–3 J
2. A paramagnetic material has 1028 atoms/m3. Its
magnetic susceptibility at temperature 350 K is
2.8 × 10–4. Its susceptibility at 300 K is:
(a) 3.267 × 10–4 (b) 3.672 × 10–4
(c) 3.726 × 10 –4 (d) 2.672 × 10 –4
3. A paramagnetic substance in the form of a cube
with sides 1 cm has a magnetic dipole moment of
20 × 10–6 J/T when a magnetic intensity of 60 ×
103 A/m is applied. Its magnetic susceptibility
is:
(a) 3.3 × 10–2 (b) 4.3 × 10 –2 The figure gives experimentally measured B vs.
(c) 2.3 × 10 –2 (d) 3.3 × 10–4 H variation in a ferromagnetic material. The
4. A bar magnet is demagnetized by inserthing it retentivity, co-ercivity and saturation,
inside a solenoid of length 0.2 m, 100 turns, and respectively, of the material are:
carrying a current of 5.2 A. The coercivity of the (a) 1.5 T, 50 A/m and 1.0 T
bar magnet is: (b) 1.5 T, 50 A/m and 1.0 T
(a) 285 A/m (b) 2600 A/m
(c) 150 A/m, 1.0 T and 1.5 T
(c) 520 A/m (d) 1200 A/m
(d) 1.0 T, 50 A/m and 1.5 T
5. A magnetic compass needle oscillates 30 times
8. An iron rod of volume 10–3 m3 and relative
per minute at a place where the dip is 45o, and 40
permeability 1000 is placed as core in a solenoid
times per minute where the dip is 30o. If B1 and
with 10 turns/cm. If a current of 0.5 A is passed
B2 are respectively the total magnetic field due
through the solenoid, then the magnetic moment
to the earth and the two places, then the ratio
of the rod will be :
B1/B2 is best given by :
(a) 1.8 (b) 0.7 (a) 50 × 102 Am2 (b) 5 × 102 Am2
2
(c) 500 × 10 Am 2 (d) 0.5 × 102 Am2
(c) 3.6 (d) 2.2

@ebooks_for_freee on Telegram
68 PHYSICS
9. A paramagnetic sample shows a net 14. The angle of dip at a place is 37° and the vertical
magnetisation of 6 A/m when it is placed in an component of the earth’s magnetic field is
external magnetic field of 0.4 T at a temperature 6 × 10–5T. The earth’s magnetic field at this place
of 4 K. When the sample is placed in an external is (tan 37° = 3/4)
magnetic field of 0.3 T at a temperature of 24 K,
(a) 7 × 10–5 T (b) 6 × 10–5 T
then the magnetisation will be :
(a) 1 A/m (b) 4 A/m (c) 5 × 10–5 T (d) 10–4 T
(c) 2.25 A/m (d) 0.75 A/m 15. A bar magnet of moment of inertia 9 × l0–5 kg m2
10. A perfectly diamagnetic sphere has a small placed in a vibration magnetometer and
spherical cavity at its centre, which is filled with oscillating in a uniform magnetic field l6p2 × l0–5T
a paramagnetic substance. The whole system is makes 20 oscillations in 15 s. The magnetic
r moment of the bar magnet is
placed in a uniform magnetic field B. Then the
(a) 3 Am2 (b) 2 Am2
field inside the paramagnetic substance is : (c) 5 Am 2 (d) 4 Am2
16. The work done in turning a magnet of magnetic
moment M by an angle of 90° from the meridian,
P is n times the corresponding work done to turn
r it through an angle of 60°. The value of n is
(a) B given by
(b) zero
r r (a) 2 (b) 1
(c) much large than | B | and parallel to B (c) 0.5 (d) 0.25
r r
(d) much large than | B | but opposite to B 17. The magnetic dipole moment of a coil is
11. A bar magnet having a magnetic moment of 5.4 × 10–6 joule/tesla and it is lined up with an
2 × 104 JT–1 is free to rotate in a horizontal plane. external magnetic field whose strength is 0.80 T.
Then the work done in rotating the coil (for q =
A horizontal magnetic field B = 6 × 10–4 T exists
180º) is
in the space. The work done in taking the magnet
(a) 4.32 mJ (b) 2.16 mJ
slowly from a direction parallel to the field to a
(c) 8.6 mJ (d) None of these
direction 60° from the field is
18. A bar magnet of length 6 cm has a magnetic
(a) 12 J (b) 6 J moment of 4 J T–1. Find the strength of magnetic
(c) 2 J (d) 0.6 J field at a distance of 200 cm from the centre of
12. The angle of dip at a certain place is 30°. If the the magnet along its equatorial line.
horizontal component of the earth’s magnetic (a) 4 × 10–6 tesla (b) 3.5 × 10–7 tesla
–8
(c) 5 × 10 tesla (d) 3 × 10–3 tesla
field is H, the intensity of the total magnetic field
is 19. A bar magnet has a length 8 cm. The magnetic
field at a point at a distance 3 cm from the centre
H 2H
(a) (b) in the broad side-on position is found to be
2 3 4 × 10–6 T. The pole strength of the magnet is
(c) (d) (a) 6.25 × 10–2 Am (b) 5 × 10–5 Am
H 2 H 3 –4
(c) 2 × 10 Am (d) 3 × 10–4 Am
13. If the dipole moment of magnet is 0.4 amp – m 2 20. A bar magnet of length 0.2 m and pole strength
and the force acting on each pole in a uniform 5 Am is kept in a uniform magnetic induction
magnetic field of induction 3.2 × 10–5 Weber/m2 field of strength 15Wbm–2 making an angle of
is 5.12 × 10–5 N, the distance between the poles 30º with the field. Find the couple acting on it
of the magnet is
(a) 7.5 Nm (b) 4.5 Nm
(a) 25 cm (b) 16 cm
(c) 12.5 cm (d) 12 cm (c) 5.5 Nm (d) 6.5 Nm

@ebooks_for_freee on Telegram
Magnetism and Matter 69

Numeric Value Answer


21. A bar magnet is demagnetized by inserthing it N S
inside a solenoid of length 0.2 m, 100 turns,
and carrying a current of 5.2 A. The coercivity
(in A/m) of the bar magnet is:
22. At some location on earth the horizontal component N S
P
of earth’s magnetic field is 18 × 10–6 T. At this
location, magnetic needle of length 0.12 m and 27. The magnetic field of earth at the equator is
pole strength 1.8 Am is suspended from its approximately 4 × 10–5 T. The radius of earth is
mid-point using a thread, it makes 45° angle
with horizontal in equilibrium. To keep this 6.4 × 106 m. Then the dipole moment (in A-m2)
needle horizontal, the vertical force (in N) that of the earth will be nearly of the order of
should be applied at one of its ends is: 28. Two tangent galvanometers having coils of the
23. A paramagnetic substance in the form of a cube same radius are connected in series. A current
with sides 1 cm has a magnetic dipole moment of flowing in them produces deflections of 60º and
20 × 10–6 J/T when a magnetic intensity of 45º respectively. The ratio of the number of turns
60 × 10 3 A/m is applied. Its magnetic in the coils is
susceptibility is: 29. Two short magnets with their axes horizontal
24. A paramagnetic material has 1028 atoms/m3. Its and perpendicular to the magnetic maridian are
magnetic susceptibility at temperature 350 K is placed with their centres 40 cm east and 50 cm
2.8 × 10–4. Its susceptibility at 300 K is: west of magnetic needle. If the needle remains
25. If the dipole moment of magnet is 0.4 amp – m2 undeflected, the ratio of their magnetic
and the force acting on each pole in a uniform moments M1 : M2 is
magnetic field of induction 3.2 × 10–5 Weber/m2 30. A certain amount of current when flowing in a
is 5.12 × 10–5 N, the distance (in cm) between properly set tangent galvanometer, produces a
the poles of the magnet is deflection of 45°. If the current be reduced by a
26. Two short bar magnets of magnetic moments
1000 Am2 are placed as shown at the corners of factor of 3 , the deflection (in degree) would
a square of side 10 cm. The net magnetic decrease by
induction (in Tesla) at P is

ANSWER KEY
–5
1 (c) 4 (b) 7 (d) 10 (b) 13 (a) 16 (a) 19 (a) 22 (6.5×10 ) 25 (25) 28 ( 3)
–4
2 (a) 5 (Bonus) 8 (b) 11 (b) 14 (d) 17 (c) 20 (a) 23 (3.3×10 ) 26 (0.1) 29 (0.51)
3 (d) 6 (b) 9 (d) 12 (b) 15 (d) 18 (c) 21 (2600) 24 (3.266×10 ) 27 (1023) 30
–4
(15)

@ebooks_for_freee on Telegram
70 PHYSICS

ELECTROMAGNETIC 20
INDUCTION

MCQs with One Correct Answer 5. A wire of length 1 m is moving at a speed of


2ms–1 perpendicular to its length in a uniform
1. A long solenoid has 500 turns. When a current magnetic field of 0.5 T. The ends of the wire are
of 2 ampere is passed through it, the resulting joined to a circuit of resistance 6W. The rate at
magnetic flux linked with each turn of the which work is being done to keep the wire moving
solenoid is 4 ×10–3 Wb. The self- inductance of at constant speed is
the solenoid is
(a) 2.5 henry (b) 2.0 henry 1 1
(a) W (b) W
(c) 1.0 henry (d) 40 henry 12 6
2. A very small square loop of wire of side l is
placed inside a large square loop of wire of side 1
(c) W (d) 1W
L (L ? l). The loop are coplanar and their centre 3
coincide. The mutual inductance of the system 6. Consider the situation shown in figure. If the
is equal to switch is closed and after some time it is opened
m0 m0 again, the closed loop will show [Just after the
(a) (l / L) (b) 8 2 (l2 / L)
4p 4p closing and opening the switch]
m0 m0 (a) a clockwise current
(c) (L / l) (d) 3 2 (L2 / l)
4p 4p pulse
3. A metal rod of length l moves perpendicularly
(b) an anticlockwise current
across a uniform magnetic field B with a velocity
v. If the resistance of the circuit of which the rod pulse
forms a part is r, then the force required to move (c) an anticlockwise current and then clockwise
the rod uniformly is pulse
B2 l 2 v Blv (d) a clockwise current and then an anticlock wise
(a) (b) current pulse.
r r
7. A coil having 100 turns and area of 0.001 metre2
B2 lv B2 l 2 v 2
(c) (d) is free to rotate about an axis. The coil is placed
r r perpendicular to a magnetic field of 1.0 weber/
4. A copper rod of length l is rotated about one
end perpendicular to the magnetic field B with metre2. If the coil is rotate rapidly through an
constant angular velocity w. The induced e.m.f. angle of 180°, how much charge will flow through
between the two ends is the coil? The resistance of the coil is 10 ohm.
1 3 (a) 0.02 coulomb (b) 0.2 coulomb
(a) Bw l 2 (b) Bwl 2
2 4 (c) 3 coulomb (d) 2 coulomb
(c) Bwl2 (d) 2Bwl2

@ebooks_for_freee on Telegram
Electromagnetic Induction 71
8. A thin circular ring of area A is perpendicular to 15. A horizontal straight wire 20 m long extending
uniform magnetic field of induction B. A small from east to west falling with a speed of 5.0 m/s, at
cut is made in the ring and a galvanometer is right angles to the horizontal component of the
connected across the ends such that the total earth’s magnetic field 0.30 × 10–4 Wb/m2. The
resistance of circuit is R. When the ring is instantaneous value of the e.m.f. induced in the
suddenly squeezed to zero area, the charge wire will be
flowing through the galvanometer is (a) 3mV (b) 4.5 mV (c) 1.5 mV (d) 6.0mV
BR AB 16. There are two long co-axial solenoids of same
(a) (b) length l. The inner and outer coils have radii r 1
A R
and r2 and number of turns per unit length n 1
(c) ABR (d) B2 A/R2 and n 2 , respectively. The ratio of mutual
9. An electron moves along the line PQ as shown inductance to the self-inductance of the inner-
which lies in the same plane as a circular loop of coil is :
conducting wire as shown in figure. What will be n1 n2 r1
the direction of the induced current initially in the (a) (b) ×
loop when electron comes closer to loop? n2 n1 r2
(a) Anticlockwise loop n2 r22 n2
(b) Clockwise (c) × (d)
n1 r12 n1
(c) Direction P Q
can not be predicted 17. Two coils ‘P’ and ‘Q’ are separated by some
(d) No current will be induced distance. When a current of 3A flows through
10. A straight conductor of length 2m moves at a coil ‘P’, a magnetic flux of 10–3 Wb passes
speed of 20 m/s. When the conductor makes an through ‘Q’. No current is passed through ‘Q’.
angle of 30° with the direction of magnetic field When no current passes through ‘P’ and a
of induction of 0.1 wbm2 then induced emf current of 2A passes through ‘Q’, the flux
(a) 4V (b) 3V (c) 1V (d) 2V through ‘P’ is:
11. The self induced emf of a coil is 25 volts. (a) 6.67 × 10–4 Wb (b) 3.67 × 10–3 Wb
When the current in it is changed at uniiform (c) 6.67 × 10 Wb
–3
(d) 3.67 × 10–4 Wb
rate from 10 A to 25 A in 1s, the change in the 18. An insulating thin rod of length l has a linear
energy of the inductance is: x
charge density r(x) = r0 on it. The rod is
(a) 740 J (b) 437.5 J(c) 540 J (d) 637.5 J l
12. In a coil of resistance 100W, a current is induced rotated about an axis passing through the
by changing the magnetic flux through it as origin (x = 0) and perpendicular to the rod. If
shown in the figure. The magnitude of change the rod makes n rotations per second, then the
in flux through the coil is time averaged magnetic moment of the rod is:
p
(a) 250 Wb (a) p n r l3 (b) n r l3
3
(b) 275 Wb
p
(c) 200 Wb (c) n r l3 (d) n r l3
4
(d) 225 Wb 19. A square frame of side 10 cm and a long straight
13. When current in a coil changes from 5 A to 2 A wire carrying current 1 A are in the plate of the
in 0.1 s, average voltage of 50 V is produced. paper. Starting from close to the wire, the frame
The self - inductance of the coil is : moves towards the right with a constant speed
(a) 6 H (b) 0.67 H (c) 3 H (d) 1.67 H of 10 ms–1 (see figure).
14. A solid metal cube of edge length 2 cm is I = 1A
moving in a positive y-direction at a constant
speed of 6 m/s. There is a uniform magnetic x
field of 0.1 T in the positive z-direction. The
potential difference between the two faces of v
the cube perpendicular to the x-axis, is:
(a) 12 mV(b) 6 mV (c) 1 mV (d) 2 mV
10 cm

@ebooks_for_freee on Telegram
72 PHYSICS

The e.m.f induced at the time the left arm of the 26. A uniform magnetic field B exists in a direction
frame is at x = 10 cm from the wire is: perpendicular to the plane of a square frame
(a) 2 mV (b) 1 mV made of copper wire. The wire has a diameter of
(c) 0.75 mV (d) 0.5 mV 2 mm and a total length of 40 cm. The magnetic
20. A coil of cross-sectional area A having n turns is field changes with time at a steady rate
placed in a uniform magnetic field B. When it is dB T
= 0.02 . Find the current (in A) induced in
rotated with an angular velocity w, the maximum dt s
e.m.f. induced in the coil will be the frame. Resistivity of copper = 1.7 × 10–8
3 W-m.
(a) nBAw (b) nBAw 27. A coil of inductance 1 H and resistance 10W is
2
1 connected to a resistanceless battery of emf 50
(c) 3nBAw (d) nBAw V at time t = 0. Calculate the ratio of the rate at
2
which magnetic energy is stored in the coil to
Numeric Value Answer the rate at which energy is supplied by the
battery at t = 0.1 s.
21. Two concentric coplanar circular loops made of 28. A long solenoid having 200 turns per cm carries a
wire, with resistance per unit length 10W/m have current of 1.5 amp. At the centre of it is placed a
diameters 0.2 m and 2 m. A time varying potential coil of 100 turns of cross-sectional area 3.14 ×
difference (4 + 2.5 t) volt is applied to the larger 10–4 m2 having its axis parallel to the field
loop. Calculate the current (in A) in the smaller produced by the solenoid. When the direction of
loop. current in the solenoid is reversed within 0.05 sec,
22. The current in a coil of self-induction 2.0 henry the induced e.m.f. (in V) in the coil is
is increasing according to i = 2 sin t2 ampere. 29. An air plane, with a 20 m wing spread is plying at
Find the amount of energy (in joule) spent during 250 m/s straight south parallel to earth¢s surface.
the period when the current changes from 0 to 2 The earth¢s magnetic field has a horizontal
ampere. component of 2 × 10–5 Wb/m2 and angle of dip
23. The self induced emf of a coil is 25 volts. When is 60°. Calculate the induced emf (in V) between
the current in it is changed at uniform rate from the plane tips.
10 A to 25 A in 1s, the change in the energy 30. If the rod is moving with a constant velocity of
(in joule) of the inductance is: 12 cm/s then the power (in watt) that must be
24. A 10 m long horizontal wire extends from North supplied by an external force in maintaining the
East to South West. It is falling with a speed of speed will be
5.0 ms–1 , at right angles to the horizontal
component of the earth’s magnetic field, of × ×
A × × ×

0.3 × 10–4 Wb/m2. The value of the induced emf × × × × ×


(in V) in wire is : Ammeter × × × v × ×

25. A conducting circular loop having a radius of × × × × ×


× × × × ×
5.0 cm, is placed perpendicular to a magnetic
field of 0.50 T. It is removed from the field in 0.50 s. × ×B × × ×

Find the average emf (in V) produced in the loop (Given B = 0.5 Tesla, l = 15 cm, v = 12 cm/s,
during this time. Resistance of rod RAB = 9.0 mW)

ANSWER KEY
–2
1 (c) 4 (a) 7 (a) 10 (d) 13 (d) 16 (d) 19 (b) 22 (4) 25 (7.85 × 10 ) 28 (0.048)
–2
2 (b) 5 (b) 8 (b) 11 (b) 14 (a) 17 (a) 20 (a) 23 (437.5) 26 (9.3 × 10 ) 29 (0.173)
–3 –3
3 (a) 6 (d) 9 (a) 12 (a) 15 (a) 18 (c) 21 (1.25) 24 (1.5 × 10 ) 27 (0.36) 30 (9 × 10 )

@ebooks_for_freee on Telegram
Alternating Current 73

21
ALTERNATING CURRENT

MCQs with One Correct Answer Z Z

(a) (b)
1. The current I passed in any instrument in alter-
nating current circuit is I = 2 sin wt amp and
potential difference applied is given by V = 5 f f
Z Z
cos wt volt then power loss over a complete cycle
(c) (d)
is in instrument is
(a) 2.5 watt (b) 5 watt
(c) 10 watt (d) zero f f

2. An alternating e.m.f. of angular frequency w is 5. For a series RLC circuit R = XL = 2XC. The
applied across an inductance. The instantaneous impedance of the circuit and phase difference
power developed in the circuit has an angular between V and I respectively will be
frequency [take phase difference between emf 5R 5R
p (a) , tan -1 (2) (b) , tan–1(1/2)
and current is ] 3 2
2
w w
(c) 5X C , tan -1 (2) (d) 5R, tan -1(1/ 3)
(a) (b) 6. In an a.c. circuit V and I are given by
4 2
(c) w (d) 2w V = 100 sin (100 t) volt
I = 100 sin (100 t + p/3) mA
3. A resistance 'R' draws power 'P' when connected
to an AC source. If an inductance is now placed The power dissipated in the circuit is
in series with the resistance, such that the (a) 104 watt (b) 10 watt
impedance of the circuit becomes 'Z', the power (c) 2.5 watt (d) 5.0 watt
drawn will be 7. An LCR circuit as shown in the figure is
connected to a voltage source V ac whose
R æRö frequency can be varied.
(a) P (b) Pç ÷ V
Z èZø The frequency,
at which the
2
æRö voltage across 24 H 2 µF 15 W
(c) P (d) Pç ÷
èZø the resistor is
maximum, is : ~
4. Which one of the following curves represents Vac = V0 sin wt
the variation of impedance (Z) with frequency f (a) 902 Hz (b) 143 Hz
in series LCR circuit? (c) 23 Hz (d) 345 Hz

@ebooks_for_freee on Telegram
74 PHYSICS

8. The primary winding of a transformer has 500 (a) 50 A (b) 45 A


turns whereas its secondary has 5000 turns. The (c) 35 A (d) 25 A
primary is connected to an A.C. supply of 20 V,
14. The power factor of an AC circuit having
50 Hz. The secondary will have an output of
resistance (R) and inductance (L) connected in
(a) 2 V, 5 Hz (b) 200 V, 500 Hz
series and an angular velocity w is
(c) 2V, 50 Hz (d) 200 V, 50 Hz
9. In an LCR circuit shown in the following figure, (a) R/ w L (b) w 2L2)1/2
R/(R2 +
what will be the readings of the voltmeter across (c) w L/R (d) R/(R2 – w 2L2)1/2
the resistor and ammeter if an a.c. source of 220V
and 100 Hz is connected to it as shown? 15. An alternating voltage v(t) = 220 sin 100Àt volt
L 100 W
is applied to a purely resistive load of 50W. The
C
time taken for the current to rise from half of
the peak value to the peak value is :
V V V A (a) 5 ms (b) 2.2 ms
300V 300 V VR (c) 7.2 ms (d) 3.3 ms
16. An emf of 20 V is applied at time t = 0 to a
220 V, 100 Hz circuit containing in series 10 mH inductor and
(a) 800 V, 8 A (b) 110 V, 1.1 A 5 W resistor. The ratio of the currents at time
(c) 300 V, 3 A (d) 220V, 2.2 A t = ¥ and at t = 40 s is close to:
10. The primary and secondary coil of a transformer (Take e2 = 7.389)
have 50 and 1500 turns respectively. If the (a) 1.06 (b) 1.15
magnetic flux f linked with the primary coil is
(c) 1.46 (d) 0.84
given by f = f0 + 4t, where f is in webers, t is time
17. For an RLC circuit driven with voltage of
in seconds and f0 is a constant, the output
voltage across the secondary coil is 1
amplitude vm and frequency w0 = the
(a) 120 volt (b) 220 volt LC
(c) 30 volt (d) 90 volt
current exhibits resonance. The quality factor, Q
11. An AC circuit has R = 100 W, C = 2 mF and L = 80
mH, connected in series. The quality factor of is given by:
the circuit is : w0 L w0 R
(a) 2 (b) 0.5 (a) (b)
R L
(c) 20 (d) 400
R CR
12. In LC circuit the inductance L = 40 mH and (c) (d)
(w0C) w0
capacitance C = 100 mF. If a voltage V(t) = 10
sin(314 t) is applied to the circuit, the current 18. A coil of inductance 300 mH and resistance 2 W
in the circuit is given as: is connected to a source of voltage 2V. The
(a) 0.52 cos 314 t (b) 10 cos 314 t current reaches half of its steady state value in
(c) 5.2 cos 314 t (d) 0.52 sin 314 t (a) 0.1 s (b) 0.05 s
(c) 0.3 s (d) 0.15 s
13. A power transmission line feeds input power
at 2300 V to a step down transformer with its 19. A series AC circuit containing an inductor (20
primary windings having 4000 turns. The mH), a capacitor (120 mF) and a resistor (60 W)
output power is delivered at 230 V by the is driven by an AC source of 24 V/50 Hz. The
transformer. If the current in the primary of the energy dissipated in the circuit in 60 s is:
transformer is 5A and its efficiency is 90%, the (a) 5.65 × 102 J (b) 2.26 × 103 J
output current would be: (c) 5.17 × 102 J (d) 3.39 × 103 J

@ebooks_for_freee on Telegram
Alternating Current 75
20. In a series resonant LCR circuit, the voltage power dissipation is 200 W, calculate the
across R is 100 volts and R = 1 kW with C = 2mF. maximum value of the back emf (in volt) arising
The resonant frequency w is 200 rad/s. At in the inductance.
resonance the voltage across L is 26. A 100 V AC source of frequency 500 Hz is
(a) 2.5 × 10–2 V (b) 40 V connected to LCR circuit with L = 8.1 mH, C =
(c) 250 V (d) 4 × 10–3 V 12.5 mF and R = 10W, all connected in series.
Find the potential (in volt) across the resistance.
Numeric Value Answer 27. A coil has a resistance of 10W and an inductance
21. A series AC circuit containing an inductor (20 of 0.4 henry. It is connected to an AC source of
mH), a capacitor (120 mF) and a resistor (60 W) 30
6.5 V, Hz. Find the average power (in watt)
is driven by an AC source of 24 V/50 Hz. The p
energy (in joule) dissipated in the circuit in 60 consumed in the circuit.
s is: 28. If i1 = 3 sin wt, i2 = 4 cos wt, and i3 = i0 sin (wt +
22. A power transmission line feeds input power 53°), find the value of i0.
i1 i2
at 2300 V to a step down transformer with its
primary windings having 4000 turns. The
i3
output power is delivered at 230 V by the
transformer. If the current in the primary of the 29. Given LCR circuit has L = 5 H, C = 80 mF, R = 40
transformer is 5A and its efficiency is 90%, the W and variable frequency source of 200 V. The
output current (in ampere) would be: source frequency (in Hz) which drives the circuit
23. An alternating voltage v(t) = 220 sin 100pt volt x
at resonance is . Find the value of x.
is applied to a purely resistive load of 50 W. p
L C
The time taken (in ms) for the current to rise
from half of the peak value to the peak value
R
is :
24. An inductor of inductance 100 mH is connected ~
in series with a resistance, a variable capacitance 30. An LCR series circuit with 100W resistance is
and an AC source of frequency 2.0 kHz. What connected to an AC source of 200 V and angular
should be the value of the capacitance (in farad) frequency 300 rad/s. When only the capacitance
so that maximum current may be drawn into the is removed, the current lags behind the voltage
circuit ? by 60°. When only the inductance is removed,
25. A 60 Hz AC voltage of 160 V impressed across the current leads the voltage by 60°. Calculate
an LR-circuit results in a current of 2 A. If the the current (in ampere) in the LCR circuit.

ANSWER KEY
1 (d) 4 (c) 7 (c) 10 (a) 13 (b) 16 (a) 19 (c) 22 (45) 25 (125) 28 (5)
2 (d) 5 (b) 8 (d) 11 (a) 14 (b) 17 (a) 20 (c) 23 (3.3) 26 (100) 29 (25)
3 (d) 6 (c) 9 (d) 12 (a) 15 (d) 18 (a) 21 (5.17×102) 24 (65×10–9) 27 (0.625) 30 (2)

@ebooks_for_freee on Telegram
76 PHYSICS

ELECTROMAGNETIC 22
WAVES

MCQs with One Correct Answer 5. A point source of electromagnetic radiation has
r r an average power output of 800 W. The maximum
1. If E and B represent electric and magnetic value of electric field at a distance 4.0 m from the
field vectors of the electromagnetic waves, then source is
the direction of propagation of the waves will (a) 64.7 V/m (b) 97.8 V/m
be along
r (c) 86.72 V/m (d) 54.77 V/m
r r
(a) B ´ E (b) E 6. Which of the following has/have zero average
r r r value in a plane electromagnetic wave?
(c) B (d) E ´ B
(a) Both magnetic and electric fields
2. In an apparatus, the electric field was found to
oscillate with an amplitude of 24 V/m. The (b) Electric field only
amplitude of the oscillating magnetic field will (c) Magnetic field only
be (d) Magnetic energy
(a) 6 × 10–6 T (b) 2 × 10–8 T 7. Which of the following is correct about the
(c) 8 × 10–8 T (d) 12 × 10–6 T electromagnetic waves?
3. A plane electromagnetic wave of wave intensity (a) they are transverse waves
10 W/m2 strikes a small mirror of area 20 cm 2,
(b) they have rest mass
held perpendicular to the approaching wave. The
radiation force on the mirror will be (c) they require medium to propagate
(a) 6.6 × 10–11 N (b) 1.33 × 10–11 N (d) they travel at varying speed through
vaccum
(c) 1.33 × 10–10 N (d) 6.6 × 10–10 N
8. In an electromagnetic wave
4. Given below is a list of E.M spectrum and its
use. Which one does not match? (a) power is transmitted along the magnetic
field
(a) U.V. rays — finger prints detection
(b) power is transmitted along the electric field
(b) I.R.. rays — for taking photography during
the fog (c) power is equally transferred along the
electric and magnetic fields
(c) X- rays — atomic structure
(d) power is transmitted in a direction
(d) Microwaves — forged document detection perpendicular to both the fields

@ebooks_for_freee on Telegram
Electromagnetic Waves 77
9. An electro magnetic wave travels along z-axis. (a) 35.2 × 10–10 J/m3
Which of the following pairs of space and time (b) 35.2 × 10–11 J/m3
varying fields would generate such a wave (c) 35.2 × 10–12 J/m3
(a) Ex, By (b) Ey, Bx (d) 35.2 × 10–13 J/m3
(c) Ez, Bx (d) Ey, Bz 14. A plane electromagnetic wave in a non-magnetic
10. Which of the following statements is true? dielectric medium is given by
(a) The frequency of microwaves is greater ur ur -7
E= E 0 (4 ´ 10 x - 50t ) with distance being in
than that of UV-rays
(b) The wavelength of IR rays is lesser than meter and time in seconds. The dielectric
that of UV-rays constant of the medium is :
(c) The wavelength of microwaves is lesser (a) 2.4 (b) 5.8
than that of IR rays (c) 8.2 (d) 4.8
(d) Gamma rays has least wavelength in the 15. An electromagnetic wave in vacuum has the
r r
electromagnetic spectrum. electric and magnetic field E and B , which are
11. For a plane electromagnetic wave, the magnetic always perpendicular to each other. The
r
field at a point x and time t is direction of polarization is given by X and that
® r
B( x, t ) = [1.2 ´10-7 sin(0.5 ´103 x of wave propagation by k . Then
r r r r r
+1.5 ´ 1011 t ) k$ ]T (a) X || B and k || B ´ E
® r r r r r
The instantaneous electric field E (b) X || E and k || E ´ B
®
r r r r r
corresponding to B is: (c) X || B and k || E ´ B
(speed of light c = 3 × 108 ms–1) r r r r r
(d) X || E and k || B ´ E
®
(a) E( x , t ) = [-36sin(0.5 ´103 x 16. Chosse the correct option relating wavelengths
V of different parts of electromagnetic wave
+ 1.5 ´1011 t ) $j ] spectrum :
m
® V (a) l visible < l micro waves < l radio waves < l X -rays
(b) E( x, t ) = [36 sin(1 ´ 103 x + 0.5 ´ 1011 t ) $j ]
m (b) l radio waves > l micro waves > l visible > l x-rays
® V (c) l x- rays < l micro waves < l radio waves < l visible
(c) E ( x, t ) = [36sin(0.5 ´ 103 x + 1.5 ´ 1011 t ) k$ ]
m (d) l visible > l x-rays > l radio waves > l micro waves
® V
(d) E ( x, t ) = [36sin(1 ´ 103 x + 1.5 ´ 1011 t )$i] 17. Photons of an electromagnetic radiation has an
m energy11 keV each. To which region of
12. The energy associated with electric field is electromagnetic spectrum does it belong ?
(UE) and with magnetic fields is (UB) for an (a) X-ray region
electromagnetic wave in free space. Then :
(b) Ultra violet region
UB (c) Infrared region
(a) UE = (b) UE > UB
2 (d) Visible region
(c) UE < UB (d) UE = UB 18. The mean intensity of radiation on the surface
13. An electromagnetic wave of frequency 1 × 1014 of the Sun is about 108 W/m2. The rms value of
hertz is propagating along z-axis. The amplitude the corresponding magnetic field is closest to :
of electric field is 4 V/m. If e0 = 8.8 × 10–12 C2/N-m2, (a) 1 T (b) 102 T
then average energy density of electric field will (c) 10 T–2 (d) 10–4 T
be:

@ebooks_for_freee on Telegram
78 PHYSICS

19. Arrange the following electromagnetic radiations 25. The magnetic field of a plane electromagnetic
per quantum in the order of increasing energy : wave is given by:
A : Blue light B : Yellow light r
B = B0 $i [ cos ( kz – wt ) ] +B1 $j cos ( kz + wt )
C : X-ray D : Radiowave.
(a) C, A, B, D (b) B, A, D, C Where B0 = 3 × 10–5 T and B1 = 2 × 10–6 T.
(c) D, B, A, C (d) A, B, D, C The rms value of the force (in newton)
20. The frequency of X-rays; g-rays and ultraviolet experienced by a stationary charge Q = 10–4
rays are respectively a, b and c then C at z = 0 is :
(a) a < b; b > c (b) a > b ; b > c 26. 50 W/m2 energy density of sunlight is normally
(c) a < b < c (d) a = b = c incident on the surface of a solar panel. Some
part of incident energy (25%) is reflected from
Numeric Value Answer the surface and the rest is absorbed. The
force (in newton) exerted on 1m2 surface area
21. A plane electromagnetic wave of frequency 50
will be (c = 3 × 108 m/s):
MHz travels in free space along the positive x-
direction. At a particular point in space and time, 27. A light beam travelling in the x-direction is
r described by the electric field
E = 6.3 ˆj V / m. The corresponding magnetic
r E= æ xö
y 300sin w ç t - ÷ . An electron is
field B , at that point is x × 10–8 k̂T. Find the è cø
value of x. constrained to move along the y-direction with
22. If t h e m a gn et i c fi eld of a p la n e a speed of 2.0 × 107 m/s. Find the maximum electric
electromagnetic wave is given by (The force (in newton) on the electron.
speed of light = 3 × 108 m/s) 28. A plane electromagnetic wave of wave intensity
é 15 æ x öù 10 W/m2 strikes a small mirror of area 20 cm 2,
B = 100 × 10–6 sin ê 2p ´ 2 ´ 10 ç t - ÷ ú held perpendicular to the approaching wave. The
ë è c øû
then the maximum electric field (in N/C) radiation force (in newton) on the mirror will be
associated with it is: 29. The magnetic field in a travell ing
23. A 27 mW laser beam has a cross-sectional area electromagnetic wave has a peak value of 20
of 10 mm2. The magnitude of the maximum electric nT. The peak value of electric field strength (in
field (in kV/m) in this electromagnetic wave is volt m–1) is
given by : 30. Light is incident normally on a completely
[Given permittivity of space Î0 = 9 × 10 –12 SI absorbing surface with an energy flux of 25
units, Speed of light c = 3 × 108 m/s] Wcm–2. If the surface has an area of 25 cm2,
24. The mean intensity of radiation on the surface the momentum (in Ns) transferred to the
of the Sun is about 108 W/m2. The rms value of surface in 40 min time duration will be:
the corresponding magnetic field (in tesla) is :

ANSWER KEY
1 (d) 4 (d) 7 (a) 10 (d) 13 (c) 16 (b) 19 (c) 22 (3×104 ) 25 (0.64) 28 (1.33×10–10 )
2 (c) 5 (d) 8 (d) 11 (a) 14 (b) 17 (a) 20 (a) 23 (1.4) 26 (20×10–8) 29 (6)
3 (c) 6 (a) 9 (a) 12 (d) 15 (b) 18 (d) 21 (2.1) 24 (6×10–4) 27 (4.8×10–7) 30 –3
(5×10 )

@ebooks_for_freee on Telegram
RAY OPTICS AND 23
OPTICAL INSTRUMENTS

MCQs with One Correct Answer 5. The focal lengths of the objective and the
eyepiece of the reflecting telescope are 225 cm
1. A vessel is half filled with a liquid of refractive and 5 cm respectively. The magnifying power of
index m. The other half of the vessel is filled with the telescope will be
an immiscible liquid of refrative index 1.5 m. The (a) 49 (b) 45
apparent depth of the vessel is 50% of the actual (c) 35 (d) 60
depth. Then m is 6. A ray of light passes through an equilateral prism
(a) 1.4 (b) 1.5 such that the angle of incidence is equal to the
(c) 1.6 (d) 1.67 angle of emergence and the latter is equal to
2. A convex lens is in contact with concave lens. 3
The magnitude of the ratio of their powers is 2/3. th of angle of prism. The angle of deviation is
4
Their equivalent focal length is 30 cm. What are (a) 25° (b) 30°
their individual focal lengths (in cm)? (c) 45° (d) 35°
(a) –15, 10 (b) –10, 15 7. An object is placed at a distance of 40 cm from a
(c) 75, 50 (d) –75, 50 convex mirror of radius of curvature 20 cm. At
3. A plano-convex lens of refractive index 1.5 and what distance from the object a plane mirror be
radius of curvature 30 cm is silvered at the curved placed so that image in the convex mirror and
surface. Now this lens has been used to form the plane mirror coincides?
image of an object. At what distance from this (a) 20cm (b) 24cm
lens an object be placed in order to have a real (c) 28cm (d) 32cm
image of size of the object? 8. Light propagates with speed of 2.2×108 m/s and
(a) 60 cm (b) 30 cm 2.4×108 m/s in the medium P and Q respectively.
(c) 20 cm (d) 80 cm The eritical angle between them is
4. A light ray is incident perpendicularly to one
æ1ö
(b) sin -1 æç ÷
11 ö
face of the prism shown in figure and is totally (a) sin -1 ç ÷
reflected if q = 45°, we conclude that the refractive 11
è ø è 12 ø
index n of the prism -1 æ 5 ö -1 æ 5 ö
(c) sin ç ÷ (d) sin ç ÷
1 è 12 ø è 11 ø
(a) n > 9. A plano-convex lens fits exactly into a plano-
2 concave lens. Their plane surfaces are parallel
q
(b) n > 2 q to each other. If lenses are made of different
1 materials of refractive indices m1 and m2 and R is
(c) n < the radius of curvature of the curved surface of
2
45° the lenses, then the focal length of the
(d) n < 2 combination is

@ebooks_for_freee on Telegram
80 PHYSICS
R R 16. A double convex lens has power P and same
(a) (b)
2 ( m1 - m 2 ) (m1 - m 2 ) radii of curvature R of both the surfaces. The
2R R radius of curvature of a surface of a plano-
(c) (d)
(m 2 - m1 ) 2 ( m1 + m 2 ) convex lens made of the same material with
10. A thin prism P1 with angle 4° and made from power 1.5 P is :
R
glass of refractive index 1.54 is combined with (a) 2R (b)
2
another prism P2 made of glass of refractive index
3R R
1.72 to produce dispersion without deviation. (c) (d)
The angle of prism P2 is 2 3
17. In a compound microscope, the focal length of
(a) 5.33° (b) 4°
objective lens is 1.2 cm and focal length of eye
(c) 2.6° (d) 3°
piece is 3.0 cm. When object is kept at 1.25 cm in
11. A thin lens made of glass (refractive index = 1.5)
front of objective, final image is formed at infinity.
of focal length f = 16 cm is immersed in a liquid
Magnifying power of the compound microscope
of refractive index 1.42. If its focal length in liquid
should be:
is fl ,then the ratio fl /f is closest to the integer:
(a) 200 (b) 100
(a) 1 (b) 9
(c) 400 (d) 150
(c) 5 (d) 17 18. Light is incident from a medium into air at two
12. A convex lens of focal length 20 cm produces possible angles of incidence (A) 20° and (B) 40°.
images of the same magnification 2 when an In the medium light travels 3.0 cm in 0.2 ns. The
object is kept at two distances x1 and x2 (x1 > x2) ray will :
from the lens. The ratio of x1 and x2 is: (a) suffer total internal reflection in both cases
(a) 2 : 1 (b) 3 : 1 (A) and (B)
(c) 5 : 3 (d) 4 : 3 (b) suffer total internal reflection in case (B)
13. Two lenses of power –15 D and +5 D are in only
contact with each other. The focal length of the (c) have partial reflection an d partial
combination is transmission in case (B)
(a) + 10 cm (b) – 20 cm (d) have 100% transmission in case (A)
(c) – 10 cm (d) + 20 cm 19. To get three images of a single object, one should
14. A ray of light AO in vacuum is incident on a have two plane mirrors at an angle of
glass slab at angle 60o and refracted at angle (a) 60º (b) 90º
30o along OB as shown in the figure. The optical (c) 120º (d) 30º
path length of light ray from A to B 20. The refractive index of a glass is 1.520 for red
is : light and 1.525 for blue light. Let D1 and D2 be
2 3 angles of minimum deviation for red and blue
(a) + 2b
a light respectively in a prism of this glass. Then,
2b (a) D1 < D2
(b) 2a + (b) D1 = D2
3
2b (c) D1 can be less than or greater than D2
(c) 2a + depending upon the angle of prism
3 (d) D1 > D2
(d) 2a + 2b
15. If we need a magnification of 375 from a Numeric Value Answer
compound microscope of tube length 150 mm
and an objective of focal length 5 mm, the focal 21. A convex lens (of focal length 20 cm) and a
length of the eye-piece, should be close to: concave mirror, having their principal axes
along the same lines, are kept 80 cm apart from
(a) 22mm (b) 12mm
each other. The concave mirror is to the right
(c) 2 mm (d) 33mm
of the convex lens. When an object is kept at

@ebooks_for_freee on Telegram
Ray Optics and Optical Instruments 81
a distance of 30 cm to the left of the convex dq
lens, its image remains at the same position q is 60° and = m. The value of m is
dn
even if the concave mirror is removed. The
maximum distance (in cm) of the object for
which this concave mirror, by itself would
produce a virtual image would be :
22. In figure, the optical fiber is l = 2 m long and 60°
has a diameter of d = 20 mm. If a ray of light q
is incident on one end of the fiber at angle q1
= 40°, the number of reflections it makes before
emerging from the other end is : (refractive
index of fiber is 1.31 and sin 40° = 0.64) 27. The magnifying power of a microscope with an
objective of 5 mm focal length is 40. The length
of its tube is 20 cm. Then the focal length (in cm)
of the eye-piece is
d 28. A glass sphere of radius 5 cm has a small bubble
q2
2 cm from its centre. The bubble is viewed along
40°
a diameter of the sphere from the side on which
23. A concave mirror for face viewing has focal it lies. How far (in cm) from the surface will it
length of 0.4 m. The distance (in metre) at appear. Refractive index of glass is 1.5.
which you hold the mirror from your face in 29. A converging beam of rays is incident on a
order to see your image upright with a diverging lens. Having passed through the lens
magnification of 5 is: the rays intersect at a point 15 cm from the lens.
24. A concave mirror has radius of curvature of 40 If the lens is removed the point where the rays
cm. It is at the bottom of a glass that has water meet will move 5 cm closer to the mounting that
filled up to 5 cm (see figure). holds the lens. Find focal length (in cm) of the
If a small particle is floating particle lens.
on the surface of water, its
image as seen, from
directly above the glass, 5 cm
is at a distance d from the
surface of water. The value
30. A prism ABC of angle 30° has its face AC
of d (in cm) is :
silvered. A ray of light incident at an angle of 45°
(Refractive index of water = 1.33) at the face AB retraces its path after refraction at
25. A ray of light falls on a glass plate of refractive face AB and reflection at face AC. The refractive
index µ = 1.5. What is the angle of incidence (in index of the material of the prism is
degree) of the ray if the angle between the
reflected and refracted rays is 90°? A
26. The monochromatic beam of light is incident
at 60° on one face of an equilateral prism of Silvered
refractive index n and emerges from the 45°
opposite face making an angle q(n) with the
normal (see the figure). For n = 3 the value of B C

ANSW ER KEY
1 (d) 4 (b) 7 (b) 10 (d) 13 (c) 16 (d) 19 (b) 22 (57000) 25 (57) 28 (2.5)
2 (a) 5 (b) 8 (b) 11 (b) 14 (d) 17 (a) 20 (a) 23 (0.32) 26 (2) 29 (30)
3 (c) 6 (b) 9 (b) 12 (b) 15 (a) 18 (b) 21 (10) 24 (8.8) 27 (2.5) 30 ( 2)

@ebooks_for_freee on Telegram
82 PHYSICS

WAVE OPTICS 24

MCQs with One Correct Answer 5. Unpolarised light is incident on a dielectric of


refractive index 3 . What is the angle of
1. If two waves represented by y1 = 4 sin wt and
incidence if the reflected beam is completely
æ pö polarised?
y2 = 3 sin ç wt + ÷ interfere at a point, then the (a) 30° (b) 45°
è 3ø
amplitude of the resulting wave will be about (c) 60° (d) 75°
(a) 7.99 (b) 6.08 6. A ray of light is incident on the surface of a glass
plate at an angle of incidence equal to Brewster’s
(c) 5.00 (d) 3.50
angel f. If m represents the refractive index of glass
2. In a double slit experiment, the screen is placed with respect to air, then the angle between the
at a distance of 1.25 m from the slits. When the reflected and the refracted rays is
apparatus is immersed in water (µw = 4/3), the (a) 90° + f (b) sin–1(m cos f)
angular width of a fringe is found to be 0.2°.
æ sin f ö
When the experiment is performed in air with (c) 90º (d) 90° – sin –1 ç ÷
same set up, the angular width of the fringe is è m ø
(a) 0.4° (b) 0.27° 7. Unpolarized light is incident on a plane sheet on
(c) 0.35° (d) 0.15° water surface. The angle of incidence for which
3. A ray of light is incident from a denser to a rarer the reflected and refracted rays are perpendicular
medium. The critical angle for total internal to each other is
reflection is qiC and Brewster’s angle of incidence æ 4ö
çè m of water = ÷ø
is qiB, such that sin qiC/sin qiB = h = 1.28. The 3
–1 æ 4 ö æ 3ö
relative refractive index of the two media is: (a) sin ç ÷ (b) tan –1 ç ÷
è 3ø è 4ø
(a) 0.2 (b) 1.4 –1 æ 4 ö –1 æ 1 ö
(c) 0.8 (d) 0.12 (c) tan ç ÷ (d) sin ç ÷
è 3ø è 3ø
4. In a Young’s double-slit experiment, let b be the 8. Two coherent sources of sound, S1 and S2 ,
fringe width, and I0 be the intensity at the central produce sound waves of the same wavelength,
bright fringe. At a distance x from the central l = 1 m, in phase. S1 and S2 are placed 1.5 m apart
bright fringe, the intensity is (see fig.). A listener, located at L, directly in front
of S2 finds that the intensity is at a minimum
æxö æxö
(a) I 0 cos çç ÷÷ (b) I 0 cos 2 çç ÷÷ when he is 2 m away from S2. The listener moves
èbø èbø away from S1, keeping his distance from S2 fixed.
æ px ö æI ö æ px ö The adjacent maximum of intensity is observed
(c) I 0 cos 2 çç ÷÷ (d) ç 0 ÷ cos 2 çç ÷÷ when the listener is at a distance d from S1. Then,
è b ø 4
è ø è b ø
d is :

@ebooks_for_freee on Telegram
Wave Optics 83
(a) 12 m 1 1 1 1
(a) (b) (c) (d)
2m
64 32 256 512
13. Two polaroids are oriented with their planes
(b) 5 m S2 L perpendicular to incident light and transmission
2m
axis making an angle of 30o with each other. What
(c) 2 m 1.5 m d fraction of incident unpolarised light is
transmitted?
(a) 15.2% (b) 9.2%
(d) 3 m S1 (c) 11.6% (d) 37.5%
9. Wavelength of light used in an optical instrument 14. Two ideal slits S1 and S2 are at a distance d apart,
and illuminated by light of wavelength l passing
are l1 = 4000 Å and l2 = 5000 Å, then ratio of
through an ideal source slit S placed on the line
their respective resolving powers l1 = 4000 Å
through S2 as shown. The distance between the
(corresponding to l1 and l2) is planes of slits and the source slit is D. A screen is
(a) 16 : 25 (b) 9 : 1 held at a distance D from the plane of the slits.
(c) 4 : 5 (d) 5 : 4 The minimum value of d for which there is darkness
10. Consider a tank made of glass (refractive index at O is
1.5) with a thick bottom. It is filled with a liquid 3lD
of refractive index m. A student finds that, (a)
2
irrespective of what the incident angle i (see
(b) lD
figure) is for a beam of light entering the liquid,
lD
the light reflected from the liquid glass interface (c)
is never completely polarized. For this to happen, 2
the minimum value of m is: (d) 3 lD
15. A parallel beam of light of wavelength l is
5 incident normally on a narrow slit. A diffraction
(a)
3 pattern is formed on a screen placed
3 perpendicular to the direction of the incident
(b) beam. At the second minimum of the diffraction
5
pattern, the phase difference between the rays
5
(c) coming from the two edges of slit is
3 (a) p (b) 2p (c) 3p (d) 4p
4 16. In a Young’s double slit experiment, the distance
(d) between the two identical slits is 6.1 times larger
3
11. For the two parallel rays AB and DE shown here, than the slit width. Then the number of intensity
BD is the wavefr ont. For what value of maxima observed within the central maximum of
wavelength of rays destructive interference takes the single slit diffraction pattern is:
place between ray DE and reflected ray CD ? (a) 3 (b) 6 (c) 12 (d) 24
(a) 3x
E 17. A single slit of width 0.1 mm is illuminated by a
A parallel beam of light of wavelength 6000 Å and
(b) 2x D
diffraction bands are observed on a screen 0.5 m
x from the slit. The distance of the third dark band
(c) x B
60°
from the central bright band is :
(d) 2 x F C Mirror
(a) 3 mm (b) 9mm
12. A beam of natural light falls on a system of 5
(c) 4.5 mm (d) 1.5 mm
polaroids, which arranged in succession such
18. In the ideal double-slit experiment, when a glass-
that the pass axis of each polaroid is turned plate (refractive index 1.5) of thickness t is
through 60° with respect to the preceding one. introduced in the path of one of the interfering
The fraction of the incident light intensity that beams (wavelength l), the intensity at the
passes through the system is position where the central maximum occurred

@ebooks_for_freee on Telegram
84 PHYSICS
previously remains unchanged. The minimum of wavelength 5000 Å is used, the minimum
thickness of the glass-plate is separation (in µm) between two points, to be
2l l seen as distinct, will be :
(a) 2 l (b) (c) (d) l
3 3 26. There are two sources kept at distances 2 l. A
19. At the first minimum adjacent to the central
large screen is perpendicular to line joining the
maximum of a single-slit diffraction pattern, the
phase difference between the Huygen’s wavelet sources. Number of maximas on the screen in
from the edge of the slit and the wavelet from this case is (l = wavelength of light)
the midpoint of the slit is ¥
p
(a) radian (b) p radian S1 S2
2 2l
p p
(c) radian (d) radian ¥
8 4
20. A parallel beam of monochromatic light of 27. A Young's double slit interference arrangement
wavelength 5000Å is incident normally on a with slits S1 and S2 is immersed in water (refractive
single narrow slit of width 0.001 mm. The light is 4
focussed by a convex lens on a screen placed in index = ) as shown in the figure. The positions
focal plane. The first minimum will be formed for 3
the angle of diffraction equal to of maximum on the surface of water are given by
(a) 0° (b) 15° (c) 30° (d) 50° x2 = p2m2l2 – d2, where l is the wavelength of
light in air (refractive index = 1), 2d is the
Numeric Value Answer separation between the slits and m is an integer.
21. In a Young's double slit experiment, the slits The value of p is
are placed 0.320 mm apart. Light of wavelength
l = 500 nm is incident on the slits. The total S1
number of bright fringes that are observed in
d
the angular range – 30° £ q £ 30° is x Air
22. In a Young’s double slit experiment, the path
d
difference, at a certain point on the screen,
S2 Water
1
betwen two interfering waves is th of
8
wavelength. The ratio of the intensity at this 28. Two waves of the same frequency have
point to that at the centre of a bright fringe is amplitudes 2 and 4. They interfere at a point where
23. In a double-slit experiment, green light (5303Å)
their phase difference is 60°. Find their resultant
falls on a double slit having a separation of 19.44
amplitude.
µm and a width of 4.05 µm. The number of bright
29. Diameter of the objective lens of a telescope
fringes between the first and the second
is 250 cm. For light of wavelength 600 nm,
diffraction minima is :
coming from a distant object, the limit of
24. Calculate the limit of resolution (in radian) of
resolution of the telescope (in radian) is
a telescope objective having a diameter of 200
30. In an interference pattern, at a point there observe
cm, if it has to detect light of wavelength 500
16th order maximum for l1 = 6000 Å. What order
nm coming from a star.
will be visible here if the source is replaced by
25. The value of numerical aperature of the
light of wavelength l2 = 4800 Å?
objective lens of a microscope is 1.25. If light
ANSW ER KEY
1 (b) 4 (c) 7 (c) 10 (b) 13 (d) 16 (c) 19 (b) 22 (0.85) 25 (0.24) 28 ( 28 )
2 (b) 5 (c) 8 (d) 11 (a) 14 (c) 17 (b) 20 (c) 23 (5) 26 (3) 29 (3 × 10– 7 )
3 (c) 6 (c) 9 (d) 12 (d) 15 (d) 18 (a) 21 (641) 24 (305 × 10– 9 ) 27 (3) 30 (20)

@ebooks_for_freee on Telegram
Dual Nature of Radiation and Matter 85

DUAL NATURE OF 25
RADIATION AND MATTER

MCQs with One Correct Answer (a) B alone (b) A alone


(c) neither A nor B (d) both A and B
1. Which of the following figures represent the
variation of particle momentum and the 5. Which is the incorrect statement of the following
associated de-Broglie wavelength? (a) Photon is a particle with zero rest mass
p p (b) Photon is a particle with zero mementum
(c) Photon travel with velocity of light in
(a) (b) vacuum
(d) Photon even feel the pull of gravity
l l 6. An electron (mass m) with initial velocity
p p r
v = v0iˆ + v0 ˆj is in an electric field
r
(c) (d) E = - E0 kˆ. If l 0 is initial de-Broglie wavelength
of electron, its de-Broglie wave length at time t
l l is given by:
2. When ultraviolet light of energy 6.2 eV incidents l0 2 l0
on a aluminimum surface, it emits photoelectrons. (a) (b)
2 2 2
If work function for aluminium surface is 4.2 eV, e E t e2 E 2 t 2
1+ 2 2 1 + 20 2
then kinetic energy of emitted electrons is m v0 m v0
(a) 3.2 × 10–19 J (b) 3.2 × 10–17 J l0 l0
(c) (d)
(c) 3.2 × 10–16 J (d) 3.2 × 10–11 J 2 2 2
e E t e2 E 2t 2
1+ 2 +
3. A small photocell is placed at a distance of 4 m 2 m2 v02 m 2 v02
from a photosensitive surface. When light falls 7. A particle ‘P’ is formed due to a completely
on the surface the current is 5 mA. If the distance inelastic collision of particles ‘x’ and ‘y’ having
of cell is decreased to 1 m, the current will become de-Broglie wavelengths ‘gx’ and ‘gy’ respectively.
(a) 10 mA (b) 40 mA If x and y were moving in opposite directions,
(c) 20 mA (d) 80 mA then the de-Broglie wavelength of ‘P’ is:
4. A and B are two metals with threshold g xg y g xg y
frequencies 1.8 × 1014 Hz and 2.2 × 1014 Hz. Two (a) (b)
identical photons of energy 0.825 eV each are gx +g y | gx -g y |
incident on them. Then photoelectrons are
emitted in (Take h = 6.6 × 10–34 Js) (c) gx -g y (d) gx +g y

@ebooks_for_freee on Telegram
86 PHYSICS

8. The stopping potential V0 (in volt) as a function 12. In an experiment on photoelectric effect, a
of frequency (v) for a sodium emitter, is shown in student plots stopping potential V 0 against
the figure. The work function of sodium, from the reciprocal of the wavelength l of the incident
data plotted in the figure, will be : light for two different metals A and B. These are
(Given : Planck’s constant (h) = 6.63 × 10–34 Js, shown in the figure.
electron charge e = 1.6 × 10–19 C)

(a) 1.82 eV
V0 Metal A
Metal B
(b) 1.66 eV

(c) 1.95 eV

(d) 2.12 eV 1/l


9. A metal plate of area 1 × 10–4 m2 is illuminated
by a radiation of intensity 16 mW/m2. The work Looking at the graphs, you can most
function of the metal is 5 eV. The energy of the appropriately say that:
incident photons is 10 eV and only 10% of it
produces photo electrons. The number of (a) Work function of metal B is greater than that
emitted photo electrons per second and their of metal A
maximum energy, respectively, will be: (b) For light of certain wavelength falling on
[1 eV = 1.6 × 10–19 J] both metal, maximum kinetic energy of
electrons emitted from A will be greater than
(a) 1014 and 10 eV (b) 1012 and 5 eV those emitted from B.
(c) 1011 and 5 eV (d) 1010 and 5 eV (c) Work function of metal A is greater than that
10. Two particles move at right angle to each other. of metal B
Their de Broglie wavelengths are l1 and l2
(d) Students data is not correct
respectively. The particles suffer perfectly
inelastic collision. The de Broglie wavelength 13. The threshold frequency for a metallic surface
l, of the final particle, is given by : corresponds to an energy of 6.2 eV and the
stopping potential for a radiation incident on
1 1 1 this surface is 5 V. The incident radiation lies in
(a) 2
= + (b) l = l1l2
l l12 l 22 (a) ultra-violet region (b) infra-red region
l2 + l2 2 1 1 (c) visible region (d) X-ray region
(c) l = (d) = +
2 l l1 l 2 14. According to Einstein’s photoelectric equation,
the plot of the kinetic energy of the emitted photo
11. The magnetic field associated with a light
electrons from a metal Versus the frequency, of
wave is given at the origin by
the incident radiation gives a straight line whose
B = B0 [sin(3.14 × 107)ct + sin(6.28 × 107)ct]. slope
If this light falls on a silver plate having a work (a) depends both on the intensity of the
function of 4.7 eV, what will be the maximum radiation and the metal used
kinetic energy of the photoelectrons? (b) depends on the intensity of the radiation
(c = 3 × 108 ms–1, h = 6.6 × 10–34J-s) (c) depends on the nature of the metal used
(a) 6.82 eV (b) 12.5 eV (d) is the same for the all metals and independent
of the intensity of the radiation
(c) 8.52 eV (d) 7.72 eV

@ebooks_for_freee on Telegram
Dual Nature of Radiation and Matter 87
15. Photoelectrons are ejected from a metal when 20. The work functions of metals A and B are in the
light of frequency u falls on it. Pick out the wrong raio 1 : 2.
statement from the following. If light of frequencies f and 2f are incident on the
(a) No electrons are emitted if u is less than surfaces of A and B respectively, the ratio of the
W/h, where W is the work function of the maximum kinetic energies of photoelectrons
metal emitted is (f is greater than threshold frequency
(b) The ejection of the photoelectrons is of A, 2f is greater than threshold frequency of B)
instantaneous. (a) 1 : 1 (b) 1 : 2
(c) The maximum energy of the photoelectrons (c) 1 : 3 (d) 1 : 4
is hu.
(d) The maximum energy of the photoelectrons Numeric Value Answer
is independent of the intensity of the light.
16. When photons of energy hn fall on an aluminium 21. Surface of certain metal is first illuminated with
plate (of work function E0), photoelectrons of light of wavelength l1 = 350 nm and then, by
maximum kinetic energy K are ejected. If the light of wavelength l2 = 540 nm. It is found
frequency of the radiation is doubled, the that the maximum speed of the photo electrons
maximum kinetic energy of the ejected in the two cases differ by a factor of (2) The
photoelectrons will be work function of the metal (in eV) is
(a) 2K (b) K
1240
(c) K + hn (d) K + E0 (Energy of photon = eV )
l ( in nm )
17. A Laser light of wavelength 660 nm is used to
weld Retina detachment. If a Laser pulse of width 22. The magnetic field associated with a light
60 ms and power 0.5 kW is used the approximate wave is given at the origin by
number of photons in the pulse are : [Take
B = B0 [sin(3.14 × 107)ct + sin(6.28 × 107)ct].
Planck’s constant h = 6.62 × 10–34 Js]
(a) 1020 (b) 1018 If this light falls on a silver plate having a work
(c) 10 22 (d) 1019 function of 4.7 eV, what will be the maximum
kinetic energy (in eV) of the photoelectrons?
18. When a metallic surface is illuminated by a light
of wavelength l, the stopping potential for the (c = 3 × 108 ms–1, h = 6.6 × 10–34J-s)
photoelectric current is 3 V. When the same 23. A metal plate of area 1 × 10–4 m2 is illuminated
surface is illuminated by light of wavelength 2l, by a radiation of intensity 16 mW/m2. The
the stopping potential is 1V. The threshold work function of the metal is 5 eV. The energy
wavelength for this surface is of the incident photons is 10 eV and only 10%
(a) 4l (b) 3.5 l of it produces photo electrons. The number of
(c) 3l (d) 2.75l emitted photo electrons per second is
19. Photoelectric emission is observed from a [1 eV = 1.6 × 10–19 J]
metallic surface for frequencies v1 and v2 of the
24. If the deBroglie wavelength of an electron is
incident light rays (v1 > v2). If the maximum
equal to 10–3 times the wavelength of a photon
values of kinetic energy of the photoelectrons
emitted in the two cases are in the ratio of 1 : k, of frequency 6 × 1014 Hz, then the speed (in m/s)
then the threshold frequency of the metallic of electron is equal to :
surface is (Speed of light = 3 × 108 m/s)
v1 - v 2 kv1 - v 2
(a) (b) Planck’s constant = 6.63 × 10–34J.s
k -1 k -1
kv2 - v1 v2 - v1 Mass of electron = 9.1 × 10–31 kg)
(c) (d)
k -1 k

@ebooks_for_freee on Telegram
88 PHYSICS

25. In a photoelectric experiment, the wavelength of [Given Planck’s constant h = 6.6 × 10–34 Js,
the light incident on a metal is changed from 300 speed of light c = 3.0 × 108 m/s]
nm to 400 nm. The decrease in the stopping 29. Light of wavelength 180 nm ejects photoelectron
potential (in V) is from a plate of a metal whose work function is 2
eV. If a uniform magnetic field of 5 × 10–5 T is
æ hc ö
çè = 1240 nm-V÷ø applied parallel to plate, what would be the radius
e
(in metre) of the path followed by electrons
26. A particle A of mass ‘m’ and charge ‘q’ is
ejected normally from the plate with maximum
accelerated by a potential difference of 50v
energy ?
Another particle B of mass ‘4m’ and charge‘q’ is
accelerated by a potential differnce of 2500V. 30. In a photoelectric effect experiment the
threshold wavelength of light is 380 nm. If the
lA wavelength of incident light is 260 nm, the
The ratio of de-Broglie wavelength is
lB
maximum kinetic energy (in eV) of emitted
27. A monochromatic source of light operating at
electrons will be:
200W emits 4 × 1020 photons per second. Find
the wavelength (in nm) of the light. 1237
Given E (in eV) =
28. A 2 mW laser operates at a wavelength of 500 l(in nm)
nm. The number of photons that will be
emitted per second is :

ANSW ER KEY
15
1 (a) 4 (b) 7 (b) 10 (a) 13 (a) 16 (c) 19 (b ) 22 (7.7) 25 (1) 28 (5 × 10 )
11
2 (a) 5 (b) 8 (b) 11 (d) 14 (d ) 17 (a) 20 (b ) 23 (10 ) 26 (14.14) 29 (0.149)
6
3 (d) 6 (c) 9 (c) 12 (d) 15 (c) 18 (a) 21 (1.8) 24 (1.45 × 10 ) 27 (400) 30 (1.5)

@ebooks_for_freee on Telegram
Atoms 89

26
ATOMS

3
MCQs with One Correct Answer (a) 15 × 106 (b) × 106
5
1. The significant result deduced from the 5
(c) × 106 (d) None of these
Rutherford's scattering experiment is that 9
6. In the Rutherford experiment, a-particles are
(a) whole of the positive charge is concentrated
scattered from a nucleus as shown. Out of the
at the centre of atom
four paths, which path is not possible?
(b) there are neutrons inside the nucleus A
(c) a-particles are helium nuclei (a) D
(d) electrons are embedded in the atom (b) B B

2. An a-particle of energy 5 MeV is scattered through (c) C C


180º by a fixed uranium nucleus. The distance of D
closest approach is of the order of (d) A
7. Electrons in a certain energy level n = n 1, can
(a) 10–12 cm (b) 10–10 cm
emit 3 spectral lines. When they are in another
(c) 10–20 cm (d) 10–15 cm energy level, n = n2. They can emit 6 spectral
3. The energy of electron in the nth orbit of lines. The orbital speed of the electrons in the
-13.6 two orbits are in the ratio of
hydrogen atom is expressed as E n = eV.
n2 (a) 4 : 3 (b) 3 : 4
The shortest wavelength of Lyman series will be (c) 2 : 1 (d) 1 : 2
(a) 910 Å (b) 5463 Å 8. Ionization energy of hydrogen atom is 13.6eV.
(c) 1315 Å (d) None of these Hydrogen atoms in the ground state are excited
4. Consider an electron in the n th orbit of a by monochromatic radiation of photon energy
hydrogen atom in the Bohr ’s model. The 12.1 eV. According to Bohr’s theory, the spectral
circumference of the orbit can be expressed in lines emitted by hydrogen will be
terms of the de Broglie wavelength l of that (a) three (b) four
electron as
(c) one (d) fwo
(a) 0.529 nl (b) nλ 9. The ionisation energy of hydrogen is 13.6 eV.
(c) (13.6) l (d) nl The energy required to excite the electron from
5. In Rutherford’s scattering experiment, the the first to the third orbit is approximately
number of a-particles scattered at 60° is 5 × 106.
The number of a-particles scattered at 120° will (a) 10.2 J (b) 12.09 × 10–6J
be (c) 19.94 J (d) 19.34 × 10–19J

@ebooks_for_freee on Telegram
90 PHYSICS
10. The energy levels of the hydrogen spectrum is 14. The energy required to remove the electron from
shown in figure. There are some transitions A, a singly ionized Helium atom is 2.2 times the
B, C, D and E. Transition A, B and C respectively energy required to remove an electron from
represent Helium atom. The total energy required to ionize
n=¥ – 0.00 eV the Helium atom completely is:
n=6 – 0.36 eV
n=5 – 0.54 eV (a) 20 eV (b) 79 eV
n=4
C
– 0.85 eV (c) 109 eV (d) 34 eV
n=3 – 1.51 eV
B D 15. The binding energy of the electron in a hydrogen
n=2 – 3.39 eV
A E atom is 13.6 eV, the energy required to remove
the electron from the first excited state of Li ++ is:
n=1 – 13.5 eV
(a) first member of Lyman series, third spectral (a) 122.4 eV (b) 30.6 eV
line of Balmer series and the second spectral (c) 13.6 eV (d) 3.4 eV
line of Paschen series 16. The electron of a hydrogen atom makes a
(b) ionization potential of hydrogen, second transition from the (n + 1)th orbit to the nth orbit.
spectral line of Balmer series, third spectral For large n the wavelength of the emitted
line of Paschen series radiation is proportional to
(c) series limit of Lyman series, third spectral (a) n (b) n 3
line of Balmer series and second spectral
(c) n 4 (d) n 2
line of Paschen series
(d) series limit of Lyman series, second spectral 17. The transition from the state n = 4 to n = 3 in a
line of Balmer series and third spectral line hydrogen like atom results in ultraviolet
of Paschen series radiation. Infrared radiation will be obtained in
the transition from :
1 2
11. An alpha nucleus of energy mv bombards a (a) 3 ® 2 (b) 4 ® 2
2
heavy nuclear target of charge Ze. Then the (c) 5 ® 4 (d) 2 ® 1
distance of closest approach for the alpha 18. Which of the following transitions in hydrogen
nucleus will be proportional to atoms emit photons of highest frequency?
1 (a) n = 1 to n = 2 (b) n = 2 to n = 6
(a) v 2 (b) (c) n = 6 to n = 2 (d) n = 2 to n = 1
m
1 1 19. In a hydrogen like atom, when an electron jumps
(c) 2 (d) from the M-shell to the L-shell, the wavelength
v Ze
of emitted radiation is l. If an electron jumps from
12. The energy required to ionise a hydrogen like
ion in its ground state is 9 Rydbergs. What is N-shell to the L-shell, the wavelength of emitted
the wavelength of the radiation emitted when radiation will be:
the electron in this ion jumps from the second 27 16
excited state to the ground state? (a) l (b) l
20 25
(a) 24.2 nm (b) 11.4 nm
(c) 35.8 nm (d) 8.6 nm 25 20
(c) l (d) l
13. In a hydrogen atom the electron makes a 16 27
transition from (n + 1)th level to the nth level. If
20. Consider an electron in a hydrogen atom,
n >> 1, the frequency of radiation emitted is
proportional to : revolving in its second excited state (having
1 1 radius 4.65 Å). The de-Broglie wavelength of
(a) (b) this electron is :
n n3
1 1 (a) 3.5 Å (b) 6.6 Å
(c) 2 (d) (c) 12.9 Å (d) 9.7 Å
n n4

@ebooks_for_freee on Telegram
Atoms 91

Numeric Value Answer


21. Taking the wavelength of first Balmer line in
hydrogen spectrum (n = 3 to n = 2) as 660 nm,
the wavelength (in nm) of the 2nd Balmer line
(n = 4 to n = 2) will be;
22. An excited He+ ion emits two photons in
succession, with wavelengths 108.5 nm and 26. Ratio of the wavelengths of first line of Lyman
30.4 nm, in making a transition to ground state. series and first line of Balmer series is
The quantum number n, corresponding to its
27. As per Bohr model, the minimum energy (in eV)
initial excited state is (for photon of wavelength
required to remove an electron from the ground
1240eV state of doubly ionized Li atom (Z = 3) is
l, energy E =
l(in nm) 28. The ionisation energy of hydrogen atom is 13.6
23. The largest wavelength in the ultraviolet region eV. Following Bohr¢s theory, the energy (in eV)
of the hydrogen spectrum is 122 nm. The smallest corresponding to a transition between the 3rd
wavelength in the infrared region of the and the 4th orbit is
hydrogen spectrum (to the nearest integer) is
24. An electron in hydrogen like atom makes a 29. If the binding energy (in eV) of the electron in a
transition from nth orbit and emits radiation hydrogen atom is 13.6 eV, the energy (in eV)
corresponding to Lyman series. If de-Broglie required to remove the electron from the first
wavelength of electron in nth orbit is equal to excited state of Li++ is
the wavelength of radiation emitted, find the 30. An electron in hydrogen atom jumps from a level n
value of n. The atomic number of atom is 11. = 4 to n = 1. The momentum (in kg m/s) of the
25. Some energy levels of a molecule are shown in recoiled atom is
the figure. The ratio of the wavelengths
r = l1 / l 2 , is given by

ANSWER KEY
1 (a) 4 (d) 7 (a) 10 (c) 13 (b) 16 (b) 19 (d) 22 (5) 25 (0.34) 28 (0.66)
2 (a) 5 (c) 8 (a) 11 (b) 14 (b) 17 (c) 20 (d) 23 (823.5) 26 (0.18) 29 (30.6)
3 (a) 6 (c) 9 (d) 12 (b) 15 (b) 18 (d) 21 (488.9) 24 (25) 27 (122.4) 30 (6.8 × 10–27)

@ebooks_for_freee on Telegram
92 PHYSICS

NUCLEI
27

MCQs with One Correct Answer 5. The nuclear radius of a nucleus with nucleon
number 16 is 3 × 10–15 m. Then, the nuclear
1. If radius of the 27
nucleus is taken to be RAl, radius of a nucleus with nucleon number 128 is
12 Al
(a) 3 × 10–15 m (b) 1.5×10–15 m
125 (c) 6 × 10 m (d) 4.5 × 10–15 m
then the radius of 53 Te nucleus is nearly: –15

6. If MO is the mass of an oxygen isotope 8O17,


5 3 MP and MN are the masses of a proton and a
(a) R Al (b) R Al
3 5 neutron respectively, the nuclear binding energy
1/3 1/3
of the isotope is
æ 13 ö æ 53 ö (a) (MO – 17MN)c2
(c) ç ÷ R Al (d) ç ÷ R Al
è 53 ø è 13 ø (b) (MO – 8MP)c2
(c) (8MP + 9MN – MO)c2
2. When a U238 nucleus originally at rest, decays
(d) MOc 2
by emitting an alpha particle having a speed ‘u’,
the recoil speed of the residual nucleus is 7. 27
If the nucleus 13 Al has nuclear radius of about
4u 3.6 fm, then 12532 Te would have its radius
(a) (b) - 4u
238 234 approximately as
(a) 9.6 f m (b) 12.0 f m
4u
(c) (d) - 4u (c) 4.8 f m (d) 6.0 f m.
234 238 8. Which of the following nuclei has lowest value
3. At any instant, the ratio of the amount of of the binding energy per nucleon :
radioactive substances is 2 : 1. If their half lives (a) 4 (b) 52
2 He 24 Cr
be respectively 12 and 16 hours, then after two
days, what will be the ratio of the substances ? (c) 152 (d) 100
62 Sm 80 Hg
(a) 1 : 1 (b) 2 : 1
9. Half lives for a and b emission of a radioactive
(c) 1 : 2 (d) 1 : 4 material are 16 years and 48 years respectively.
4. A nucleus disintegrates into two nuclear parts When material decays giving a and b emission
which have their velocities in the ratio 2 : 1. Ratio simultaneously, time in which 3/4th material
of their nuclear sizes will be decays is
(a) 21/3 : 1 (b) 1 : 31/2 (a) 29 years (b) 24 years
1/2
(c) 3 : 1 (d) 1 : 21/3 (c) 64 years (d) 12 years

@ebooks_for_freee on Telegram
Nuclei 93
10. The initial activity of a certain radioactive 16. The activity of a radioactive sample falls from
isotope was measured as 16000 counts min –1 700 s –1 to 500 s –1 in 30 minutes. Its half life is
and its activity after 12 h was 2100 counts close to:
min–1, its half-life, in hours, is nearest to [Given
(a) 72 min (b) 62 min
loge (7.2) = 2]
(c) 66 min (d) 52 min
(a) 9.0 (b) 6.0
17. The energy spectrum of b-particles [number N(E)
(c) 4.0 (d) 3.0
as a function of b-energy E] emitted from a
11. The radius R of a nucleus of mass number A can
radioactive source is
be estimated by the formula R = (1.3 × 10–15)
A1/3 m. It follows that the mass density of a
nucleus is of the order of : (a) N(E)

( M prot. @ M neut. ; 1.67 ´ 10 -27


kg)
E
E0
(a) 103 kg m–3 (b) 1010 kg m–3
(c) 1024 kg m–3 (d) 1017 kg m–3
12. Find the Binding energy per neucleon for 120 (b) N(E)
50 Sn.
Mass of proton mp = 1.00783 U, mass of neutron E
E0
mn = 1.00867 U and mass of tin nucleus mSn =
119.902199 U. (take 1U = 931 MeV)
(a) 7.5 MeV (b) 9.0 MeV (c) N(E)

(c) 8.0 MeV (d) 8.5 MeV


E
13. In a reactor, 2 kg of 92U235 fuel is fully used up in E0

30 days. The energy released per fission is 200


MeV. Given that the Avogadro number, N = 6.023
(d) N(E)
× 1026 per kilo mole and 1 eV = 1.6 × 10–19 J. The
power output of the reactor is close to: E
E0
(a) 35 MW (b) 60 MW
18. The decay constants of a radioactive substance
(c) 125 MW (d) 54 MW for a and b emission are la and lb respectively.
14. A piece of bone of an animal from a ruin is found If the substance emits a and b simultaneously,
to have 14C activity of 12 disintegrations per then the average half life of the material will be
minute per gm of its carbon content. The 14C
2TaTb
activity of a living animal is 16 disintegrations
(a) Ta + Tb (b) Ta + Tb
per minute per gm. How long ago nearly did the
animal die? (Given half life of 14C is t1/2 = 5760 Ta Tb
years) (c)Ta + Tb (d)
1
2
Ta + Tb ( )
(a) 1672 years (b) 2391 years
19. The half-life of a radioactive element A is the
(c) 3291 years (d) 4453 years
same as the mean-life of another radioactive
15. The binding energy per nucleon of deuteron element B. Initially both substances have the
( H) and helium nucleus ( He) is 1.1 MeV
2
1
4
2
same number of atoms, then :
(a) A and B decay at the same rate always.
and 7 MeV respectively. If two deuteron nuclei
react to form a single helium nucleus, then the (b) A and B decay at the same rate initially.
energy released is (c) A will decay at a faster rate than B.
(a) 23.6 MeV (b) 26.9 MeV (d) B will decay at a faster rate than A.
(c) 13.9 MeV (d) 19.2 MeV

@ebooks_for_freee on Telegram
94 PHYSICS

20. In a radioactive decay chain, the initial nucleus 25. The count rate from a radioactive sample falls
is 232 from 4.0× 106 per second to 1 × 106 per second
90 Th . At the end there are 6 a-particles and
4 b-particles which are emitted. If the end nucleus in 20 hour. What will be the count rate per second
A , A and Z are given by : 100 hour after the beginning?
is ZX
26. In an ore containing uranium, the ratio of U238
(a) A = 208 ; Z = 80 (b) A = 202 ; Z = 80
to Pb206 nuclei is 3. Calculate the age of the ore,
(c) A = 208 ; Z = 82 (d) A = 200; Z = 81
(in year) assuming that all the lead present in the
Numeric Value Answer ore in the final stable product of U238. Take the
half life of U238 to be 4.5 × 109 year.
21. Using a nuclear counter the count rate of
emitted particles from a radioactive source is 27. In a nuclear reactor U235 undergoes fission
measured. At t = 0 it was 1600 counts per liberating 200 MeV of energy. The reactor has
second and t = 8 seconds it was 100 counts 10% efficiency and produces 1000 MW power.
If the reactor is to function for 10 year, find the
per second. The count rate observed, as
total mass (in kg) of the uranium required.
counts per second, at t = 6 seconds is
28. The disintegration rate of a certain radioactive
22. The ratio of the mass densities of nuclei of
40 sample at any instant is 4750 disintegrations per
Ca and 16O is
minute. Five minutes later the rate becomes 2700
23. The activity of a freshly prepared radioactive disintegrations per minute. Calculate half life of
sample is 1010 disintegrations per second, whose the sample (in minute)
mean life is 109 s. The mass of an atom of this 29. The mass defect for the nucleus of helium is
radioisotope is 10–25 kg. The mass (in mg) of the 0.0303 a.m.u. What is the binding energy per
radioactive sample is nucleon for helium in MeV?
24. The half life of radon is 3.8 days. After how many 30. If the radius of a nucleus 256X is 8 fermi, then
days will only one twentieth of radon sample be the radius (in fermi) of 4He nucleus will be
left over?

ANSWER KEY
3
1 (a) 4 (d) 7 (d) 10 (c) 13 (b) 16 (b) 19 (d) 22 (1) 25 (3.91 × 10 ) 28 (6.1)
9
2 (c) 5 (c) 8 (a) 11 (d) 14 (b) 17 (c) 20 (c) 23 (1) 26 (1.868 × 10 ) 29 (7)
4
3 (a) 6 (c) 9 (b) 12 (d) 15 (a) 18 (c) 21 (200) 24 (16.45) 27 (3.8 × 10 ) 30 (2)

@ebooks_for_freee on Telegram
Semiconductor Electronics: Materials, Devices and Simple Circuits 95

SEMICONDUCTOR
ELECTRONICS: 28
MATERIALS, DEVICES
AND SIMPLE CIRCUITS

MCQs with One Correct Answer 5. The following circut represents

1. In a p-type semiconductor the acceptor level is A


situated 60 meV above the valence band. The
Y
maximum wavelength of light required to
produce a hole will be B
(a) 0.207 × 10–5 m (b) 2.07 × 10–5 m
–5
(a) OR gate (b) AND gate
(c) 20.7 × 10 m (d) 2075 × 10–5 m
(c) NAND gate (d) None of these
2. A half-wave rectifier is being used to rectify an
alternating voltage of frequency 50 Hz. The 6. What is the conductivity (in mho m –1 )of a
number of pulses of rectified current obtained in semiconductor if electron density = 5 × 1012/
one second is cm3 and hole density = 8 × 1013/cm3 (µe = 2.3 m2
(a) 50 (b) 25 V–1 s–1, µh = 0.01 m2V–1 s–1)
(c) 100 (d) 2000 (a) 5.634 (b) 1.968
3. Which of the junction diodes shown below are (c) 3.421 (d) 8.964
forward biased? 7. In the energy band diagram of a material shown
–10 V +10 V below, the open circles and filled circles denote
holes and electrons respectively.
R R
(a) (b) The material is
Ec
–5 V +5 V (a) an insulator
(b) a metal Eg
R R
–5 V (c) an n-type EA
–10 V
semiconductor
(c) (d) Ev
(d) a p-type
semiconductor
4. The current gain in the common emitter mode of 8. A potential barrier of 0.50 V exists across a p-n
a transistor is 10. The input impedance is 20kW junction. If the depletion region is 5.0 × 10–7 m
and load of resistance is 100kW. The power gain wide, the intensity of the electric field in this
is region is
(a) 300 (b) 500 (a) 1.0 × 106 V/m (b) 1.0 × 105 V/m
(c) 200 (d) 100 (c) 2.0 × 105 V/m (d) 2.0 × 106 V/m

@ebooks_for_freee on Telegram
96 PHYSICS

9. In case of a common emitter transistor amplifier, 16. The V–I characteristic of a diode is shown in the
the ratio of the collector current to the emitter figure. The ratio of forward to reverse bias
current Ic /Ie is 0.96. The current gain of the resistance is :
amplifier is I(mA)
(a) 6 (b) 48
(c) 24 (d) 12 20
10. In common emitter amplifier, the current gain is 62. 15
The collector resistance and input resistance are 5
kW an 500 W respectively. If the input voltage is 10
0.01 V, the output voltage is –10
(a) 0.62 V (b) 6.2 V .7 .8 V (Volt)
1m A
(c) 62 V (d) 620 V
11. With increasing biasing voltage of a
photodiode, the photocurrent magnitude :
(a) 10 (b) 10–6
(a) remains constant 6
(c) 10 (d) 100
(b) increases initially and after attaining
17. In the middle of the depletion layer of a reverse-
certain value, it decreases
biased p-n junction, the
(c) Increases linearly
(a) electric field is zero
(d) increases initially and saturates finally (b) potential is maximum
12. If a semiconductor photodiode can detect a photon (c) electric field is maximum
with a maximum wavelength of 400 nm, then its (d) potential is zero
band gap energy is: 18. The electrical conductivity of a semiconductor
Planck's constant, h = 6.63 × 10–34 J.s. increases when electromagnetic radiation of
Speed of light, c = 3 × 108 m/s wavelength shorter than 2480 nm is incident on
(a) 1.1 eV (b) 2.0 eV it. The band gap in (eV) for the semiconductor is
(c) 1.5 eV (d) 3.1 eV (a) 2.5 eV (b) 1.1 eV
13. A red LED emits light at 0.1 watt uniformly (c) 0.7 eV (d) 0.5 eV
around it. The amplitude of the electric field of 19. Which of the following gives a reversible
the light at a distance of 1 m from the diode is : operation?
(a) 5.48 V/m (b) 7.75 V/m
(a)
(c) 1.73 V/m (d) 2.45 V/m
14. The forward biased diode connection is:
+2V –2V (b)
(a)
–3V –3V
(b) (c)
2V 4V
(c)
–2V +2V (d)
(d)
15. If the ratio of the concentration of electrons to
7 20. When a diode is forward biased, it has a voltage
that of holes in a semiconductor is and the drop of 0.5 V. The safe limit of current through
5
7 the diode is 10 mA. If a battery of emf 1.5 V is
ratio of currents is , then what is the ratio of used in the circuit, the value of minimum
4
their drift velocities? resistance to be connected in series with the
5 4 diode so that the current does not exceed the safe
(a) (b)
8 5 limit is :
5 4 (a) 300 W (b) 50 W
(c) (d) (c) 100 W (d) 200 W
4 7

@ebooks_for_freee on Telegram
Semiconductor Electronics: Materials, Devices and Simple Circuits 97

Numeric Value Answer 27. In a photodiode, the conductivity increases


when the material is exposed to light. It is found
21. Mobility of electrons in a semiconductor is that the conductivity changes only if the
defined as the ratio of their drift velocity to wavelength is less than 620 nm. What is the band
the applied electric field. If, for an n-type gap (in eV)?
semiconductor, the density of electrons is 28. When the base current in a transistor is changed
1019 m –3 and their mobility is 1.6m2/(V.s) from 30 µA to 80 µA, the collector current is
then the resistivity (in W m) of the changed from 1.0 mA to 3.5 mA. Find the current
semiconductor (since it is an n-type gain b.
semiconductor contribution of holes is 29. The transfer characteristic curve of a transistor,
ignored) is having input and output resistance 100 W and
22. Ge and Si diodes start conducting at 0.3 V 100 k W respectively, is shown in the figure.
and 0.7 V respectively. In the following figure The Voltage gain is
if Ge diode connection are reversed, the value
of Vo (in volt) changes by : (assume that the
Ge diode has large breakdown voltage)
Ge
Vo

12 V Si 5K

23. Copper, a monovalent, has molar mass 63.54 g/


mol and density 8.96 g/cm3. What is the number
density n (in m–3) of conduction electron in
copper?
24. An LED is constructed from a p-n junction
based on a certain Ga-As -P semiconducting 30. The circuit shown below contains two ideal
material whose energy gap is 1.9 eV. What is diodes, each with a forward resistance of 50 W. If
the wavelength (in nm) of the emitted light? the battery voltage is 6V, the current through the
25. For the circuit shown below, the current (in 100 W resistance (in Ampere) is :
mA) through the Zener diode is: D1 150 W
5 kW

75 W

120 V 50 V 10 kW
D2

100 W
26. In half-wave rectification, what is the output
frequency (in Hz) if the input frequency is 50 6V
Hz?
ANSWER KEY
1 (b) 4 (b) 7 (d) 10 (b) 13 (d) 16 (b) 19 (d) 22 (0.04) 25 (9) 28 (50)
2 (b) 5 (d) 8 (a) 11 (d) 14 (a) 17 (a) 20 (c) 23 (8.49 × 10 ) 26 (50) 29 (20 × 103 )
26

3 (a) 6 (b) 9 (c) 12 (d) 15 (c) 18 (d) 21 (0.04) 24 (650) 27 (2.0) 30 (0.02)

@ebooks_for_freee on Telegram
98 PHYSICS

COMMUNICATION 29
SYSTEMS

MCQs with One Correct Answer (a) w c and wc2 + w 2m


1. 100% modulation in FM means (b) wc , wc + wm and wc - wm
(a) actual frequency deviation > maximum (c) wc and w m
allowed frequency deviation
(d) wc and wc wm
(b) actual frequency deviation = maximum
allowed frequency deviation 5. The fundamental radio antenna is a metal rod
which has a length equal to
(c) actual frequency deviation ³ maximum
(a) l in free space at the frequency of operation
allowed frequency deviation
(b) l/2 in free space at the frequency of
(d) actual frequency deviation < maximum
operation
allowed frequency deviation (c) l/4 in free space at the frequency of
2. If a carrier wave c(t) = A sin wct is amplitude operation
modulated by a modulator signal m(t) = A sin (d) 3l/4 in free space at the frequency of
wmt then the equation of modulated signal operation
[Cm(t)] and its modulation index are respectively 6. An audio signal represented as 25 sin 2p (2000 t)
amplitude modulated by a carrier wave : 60 sin
(a) Cm (t) = A (1 + sin wm t) sin wc t and 2
2p(100, 000)t. The modulation index of the
(b) Cm (t) = A (1 + sin wm t) sin wm t and 1 modulated signal is
(c) Cm (t) = A (1 + sin wm t) sin wc t and 1 (a) 25% (b) 41.6 %
(d) Cm (t) = A (1 + sin wc t) sin wm t and 2 (c) 50 % (d) 75 %
3. The maximum line-of-sight distance dM between 7. Intensity of electric field obtained at receiver
two antennas having heights hT and hR above antenna for a space wave propagation is
the earth is
(a) directly proportional to the perpendicular-
(a) R ( hT + h R ) (b) 2R ( h T + h R ) distance from transmitter to antenna
(c) Rh T + 2Rh R (d) 2Rh T + 2Rh R (b) inversely proportional to the perpendicular-
distance from transmitter to antenna
4. Given the electric field of a complete amplitude
modulated wave as (c) directly proportional to the square
perpendicular-distance from transmitter to
æ ö
ˆ c ç 1 + Em cos wm t ÷ cos wc t .
®
E = iE antenna
è Ec ø (d) inversely proportional to the square
Where the subscript c stands for the carrier wave perpendicular-distance from transmitter to
and m for the modulating signal. The frequencies antenna
present in the modulated wave are

@ebooks_for_freee on Telegram
Communication Systems 99
8. For sky wave propagation of a 10 MHz signal, (c) same length antenna will work for both
what should be the minimum electron density in (d) information given is not enough to say
ionosphere which one to use for which
(a) ~ 1.2 × 1012 m–3 (b) ~ 106 m–3 14. An AM- signal is given as
(c) ~ 1014 m–3 (d) ~ 1022 m–3
xAM (t) = 100 [p(t) + 0.5g(t)] cos wct
9. An amplitude modulated wave is represented
by the expression vm = 5(1 + 0.6 cos 6280t) sin(211 in interval 0 £ t £ 1. One set of possible values of
× 104t) volts the modulating signal and modulation index
The minimum and maximum amplitudes of the would be
amplitude modulated wave are, respectively : (a) t, 0.5 (b) t, 1.0
3 5 (c) t, 1.5 (d) t2, 2.0
(a) V, 5 V (b) V, 8 V 15. A device with input x(t) and outputy(t) is
2 2
(c) 5 V, 8 V (d) 3 V, 5 V characterized by: y(t) = x2.
10. In an amplitude modulator circuit, the carrier An FM signal with frequency deviation of 90
wave is given by, C(t) = 4 sin (20000 pt) while kHz and modulating signal bandwidth of 5 kHz
modulating signal is given by, m(t) = 2 sin (2000 is applid to this device. The bandwidth of the
pt). The values of modulation index and lower output signal is
side band frequency are : (a) 370 kHz (b) 190 kHz
(a) 0.5 and 10 kHz (b) 0.4 and 10 kHz (c) 380 kHz (d) 95 kHz.
(c) 0.3 and 9 kHz (d) 0.5 and 9 kHz 16. A sinusoidal carrier voltage of frequency 10 MHz
11. Television signals on earth cannot be received and amplitude 200 volts is amplitude modulated
at distances greater than 100 km from the by a sinusoidal voltage of frequency 10 kHz
transmission station. The reason behind this is producing 40% modulation. Calculate the
that frequency of upper and lower sidebands.
(a) the receiver antenna is unable to detect (a) 10010 kHz, 9990 kHz
the signal at a disance greater than 100 km
(b) 1010 kHz, 990 kHz
(b) the TV programme consists of both audio and
(c) 10100 Hz,9990 Hz
video signals
(d) 1010 MHz, 990 MHz
(c) the TV signals are less powerful than radio
signals 17. A modulated signal Cm(t) has the form Cm(t) =
30 sin 300pt + 10 (cos 200pt – cos 400pt). The
(d) the surface of earth is curved like a sphere carrier frequency fc, the modulating frequency
12. Sinusoidal carrier voltage of frequency 1.5 MHz (message frequency) fw and the modulation indix
and amplitude 50 V is amplitude modulated by m are respectively given by :
sin usoidal voltage of frequency 10 kHz 1
producing 50% modulation. The lower and upper (a) fc = 200 Hz; fw = 50 Hz; m =
2
side-band frequencies in kHz are 2
(a) 1490, 1510 (b) 1510, 1490 (b) fc = 150 Hz; fw = 50 Hz; m =
3
1 1 1 1 1
(c) , (d) , (c) fc = 150 Hz; fw = 30 Hz; m =
1490 1510 1510 1490 3
13. A radio station has two channels. One is AM at 1
(d) fc = 200 Hz; fw = 30 Hz; m =
1020 kHz and the other FM at 89.5 MHz. For 2
good results you will use 18. Long range radio transmission is possible when
the radio waves are reflected from the
(a) longer antenna for the AM channel and shorter
for the FM ionosphere. For this to happen the frequency of
the radio waves must be in the range:
(b) shorter antenna for the AM channel and longer
for the FM (a) 80 - 150 MHz (b) 8 - 25 MHz
(c) 1 - 3 MHz (d) 150 - 1500 kHz

@ebooks_for_freee on Telegram
100 PHYSICS
19. An AM wave is expressed as e = 10 (1 + 0.6 cos layer up to a range of 250 km, the critical
2000 p t) cos 2 × 108 pt volts, the minimum and frequency (in MHz) of the layer is
maximum value of modulated carrier wave are 25. Consider the following amplitude modulated
respectively. (AM) signal , where fm < B
(a) 10 V and 20 V (b) 4V and 8V
xAM (t) = 10 (1 + 0.5 sin 2pfmt) cos 2pfct
(c) 16 V and 4V (d) 8 V and 20 V
The average side-band power for the AM signal
20. When radio waves passes through ionosphere, given above is
phase difference between space current and
capacitive displacement current is 26. An audio signal consists of two distinct sounds:
one a human speech signal in the frequency
(a) 0 rad (b) (3p /2) rad
band of 200 Hz to 2700 Hz, while the other is a
(c) (p/2) rad (d) p rad high frequency music signal in the frequency
band of 10200 Hz to 15200 Hz. The ratio of the
Numeric Value Answer
AM signal bandwidth required to send both the
21. The electron density of a layer of ionosphere at signals together to the AM signal bandwidth
a height 150 km from the earth's surface is 9 × requried to send just the human speech is :
109 per m3. For the sky transmission from this 27. For an AM wave, the maximum voltage was
layer up to a range of 250 km, the critical found to be 10 V and minimum voltage was 4 V.
frequency (in MHz) of the layer is The modulation index of the wave is
22. An amplitude modulated voltage is expressed 28. A broadcast radio transmitter radiates 12 kW
as e = 10 (1 + 0.8 cos 2000 pt) cos 3 × 106 pt volt when percentage of modulation is 50%, then the
The peak value (in V) of carrier wave is unmodulated carrier power (in kW) is
23. The rms value of a carrier voltage is 100 volts. 29. The area (in km2) of the region covered by the
Compute its rms value (in V) when it has been TV broadcast by a TV tower of 100 m height is
amplitude modulated by a sinusoidal audio (Radius of the earth = 6.4 × 106 m)
voltage to a depth of 30%. 30. 12 signals each band limited to 5 kHz are to be
24. The electron density of a layer of ionosphere at transmitted by frequency-division multiplexer. If
a height 150 km from the earth's surface is 9 × AM-SSB modulation guard band of 1 kHz is used
109 per m3. For the sky transmission from this then the bandwidth (in kHz) of multiplexed signal
is

ANSWER KEY
1 (b) 4 (b) 7 (d) 10 (d) 13 (b) 16 (a) 19 (c) 22 (10) 25 (6.25) 28 (9.6)
3
2 (c) 5 (c) 8 (a) 11 (d) 14 (a) 17 (b) 20 (a) 23 (104.5) 26 (6) 29 (1.28 p ´10 )
3 (d) 6 (b) 9 (b) 12 (a) 15 (c) 18 (b) 21 (2.7) 24 (2.7) 27 (0.43) 30 (71)

@ebooks_for_freee on Telegram
Physical World, Units and Measurements 101

CHAPTER
Physical World, Units and
1 Measurements
1. (c) [x] = [bt2]. Hence [b] = [x /t2] = km/s2. A Force é 0 0 0 ù
9. (c) = = ëM L T û
2. (b) According to question B Force
angle 1
E y µ J x BZ Ct = angle Þ C = = = T -1
time T
\ Constant of proportionality angle 1
Dx = angle Þ D = = = L-1
Ey C m3 distance L
K= = =
BZ Jx J x As C T -1 é 0
\ = = ë M L T -1 ùû
E I D L- 1
[As = C (speed of light) and J = ] 10. (a) Number of significant figures in 23.023
B Area
=5
3. (a) The mean value of refractive index,
Number of significant figures in 0.0003 = 1
1.34 + 1.38 + 1.32 + 1.36 Number of significant figures in 2.1 × 10–3 = 2
m= = 1.35 So, the radiation belongs to X-rays part of the
4
and spectrum.
| (1.35 - 1.34) | + | (1.35 - 1.38) | + | (1.35 - 1.32) |
11. (d) No. of divisions on main scale = N
No. of divisions on vernier scale = N + 1
+ | (1.35 - 1.36) |
Dm = size of main scale division = a
4
Let size of vernier scale division be b
= 0.02 then we have
Dm 0.02 aN
Thus ´ 100 = ´ 100 = 1.48 aN = b (N + 1) Þ b =
m 1.35 N +1
4. (b) Here, b and x2 = L2 have same dimensions aN
Least count is a – b = a –
x2 L2 N +1
Also, a = = = M -1T 1 é N + 1 - N ù a
E ´t (
MLT 2 -2
T ) = aê
ë N +1 û
ú = N +1
a × b = [M–1 L2T1]
5. (a) F F l T2
12. (a) Surface tension, T = = . .
6. (b) F = ma = r volume ‘a’ l l l T2
To write volume in terms of ‘a’ and ‘f ’ T2
3 (As, F.l = K (energy); = V -2 )
2
æ Lö 6 3 -6 l
Volume = L3 = ç 2 ÷ T = a f Therefore, surface tension = [KV–2T–2]
èT ø
13. (a) X = 5YZ2
\ F = r a4 f –6
7. (b) X
ÞY µ 2 ...(i)
8. (d) Reynold’s number Z
= Coefficient of friction = [M0L0T0] Q Q2 [ A2T 2 ]
Curie is the unit of radioactivity (number of atoms X = Capacitance = = =
decaying per second) and frequency also has
V W [ ML2T -2 ]
the unit per second. X = [M–1L–2T4A2]
Q W F
Latent heat = and Gravitation potential = . Z=B= [Q F = ILB]
m m IL

@ebooks_for_freee on Telegram
102 PHYSICS

Z = [MT–2A–1]
G = é M -1L3T -2 ù and me = é M1L0T 0 ù
-1 -2 4 2
ë û ë û
[M L T A ]
Y= e2
[ MT -2 A-1 ]2 Now,
2 pe0 Gm e2
Y = [M–3L–2T8A4] (Using (i))
14. (c) Relative error in Surface area, 2
é M0 L0 T1A1 ù
Ds Dr ë û
= 2 ´ = a and relative error in volume, = 2
s r 2 p é M -1L-3T 4 A 2 ù é M -1L3T -2 ù é M1L0 T 0 ù
Dv Dr ë ûë ûë û
= 3´
v r éT 2A 2 ù
\ Relative error in volume w.r.t. relative error in ë û
= -1-1+ 2 -3+ 3 4 - 2 2 ù
area, é
2p M L T A
ë û
Dv 3
= a
v 2 éT 2 A 2 ù
ë û 1
15. (d) 30 Divisions of V.S. coincide with 29 = =
divisions of M.S. 2 p é M L T A ù 2p
0 0 2 2
ë û
29
\ 1 V.S.D = MSD 1
30 Q is dimensionless thus the combination
L.C. = 1 MSD – 1VSD 2p
29 e2
= 1 MSD - MSD would have the same value in
30 2 pe 0 Gm e2
1
= MSD different systems of units.
30 18. (d) Average diameter, dav = 5.5375 mm
1
= ´ 0.5° = 1 minute. Deviation of data, Dd = 0.07395 mm
30
As the measured data are upto two digits after
é e0 ù e 02 é e0 ù
decimal, therefore answer should be in two digits
16. (d) ê m ú = m e = ê ú
ëê 0 ûú 0 0 êë m 0 e 0 úû after decimal.
é 1 ù \ d = (5.54 ± 0.07) mm
= e0C[LT–1]×[e0] êQ = Cú
19. (c) Dimension of Force F = M1L1T–2
êë m0 e0 úû
q 2 Dimension of velocity V = L1T–1
Q F=
4pe 0 r 2 Dimension of work = M1L2T–2
Dimension of length = L
[ AT ]2
Þ [e 0 ] = -2 2
= [ A2 M -1 L-3T 4 ] Moment of inertia = ML2
[ MLT ] ´ [ L ]
é e ù IFv 2
\x =
\ ê 0 ú = [ LT -1 ] ´ [ A2 M -1 L-3T 4 ] WL4
êë m 0 úû
(M1L2 )(M1L1T -2 )(L1T -2 )2
-1 -2 3
= [M L T A ] 2 =
(M1L2 T -2 )(L4 )
17. (b) The dimensional formulae of
M1L-2 T -2
= = M1L-1T -2 = Energy density
e = é M0 L0T1A1 ù L3
ë û 20. (c) Take T µ ra Mb Gc and solving we get
e0 = é M -1L3T 4 A 2 ù 3
a= .
ë û
2

@ebooks_for_freee on Telegram
Physical World, Units and Measurements 103
21. (0.2) The current voltage relation of diode is DV DI
where, ´ 100 = ´100 = 3%
1000 V /T I
I = (e - 1) mA (given) V
DR DV DI
When, I = 5mA, e1000 V /T = 6mA then, ´ 100 = ´ 102 + ´ 102
R V I
1000 = 3% + 3% = 6%
Also, dI = (e1000 V /T

T 26. (4.6) We have
Error = ± 0.01 (By exponential function)
3.8 ´ 10 –6 + 4.2 ´ 10 –5
1000
= (6 mA) ´ ´ (0.01) = 0.2 mA = 0.38 × 10–5 + 4.2 × 10–5 = 4.58 ×10–5
300
= 4.6 × 10–5 (upto 2 significant figures)
22. (32) Given, P = a1/2 b2 c2 d–4,
Maximum relative error, M
27. (–1) µ P x v y Þ M L-2T -1
DP 1 Da Db Dc Dd At
= +2 +3 +4
P 2 a b c d x y
1
= ´ 2 + 2 ´ 1 + 3 ´ 3 + 4 ´ 5 = 32%
= éë M L-1T -2 ùû éë L1T -1 ûù
2
23. (40) Density of material in SI unit, = M x L- x + y T -2 x - y
128kg x = 1, –x + y = –2 and –2x – y = –1
= From here, we get y = –1. Thus, x ÷ y = –1
m3
Density of material in new system 28. (20) Required percentage
128 ( 50 g )( 20) 128 = 2´
0.02 1
´ 100 + 100 +
0.01
´ 100
= = ( 20) = 40 units 0.24 30 4.80
( 25 cm ) ( 4)
3 3 64
= 16.7 + 3.3 + 0.2 = 20%
24. (0.001) When screw on a screw-gauge is given
F L dyne 10 -5 N
six rotations, it moves by 3mm on the main scale 29. (0.1) Y = . = =
3 A DL cm 2 10-4 m 2
\ Pitch = = 0.5 mm
6 = 0.1 N / m2
Pitch 0.5 mm
\ Least count L.C. = =
CSD 50 L
30. (3) As, g = 4p2
=
1
mm = 0.01 mm = 0.001cm T2
100 Dg DL DT
25. (6) According to ohm’s law, V = IR So, ´100 = ´100 + 2 ´100
g L T
V
R= 0.1 1
I = ´100 + 2 ´ ´100 = 2.72 ; 3%
20 90
Absolute error
\ Percentage error = ´102
Measurement

@ebooks_for_freee on Telegram
104 PHYSICS

CHAPTER

Motion in a Straight Line


2
1. (d) Given x = ae–at + bebt 6. (b) Average velocity
dx v +v +v 3+4+5
= 1 2 3 = = 4 m/s
Velocity, v = = –aae–at + bbebt 3 3
dt
7. (b) Distance along a line i.e., displacement (s)
aa bt = t3 (Q s µ t3 given)
= - at + bbe By double differentiation of displacement, we get
e
i.e., go on increasing with time. acceleration.
2. (d) In (a), at the same time particle has two ds dt 3 dv d 3t 2
positions which is not possible. In (b), particle V= = = 3t 2 and a = = = 6t
dt dt dt dt
has two velocities at the same time. In (c), speed a = 6t or a µ t
is negative which is not possible. 8. (d) Let 'S' be the distance between two ends
a 'a' be the constant acceleration
3. (c) We have, Sn = u + (2n - 1) As we know v2 – u2 = 2aS
2
a v2 - u2
or 65 = u + (2 ´ 5 - 1) or, aS =
2 2
9 Let v be velocity at mid point.
or 65 = u + a ..... (1) 2 2 S
2 Therefore, vc - u = 2a
a 2
Also, 105 = u + (2 ´ 9 - 1) 2 2
2 v -u u 2 + v2
17 vc2 = u 2 + Þ vc =
or 105 = u + a ..... (2) 2 2
2 9. (b) Time taken by same ball to return to the
Equation (2) – (1) gives,
2u 2 ´ 20
17 9 hands of juggler = = = 4 s. So he is
40 = a - a = 4a or a = 10 m/s2. g 10
2 2
Substitute this value in (1) we get, throwing the balls after each 1 s. Let at some
instant he is throwing ball number 4. Before 1 s
9
u = 65 - ´ 10 = 65 - 45 = 20 m / s of it he throws ball. So height of ball 3 :
2 1
\ The distance travelled by the body in 20 s is, h3 = 20 × 1 – 10(1)2 = 15 m
1 1 2
s = ut + at 2 = 20 ´ 20 + ´ 10 ´ (20) 2 Before 2s, he throws ball 2. So height of ball 2 :
2 2
= 400 + 2000 = 2400 m. 1
h2 = 20 × 2 – 10(2)2 = 20 m
2
4. (c) Let vA and vB are the velocities of two Before 3 s, he throws ball 1. So height of ball 1 :
bodies. 1
In first case, v A + vB = 6m/s .....(1) h1 = 20 × 3 – 10(3)2 = 15 m
2
v
In second case, v A – B = 4m/s .....(2) 1 2
10. (d) Distance from A to B = S = ft1
From (1) & (2) we get, vA = 5 m/s and vB =1 m/s. 2
Distance from B to C = ( ft1 ) t
5. (b) Distance in last two second
1 u2 ( ft1 )2
= × 10 × 2 = 10 m. Distance from C to D = =
2 2a 2( f / 2)
1
Total distance = × 10 × (6 + 2) = 40 m. = ft12 = 2 S
2

@ebooks_for_freee on Telegram
Motion in a Straight Line 105
A f B C f /2 D 14. (a) In first case
t1 t 2t 1 u
u1 = u ; v1 = , s1 = 3 cm, a1 = ?
2
15 S
Þ S + f t1t + 2 S = 15 S Using, v12 - u12 = 2a1s1 ...(i)
2
Þ f t1t = 12 S ............. (i) æ uö 2
çè ÷ø - u = 2 × a × 3
1 2 2
f t1 = S ............ (ii)
2 –u2
t Þ a=
Dividing (i) by (ii), we get t1 = 8
6 In second case: Assuming the same retardation
2
1 ætö f t2
Þ S= fç ÷ = -u 2
2 è 6ø 72 u2 = u /2 ; v2 = 0 ; s2 = ?; a2 =
11. (a) v = a x , 8
dx dx v22 - u 22 = 2a2 ´ s2 ...(ii)
Þ =a x Þ = a dt
dt x
u2 æ –u2 ö
Integrating both sides, \ 0- = 2ç ÷ ´ s2
x t x 4 è 8 ø
dx é2 x ù
ò x = a ò dt ; ê 1 ú = a[t ]t0 Þ s2 = 1 cm
0 0 ë û0 15. (b) For downward motion v = –gt
a2 2 The velocity of the rubber ball increases in
Þ 2 x = at Þ x = t downward direction and we get a straight line
4
12. (c) Person’s speed walking only is between v and t with a negative slope.
1 "escalator" 1 2
Also applying y - y0 = ut + at
60 second 2
Standing the escalator without walking the speed 1 2 1 2
We get y - h = - gt Þ y = h - gt
1 "escalator" 2 2
is The graph between y and t is a parabola with y =
40 second
Walking with the escalator going, the speed add. h at t = 0. As time increases y decreases.
1 1 15 For upward motion.
So, th e per son’s speed is + = The ball suffer elastic collision with the
60 40 120
horizontal elastic plate therefore the direction of
"escalator"
velocity is reversed and the magnitude remains
second the same.
120
So, the time to go up the escalator t = = 24
5 Here v = u – gt where u is the velocity just after
second. collision.
13. (c) Speed on reaching ground u As t increases, v decreases. We get a straight line
v= u 2 + 2 gh between v and t with negative slope.
H 1 2
Now, v = u + at Also y = ut - gt
2
Þ u 2 + 2 gh = -u + gt All these characteristics are represented by
u graph (b).
Time taken to reach highest point is t = , 16. (b) x = at + bt2 – ct3
g
u + u 2 + 2 gH nu dx d
= (at + bt 2 + ct 3 )
Þt = = (from question) Velocity, v =
g g dt dt
= a + 2bt – 3ct2
Þ 2gH = n(n –2)u2

@ebooks_for_freee on Telegram
106 PHYSICS

dv d 22. (80) In first case speed,


Acceleration, = (a + 2bt - 3ct 2 )
dt dt 5 50
u = 60 ´ m/s = m/s
æb ö 18 3
or 0 = 2b – 3c × 2t \ t = ç ÷ d = 20m,
è 3c ø
Let retardation be a then
2 æ b2 ö
æb ö æb ö (0)2 – u2 = –2ad
and v = a + 2b ç ÷ - 3c ç ÷ = ç a + 3c ÷
è 3c ø è 3c ø è ø or u2 = 2ad …(i)
5
17. (c) 4 m/sec2 2 m/sec2 In second case speed, u¢ = 120 ´
Car Bus 18
100
= m/s
200 m 3
Given, uC = uB = 0, aC = 4 m/s2, aB = 2 m/s2 and (0)2 – u¢2 = –2ad¢
hence relative acceleration, aCB = 2 m/sec2 or u¢2 = 2ad¢ …(ii)
1 (ii) divided by (i) gives,
Now, we know, s = ut + at 2
2 d'
1 2 4= Þ d ' = 4 ´ 20 = 80m
200 = ´ 2t Q u = 0 d
2 u
Hence, the car will catch up with the bus after 23. (20)
time 8 A
t = 10 2 second
18. (d) Distance, PQ = vp × t (Distance = speed
× time)
Distance, QR = V.t Q vP P B t
o O 5
PQ 60
cos 60° = Distance travelled = Area of speed-time graph
QR v
1
1 vp ´ t v = ´ 5 ´ 8 = 20 m
= Þ vp = 2
2 V.t 2 R (Observer)
24. (3) Distance X varies with time t as
19. (a)
20. (b) The slope of v-t graph is constant and x2 = at2 + 2bt + c
velocity decreasing for first half. It is positive dx
and constant over next half. Þ 2x = 2at + 2b
dt
21. (293) Initial velocity of parachute
dx dx ( at + b)
after bailing out, Þx = at + b Þ =
dt dt x
u= 2gh 2
d 2 x æ dx ö
u = 2 ´ 9.8 ´ 50 = 14 5 50 m Þx +ç ÷ =a
dt 2 è dt ø
The velocity at ground, v
2 2
v = 3m/s a = - 2 m / s2 æ dx ö æ at + b ö
2 a -ç ÷ a -ç ÷
d x è dt ø = è x ø
v2 - u2 32 - 980 Þ =
S= = dt 2 x x
2´2 4 3m / s 2
» 243 m ax 2 - ( at + b ) ac - b 2
Initially he has fallen 50 m. = =
x3 x3
\ Total height from where
he bailed out = 243 + 50 = 293 m Þ a µ x–3 Hence, n = 3

@ebooks_for_freee on Telegram
Motion in a Straight Line 107

25. (08.00) Let the ball takes time t to reach the Solving (i) and (ii) fot t to be real
ground u2
1 2 b= a+ ,
Using, S = ut + gt 2d
2
52
1 2 Þb = 2.5 + = 5 ms-2
Þ S = 0 ´ t + gt 2´ 5
2 28. (50) The distance travel in n th second is
Þ 200 = gt2 [Q 2S = 100m] Sn = u + ½ (2n–1)a ....(1)
so distance travel in tth & (t+1)th second are
200 St = u +½ (2t–1)a ....(2)
Þt= …(i)
g St+1= u+½ (2t+1)a ....(3)
As per question,
1
In last s, body travels a distance of 19 m, so in St+St+1 = 100 = 2(u + at) ....(4)
2 Now from first equation of motion the velocity,
æ 1ö of particle after time t, if it moves with an
ç t – ÷ distance travelled = 81 accleration a is
è 2ø
2
v=u+at ....(5)
1 æ 1ö where u is initial velocity
Now, g ç t – ÷ = 81
2 è 2ø So from eq(4) and (5), we get v = 50cm./sec.
2 29. (49) Sn = Distance covered in n th sec
æ 1ö a
\ g ç t – ÷ = 81´ 2 Þ Sn = u + (2n - 1)
è 2ø 2
Putting a = – g and n = 5, we get
æ 1ö 81´ 2 g 9g
Þ çt – ÷ = Þ S5 = u - ´ 9 = u - ...(1)
è 2ø g 2 2
Distance covered in two continuous seconds
1 1
\ = ( 200 – 81 ´ 2) using (i) can only be equal when the body reaches the
2 g highest point after the fifth second and comes
Þ g = 2(10 2 – 9 2) down in the sixth second for which u = 0 and
n = 1.
Þ g =2 2 g g
Þ S6 = 0 + (2 ´1 - 1) = ...(2)
\ g = 8 m/s2 2 2
dv Equating (1) and (2)
26. (2) Given, = -2.5 v
dt 9g g 10g
or, u - = Þu= = 49 m / s
dv 2 2 2
Þ = – 2.5 dt 30. (10) The only force acting on the ball is the force
v
Integrating, of gravity. The ball will ascend until gravity
0 -½ t reduces its velocity to zero and then it will
ò6.25 v dv = -2.5ò dt
0 descend. Find the time it takes for the ball to
0 reach its maximum height and then double the
é v +½ ù
= -2.5 [ t ]0
t
Þ ê (½) ú time to cover the round trip.
ëê ûú 6.25 Using vat maximum height = v0 + at = v0 – gt, we
Þ – 2(6.25)½ = – 2.5t get:
Þ – 2 × 2.5 = –2.5t 0 m/s = 50 m/s – (9.8 m/s2) t
Þ t = 2s Therefore,
1 t = (50 m/s)/(9.8 m/s2) ~ (50 m/s)/ (10 m/s2) ~ 5s
27. (5) For rat s = bt 2 .... (i)
2 This is the time it takes the ball to reach its
1 2 maximum height. The total round trip time is
for cat s = d = ut + a t .... (ii)
2 2t ~ 10s.

@ebooks_for_freee on Telegram
108 PHYSICS

CHAPTER

Motion in a Plane
3
1. (d) | B | = 7 2 + (24)2 = 625 = 25 v 2 9t 4
Radial acceleration ac = =
Unit vector
Ù Ù in the direction of A will be R R
Ù 3i +4 j At t = 2s, at = 6 ´ 2 = 12 m/s2
A =
5
So, required vector 9 ´ 16
Ù Ù
ac = = 7.2 m/s2
æ3i + 4 jö 20
Ù Ù
= 25 ç ÷ = 15 i + 20 j \ Resultant acceleration
ç 5 ÷
è ø 2 2
H1 u sin q / 2g = at2 + ac2 = (12) 2 + (7.2) 2 = 144 + 51.84
2. (d) = 2 2 = tan 2 q
H 2 u sin (90º -q) / 2g = 195.84 = 14 m/s2
ur uur r 2 r 2 ur uur
u 2 sin 2 q 7. (d) | A + B |2 = | A | + | B | + 2A . B
3. (a) H1 =
2g
= A 2 + B2 + 2AB cos q
r r r uur ur uur
u 2 sin 2 (90° - q) u 2 cos 2 q | A - B |2 = | A |2 + | B |2 - 2 A . B
and H 2 = =
2g 2g
= A 2 + B2 - 2AB cos q
2 2 2 2 2 2 2
u sin q u cos q (u sin 2q ) R So, A2 + B2 + 2AB cosq = A2 + B2 – 2AB cos q
H 1H 2 = ´ = =
2g 2g 16 g 2
16 4AB cos q = 0 Þ cos q = 0 \ q = 90º
So, angle between A & B is 90º.
\ R = 4 H1 H 2 8. (c)
4. (b) Comparing the given equation with 9. (a) The angle for which the ranges are same is
complementary.
gx 2 Let one angle be q, then other is 90° – q
y = x tan q - , we get
2u 2 cos 2 q 2u sin q 2u cos q
T1 = , T2 =
g g
tan q = 3 or q = 60°.
4u 2 sin q cos q u 2 sin 2 q
5. (b) Circumference of circle is 2 pr = 40 m T1T2 = = 2 R (Q R = )
g g
Total distance travelled in two revolution is 80m. Hence it is proportional to R.
Initial velocity u = 0, final veloctiy v = 80 m/sec 10. (a) Horizontal component of velocity vx = 500
m/s and vertical component of velocity while
so from v2 = u2 + 2 as
striking the ground.
Þ (80)2 = 02 + 2 × 80 × a Þ a = 40 m/sec2 uv = 0 + 10 × 10 = 100 m/s
6. (d) s = t3 + 5 A u = 500 m/s
ds
Þ velocity, v = = 3t 2
dt
dv 500 m/s
Tangential acceleration at = = 6t B q
dt

@ebooks_for_freee on Telegram
Motion in a Plane 109
\ Angle with which it strikes the ground 14. (b) ĵ (North)
æu ö æ 100 ö -1 æ 1 ö
q = tan -1 ç v ÷ = tan -1 ç ÷ø = tan çè ÷ø
è xø
u è 500 5
B
r
11. (a) Given : u = 5 ˆj m/s
rBA
r iˆ (East)
Acceleration, a = 10iˆ + 4 ˆj and A
r
final coordinate (20, y0) in time t. vA = 30iˆ + 50 ˆj km/hr
r
1 vB = (-10iˆ) km/hr
S x = ux t + ax t 2 [Q ux = 0]
2
rBA = (80iˆ + 150 ˆj ) km
1
Þ 20 = 0 + ´ 10 ´ t 2 Þ t = 2 s r r r
vBA = vB - vA = -10iˆ - 30iˆ - 50iˆ = 40iˆ - 50 ˆj
2
1 ( rr )(· vr )
tminimum = BA BA
S y = u y ´ t + ayt 2
( vrBA )
2 2

1
y0 = 5 ´ 2 + ´ 4 ´ 22 = 18 m (80iˆ + 150 ˆj )( -40iˆ - 50 ˆj )
2
=
® (10 41) 2
12. (d) r = 15t 2iˆ + (4 - 20t 2 ) ˆj
10700 107
® \t= = = 2.6 hrs.
® d r 10 41 ´ 10 41 41
v = = 30tiˆ - 40tjˆ
dt p p
15. (c) From, q = wt = w =
® 2w 2
®
Acceleration, a = d v = 30iˆ - 40 ˆj So, both have completed quater circle
dt
\ a = 302 + 402 = 50 m/s 2
wR1 A
r r
13. (a) Let magnitude of two vectors A and B = a
r r X
| A + B | = a 2 + a 2 + 2a 2 cos q and
r r
| A – B | = a 2 + a 2 – 2a 2 éëcos (180° – q ) ùû wR2 B
= a + a – 2a cos q
2 2 2
Relative velocity,
and accroding to question,
r r r r
| A + B| = n | A – B |
( )
v A – v B =wR1 –iˆ - wR 2 ( –i ) =w ( R 2 – R1 ) i
16. (d) v = k(yi + xj)
a 2 + a 2 + 2a 2 cos q v = kyi + kxj
or, 2 2 = n2
a + a – 2a 2 cos q
dx dy
= ky, = kx
Þ
a 2 (1+ 1 + 2cos q )
n2 Þ
(1+ cos q ) = n 2 dt dt
a (1+ 1– 2cos q )
2
(1– cos q) dy
=
dy dt
´
dx dt dx
using componendo and dividendo theorem, we
dy kx
=
get q = cos –1 æç n –1 ö÷
2
dx ky
è n +1 ø
2
ydy = xdx ...(i)

@ebooks_for_freee on Telegram
110 PHYSICS

Integrating equation (i)


B2 B
Hence, from (i) = A2 + B2 – 2A2 Þ A= 3
ò ydy = ò x × dx 4 2
y2 = x2 + c
A 3
17. (d) Given, Position vector, Þ cos q = – =– \ q = 150°
B 2
r
r = cos wtiˆ + sin wt ˆj
r 2h 5
r dr 22. (10) S = u ´ Þ 10 = u 2 ´
Velocity, v = = w (– sin wtiˆ + cos wt ˆj ) g 10
dt
Acceleration, Þ u = 10 m/s
r p
r dv 23. (8) Given, w = 2 rad s–1, r = 2 m, t = s
a= = - w 2 (cos wtiˆ + sin wt ˆj )
dt 2
r r p
a = -w 2 r Angular displacement, q = wt = 2 ´ = p rad
r 2
\ a is antiparallel to rr Linear velocity, v = r × w = 2 × 2 = 4 m s–1
r r r r
Also v . r = 0 \v ^ r \ change in velocity, Dv = 2v = 2 × 4 = 8 m s–1
Thus, the particle is performing uniform circular r r
24. ( 7 2 ) Given u = 3iˆ + 4 ˆj , a = 0.4iˆ + 0.3 ˆj ,
motion.
t = 10 s
18. (c) From question,
Horizontal velocity (initial), r r r
v = u + at = 3iˆ + 4 ˆj + (0.4iˆ + 0.3 ˆj) ´ 10
40
ux = = 20m/s = 7iˆ + 7 ˆj
2
1 r
Vertical velocity (initial), 50 = uy t + gt2 \ | v |= 72 + 72 = 7 2 units
2
1 25. (–0.5) For two vectors to be perpendicular to
Þ uy × 2 + (–10) ×4 or,, 50 = 2uy – 20 each other
2
70 ® ®
or, uy = = 35m / s A × B =0
2 Ù Ù Ù Ù Ù Ù
u y 35 7 7 ( 2 i + 3 j+ 8 k ) · ( 4 j- 4 i + a k ) = 0
\ tan q = = = Þ Angle q = tan–1
u x 20 4 4 –8 + 12 + 8a = 0
19. (c) x + u2cos q2t = u1 cos q1 t 4 1
x
a=- =-
\ t= ...(i) 8 2
u1 cos q1 - u2 cos q2
r r 1r
Also u1 sinq1 = u2 sin q2 ...(ii) 26. ( 20 2 ) As S = ut + at 2
After solving above equations, we get 2
r
x sin q 2 ˆ + 1 (4iˆ+ 4 ˆj)4
S = (5iˆ + 4j)2
t= . 2
u1 sin(q 2 - q1)
20. (c) = 10iˆ + 8jˆ + 8iˆ+ 8 ˆj
B r r
21. (150) = A 2 + B2 + 2AB cos q ... (i) rf - ri = 18iˆ + 16jˆ
2 r r r
B sin q [as s = change in position = rf - ri ]
\ tan 90° = Þ A + B cos q = 0 r
A + B cos q rr = 20iˆ + 20ˆj
cos q = –
A r
\ | rr |= 20 2
B

@ebooks_for_freee on Telegram
Motion in a Plane 111
r
1 2 29. (195) Given : F = (iˆ + 2 ˆj + 3kˆ) N
27. (60) Using S = ut + at
2 r
And, r = [(4iˆ + 3 ˆj - kˆ) - (iˆ + 2 ˆj + kˆ)]
1
y = u y t + a y t 2 (along y Axis)
2 = 3iˆ + ˆj - 2kˆ
1 Torque,
Þ 32 = 0 ´ t + (4)t 2
2 r r
t = r ´ F = (3iˆ + ˆj - 2kˆ) ´ (iˆ + 2 ˆj + 3kˆ)
1
Þ ´ 4 ´ t 2 = 32
2 iˆ ˆj kˆ
Þ t=4s t = 3 1 -2 = 7iˆ - 11 ˆj + 5kˆ
1 1 2 3
S x = u xt + a xt 2
2 r
(Along x Axis) Magnitude of torque, | t | = 195.
1 r r r r r
Þ x = 3 ´ 4 + ´ 6 ´ 4 2 = 60 30. (90) Given, R = P Þ P + Q = P
2
28. (580) For pariticle ‘A’ For particle ‘B’
2P + Q
XA = –3t2 + 8t + 10 YB = 5 – 8t3 2P
r r
VA = (8 – 6t )iˆ VB = –24t 2 ˆj
r r a q
a A = –6iˆ aB = -48tjˆ
Q
At t = 1 sec
r r P2 + Q2 + 2PQ. cosq = P2
VA = (8 – 6t )iˆ = 2iˆ and vB = –24 ˆj Þ Q + 2P cosq = 0
r r r
\ V B / A = – v A + vB = –2iˆ – 24 ˆj Q
Þ cos q = – ..(i)
2P
\ Speed of B w.r.t. A, v = 22 + 242 2 P sin q
tan a = = ¥ (Q 2 P cos q + Q = 0)
= 4 + 576 = 580 Q + 2 P cos q
\ v = 580 (m/s) Þ a = 90°

@ebooks_for_freee on Telegram
112 PHYSICS

CHAPTER

Laws of Motion
4
1. (b) From Newton’s second law 5. (d) Equation of motion when the mass slides
dp down
= F = kt Mg sin q – f = Ma f
A
dt
Integrating both sides we get, Þ 10 – f = 6 kg
(M = 2 kg, a = 3 m/s2, 2
T
3p T é t2 ù q = 30° given) Ma
dp = ò kt dt Þ [ p ]
3p
òp 0 p
=kê ú
êë 2 úû 0 \ f = 4N C
q 30°
B
Equation of motion when the block is pushed up
kT 2 p Let the external force required to take the block
Þ 2p = ÞT = 2
2 k up the plane with same acceleration be F
2. (b) In the absence of air resistance, if the rocket Ma A
moves up with an acceleration a, Thrust (F) F – Mg sin q – f = Ma f
k g
then thrust Þ F – 10 – 4 = 6 2
F = mg + ma F
F = 20 N q 30°
6. (d) f = µ(M + m) g C B
\ F = m ( g + a)
a
= 3.5 × 104 ( 10 + 10) f µ( M + m) g
a= = = µg
= 7 × 105 N M +m ( M + m)
mg
3. (b) From figure, = 0.05 × 10 = 0.5 ms–2
Acceleration a = Ra …(i) Initial momentum 0.05V
a V0 = =
and mg – T = ma …(ii) R (M + m) 10.05
From equation (i) and (ii) m = 50g M = 10 kg
æ aö T
T × R = mR2a = mR2 çè ÷ø n
R T V0
or T = ma
m a
Þ mg – ma = ma mg
g
Þ a=
2
4. (d) Horizontal force, N = 10 N. f = mN
Coefficient of
v2 – u2 = 2as
friction m = 0.2.
10N 10N 10N 0 – u2 = 2as
The block will be u2 = 2as
2
stationary so long as W æ 0.05v ö
Force of friction = weight of block ç ÷ = 2 ´ 0.5 ´ 2
è 10.05 ø
\ mN = W Solving we get v = 201 2
Þ 0.2 × 10 = W Object falling from height H.
Þ W = 2N V
= 2 gH
10

@ebooks_for_freee on Telegram
Laws of Motion 113

201 2 12. (a) Let T be the tension in the string.


= 2 ´10 ´ H \ 10g – T = 10a ....(i)
10
H = 40 m = 0.04 km T – 5g = 5a ....(ii)
o
T
7. (d) At equilibrium, 45 T
Adding (i) and (ii), 5g
mg 100 g
tan 45° = = 5g = 15a Þ a = m / s2 10g
F F F 3
45o 2g
\ F = 100 N So, relative acceleration of separation =
1002N 3
8. (d) Given, q = 45°, r = 0.4 m, g = 10 m/s So, velocity of separation = v
2
mv
T sin q = ...... (i) æ 2g ö 2g
r T q = 0 + ç ÷ ´1 = = 20/3 m/s
T cos q = mg ...... (ii) è 3 ø 3
13. (d) Making free body-diagrams for m & M,
From equation (i) & (ii) we have, M
m
v2 K
F
tan q =
rg
N N
v = rg Q q = 45°
2
a
Hence, speed of the pendulum in its circular T T M
path, m F

v = rg = 0.4 ´ 10 = 2 m/s mg Mg
9. (c) Mass (m) = 0.3 kg we get T = ma and F – T = Ma
Force, F = m.a = –kx where T is force due to spring
Þ ma = –15x Þ F – ma = Ma or,, F = Ma + ma
Þ 0.3a = –15x F
15 -150 \ a= .
x= x = - 50 x M +m
Þ a= – Now, force acting on the block of mass m is
0.3 3
a = –50 × 0.2 = 10m/s2
æ F ö = mF
10. (a) When forces F1, F2 and F3 are acting on ma = m ç .
è M + m ÷ø m + M
the particle, it remains in equilibrium. Force F2
and F3 are perpendicular to each other, 14. (d) At equilibrium T = Mg
F1 = F2 + F3 T

\ F1 = F22 + F32 T=Mg


The force F1 is now removed, so, resultant of F2 T
mg T
and F3 will now make the particle move with
force equal to F1.
F
F Mg F1=(m+M)g
Then, acceleration, a = 1
m F.B.D. of pulley F1 = (m + M) g
11. (c) • For the man standing in the lift, the The resultant force on pulley is
acceleration of the ball
r r r
abm = ab - am Þ abm = g – a F= F12 + T 2 = [ (m + M )2 + M 2 ] g
Where 'a' is the acceleration of the mass (because 15. (a) 16. (d)
the acceleration of the lift is 'a' ) Length of the chain hanging from the table
• For the man standing on the ground, the µs =
Length of the chain lying on the table
acceleration of the ball
r r r l /3 l /3 1
abm = ab - am Þ abm = g – 0 = g = = =
l - l / 3 2l / 3 2

@ebooks_for_freee on Telegram
114 PHYSICS
17. (a) For the maximum possible value of a, a2
mg sin a will also be maximum and equal to the
frictional force. f1=mg
2m
In this case f is the limiting friction. The two 3mg
forces acting on the insect are mg and N. Let us f2=3mg
resolve mg into two components.
3mg - mg
mg cos a balances N. a1 = = 20 m / s 2
mg sin a is balanced by the frictional force. m
4mg - 3mg
\ N = mg cos a m=1/3 a
a2 = = 5 m / s2
2m
f = mg sin a a + a 2 25 X
f N
\ a pulley = 1 = =
But f = µN 2 2 2
mgcosa
= µ mg cos a
a Hence, X = 25 Þ X = 5
mg mgsina 22. (0.98) Limiting friction between block and slab
1
\ µ mg cos a = mg sin a Þ cot a = = µsmAg = 0.6 ×10 × 9.8 = 58.8 N
µ
Þ cot a = 3 But applied force on block A is 100 N. So the
18. (d) Let the velocity of the ball just when it block will slip over a slab.
leaves the hand is u then applying, Now kinetic friction works between block and
v2 – u2 = 2as for upward journey slab
Þ -u 2 = 2( -10) ´ 2 Þ u 2 = 40 Fk = µkmAg = 0.4 × 10× 9.8 = 39.2 N
Again applying v2 – u2 = 2as for the upward This kinetic friction helps to move the slab
journey of the ball, when the ball is in the hands \ Acceleration of slab
of the thrower, 39.2 39.2
v2 – u2 = 2as = = = 0.98 m / s 2
mB 40
Þ 40 - 0 = 2 (a) 0.2 Þ a = 100 m/s2 23. (0.5) Both blocks will move with same
\ F = ma = 0.2 ´ 100 = 20 N acceleration (a) given by
Þ N - mg = 20 Þ N = 20 + 2 = 22 N F 4 4
19. (d) The inclination of person from vertical is a= = = = 0.5 m / s 2
m1 + m 2 5+3 8
given by
24. (3.47) a = g(sin q - µcos q)
v2 (10)2 1
tan q = = = \q = tan -1 (1/ 5)
rg 50 ´ 10 5 = 9.8(sin 45° - 0.5cos 45°)
20. (c) Let T be the tension in the branch of a tree 4.9
when monkey is descending with acceleration = m / sec 2
a. Then mg – T = ma; and T = 75% of weight of 2
monkey 25. (71.8) Tension in the string when the body is at
75 the top of the circle (T)
mg – mg = ma
100 mv 2
= - mg = 71.8 N
1 g r
Þ mg = ma Þ a = 26. (346) Acceleration of block while moving up
4 4 an inclined plane,
21. (b) The F.B.D. of both blocks is as shown.
a1 a1 = g sin q + mg cos q
Þ a1 = g sin 30° + mg cos30°
m 3mg g mg 3
= + ...(i) (Q q = 30o)
f1=mg 2 2

@ebooks_for_freee on Telegram
Laws of Motion 115
N2
Using v 2 - u 2 = 2a( s )

2
F
f2
Þ v02 - 02 = 2a1 ( s) (Q u = 0)
Þ v02 - 2a1 ( s ) = 0
mg sin q
v2 mg cos q
mg
Þs= 0 ...(ii) q
a1
When the body slides up the inclined plane, then
Acceleration while moving down an inclined
plane mg sin q + f1 = F1
or, F1 = mg sin q + mmg cos q
a2 = g sin q - mg cos q
When the body slides down the inclined plane,
Þ a2 = g sin 30° - mg cos 30° then
g m 3 mg sin q – f 2 = F2
Þ a2 = - g ...(iii)
2 2 or F2 = mg sin q – mmg cos q
Using again v 2 - u 2 = 2as for downward motion F1 sin q + m cos q
2 \ =
æv ö v02 F2 sin q - m cos q
Þ ç 0 ÷ = 2a2 ( s) Þ s = ...(iv)
è 2ø 4a2 F1 tan q + m 2m + m 3m
Þ = = = =3
Equating equation (ii) and (iv) F2 tan q - m 2m - m m
v02 v2 29. (192) Acceleration produced in upward
= 0 Þ a1 = 4a2 direction
a1 4a2
F
g mg 3 æg m 3ö a=
Þ + = 4ç - ÷ M1 + M 2 + Mass of metal rod
2 2 ç2 2 ÷ø
è
480
Þ 5 + 5 3m = 4(5 - 5 3m) = = 12 ms -2
(Substituting, g = 10 m/s2) 20 + 12 + 8
Þ 5 + 5 3m = 20 - 20 3m Þ 25 3m = 15 Tension at the mid point

3 346 æ Mass of rod ö


Þm= = 0.346 = T = ç M2 + ÷a
5 1000 è 2 ø
I 346 = (12 + 4) × 12 = 192 N
So, =
1000 1000 30. (24.5) When lift is stationary, W1 = mg ...(i)
27. (30) The maximum velocity of the car is When the lift descends with acceleration, a
W2 = m(g – a)
vmax = mrg
Here m = 0.6, r = 150 m, g = 9.8 49
W2 = (10 – 5) = 24.5 N
10
vmax = 0.6 ´150 ´9.8 ; 30m / s
28. (3) N1 F1

mg sin q a
f1 mg cos q
q mg mg

@ebooks_for_freee on Telegram
116 PHYSICS

CHAPTER

Work, Energy and Power


5
1. (c) Given : retardation µ displacement where hn is the height to which the ball rises
dv after nth rebounding.
i.e., a = - kx But a = v
dx un h
k = proportional constant Hence, e n < < n
v2 x u0 h0
vdv
\
dx
= - kx Þ ò v dv = - ò kxdx
v–u
v1 0 4. (c) Acceleration (a) =
t
(v ) kx2
2
2
- v12 = - (0 , 50)
2
= < ,5 m / s 2
(10 , 0)
1 æ -x2 ö
Þ m ( v22 - v12 ) = m ç
1
÷k u = 50 m/s
2 2 è 2 ø \ v = u + at = 50 – 5t
\ Loss in kinetic energy, DK µ x 2 Velocity in first two seconds t = 2
2. (b) We know that F × v = Power v2 - u 2
\ F ´ v = c where c = constant v(at t <2) < 40 m / s ; s =
2a
dv æ mdv ö
v2 - u 2
\m ´v = c çè\ F = ma = ÷
dt dt ø W = F.s = ma
v t
2a
1 2 1
\ mò vdv = c ò dt \ mv = ct W < (402 , 502 ) ´10 < ,4500 J
2 2
0 0
2c 1 2 5. (b) Work done in fulling the hanging portion
\ v= ´t
m mgL2
(L/n) on the table W = ; mass of hanging
dx 2c 1 2 dx 2n2
\ = ´t where v =
dt m dt portion of chain
x t M
2c 1
\ ò dx = m
´ ò t 2 dt m=
L
Putting the values and solving we
0 0
3
get, W = 3.6 J
2c 2t 2 3 a B
x=
m
´
3
Þ xµt 2 6. (a) U = 12 – 6
x x
3. (a) In the question, the velocity of the earth
before and after the collision may be assumed dU a 6b æ 2a ö
F=– = +12 13 – 7 = 0 Þ x = ç ÷ 1/6
zero. Hence, coefficient of restitution will be, dx x x è b ø
U (x = ¥) = 0
u1 u 2 u3 u un
en < ´ ´ ´ .......´ n = a b b2
u 0 u1 u 2 u n ,1 u0 Uequilibrium = 2
– = –
æ 2a ö æç 2a ö÷ 4a
where un is the velocity after nth rebounding ç ÷ è b ø
and u0 is the velocity with which the ball strikes è bø
the earth first time. æ b2 ö b2
\ U (x = ¥) – Uequilibrium = 0 – çç = 4a ÷÷ = 4a .
un 2 ghn è ø
Hence, e n < <
u0 2 gh0

@ebooks_for_freee on Telegram
Work, Energy and Power 117
7. (a) By conservation of energy As the collision is elastic and the particles have
1 equal masses, the velocities will interchange as
mg (3h) = mg (2h) + mv 2 (v = velocity at B)
2 shown in the figure. According to the same
1 2 reasoning as above, the 2nd collision will take
mgh = mv Þ v = 2gh
2 place at C an d the velocities will again
From free body diagram of block at B interchange.
mv2 With the same reasoning the 3rd collision will
B h
occur at the point A. Thus there will be two elastic
collisions before the particles again reach at A.
10. (b) Spring constant, k = 5 × 103 N/m
N
mg Let x1 and x2 be the initial and final stretched
mv2 position of the spring, then
N + mg = = 2mg ;
N = mg
h 1
(
Work done, W = k x22 - x12
2
)
1
8. (c) Volume of water to raise = 22380 l = 22380 × = ´ 5 ´ 103 é(0.1)2 - (0.05) 2 ù
10–3m3 2 ë û
mgh V rgh V rgh 5000
P= = Þt= = ´ 0.15 ´ 0.05 = 18.75 Nm
t t P 2
22380 ´ 10-3 ´103 ´10 ´ 30 11. (c) We know area under F-x graph gives the
t= = 15 min work done by the body
10 ´ 746
9. (c) Let the radius of the circle be r. Then the 1
two distance travelled by the two particles \W= ´ (3 + 2) ´ (3 - 2) + 2 ´ 2
2
before first collision is 2pr. Therefore = 2.5 + 4 = 6.5 J
2v × t + v × t = 2pr Using work energy theorem,
A
v 2v D K.E = work done
\ D K.E = 6.5 J
12. (a) Let u be the initial velocity of the bullet of
mass m.
After passing through a plank of width x, its
velocity decreases to v.
where t is the time taken for first collision to occur. 4 4 u(n - 1)
\ u–v= or, v = u - =
2 pr n n n
\ t= If F be the retarding force applied by each plank,
3v then using work – energy theorem,
\ Distance travelled by particle
1 2 1 2 1 2 1 2 ( n - 1)
2
2 pr 2pr Fx = mu – mv = mu – mu
with velocity v is equal to v ´ = .
3v 3 2 2 2 2 n2
é 2ù
2 ê1 - ( n - 1) ú
Therefore the collision occurs at B. 1
A A = 2 mu ê
ë n2 ú
û
1 æ 2n - 1 ö
2v
120°
v Fx = mu 2 ç ÷
2 è n2 ø
120°
Let P be the number of planks required to stop
B B C
the bullet.
Total distance travelled by the bullet before
v 2v
coming to rest = Px

@ebooks_for_freee on Telegram
118 PHYSICS

Using work-energy theorem again, 16. (d) Considering conservation of momentum


1 along x-direction,
F ( Px ) = mu 2 - 0 mv = mv1 cos q ...(1)
2
where v1 is the velocity of second mass
é1 ( 2n - 1) ù 1 2
or, P ( Fx ) = P ê mu 2 ú = mu In y-direction,
ë2 n2 û 2 mv
0= - mv1 sin q
n2 3
\ P=
2n - 1 mv
13. (c) K.E. µ t or m1v1 sin q = ...(2)
3
K.E. = ct [Here, c = constant] v/ 3
1 2
Þ mv = ct v
2 m v
(mv )2 q
v1 cosq
Þ = ct
2m
v1
p2
Þ = ct (Q p = mv) v1 sinq
2m
Squaring and adding eqns. (1) and (2) we get
Þ p = 2ctm
v2 2
dp d ( 2 ctm ) v12 = v2 + Þ v1 =
v
Þ F= = 3 3
dt dt 17. (b) As we know, dU = F.dr
1 r
Þ F = 2 cm ´ ar 3
2 t U = ò a r 2 dr = ...(i)
3
0
1
Þ F µ mv 2
t As, = ar 2
14. (c) mv = (m + M) V’ r
m2v2 = mar3
mv mv v 1 3
or v = = =
m + M m + 4m 5 or, 2m(KE) = ar ...(ii)
2
Using conservation of ME, we have Total energy = Potential energy + kinetic
2 energy
1 2 1 æ vö
mv = ( m + 4m) ç ÷ + mgh Now, from eqn (i) and (ii)
2 2 è 5ø Total energy = K.E. + P.E.
2 v2 ar 3 ar 3 5 3
or h = = + = ar
5 g 3 2 6
15. (b) When the spring gets compressed by 18. (b)
length L. Before Collision After Collision
K.E. lost by mass M = P.E. stored in the V0 V1 V2
compressed spring. m m Þ m m
1 1 Stationary
Mv 2 = k L2
2 2 M 1 2 1 2 3æ1 ö
mv1 + mv2 = ç mv02 ÷
k 2 2 2è2 ø
Þ v= ×L
M 3
Momentum of the block, = M × v Þ v12 + v 22 = v20 ....(i)
2
k From momentum conservation
=M× ×L = kM × L mv0 = m(v1 + v2) ....(ii)
M

@ebooks_for_freee on Telegram
Work, Energy and Power 119
Squarring both sides, 4 5
(v1 + v2)2 = v02 DE = 3mv2 - mv 2 = mv 2 = 55.55%
3 3
Þ v12 + v22 + 2v1v2 = v02
Percentage loss in energy during the collision ; 56%
v20 22. (0.4) In an elastic head-on collision, of two equal
2v1v2 = -
2 masses their kinetic energies or velocities are
3 2 v2 exchanged. Hence when the first ball collides
(v1 - v 2 ) 2 = v 21 + v 22 - 2v1v 2 = v0 + 0 with the second ball at rest, the second ball
2 2
Solving we get relative velocity between the two attains the speed of 0.4 m/s and the first ball
particles comes to rest. This process continues. Thus the
v1 - v2 = 2v0 velocity of the last ball is 0.4 ms–1.
v
d 2s 2 23. (45) ®u ®q
19. (b) Acceleration = = 2t ms 1
dt 2 m m q2
2 2 2 v
w = ò ma.ds = ò0 3´ 2t ´ t dt
0 Applying law of conservation of momentum
Solving we get along horizontal and vertical directions, we get,
w = 24J mv sin q1 – mv sin q2 = 0
20. (c) Let m = mass of boy, M = mass of man i.e., q1 = q2 ....... (i)
v = velocity of boy, V = velocity of man Also, mu = mv (cos q1 + cos q2)
1 1 é1 ù = 2 mv cos q
MV 2 = ê mv 2 ú ...(i) [q1 = q2 = q (say)]
2 2 ë2 û
u
1 é1 ù cos π < ...... (ii)
M (V + 1) = 1 ê mv 2 ú
2
...(ii) 2v
2 ë2 û According to law of conservation of KE,
1
Dividing eq (i) by (ii) we get, V = 1 2 1 2 1 2
2 -1 mu < mv ∗ mv
2 2 2
21. (56) Y
pf = 3 m V or u 2 < 2v 2 or u < ∋ 2 ( v ..... (iii)
m
2v pi 45°
X From eqn. (ii) and eqn. (iii), cos π <
∋ 2(v
2v
1
v
2m
or cos π < or π =45°
2
Initial momentum of the system Hence, q1 = q2 = 45°
24. (15) While moving uphill power
pi = [m(2V) 2 ´ 2m(2V) 2 ] æ wö
P = ç w sin q + ÷10 [Q q is very small]
= 2m ´ 2V è 20 ø
æw wö 3w
Final momentum of the system = 3mV P = ç + ÷10 =
By the law of conservation of momentum è 10 20 ø 2
2 2v
2 2mv = 3mV Þ = Vcombined
3
q
Loss in energy os
wc 1
1 1 1 tanq = 10
DE = m1V12 + m2V22 - (m1 + m2 )Vcombined
2
2 2 2

@ebooks_for_freee on Telegram
120 PHYSICS

While moving downhill, Given, m = 1 kg,


P 3w æ w w ö Dh = h2 – h1 = 2 – 1 = 1m
= = - V
2 4 çè 10 20 ÷ø
\ KE = 1 × 10 × 1 = 10 J
3 v
= Þ v = 15 m / s 29. (18) Given, Mass of the body, m = 2 kg
4 20
Power delivered by engine, P = 1 J/s
\ Speed of car while moving downhill v = 15 m/s.
25. (2) U = 6x + 8y Time, t = 9 seconds
Power, P = Fv
¶U
Fx = - = -6 Þ a x = -3 m / s 2 Þ P = mav [Q F = ma]
¶x
t = 0, x = 6 ; t = t0, x = 0 (i.e. at y-axis) dv æ dv ö
1 1 Þm v=P çèQ a = ÷ø
0 - 6 = - a x t 2 [ s = s0 + ut + at 2 ] dt dt
2 2
12 P
2
= t Þ t = 2 sec. Þ v dv = dt
3 m
Note : Although the particle will have acceleration Integrating both sides we get
along y-direction also but time will be same. v t
P
Þ ò v dv =
m ò0
26. (3) If AC = l then according to question, BC = dt
2l and AB = 3l. 0
1/ 2
B v 2 Pt æ 2 Pt ö
Þ = Þv=ç
2 m è m ÷ø
Smooth
Rough 3lsinq dx 2 P 1/ 2 æ dx ö
m
C Þ = t çèQ v = ÷ø
dt m dt
q
x t
A 2 P 1/ 2
Þ ò dx =
m ò0
t dt
Here, work done by all the forces is zero. 0
Wfriction + Wmg = 0
2 P t 3/ 2 2 P 2 3/ 2
mg (3l )sin q - mmg cos q(l ) = 0 \ Distance, x = = ´ t
m 3/ 2 m 3
Þ mmg cos ql = 3mgl sin q 2 ´ 1 2 3/ 2 2
Þx= ´ ´ 9 = ´ 27 = 18.
Þ m = 3tan q = k tan q 2 3 3
m
\k = 3 30. (120) v0
27. (150.00) From work energy theorem,
q 2m
1
W = F × s = DKE = mv 2 q v0/2
2
Here V 2 = 2 gh v0
m
2 1 15
\F ×s = F ´ = ´ ´ 2 ´ 10 ´ 20 Momentum conservation along x direction,
10 2 100
\ F = 150 N. v0 1
2mv0 cos q = 2m Þ cos q = or q = 60°
2 2
28. (10.00) Kinetic energy = change in potential Hence angle between the initial velocities of the
energy of the particle, two bodies
KE = mgDh = q + q = 60° + 60° = 120°.

@ebooks_for_freee on Telegram
System of Particles and Rotational Motion 121

CHAPTER
System of Particles and
6 Rotational Motion
m1x1 + m 2 x 2 + m 3 x3 m1x1 + m 2 x 2 + m3 x3
1. (b) x cm = \ XCM =
m1 + m 2 + m3 m1 + m 2 + m3
Þ m1x1 + m2x2 + m3x3 = xcm (m1 + m2 + m3) L 5L
M´ + M´L + M´ 11L
Similarly, = 2 4 =
M+M+M 12
m1y1 + m2y2 + m3y3 = ycm (m1 + m2 + m3)
4. (a) Angular momentum,
m1z1 + m2z2 + m3z3 = zcm (m1 + m2 + m3) L0 = mvx sin 90° 0.6m
o
Given, m1 = 1kg, m2 = 2kg, m3 = 3kg = 2 × 0.6 × 12 × 1 × 1
xcm = ycm = zcm = 3m [As V = rw, sin 90° = 1,
0.8m 1m=x
\ x1 + 2x2 + 3x3 = 18 r = 0.6 m, w = 12 rad/s]
y1 + 2y2 + 3y3 = 18 So, L0 = 14.4 kgm2/s O
z1 + 2z2 + 3z3 = 18
Now, if m4 = 4 kg is introduced in the system, 5. (b) As the rod is bent at the middle.
x + 2x 2 + 3x 3 + 4x 4 Moment of intertia of B
xcm = 1 =1 each part about one
1+ 2 + 3 + 4 L
end O,
18 + 4x 4 2
Þ = 1 Þ x4 = – 2 1æMöæLö
2
10 = ç ÷ç ÷
Similarly, y4 = – 2; z4 = – 2 3è 2 ø è 2 ø 60°
Thus total moment of O L
A
r
inertia through the 2
2. (c) o o
middle point O
From conservation of angular momentum about 1æMöæLö
2
1 æMöæLö
2
ML2
any fix point on the surface, = ç ÷ç ÷ + ç ÷ç ÷ =
3è 2 ø è 2 ø 3è 2 øè 2ø 12
mr2w0 = 2mr2w
6. (a) Given, q = 30°, v = 5 m/s
w0 r
Þ w = w0/2 Þ v =
2
[Q v = rw ] Let h be the height on the plane upto which
the cylinder will go up.
3. (d)
L \ From conservation of energy,
L 4 1 2 1 2
2 Total K.E = P.E Þ mv + I w = mgh
2 2
Þ 1 mv2 + 1 æç 1 mr 2 ö÷ w2 = mgh éê Icyl = 1 mr 2 ùú
L 2 2è2 ø ë 2 û
x1
x2 3 2
Þ mv = mgh [using v = rw]
x3 4
L 5L 3v 2 3 ´ 52
x1 = , x 2 = L, x 3 = Þ h= = = 1.913 m.
2 4 4 g 4 ´ 9.8

@ebooks_for_freee on Telegram
122 PHYSICS
Let, s = distance moved up by the cylinder on mR 2 a é a ù ma
the inclined plane. = fR êQ a = R ú = f ...(ii)
2 R ë û 2
h Put this value in equation (i),
\ sin q =
s s ma 3ma
h h ma = F – or F =
Þ s= 5m/s 2 2
sin q q 10. (a) Here kinetic friction force will balance the
1.913 force of gravity. So it will rotate at its initial
= = 3.826 m
sin 30° position and its angular velocity becomes zero
Time taken to return to the bottom = t (friction also becomes zero), so it will move
æ k2 ö downwards. /2
2s ç1 + 2 ÷ æ 1ö g
ç ÷ 2 ´ 3.826 ç1 + ÷ N
N =m
è r ø = è 2ø B f =µ
= = 1.53s.
g sin q 9.8sin 30° mg
7. (d) l 2 30º
A B
11. (b) I = 1.2 kg m2, Er = 1500 J,
P a = 25 rad/sec2, w1 = 0, t = ?
(0, l) 2l
F As Er = 1 Iw 2 ,
2
(0, 2 ) 2E r 2 ´ 1500
C w= = = 50 rad / sec
To have linear motion, the force F has to be I 1.2
applied at centre of mass. From w2 = w1 + at
i.e. the point ‘P’has to be at the centre of mass
m1y1 + m 2 y 2 m ´ 2 l + 2m ´ l 4l
50 = 0 + 25 t, \ t = 2 seconds
y CM = = = 12. (c) Let v be the velocity of the centre of mass
m1 + m 2 3m 3
of the sphere and w be the angular velocity of
8. (b) Moment of Inertia of complete disc about
the body about an axis passing through the
MR 2 centre of mass.
'O' I total =
2 J = Mv
Radius of removed disc = R/4
\ Mass of removed disc = M/16 [As M µ R2] 2
J(h – R) = MR2 × w
M.I of removed disc about its own axis (O') 5
1 Mæ Rö
2
MR 2 2
= From the above two equations, v(h – R) = r2w
çè ÷ø = 5
2 16 4 512
M.I of removed disc about O From the condition of pure rolling, v = Rw
2 2
I removed disc = I cm + mx 2 = MR + M æç 3R ö÷ h-R=
2R
Þh =
7R
512 16 è 4 ø 5 5
19 MR 2 13. (d) By conservation of angular momentum,
=
512
M.I of remaining disc I1w1 = I2w 2
2 2 2
237
Iremaining = MR - 19 MR 2 = MR 2 I1 = mR12 ; I2 = mR 22
2 512 512 5 5
9. (c) From figure, a 2
2 æR ö 1 I
ma = F – f ....(i) I2 = m ç 1 ÷ Þ I2 = 2 I1 Þ 1 = n 2
And, torque t = Ia Mass = m
F 5 è nø n I2
O
mR2 I1w1
a = fR \ w2 = = n 2 w1 = n 2 w [ Q w1 = w ]
2 f
I2

@ebooks_for_freee on Telegram
System of Particles and Rotational Motion 123
14. (d) Moment of inertia of system about YY' from (1) (2) and (3)
I = I 1 + I2 + I3 Y mRa
F– = maR
1 3 3 2
= MR 2 + MR 2 + MR 2
2 2 2 3
1 F = mRa
7 2 2
= MR 2F
2 2 3 a=
t 30 3mR
15. (a) a = = = 15 rad /s 2 19. (b) 40
I 2 Y'
1 2 1 a =Ra
\ q = w0 t + at = 0 + ´ (15) ´ (10) 2 = 750 rad O
2 2 f
16. (b) For translational motion, P
mg – T = ma .....(1) m R From newton’s second law
For rotational motion, 40 + f = m (Ra) .....(i)
T.R = I a Taking torque about 0 we get
1 T
Þ T.R = mR a
2 40 × R – f × R = Ia
2 m 40 × R – f × R = mR2 a
Also, acceleration, a = Ra 40 – f = mR a ...(ii)
1 1 mg
\ T = mRa = ma Solving equation (i) and (ii)
2 2 40
Substituting the value of T is equation (1) we get a= =16rad / s 2
1 2 mR
mg - ma = ma Þ a = g 20. (b) Let s be the mass per unit area of the disc.
2 3
17. (c) For sphere, Then the mass of the complete 2R
1 2 1 disc = s(p(2R)2)
mv + Iw2 = mgh The mass of the removed O R
2 2
2
1 2 1 æ2 2öv disc = s (pR 2 ) = psR 2
or mv + ç mR ÷ 2 = mgh
2 2 è5 øR Let us consider the above situation to be a
2
7v complete disc of radius 2R on which a disc of
or h = radius R of negative mass is superimposed. Let
10 g
For cylinder O be the origin. Then the above figure can be
redrawn keeping in mind the concept of centre
1 2 1 æ mR 2 ö 3v 2
of mass as :
mv + ç ÷ = mgh ' or h ' =
2 2 çè 2 ÷ø 4g 2 R
2 4ps R
h 7v / 10 g 14 2
\ h' = 2 = O –ps R

18. (d)
3v / 4 g 15
xc.m =
( 6p( 2R) ) ´ 0 + ( -6( pR )) R
2 2

4psR2 - psR2
-psR 2 ´ R
\ xc.m =
3psR2
R 1
\ xc.m = - = aR Þ a =
3 3
F – fr = ma ...(i)
1 2
mR 2 21. (16) q = w0 t + at Þ q = 100 rad
frR = Ia = a ...(ii) 2
2
for pure rolling 100
\ Number of revolutions = = 16 (approx.)
a = aR ...(iii) 2p

@ebooks_for_freee on Telegram
124 PHYSICS

MR 2 Using principal of conservation of angular


22. (0.03) Required M.I. = + MR 2 momentum we have
2 r r
3 3 Li = L f Þ mvL = I w
= MR 2 = ´ 2 ´ (0.1) 2 = 0.03 kg m 2
2 2
23. (–3) æ ML2 ö
Þ mvL = ç + mL2 ÷ w
24. (3) A 2v
v B è 3 ø
upper lower æ 0.9 ´ 12 ö
cylinder
wup
cylinder
wlower Þ 0.1 ´ 80 ´ 1 = ç + 0.1 ´ 12 ÷ w
è 3 ø
v=0 æ 3 1ö 4
v B D Þ8=ç + ÷wÞ8= w
3v v wup è 10 10 ø 10
wup = ; wlower = Þ =3 Þ w = 20 rad/sec.
2R 2R wlower
25. (2.08) 29. (9.00) Here M0 = 200 kg, m = 80 kg
26. (23.00) Let s be the mass density of circular Using conservation of angular momentum,
disc. Li = Lf
Original mass of the disc, m0 = pa2 s M0
a2 Y
Removed mass, m = s
4 m
Remaining, mass,
a
æ a2 ö 2
m ' = ç pa 2 - ÷ s
è 4ø X
a 1
æ 4p - 1ö 2 I1w1 = I 2 w 2
= a2 ç s
è 4 ÷ø æ M R2
I1 = ( I M + I m ) = ç 0 + mR 2 ÷
ö
New position of centre of mass è 2 ø
a2 a 1
m x - mx pa 2 ´ 0 -
´ 2
I 2 = M 0 R and w1 = 5 rpm
X CM = 0 0 = 4 2 2
m0 - m a2 æ M R2 ö 5
pa 2 - \w2 = çç 0 + mR 2 ÷÷ ´
4 2
-a 3 / 8 -a -a a è 2 ø M 0R
= = = =- 2
æ 1 ö 2 2(4p - 1) 8p - 2 23 5R 2 (80 + 100)
çè p - ÷ø a = 2 ´ = 9 rpm.
4 R 100
\ x = 23 30. (25) Moment of inertia of the system about axis
27. (2) Using v2 = u2 + 2gy [\u = 0 at (0,0)] XE. X F
v2 = 2gy [\v = wx] rF

Y
w a a
Þy
X 60°
(0,0)
E a G
w x (2 ´ 2p) ´ (0.05)
2 2 2 2
Þ y= = ; 2cm rG
2g 20
I = I E + I F + IG
28. (20) w
(M, L) Þ I = m(rE )2 + m(rF )2 + m(rG )2
2
æaö 5 25
m v
Þ I = m ´ 02 + m ç ÷ + ma 2 = ma 2 = ma 2
è2ø 4 20
Before collision After collision \ N = 25.

@ebooks_for_freee on Telegram
Gravitation 125

CHAPTER

Gravitation
7
- G mm ' 1
1. (c) F = KR–n = MRw2 Þ w2 = KR–(n+1) 8. (c) ´ 3 + m ' ve2 = 0
r 2
-( n +1)
-3Gm 1
or w = K' R 2 [where K' = K1/2, a constant] or + ve2 = 0
a
( / cos30°) 2
-(n +1) (n +1) 2
2p
aR 2 \T a R 2 6 3 Gm
T ve = .
a
Gm ( mdx )
2. (b) Q dF = 9. (c) W= m (V2 – V1)
x2
dx é GM1 GM 2 ù
x when, V1 = -ê + ú,
ë a 2a û

é GM 2 GM1 ù
x V2 = -ê + ú
ë a 2a û
L+ a Gm( M 2 - M1)
F= ò (
Gm A + Bx 2 dx ) \ W=
a 2
( 2 - 1) .
a
æ dö g æ dö
é æ1 1 ö ù 10. (b) g ' = g ç1 - ÷ Þ = g ç1 - ÷
F = Gm ê A ç - ÷ + BL ú è R ø n è Rø
ë è a a + L ø û
æ n -1 ö
3. (b) Gravitational force will be due to M1 only. Þd =ç ÷R
4. (b) è n ø
GMm æ GMm ö
11. (a) Potential at the given point = Potential at the
5. (d) DU = U f - U i = - - ç- ÷ point due to the shell + Potential due to the particle
nR + h è R ø
GM 4GM 5GM
n GMm æ n ö = - - = -
= . =ç ÷ mgR a a a
n +1 R è n +1 ø
6. (c) 12. (b)
13. (b) V is the orbital velocity. If Ve is the escape
mv 2 GmM GM velocity then Ve = 2 V . The kinetic energy at
7. (d) = g= 2
(R + x ) (R + x ) 2 also R the time of ejection

mv 2 æ GM ö R 2 1 1
\ = mç KE = mVe2 = m( 2 V )2 = mV 2
(R + x) è R 2 ÷ø (R + x) 2 2 2
14. (a) Volume of removed sphere
mv 2 R2 3
\ = mg 4 æRö 4 æ 1ö
(R + x ) (R + x ) 2 Vremo = p ç ÷ = pR3 ç ÷
3 è2ø 3 è 8ø
æ gR 2 ö
1/ 2 Volume of the sphere (remaining)
gR 2
Þ v = çç ÷
2
\v = 4 3 4 3æ1ö 4 æ7ö
R+x R + x ÷ Vremain = pR - pR ç ÷ = pR 3 ç ÷
è ø 3 3 è8ø 3 è8ø

@ebooks_for_freee on Telegram
126 PHYSICS
Therefore mass of sphere carved and remaining 18. (d) Due to complete solid sphere, potential at
1 7 point P
sphere are at respectively M and M.
8 8 -GM é 2 æ R ö

Therefore, gravitational force between these two Vsphere = ê3R - ç ÷ ú
sphere, 2R 3 êë è2ø úû
7M 1
GM m G 8 ´ 8 M -GM æ 11R 2 ö GM
7 GM 2 = ç ÷ = -11
F= = = 3 ç 4 ÷ 8R
2
64 ´ 9 R 2 2R è ø
r (3R )2
41 GM 2 Solid
; sphere
3600 R 2
15. (a) As two masses revolve about the common
centre of mass O.
\ Mutual gravitational attraction = centripetal P
force
Cavity
Gm 2 R
= mw 2 R
( 2 R )2
Gm m m Due to cavity part potential at point P
Þ = w2 O
4R 3 GM
Gm 3 8 3GM
Þ w= Vcavity = - =-
4 R3 2 R 8R
If the velocity of the two particles with respect 2
to the centre of gravity is v then So potential at the centre of cavity
v = wR = Vsphere - Vcavity
Gm Gm 11GM æ 3 GM ö -GM
v= 3
´R = =- -ç- ÷=
4R 4R 8R è 8 R ø R
16. (d) Value of g with altitude is, 19. (c) Let P be the point where gravitational field
é 2h ù is zero.
gh = g ê1 - ú ;
ë Rû Gm 4Gm
Value of g at depth d below earth’s surface, \ x 2 = (r - x )2
é dù 1 2 r
gd = g ê1 - ú Þ = Þ r – x = 2x Þ x=
x r-x 3
ë Rû m P 4m
Equating gh and gd, we get d = 2h x
17. (c) Initial gravitational potential energy, Ei = r
GMm Gravitational potential at P,
– Gm 4Gm 9Gm
2R V =- - =-
Final gravitational potential energy, r 2r r
GMm / 2 GMm / 2 3 3
GMm GMm
Ef = – – = – – 20. (c) Let area of ellipse abcd = x
æ ö
R æ 3 R ö 2R 6R Area of SabcS =
2ç ÷ 2ç ÷
è2ø è 2 ø x x
4GMm 2GMm + (i.e., ar of abca + SacS)
= – =- 2 4
6R 3R (Area of half ellipse + Area of triangle)
\ Difference between initial and final energy,
3x
GMm æ 2 1 ö GMm =
Ef – Ei = ç– + ÷ = – 4
R è 3 2ø 6R

@ebooks_for_freee on Telegram
Gravitation 127
c
GM m GM (2m) GMm
28. (0.33) Fmin = 2
- 2 =
r (2r ) 2r 2
d b
S GMm GM (2 m) 3 GMm
and Fmax = + =
r2 (2r ) 2 2 r2
a
3x x Fmin 1
Area of SadcS = x - = \ Fmax = 3 .
4 4
Area of SabcS 3x / 4 t1 29. (2) As we know, Gravitational force of attraction,
= =
Area of SadcS x / 4 t2 GMm
F=
t1 R2
= 3 or, t = 3t
t2 1 2
GM e m GM e M s
F1 = 2
and F2 =
21. (129) From Kepler's law of periods, r1 r22
3/2 3/ 2 2GM e m GM e M s
T2 = T1 æç
R2 ö
÷ = 365 æ R / 2 ö DF1 = Dr1 and DF2 = Dr2
è R1 ø ç R ÷
è ø r13 r23
1
= 365 × = 129 days. DF1 mDr1 r23 æ m ö æ r23 ö æ Dr1 ö
2 2
= 3 =ç ÷ç ÷ç ÷
22. (0.5) F µ xM ´ (1 – x )M = xM 2 (1 - x ) DF2 r1 Ms Dr2 è Ms ø è r13 ø è Dr2 ø
dF Using Dr1 = Dr2 = 2 Rearth; m = 8 × 1022 kg;
For maximum force, =0
dx Ms = 2 × 1030 kg
dF r1 = 0.4 × 106 km and r2 = 150 × 106 km
Þ = M 2 - 2 xM 2 = 0 Þ x = 1/ 2
dx 3
DF1 æ 8 ´ 1022 ö æ 150 ´ 106 ö
GM =ç ÷ç ÷ ´1 @ 2
23. (0.15) g = 2 DF2 è 2 ´ 1030 ø è 0.4 ´ 106 ø
R
(Given Me = 81 Mm, Re = 3.5 Rm) 30. (0.56) According to question, gh = g d = g1
gm
Substituting the above values, = 0.15
ge h = R/2
24. (22.4)
GMm d
25. (0.70) Potential energy at height R = -
2R
ve be the escape velocity, then (R-d)
1 GM GM
mve 2 = .m Þ v e = GM GM ( R - d )
2 2R R gh = and g d =
æ Rö
2 R3
or, v e = gR çè R + ÷ø
2
Now, v = gR ' Þ v = 2gR [Q R ' = 2R]
GM GM ( R - d ) 4 (R - d )
v 2
= 3
Þ =
So, v = 2ve or, v e = æ 3R ö R 9 R
2 çè ÷ø
1 2
Comparing it with given equation, f = .
2 Þ 4 R = 9 R - 9d Þ 5 R = 9 d
26. (0.40) d 5
\ =
27. (2) R 9

@ebooks_for_freee on Telegram
128 PHYSICS

CHAPTER

Mechanical Properties of Solids


8
rq 6mm ´ 30° 5. (c)
1. (b) rq = lf Þ f = = = 0.18° 6. (a) From the graph l = 10–4m, F = 20 N
l 1m
2. (a) P = 80 atm = 80 × 1.013 × 105 Pa A = 10–6m2, L = 1m
1 FL 20 ´ 1
Compressibility = = 45.8 × 10–11/ Pa; Density \Y = = -6
B Al 10 ´ 10-4
of water at the surface = r = 1.03 × 103 kg/m3
Let r' = density of water at a given depth; = 20 ´ 1010 = 2 ´ 1011 N/m 2
V' = Vol. of water of mass M at given depth 7. (a) Force, F = A × Y × strain
V = Vol. of same mass of water at the surface = 1 × 10–4 × 2 × 1011 × 0.1
= 2 × 106 N
M M
\ V= and V ' = 8. (b) Initial length (circumference) of the ring
r r' = 2pr
DV r 1.03 ´ 10 3 Final length (circumference) of the ring = 2pR
Vol. strain = = 1- =1–
V r' r' Change in length = 2pR – 2pr
P DV P change in length 2p(R–r) R - r
Also B = \ = strain = = =
DV / V V B original length 2pr r
= 80 × 1.013 ×105 × 45.8 × 10–11 = 3.712 × 10–3 F/A F/A
Now Young's modulus E = =
1.03 ´ 10 3 l / L (R - r ) / r
\ 1- = 3.712 × 10–3
r' æ R-r ö
\ F = AE ç ÷
1.03 ´ 10 3 è r ø
Þ r' = = 1.034 × 103 kg/m3 9. (b)
1 - 3.712 ´ 10 -3
stress 10. (c) If l is the original length of wire, then
3. (a) tan(90° - q) =
strain change in length of wire with tension T1,
4. (c) As shown in the figure, the wires will have
Dl 1 = (l 1 - l)
the same Young’s modulus (same material) and
the length of the wire of area of cross-section 3A Change in length of wire with tension T2,
will be l/3 (same volume as wire 1). Dl 2 = (l 2 - l)
T l T l
l Now, Y = 1 ´ = 2´
A Dl 1 A Dl 2
A Y T l -T l
Þ l= 2 1 1 2
Wire (1) T2 - T1
11. (c) From formula,
3A Y FL 4 FL
Increase in length DL = =
AY pD 2Y
l/3 2 2
Wire (2) DLS FS æ DC ö YC LS 7 æ 1 ö æ1ö
= ç ÷ = ´ç ÷ ç ÷q
DLC FC è DS ø YS LC 5 è pø èsø
F l F' l
´ = ´ Þ F ' = 9F 7q
A Dx 3 A 3Dx =
(5sp 2 )

@ebooks_for_freee on Telegram
Mechanical Properties of Solids 129
12. (c) Here, 22
m = 14.5 kg, l = r = 1m, A = p (r22 – r12) = [(0.60)2 – (0.30)2]
7
w = 2rps = 2 × 2p rad/s
A = 0.065 × 10–4 m2 stress F / A F
Tension in the wire at the lowest position on Compressional strain = = =
Y Y AY
the vertical circle = F = mg + mrw2
2
æ 22 ö 50,000 ´ 9.8
= 14.5 × 9.8 + 14.5 × 1 × 4 × ç ÷ × 4 =
è 7 ø 22
4 ´ ´ [(0.60) 2 - (0.30) 2 ] ´ 2 ´1011
= 142.1 + 2291.6 = 2433.7 N 7
Fl Fl 2433.7 ´ 1 = 7.21× 10–7
Y= Þ Dl = =
ADl AY 0.065 ´ 10 - 4 ´ 2 ´ 1011 æ 2mM ö
= 1.87 × 10–3 m = 1.87 mm 16. (b) Tension in the wire, T = ç ÷g
13. (a) For steel wire: Total force = F1 = (4 + 6) èm+Mø
kgwt. = 10 kg wt = 10 × 9.8N Force / Tension 2mM
Stress = = g
0.25 Area A(m + M)
l1 = 1.5m , r1 = cm = 0.125 × 10–2 m;
2 2(m ´ 2m)g 4mg
= = (M = 2 m given)
Y1 = 2 × 1011 Pa, Dl1 = ? A(m + 2m) 3A
For brass wire, F2 = 6 kg wt. = 6 × 9.8N, 6
r2 = 0.125 × 10–2m 17. (a) Here, r = = 3mm = 3 × 10–3m;
2
Y2 = 0.91 × 1011 Pa, l2 = 1m Max. stress = 6.9 × 107 Pa
Max. load on a rivet
Fl Fl Fl
Q Y= \ Dl = = = Max. stress × area of cross section
ADl AY p r 2Y
For steel, 22
= 6.9 × 107 × × (3 × 10–3)2
Fl 10 ´ 9.8 ´ 1.5 ´ 7 7
Dl1 = 121 = \
p r1 Y1 22 ´ (0.125 ´ 10 -2 ) 2 ´ 2 ´ 1011 Max. tension = 4 × max. load
= 1.49 × 10–4 m 22
For Brass, = 4 × 6.9 × 107 × × 9 × 10–6
7
F2l2 6 ´ 9. 8 ´ 1 ´ 7 = 7.8 × 103 N
Dl2 = 2
= -2 2 11
p r2 Y2 22 ´ (0.125 ´10 ) ´ 0.91´10 18. (a)
= 1.3 × 10–4 m
Normal force N N
14. (a) Given: 19. (d) Stress < < <
A = 0.1 × 0.1 = 10–2 m2 F = mg = 100 × 10N; Area A (2p a)b
Stress = B×strain
DL Shearing stress
Shearing strain = = N 2paΧa ≥ b
L Shear modulus <B
FL (2pa)b pa 2 b
\ DL F / A
= Þ DL =
L h Ah (2pa) 2 Χab 2
ÞN<B
100 ´10 ´ 0.1 pa 2 b
= Þ DL = 4 × 10–7 m Force needed to push the cork.
10 - 2 ´ 25 ´109
mg f < m N < m 4pbΧaB = (4pmBb)Da
15. (c) Load on each column F =
4 20. (d) Yc ´ ( DLc / Lc ) = Ys ´ (DLs / Ls )
50,000 ´ 9.8 æ 1´ 10 -3 ö æ DL ö
= N Þ 1´1011 ´ ç ÷ = 2 ´1011 ´ ç s ÷
4 ç 1 ÷ è 0.5 ø
è ø

@ebooks_for_freee on Telegram
130 PHYSICS

0.5 ´10-3 T ml w 2
\ DLs = = 0.25 mm s= =
2 A A
Therefore, total extension of the composite wire
= DLc + DLs = 1 mm + 0.25 mm = 1.25 mm ml w 2
£ 48 ´ 107 Þ w 2 £
48 ´ 107 A ( )
21. (0.04) rq = Lf Þ 10-2 ´ 0.8 = 2 ´f A ml
Þ f = 0.004

22. (2 × 109) K=
P
=
hrg
Þ w2 £
( 48 ´ 10 )(10 ) = 16
7 -6

D V / V DV / V 10 ´ 3
200 ´ 103 ´10 Þ wmax = 4 rad/s
= = 2 ´ 109
0.1/ 100 27. (100) Breaking force a area of cross section
23. (0.03) If side of the cube is L then of wire
dV dL Load hold by wire is independent of length of
V = L3 Þ = 3 the wire.
V L
\ % change in volume = 3 × (% change in length) 28. (1.41) If force F acts along the length L of the
= 3 × 1% = 3% wire of cross-section A, then energy stored in
unit volume of wire is given by
DV
\ Bulk strain = 0.03 1
V Energy density = stress × strain
24. (2.026 × 109) Here, 2
DV = 100.5 – 100 = 0.5 litre = 0.5 × 10–3 m3; 1 F F
P = 100 atm = 100 × 1.013 × 105 Pa = ´ ´
2 A AY
V = 100 litre = 100 × 10–3m3
P PV æ F X ö
Bulk modulus = B = = çèQ stress = and strain = ÷
DV / V DV A AY ø
100 ´ 1.013 ´ 10 5 ´ 100 ´ 10 -3
= 1 F2 1 F 2 ´ 16 1 F 2 ´ 16
0.5 ´ 10 -3 = = =
2 A2Y 2 (pd 2 )2 Y 2 pd 4Y
Þ B = 2.026 × 109 Pa
Force If u1 and u2 are the densities of two wires, then
25. (200) Breaking stress =
area 4
u1 æ d 2 ö d1 d
= ( 4) Þ 1 = 2 :1
14
The breaking force will be its own weight. =ç ÷ Þ
F = mg = Vrg = area × lrg u2 è d1 ø d2 d2
area ´ l ´ rg 29. (1.75) D1 = D2
Breaking stress = 6 ´ 10 6 =
area
Fl1 Fl2 21.5
6 ´ 106 or = or = 2
or l= = 200 m. pr12 y1 pr22 y2 2
R ´7 2 ´4
3 ´ 103 ´ 10
26. (4) Given : Wire length, l = 0.3 m \ R = 1.75 mm

Mass of the body, m = 10 kg F 400 ´ 4


30. (1.15) Stress = = 2
= 379 ´106 N/m 2
A pd
Breaking stress, s = 4.8 × 107 Nm–2
Area of cross-section, a = 10–2 cm2 400 ´ 4
Þ d2 =
Maximum angular speed w = ? 379 ´ 106 p
d = 1.15 mm
T = Mlw2

@ebooks_for_freee on Telegram
Mechanical Properties of Fluids 131

CHAPTER
Mechanical Properties
9 of Fluids

1. (d) Weight of submerged part of the block Since height of water column is constant
1 therefore, water inflow rate (Qin)
W = v (Density of water) g ...(i) = water outflow rate
3 Qin = 10–4 m3s–1
Excess weight, = weight of water having
2 Qout = Au = 10–4 × 2gh
volume of the block.
3 \ 10 = 10 ×
–4 –4
20 ´ h
2
W ' = v (Density of water) g ...(ii) 1
3 \h = m = 5 cm
Dividing (ii) by (i), 20
W' 2/3 7. (c) Volume of 8 small droplets = Volume of 1
= big drop
W 1/ 3
\ W ' = 2W Þ W ' = 2 ´ 6 = 12 kg 4 3 4 R
Þ pr ´ 8 = pR 3 Þ r=
2. (b) The maximum force, which the bigger piston 3 3 (8)1 / 3
can bear, m = 3000 kg, F = 3000 × 9.8 N
Work done = (Change in area) × surface tension
Area of piston, A = 425 cm2 = 425 × 10–4 m2
\ Maximum pressure on the bigger piston, = (4pr 2 n - 4pR 2 )T
F 3000 ´9.8 ìï æ R 2 ö ü
P= = = 6.92 × 105 Pa 2ï
A 425´10,4 = í4pç 2 / 3 ÷ ´ 8 - 4pR ýT = 4pR 2 T
ïî çè (8) ÷
ø ïþ
\ The maximum pressure the smaller piston
can bear is 6.92 × 105 Pa. 8. (a) Since, soap film has two free surfaces, so
total length of the film = 2l = 2 × 30 = 60 cm = 0.6
3. (a)
m.
4.
1
2
( 2 2
)
(c) P2 - P1 = r v2 - v1 = 7500 Nm–2 Total force on the slider due to surface tension,
F = S × 2l = S × 0.6 N
rvd 4Q
5. (d) Reynold's no. NR = and V = 2 In equilibrium, F = mg
h d p
1.5´10,2
6. (a) \ S × 0.6 = 1.5 × 10–2 Þ S = = 2.5
0.6
× 10–2 Nm–1
9. (a) Weight of the liquid column = T cosq × 2pr.
Q in
For water q= 0°. Here weight of liquid column
W = 7.5 × 10–4N and T = 6 × 10–2N/m. Then
circumference, 2pr = W/T = 1.25 × 10–2m
10. (b) When the bubble gets detached,
h Bouyant force = force due to surface tension
4 3
2prTsin q = pR rw g
3
Qout

@ebooks_for_freee on Telegram
132 PHYSICS
15. (c) Pressure at interface A must be same from
both the sides to be in equilibrium.

R q
R d2 Rsina
r a

a Rcosa R
Rsina – Rsin a
T(2pr) sinq A
d1
r 4
Þ T´ ´ 2pr = pR 3rw g \ ( R cos a + R sin a )d2 g
R 3
= ( R cos a - R sin a )d1 g
2T 4p R 3
or, (p r 2 ) = rw g d1 cos a + sin a 1 + tan a
R 3T
Þ d = cos a - sin a = 1 - tan a
2
2rw g
Þ r = R2 16. (a) When the ball attains terminal velocity
3T Weight of the ball = Buoyant force + Viscous
11. (a) force
12. (a) As A1v1 = A2v2 (Principle of continuity) Fv B=Vr2 g

or, l 2 2 gh = pr 2 2 g ´ 4h

(Efflux velocity = 2gh )

l2 l2 l
\ r2 =or r = =
W=V r1g
2p 2 p 2p
13. (c) Given,
Density of gold, rG = 19.5 kg/m3 \ V r1 g = V r2 g + kvt2 Þ Vg ( r1 – r2 ) g = kvt2
Density of silver, r5 = 10.5kg/m3
Vg (r1 - r 2 )
Density of liquid, s = 1.5kg/m3 Þ vt =
k
2 r 2 (r - s ) g 17. (d) As liquid 1 floats over liquid 2. The lighter
Terminal velocity, vT =
9h liquid floats over heavier liquid. So, r1 < r2
vT (10.5 - 1.5) 9 Also r3 < r2 because the ball of density r3 does
\ 2
= Þ vT = 0.2 ´ not sink to the bottom of the jar.
0.2 (19.5 - 1.5) 2 18
Also r3 > r1 otherwise the ball would have
\ vT = 0.1 m/s floated in liquid 1. we conclude that
2
14. (d) The volume of liquid flowing through both r1 < r3 < r2.
the tubes i.e., rate of flow of liquid is same. 18. (d) Pressure at interface A must be same from
Therefore, V = V1= V2 both the sides to be in equilibrium.
pP1r14 pP2 r24
i.e., =
8hl1 8hl2
P1r14 P2r24 R d2 Rsinq
or = q
l1 l2
R
Q P2 = 4 P1 and l2 = l1/4 q Rcosq
Rsina – Rsin q
P1r14 4P1r24 r4 A
= Þ r24 = 1 d1
l1 l1 4 16
r2 = r1 2 \ (R cos q + R sin q)r2 g

@ebooks_for_freee on Telegram
Mechanical Properties of Fluids 133

= (R cos q - R sin q)r1g The terminal velocity vf of the ball is attained


when net force on the ball is zero.
d1 cos q + sin q 1 + tan q
Þ d = cos q - sin q = 1 - tan q \ Viscous force 6phr vf = Weff
2
Þ r 1 – r1 tan q = r2 + r2 tan q 2
When the ball acquires rd of its maximum
Þ (r1 + r2) tan q = r1 – r2 3
2
ær –r ö velocity vf the viscous force is = Weff
\ q = tan –1 ç 1 2 ÷ 3
è r1 + r2 ø
2 1
19. (c) Hence net force is Weff - Weff = Weff
20. (a) Weight of cylinder = upthrust due to both 3 3
liquids \ Required acceleration is a/3
æA 3 ö æ A Lö 4T
V ´ D ´ g = ç ´ L÷ ´ d ´ g + ç ´ ÷ ´ 2d ´ g
è5 4 ø è 5 4ø 23. (8) Inside pressure must be
r
æA ö A´ L ´ d ´ g D d greater than outside pressure in bubble.
Þ çè ´ L÷ø ´ D ´ g = Þ =
5 4 5 4
pa
5
\D= d
4 pa
21. (6) Initially, the pressure of air column above
water is P 1 = 10 5 Nm –2 and volume
This excess pressure is provided by charge on
V1 = (500 - H ) A , where A is the area of cross- bubble.
section of the vessel.
Finally, the volume of air column above water is 4T s 2
=
300 A. If P2 is the pressure of air then r 2e 0
200
P2 + rgh = 105 \ P2 + 103 ´ 10 ´ = 105 4T Q2 é Q ù
1000 Þ = ê s= ú
r 2 4
16p r ´ 2e 0 ë 4 pr 2 û
\ P2 = 9.8 ´ 104 N / m 2
As the temperature remains constant Q = 8pr 2rTe 0

105 ´ (500 - H ) A = (9.8 ´ 10 4 ) ´ 300 A 24. (20) Water fills the tube entirely in gravityless
condition i.e., 20 cm.
Þ H = 206 mm 25. (20) Given, Height of cylinder, h=20 cm
\ The fall of height of water level due to the Acceleration due to gravity, g=10 ms–2
opening of orifice = 206 – 200 = 6 mm Velocity of efflux
22. (3) v = 2gh
vf 2 2
v = W
Where h is the height of the free surface of liquid
3 f 3 eff from th e hole
Þ v= 2 ´10 ´ 20 = 20 m / s
W eff = vf
26. (2)
T 30°
Weff
Weff T cos 30°
When the When the ball attains
ball is released terminal velocity When the ball attains
2/3 of terminal velocity Fe
T sin 30°
When the ball is just released, the net force on
mg
ball is Weff ( = mg - buoyant force)

@ebooks_for_freee on Telegram
134 PHYSICS

Fe = T sin 30° Surface tension,


mg = T cos30° rhrg 15 ´10-5 ´15 ´10-2 ´ 900 ´10
T= =
2 2
Fe = 101 milli newton m–1
Þ tan 30° = ...(1)
mg 28. (0.1) Given: Radius of air bubble,
In liquid, r = 0.1 cm = 10–3 m
Fe ' = T 'sin 30° ...(A) Surface tension of liquid,
S = 0.06 N/m = 6 × 10–2 N/m
mg = FB + T 'cos 30° Density of liquid, r = 103 kg/m3
But FB = Buoyant force Excess pressure inside the bubble,
rexe = 1100 Nm–2
30° Depth of bubble below the liquid surface,
FB T¢ h=?
As we know,
T¢ cos 30°
2s
F¢ e rExcess = hrg +
T¢ sin 30° r
2 ´ 6 ´ 10-2
mg Þ 1100 = h × 103 × 9.8 +
10-3
= V (d - r) g = V (1.6 - 0.8) g = 0.8 Vg Þ 1100 = 9800 h + 120
Þ 9800h = 1100 – 120
m 0.8 mg mg
= 0.8 g= = 980
d 1.6 2 Þ h= = 0.1 m
9800
mg
\ mg = + T ' cos 30°
2 29. (16) (54–x)
mg 54 cm P
Þ = T 'cos 30° ...(B) 8 cm
2 x

2 Fe'
From (A) and (B), tan 30° =
mg
Hg
From (1) and (2)
Fe 2 Fe'
= (2) Length of the air column above mercury in the
mg mg
tube is,
Þ Fe = 2 Fe' P + x = P0
Þ P = (76 – x)
If K be the dielectric constant, then
Þ 8 × A × 76 = (76 – x) × A × (54 – x)
F \ x = 38
Fe' = e
K Thus, length of air column
= 54 – 38 = 16 cm.
2 Fe
\ Fe = Þ K=2 18000
K 30. (10–2 ) h = 10–2 poise; v = 18 km/h = =5
3600
27. (101)
m/s, l = 5 m
Given : Radius of capillary tube,
v
r = 0.015 cm = 15 × 10–5 mm Strain rate =
h = 15 cm = 15 × 10–2 mm l
\ Shearing stress = h × strain rate
2T cos q
Using, h = [cos q = cos0° = 1] -2 5
rgr = 10 ´ = 10–2 Nm–2
5

@ebooks_for_freee on Telegram
Thermal Properties of Matter 135

CHAPTER

Thermal Properties of Matter


10
1. (c) The lengths of each rod increases by the
same amount t1
\ Dla = Dls Þ l1aat = l2ast t2 t1 t2
l1
l1 as l l
Þ = l2
l1 + l 2 a a + a s
2. (a) We know that, With temperature rises height of liquid in each
C F - 32 9 arm increases i.e, l1 > l and l2 > l
= or F = C + 32
100 180 5 l1 l2
Equation of straight lines is, Also l = =
1 + g t1 1 + g t2
y = mx + c Y
­
Hence, m = (9/5), positive F l1 – l2
and c = 32 positive. Þ l1 + g l1t2 = l2 + g l2 t1 Þ g = .
l2 t1 – l1t2
The graph is
X 7. (d) According to principle of calorimetry,
shown in figure. O C®
Qgiven = Qused
3. (b) Young's modulus 0.2 × S × (150 – 40) = 150 × 1 × (40 – 27)
Thermal stress F A + 25 × (40 – 27)
= =
Strain DL L 13 ´ 25 ´ 7
S= = 434 J/kg-°C
F æ DL ö 0.2 ´ 110
Y= çèQ = a D q÷
A. a. Dq L ø 8. (c) Applying Wein's displacement law,
Force developed in the rail F = YAaDq lmT = constant
= 2 × 1011 × 40 × 10–4 × 1.2 × 10–5 × 10 5000 Å × (1227 + 273) = (2227 + 273) × lm
= 9.6 × 104 ; 1 × 105 N 5000 ´ 1500
4. (b) Due to volume expansion of both liquid and lm = = 3000Å
2500
vessel, the change in volume of liquid relative to
container is given by l 2l t¢ l/A
9. (d) tµ
and t ' µ Þ =4
DV = V0 [ g L - g g ]Dq A A/2 t l/A
Given V0 = 1000cc, a g = 0.1´ 10-4 / °C t¢ = 4 × t Þ t¢ = 48s
10. (c) Heat lost by He = Heat gained by N2
\ g g = 3a g = 3 ´ 0.1´10-4 / °C
n1Cv1 DT1 = n2Cv2 DT2
= 0.3 ´ 10 -4 / °C
3 é7 ù 5 é
\ DV = 1000[1.82 ´ 10-4 - 0.3 ´10 -4 ] ´ 100 R T - Tf ù
2 êë 3 0 úû = 2 R ëT f - T0 û
= 15.2cc
3
5. (b) 7T0 - 3T f = 5T f - 5T0 Þ T f = T0 .
6. (a) Suppose, height of liquid in each arm before 2
rising the temperature is l. 11. (c)

@ebooks_for_freee on Telegram
136 PHYSICS

12. (c) KA[(T - dT ) - T ] T - dT


=
dr dr
H1 H1
- KAdT
= ·
T1
dr r1 r

H2 dT
H H
= -4pKr 2 T2
The given arrangement of rods can be redrawn dr
r2
2
as follows (Q A = 4pr )
2K1K2
K= To measure the radial rate of heat flow, integration
K1+K2
technique is used, since the area of the surface
K1 K2 through which heat will flow is not constant.
l l r2 T2
æ dQ ö 1
Then, ç
è dt ÷ø ò r2 dr = -4 pK ò dT
r1 T1
K3 dQ é 1 1 ù
It is given that H1 = H2 ê - ú = -4pK [T2 - T 1 ]
dt ë r1 r2 û
KA(q1 - q2 ) K3 A(q1 - q2 ) dQ -4pKr1r2 (T2 - T1 )
Þ = or =
2l l dt (r2 - r1 )
K K K r r
Þ K3 = = 1 2 \
dQ
µ 1 2
2 K1 + K2 dt (r2 - r1 )
13. (b) By Newton's law of cooling, 15. (a) Let required temperature = T°C

q1 - q 2 é q1 + q 2 ù M.P. B.P.
= -k ê 2 - q0 ú ....(1) o o o
t ë û 0C TC 100 C
A sphere cools from 62°C to 50°C in 10 min.
x0 x0
62 - 50 é 62 + 50 ù 3
= -k êë 2 - q0 úû ....(2)
10 x0
2
Now, sphere cools from 50°C to 42°C in next
10 min. x0
6
50 - 42 é 50 + 42 ù
= -k ê - q0 ú ....(3) x 0 x 0 x0
10 ë 2 û Þ T° C = – =
2 3 6
Dividing eqn. (2) by (3) we get,
æ x ö
& ç x 0 – 0 ÷ = (100 – 0°C)
56 - q0 1.2 è 3 ø
=
46 - q 0 0.8 or 0.4q0 = 10.4 hence q0 = 26°C 2x 0 300
Þ =100 Þ x 0 =
14. (d) Consider a concentric spherical shell of 3 2
thickness (dr) and of radiius (r) and let the x 0 150
Þ T° C = = = 25° C
temperature of inner and outer surfaces of this 6 6
shell be T and (T – dT) respectively. 16. (a) Let Q be the temperature at a distance x
dQ from hot end of bar. Let Q is the temperature of
= rate of flow of heat through it hot end.
dt

@ebooks_for_freee on Telegram
Thermal Properties of Matter 137
The heat flow rate is given by 20. (a) According to newton's law of cooling
dQ kA(q1 - q) dq
= = - k(q - q0 )
dt x dt
x dQ dq
Þ q - q = x dQ Þ q = q1 - Þ = - kdt
1
kA dt kA dt (q - q0 )
Thus, the graph of Q versus x is a straight line with q t
a positive intercept and a negative slope. dq
The above equation can be graphically
Þ ò (q - q 0 )
= - k ò dt
q0 q
represented by option (a).
17. (b) As 1g of steam at 100°C melts 8g of ice at Þ log(q - q0 ) = -kt + c
0°C. Which represents an equation of straight line.
10 g of steam will melt 8× 10 g of ice at 0°C Thus the option (a) is correct.
Water in calorimeter = 500 + 80 + 10g = 590g
18. (a) Change in length in both rods are same i.e. 21. (9 × 10–6) g real = g app. + g vessel
Dl1 =D l 2 So ( g app. + g vessel )glass = ( g app + g vessel )steel
la1D q1 = la 2 Dq 2 -6
Þ 153 ´10 + ( g vessel )glass
a1 Dq 2 é a1 4 ù = 144 ´10-6 + ( g vessel )steel
=
a 2 Dq1 êQ a = 3 ú
ë 2 û Further,
4 q – 30 ( g vessel )steel = 3a = 3 ´ (12 ´10-6 )
=
3 180 – 30 = 36 ´ 10-6 / °C
-6
q = 230°C Þ 153 ´10 + ( g vessel )glass
19. (a) L = 144 ´ 10-6 + 36 ´ 10 -6
100°C Copper Steel 0°C Þ ( g vessel )glass = 3a = 27 ´10-6 / °C

Þ a = 9 ´ 10 -6 / °C
L/2 L/2
Let conductivity of steel Ksteel = k then from 1
22. (25) Time lost/gained per day = a Dq´ 86400
question 2
Conductivity of copper Kcopper = 9k second
qcopper = 100°C 1
qsteel = 0°C 12 = a (40 – q) ´ 86400 .... (i)
2
L 1
lsteel = lcopper = 4 = a (q – 20) ´ 86400 ....(ii)
2 2
From formula temperature of junction; 40 – q
K copper qcopper lsteel + K steel qsteel lcopper On dividing we get, 3 =
q= q – 20
K copper lsteel + Ksteel lcopper 3q – 60 = 40 – q
4q = 100 Þ q = 25°C
L L 23. (0.33) For slab in series, we have
9k ´ 100 ´ + k ´ 0´
= 2 2 x 4x 3x
L L Req = R1 + R2 = + =
9k ´ + k ´ KA 2KA KA
2 2 Now, in a steady state rate of heat transfer
900 through the slab is given by
kL
dQ T2 - T1 (T2 - T1 )
= 2 = 90°C = = KA
10kL dt R eq 3x …(i)
2

@ebooks_for_freee on Telegram
138 PHYSICS

dQ æ A(T2 - T1 )K ö 28. (64) From stefan's law, energy radiated by sun


Given =ç ÷ø f …(ii) per second
dt è x
Comparing (i) and (ii), we get f = 1/3 E = s AT 4 ;
24. (10) Rate of cooling µ temperature difference
\ A µ R2
between system and surrounding.
As the temperature difference is halved, so the \ E µ R 2T 4
rate of cooling will also be halved.
So time taken will be doubled E2 R22 T24
\ =
25. (20.00) E1 R12 T14
Volume capacity of beaker, V0 = 500 cc put R2 = 2R, R1 = R ; T2 = 2T, T1 = T
Vb = V0 + V0 g beaker DT
E (2R )2 (2T ) 4
When beaker is partially filled with Vm volume of Þ 2 = = 64
mercury,
E1 R 2T 4
29. (1) From stefan's law, the energy radiated per
Vb1 = Vm + Vm g m DT second is given by E = esT4 A
Here, T = temperature of the body
Unfilled volume (V0 - Vm ) = (Vb - Vm1 )
A = surface area of the body
Þ V0 g beaker = Vm g M For same material e is same. s is stefan's constant
Let T1 and T2 be the temperature of two spheres.
V0 g beaker A1 and A2 be the area of two spheres.
\Vm =
gM
E1 T14 A1 T14 4pr12
-6
\ = =
500 ´ 6 ´ 10 E2 T24 A2 T24 4pr22
or, Vm = = 20 cc.
1.5 ´ 10-4
26. (40) Using the principal of calorimetry (4000)4 ´12 1
= =
4 2 1
Mice Lf + mice (40 – 0) Cw (2000) ´ 4

= mstream Lv + mstream (100 – 40) Cw 30. (6.28) Dtemp = Dload


and A = pr2 = p(10–3)2 = p × 10–6
Þ M (540) + M × 1 × (100 – 40)
FL
= 200 × 80 + 200 × 1 × 40 L a DT =
AY
Þ 600 M = 24000
F ´ 0.2
Þ M = 40g or 0.2 × 10–5 × 20 =
( p ´ 10 -6 ) ´ 1011
27. (1) From Newton’s law of cooling
dQ f
- µ (Dq) \ F = 20p N \ m = = 2p = 6.28 kg
dt g

@ebooks_for_freee on Telegram
Thermodynamics 139

CHAPTER

Thermodynamics
11
1. (b) Work done is not a thermodynamical R (T - T2 )
n = 1, T1 = T Þ = 6R
function. 5 / 3 -1
2. (a) Þ T2 = (T – 4)K
7. (b) Suppose amount of water evaporated be M
gram.
Then (150 – M) gram water converted into ice.
So, heat consumed in evoporation = Heat
released in fusion
DU remains same for both paths ACB and ADB M × Lv = (150 – M) × Ls
DQACB = DWACB + DUACB M × 2.1 × 106 = (150 – M) × 3.36 × 105
Þ 60 J = 30 J + DUACB Þ M ; 20 g
Þ UACB = 30 J T
8. (d) Efficiency of engine A, h1 = 1 - ,
\ DUADB = DUACB = 30 J T1
DQADB = DUADB + DWADB T2
Efficiency of engine B, h2 = 1 -
= 10 J + 30 J = 40 J T
3. (c) Q = mL = 1 × L = L; W = P(V2 – V1) T T2
Now Q = DU + W Here, h1 = h2 \ T = T Þ T = T1T2
1
or L = DU + P(V2 – V1) Q 2 T2
\ DU = L – P(V2 – V1) 9. (a) =
Q1 T1
4. (a) dQ = dU + dW
or nCdT = nCvdT + PdV TQ 375 ´ 600
or Q 2 = 2 1 = = 450 J
P æ dV ö T1 500
\ C = Cv + ç ÷ 10. (a)
n è dT ø
Differentiating TV2 = constant, w.r.t. T, we get 1
11. (a) Initially the efficiency of the engine was
dV V 6
= – 1
dT 2T
which increases to when the sink temperature
P RT 3
Also, PV = nRT Þ = reduces by 62ºC.
n V
RT æ V ö 3R R 1 T
h= = 1 - 2 , when T = sink temperature
Now, C = Cv + ´ç– ÷ = 2 – 2 = R. 6 T1 2
V è 2T ø
5. (c) Curve A, B shows expansion. For expansion T1 = source temperature
of a gas, 5
Wisothermal > Wadiabatic Þ T2 = T1
6
Pisothermal > Padiabatic Secondly,
Tisothermal > Tadiabatic 1 T - 62 T 62 5 62
= 1- 2 =1 - 2 + =1 - +
5 3 T1 T1 T1 6 T1
6. (a) T1 = T, W = 6R joules, g =
3 or, T1 = 62 × 6 = 372K =372– 273 = 99ºC
1 1 - P2V2 = nRT1 - nRT2 = nR (T1 - T2 )
PV 5
´ 372 = 310 K = 310 – 273 = 37°C
W= & T2 =
g -1 g -1 g -1 6

@ebooks_for_freee on Telegram
140 PHYSICS
12. (b) Adiabatic modulus of elasticity of gas, 17. (b) In VT graph
K a = -gP ab-process : Isobaric, temperature increases.
bc process : Adiabatic, pressure decreases.
Isothermal, Ki = – P
cd process : Isobaric, volume decreases.
Ka Ka da process : Adiabatic, pressure increases.
\ Ki = = The above processes correctly represented in
g Cp / Cv
P-V diagram (b).
2.1´105 é Cp ù 18. (d) For isothermal process :
= = 1.5 ´105 N / m 2 . êQ g = ú
1.4 ë Cv û PV = Pi .2V
3 P = 2Pi ...(i)
13. (a) PV3/2 = K, log P + log V = log K
2 For adiabatic process PV = Pa (2V)g
g

DP 3 DV
+ =0 (Q for monatomic gas g= 5 3 )
P 2 V
5 5
DV 2 DP DV æ 2 ö æ 2 ö 4 or, [From (i)]
=- or = ç- ÷ ç ÷ = - 2Pi V 3 = Pa (2V) 3
V 3 P V è 3øè3ø 9 -2
14. (a) For a perfect gas, PV = µRT Pa 2 P
Þ = 5 Þ a =2 3
µRT 2 ´ 8.31 ´ (273 + 27) Pi Pi
P1 = = 23
V 20 ´ 10-3
5 2 19. (b) Let the initial pressure of the three samples
P1 = 2.5 × 10 N/m
V1 V2 be PA, PB and PC then
At constant pressure, T = T
1 2
PA (V )
3/ 2
= ( 2V )
3/ 2
P (Q PB = P )
æ V2 ö
\ T2 = ç V ÷ T1 = 2 ´ 300 = 600 K or PA = P(2)3/2
è ø 1
PC(V) = P(2V) or PC = 2P \ PA : P : PC
The gas now undergoes an adiabatic change.
T1 = 600 K, T2 = 300 K, V1 = 40 lit., V2 = ? = ( 2)
3/2
:1: 2 = 2 2 :1: 2
g – 1 = 5/3 – 1 = 2/3
20. (d) T2 = 7°C = (7 + 273) = 280 K
T1 V1g – 1 = T2 V2g – 1
T2 T
600 (40)2/3 = 300 (V2)2/3 h =1- Þ 2 =1- h
T1 T1
(2)3/2 × 40 = V2 or V2 = 112.4 lit.
15. (a) U = a + bPV 50 50 1
= 1- = =
In adiabatic change, 100 100 2
nR nR \ T1 = 2 × T2 = 2 × 280 = 560 K
dU = – dW = (T2 - T1 ) = (d T )
g -1 g -1 New efficiency, h'= 70%
1 b +1 T2 70 30 3
\ b= Þg = . \ = 1 - h' = 1 - = =
g -1 b T1 100 100 10
16. (d) Work done by the system in the cycle ' 10 2800
\ T1 = ´ 280 = = 933 .3 K
= Area under P-V curve and V-axis 3 3
1 \ Increase in the temperature of high temp.
= (2P0 - P0 )(2V0 - V0 ) +
2 reservoir = 933.3 – 560 = 373.3 K
é æ 1ö ù 21. (402) T = 27°C = 300 K
ê - çè 2 ÷ø (3P0 - 2P0 )(2V0 - V0 ) ú
ë û 5 8 V1 27
g= ; V2 = V1 ; =
P0 V0 P0 V0 3 27 V2 8
= - =0
2 2

@ebooks_for_freee on Telegram
Thermodynamics 141

From adiabatic process we know that Þ T1 = 1000 K


T1V1g -1 = T2V2g -1 60 T
Also, = 1 - 2 Þ T2 = 400 K
g -1 5
-1 100 1000
T2 æ V1 ö æ 27 ö 3
=ç ÷ =ç ÷ 26. (46) For adiabatic process, TV g -1 = constant
T1 è V2 ø è 8ø
T2 9 9 9 or, T1V1g -1 = T2V2g -1
= Þ T2 = ´ T1 = ´ 300 = 675 K
T1 4 4 4
V1 7
T2 = 675 – 273°C = 402°C T1 = 20°C + 273 = 293 K , V2 = and g =
10 5
1
22. (1.5) As P µ 1.5 , So PV1.5 = constant æV ö
g -1
V T1 (V1 ) g -1 = T2 ç 1 ÷
\ g = 1.5 (Q Process is adiabatic) è 10 ø
Cp 2/5
As we know, =g æ 1ö
Cv Þ 293 = T2 ç ÷ Þ T2 = 293(10) 2/ 5 ; 736 K
è 10 ø
Cp
\ = 1.5 DT = 736 - 293 = 443 K
Cv
During the process, change in internal energy
23. (1.5) P µ T 3 Þ PT -3 = constant ....(i) 5
DU = NCV DT = 5 ´ ´ 8.3 ´ 443
But for an adiabatic process, the pressure 2
temperature relationship is given by ; 46 ´103 J = X kJ
g
\ X = 46 .
P1-g T g = constant Þ PT
1-g
= constt. ....(ii)
From (i) and (ii) 27. (1818) For an adiabatic process,
TVg–1 = constant
g 3
= -3 Þ g = -3 + 3g Þ g =
1- g 2 \ T1V1g –1 = T2V2g –1
24. (1.67 × 105) DH = mL = 5 × 336 × 103 = Qsink 1.4 -1
æ ö
Qsink T çV ÷
< sink Þ T2 = (300) ´ ç 1 ÷
Qsource Tsource çç V1 ÷÷
Tsource è 16 ø
[ Qsource < ´ Qsink
Tsink Þ T2=300×(16) 0.4

Ideal gas equation, PV = nRT


Energy consumed by freezer
nRT
[ w output < Qsource , Qsink \ V=
P
Þ V = kT (since pressure is constant for
æT ö
= Qsink ç source - 1÷ isobaric process)
è Tsink ø So, during isobaric process
V2 = kT2 ...(i)
Given: Tsource < 27°C ∗ 273 < 300K,
2V2 = kTf ...(ii)
Tsink < 0°C ∗ 273 < 273 k
Dividing (i) by (ii)
3 æ 300 ö
Woutput = 5 ´ 336 ´ 10 ç - 1÷ = 1.67 ´ 105 J 1 T2
è 273 ø =
2 Tf
T2 50 500
25. (400) h = 1 - T or 100 = 1 - T Tf = 2T2 = 300 × 2 × (16)0.4 =1818 K
1 1

@ebooks_for_freee on Telegram
142 PHYSICS
28. (600.00) Tl + T3
Given; T1 = 900 K, T2 = 300K, W = 1200 J \T2 =
2
T2 W 600 + 400
Using, 1 – = =
T1 Q1 2
= 500K
300 1200
Þ 1– = Work done W
900 Q1 30. (980) Efficiency, h = =
Heat absorbed SQ
2 1200
Þ = Þ Q1 = 1800 Q1 + Q2 + Q3 + Q4
3 Q1 = = 0.5
Q1 + Q3
Therefore heat energy delivered by the engine
to the low temperature reservoir, Q2 = Q1 – W Here, Q1 = 1915 J, Q2 = – 40 J and Q3 = 125 J
= 1800 – 1200 = 600.00 J 1915 - 40 + 125 + Q4
\ = 0.5
T1 – T2 w A 1915 + 125
29. (500) hA = =
Tl Q1 Þ 1915 - 40 + 125 + Q4 = 1020
T2 – T3 WB Þ Q4 = 1020 - 2000
and, hB = =
T2 Q2 Þ Q4 = -Q = -980 J
According to question,
Þ Q = 980 J
WA = WB
Q1 T1 T2 - T3 T1
\ = ´ =
Q 2 T2 T1 - T2 T2

@ebooks_for_freee on Telegram
Kinetic Theory 143

CHAPTER

Kinetic Theory
12
N A PA VA TB 5R 1 7 R
1. (d) PV = NkT Þ = ´ n1C p1 + n2C p2 4´ + ´
N B PB VB TA and C p = = 2 2 2
N P ´ V ´ (2T ) 4 (n1 + n2 ) æ 1ö
Þ A = = çè 4 + ÷ø
V 2
NB 2P ´ ´ T 1 7
4 10 R + R
2. (b) For an ideal gas PV = constant i.e. PV = 4 = 47 R
doesn’t vary with V. 9 18
3. (b) 2
4. (a) According to given Vander Waal's equation C p 47 R 18
\Þ = ´ = 1.62
nRT an 2 Cv 18 29 R
P= - 2
V - nb V 7. (c) Mean free path of a gas molecule is given
Work done, by
V2 V2 V
2 1
dV dV l=
W= ò PdV = nRT ò
V - nb
- an2 ò 2
2pd 2 n
V1 V1 V V 1 Here, n = number of collisions per unit volume
V
é1ù 2 d = diameter of the molecule
= nRT [ loge (V - nb)]
V2
+ an2 ê ú
V1
ëV ûV1 If average speed of molecule is v then
l
æ V - nb ö 2 éV1 - V2 ù Mean free time, t =
= nRT loge ç 2 ÷ + an ê ú v
è V1 - nb ø ë V1V2 û 1 1 M
5. (d) Length of air column on both side is Þ t= 2
= 2 3RT
2pnd v 2pnd
100 –10
= 45 cm when one side is at 0°C and æ 3RT ö
2 çQ v = M ÷
the other is at 273°C è ø
2
The pressure must be same on both sides. Hence M \ t1 = M 1 d
´ 2
l1 l2 l l2 l \ tµ
= Þ 1 = Þ l1 = 2 d 2 t 2 d12 M2
T1 T2 273 (273 + 273) 2 2
Also l1 + l2 = 90 cm Þ l1 = 30 cm 40 æ 0.1 ö
= ´ç ÷ = 1.09
and l2 = 60 cm 140 è 0.07 ø
Applying gas equation to the side at 0°C, we get 8. (c) For a given pressure volume will be more if
P1l1 Pl P ´ 30 76 ´ 45 temperature is more (charles’s law) From graph
= Þ 1 = V2 > V1 \T2 > T1
T1 T 273 273 + 31
9. (d) K.E. of molecules is dependends on
Þ P1 = 102.4 cm of Hg. temperature. Since,
E k1 T T 1
n1Cv1 + n2Cv2 = 1 = = =1:1
6. (a) For mixture of gas, Cv = E k 2 T2 T 1
n1 + n2
(Since temperature of jar is same)
3 1 5 5 Hence K.E. of H2 and O2 will be found in ratio
4 ´ R + ´ R 6R + R
= 2 2 2 = 4 = 29 R ´ 2 = 29 R 1 : 1.
æ 1ö 9 9´ 4 18 10. (c) As temperature is constant during the
çè 4 + ÷ø 2 process, Þ Vrms remains constant so DVrms = 0
2

@ebooks_for_freee on Telegram
144 PHYSICS
11. (a) When temperature is same according to 3 819K B
kinetic theory of gases, kinetic energy of K.E. = K B ( 273) =
molecules will be same. 2 2
2
K.E. = P.E.
1 æ1ö 1 2 819K B
K.E. = ´ 32 ´ ç ÷ = ´ 2 ´ v Þ = Mgh or h =
819K B
2 2
è ø 2 2 2Mg
RMS velocity of hydrogen molecules
= 2 km/sec. 18. (c) From P-V graph,
12. (b) 1
P µ , T = constant
13. (d) Kinetic energy of each molecule, V
3 and Pressure is increasing from 2 to 1
K.E. = K B T
2 3RT
In the given problem, 19. (a) Q C=
M
Temperature, T = 0°C = 273 K
3 ´ 8.314 ´ 300
Height attained by the gas molecule, h = ? (1930 )2 =
3 819K B M
K.E. = K B ( 273) = 3 ´ 8.314 ´ 300
2 2 M= » 2 ´10 -3 kg
K.E. = P.E. 1930 ´1930
819K B The gas is H2.
Þ = Mgh or h = 819K B
2 2Mg n1
20. (a) Number of moles of first gas =
5 NA
14. (d) Cv for hydrogen = R ; n2
2 Number of moles of second gas =
3R NA
Cv for helium = n3
2 Number of moles of third gas =
6R NA
Cv for water vapour = = 3R If there is no loss of energy then
2
P1V1 + P2V2 + P3V3 = PV
5 3
4 ´ R + 2 ´ R + 1 ´ 3R 16 n1 n n
2 2 = R RT1 + 2 RT2 + 3 RT3
\ (Cv)mix = NA NA NA
4 + 2 +1 7
\ Cp = Cv + R n1 + n2 + n3
= RTmix
NA
16 23
Cp = R + R or C p = R n1T1 + n2T2 + n3T3
7 7 Tmix = n1 + n2 + n3
15. (a)
21. (2) Degree of freedom is the number of inde-
16. (c) vrms = ve
pendent variables required to define body’s
3RT 3kT motion completely. Here f = 2 (1 translational + 1
= 11.2 ´ 103 or = 11.2 ´ 103
M m rotational)
22. (1.2) Internal energy of n moles of an ideal gas at
3 ´ 1.38 ´ 10-23 T temperature T is given by
or -3
= 11.2 ´ 103 \ v = 104 K
2 ´ 10 f
U = nRT (f = degrees of freedom)
17. (d) Kinetic energy of each molecule, 2
3 U1 = U2 Þ f1n1T1 = f2n2T2
K.E. = K B T n1 f 2T2 3 ´ 2 6
2 \ n = f T = 5 ´1 = 5
In the given problem, 2 1 1
Temperature, T = 0°C = 273 K Here f2 = degrees of freedom of He = 3
Height attained by the gas molecule, h = ? and f1 = degrees of freedom of H2 = 5

@ebooks_for_freee on Telegram
Kinetic Theory 145

5 7 27. (150) In first case,


23. (1.5) g 1 = g2 = From ideal gas equation
3 5
n1 = 1, n2 = 1 PV = nRT
P DV + V DP = 0 (As temperature is constant)
n1 + n2 n n
= 1 + 2 DP
g -1 g1 - 1 g 2 - 1 DV = - V ...(i)
P
1 +1 1 1 3 5 In second case, using ideal gas equation again
Þ = + = + =4
g -1 5 7
-1 -1 2 2 P DV = -nR DT
3 5 nR DT
2 3 DV = - ...(ii)
\ =4 Þ g= P
g -1 2 Equating (i) and (ii), we get
24. (925) Using equipartition of energy, we have nR DT DP V
6 =- V Þ DT = DP
KT = mCT P P nR
2 Comparing the above equation with
3 ´1.38 ´10 –23 ´ 6.02 ´1023 | DT | = C | DP | , we have
C=
27 ´10 –3 V DT 300 K
C= = = = 150 K/atm
\ C = 925 J/kgK nR DP 2 atm
25. (–2.5 × 1025) Given: Temperature Ti = 17 + 273 28. (41) Room mean square speed is given by
= 290 K 3RT
Temperature Tf = 27 + 273 = 300 K vrms =
M
Atmospheric pressure, P0 = 1 × 105 Pa
Here, M = Molar mass of gas molecule
Volume of room, V0 = 30 m3
T = temperature of the gas molecule
Difference in number of molecules, nf – ni = ?
Using ideal gas equation, PV = nRT(N0), We have given vN2 = vH2
N0 = Avogadro's number
3RTN2 3RTH 2
PV \ =
Þ n= (N ) M N2 M H2
RT 0
TH2 573
P0V0 æ 1 1 ö Þ = Þ TH2 = 41 K
\ nf – ni = ç - ÷N 2 28
R è T f Ti ø 0
V1rms M2
1 ´ 105 ´ 30 æ 1 1 ö 29. (3.16) Using V = M
= ´ 6.023 ´ 10 23 ç - ÷ 2rms 1
8.314 è 300 290 ø
= – 2.5 × 1025 Vrms ( He ) M Ar 40
Vrms ( Ar )
= M He = 4 = 3.16
26. (5) Using ideal gas equation, PV = nRT
30. (266.67) Here work done on gas and heat
1 1 = nR ´ 250 [Q T1 = 250 K] ...(i)
Þ PV
supplied to the gas are zero.
5n Let T be the final equilibrium temperature of the
P2 (2V1 = R ´ 2000
4 gas in the vessel.
[Q T2 = 2000 K] ...(ii) Total internal energy of gases remain same.
Dividing eq. (i) by (ii), i.e., u1 + u2 = u '1 + u '2
P1 4 ´ 250 P 1 or, n1Cv DT1 + n2Cv DT2 = (n1 + n2 )CvT
= Þ 1 =
2 P2 5 ´ 2000 P2 5 Þ (0.1)Cv (200) + (0.05)Cv (400) = (0.15)CvT
P2
\ = 5. 800
P1 \T = = 266.67 K
3

@ebooks_for_freee on Telegram
146 PHYSICS

CHAPTER

Oscillations
13
1. (d) The motion of particle is S.H.M. with 2p
at t = 0; y = A sin f = A sin
x = A sin wt + B cos wt 3
= a sin (wt + q) = A sin 120° = 0.87 A [Q sin 120° ; 0.866]

Where a = A 2 + B 2 , A = a cosq, B = a sinq, Graph (a) depicts y = 0.87A at t = 0


q = tan–1 B/A. 7. (b) At the topmost position normal reaction
2. (b) The particles will meet at the mean position =0
when P completes one oscillation and Q \ amax = g
completes half an oscillation Þ w2A = g
vP aw P TQ 6 2 g
So v = aw = T = 3 = 1 Þw =
Q Q P A
3. (b) g
=
0.05
1 2 2 E1 1 2 E2
E1 = kx Þ x = , E2 = ky 2 Þ y = g 1
2 k 2 k now, vmax = wA = ´ 0.05 = m/s
0.05 2
1 2E 8. (d)
and E = k ( x + y )2 Þ x + y =
2 k dq
9. (b) = 2 \ q = 2t
dt
2 E1 2 E2 2E Let BP = a,
Þ + = Þ E1 + E2 = E
k k k \ x = OM = a sin q = a sin (2t)
4. (a) When the mass m1 is removed, only mass dx
Þ =a ´ 2cos (2t)
m2 remains. Therefore, its angular frequency is dt
d2x
k Þ 2 = – 4a sin 2t
w= dt
m2
5. (d) In damped oscillation, amplitude goes on Hence M executes SHM within the given time
decaying exponentially. period and its acceleration is opposite to x that
means towards left.
a = a0e–bt where b = damping coefficient.
10. (a) As the particle (P) is executing circular
a
Initially, 0 = a0e –b×100T , T = time of one motion with radius B. y
3
Consider angular velocity p(t=0)
1
oscillation or = e–100bT
3
of the particle executing r Q
circular motion is w and q q
Finally, a = a0e–b×200T or a = a0[e–100bT]2 90–q
x
when it is at Q makes an O R
é1ù
2
angle q as shown in the B
or a = a0 ´ ê ú [from (i)] or a = a0/9. diagram.
ë3û
6. (a) Displacement y(t) = A sin (wt + f) Clearly, q = wt
[Given] Now, we can write
2p OR = OQ cos(90 – q) (Q OR = X )
For f =
3

@ebooks_for_freee on Telegram
Oscillations 147

x = OQ sin q = OQ sin wt
g A
= r sin wt [Q OQ = r ] For N = 0, w = , T = 2p
A g
2p æ 2p ö
\ x = B sin t = B sin ç t ÷ For N to be positive, T must be greater than
T è 30 ø
A
æ 2p ö 2p
Þ x = B sin çè t ÷ø g
30
Hence, this equation represents SHM. l l
11. (a) The displacement of a particle in S.H.M. is 15. (a) t = 2p ; t 0 = 2p
g eff g
given by :
Buoyant
y = a sin (wt + f) force 1000 Vg
dy
velocity = = wa cos (wt + f)
dt 4
p ´1000 Vg
3
given f = Weight
4
The velocity is maximum when the particle æ4 ö 1000
æ dy ö Net force = ç - 1÷ ´1000 Vg = Vg
è3 ø 3
passes through the mean position i.e., çè dt ÷ø
max 1000 Vg g
= wa g eff = =
4 4
The kinetic energy at this instant is given by 3 ´ ´ 1000 V
3
2
1 æ dy ö 1 l
m çè ÷ø = mw2 a2 = 8 × 10–3 joule \ t = 2p Þ t = 2t0
2 dt max 2 g/4
1 16. (b) Equation of displacement is given by x = A
or × (0.1) w2 × (0.1)2 = 8 × 10–3 Þ sin(wt + f)
2
F0 F0
w= ±4 where A = =
Substituting the values of a, w and f in the 2
m (w 0 - w )
2 2 m (w 0 2 - w 2 )
equation of S.H.M., we get y = 0.1 sin (± 4t + p /
here damping effect is considered to be zero
4) metre.
12. (a) KE and PE completes two vibration in a time 1
\x µ
during which SHM completes one vibration. 2 2
m (w 0 - w )
Thus frequency of PE or KE is double than that bt
of SHM. –
17. (d) As we know, E = E0 e m
13. (a) Two perpendicular S.H.Ms are b15
x = a1 cos wt ....(1) –
Þ 15 = 45e m
and y = a2 cos2 wt ....(2) [As no. of oscillations = 15 so t = 15sec]
x b15
From eqn (1) a = cos wt 1 –
1 =e m
3
y 2
and from eqn (2) a = cos 2 wt =[2cos wt –1] Taking log on both sides
b
=
1
ln3
2 m 15
2
æ xö
y = 2a 2 ç ÷ – a 2 18. (a) (i) Restoring force = –k1x – k2x;
è a1 ø
14. (b) Minimum normal reaction is, N = mg – 1 k1 + k 2
keq = k 1 + k 2 \ f =
mAw2 2p m

@ebooks_for_freee on Telegram
148 PHYSICS
(ii) Here, extensions are different. Total Þ 4 w – 36 = 0 (Q derivative is zero)
extension = x Þ w=9
= x1 + x2 1
24. (0.33) Kinetic energy, k = mw 2 A 2 cos2 wt
1 1 1 2
Þ = +
keq k1 k2 1
k1k 2 Potential energy, U = mw 2 A 2 sin 2 wt
1 2
\ f =
2p m(k1 + k 2 ) k 2 2 p 1
= cot wt = cot (210) =
k1 + k 2 U 90 3
So, ratio = 25. (3.5) Since system dissipates its energy gradually,
k1 k2
dU and hence amplitude will also decreases with time
19. (b) U = U 0 + ax 2 Þ F = - = -2ax according to
dx
F 2a a = a0 e–bt/m ....... (i)
a= =- x Q Energy of vibration drop to half of its initial
m m
2 2a 2p m value (E0), as E µ a2 Þ a µ E
Þw = ÞT= = 2p
a -2
m w 2a a = 0 Þ bt = 10 t = t
l 2 m 0.1 10
20. (c) As we know, time period, T = 2p From eqn (i),
g a0
When additional mass M is added then = a 0e - t 10
2
t
l + Dl 1
TM = 2 p = e - t 10 or 2 = e10
g 2
t
2
l + Dl ln 2 = \ t = 3.5 seconds
l + Dl æ TM ö 10
TM = or ç ÷ =
l è T ø l 26. (7) Amplitude of vibration at time t = 0 is given
T
2 by
æ TM ö Mg é Mgl ù
or, ç ÷ = 1 + Ay êQ Dl = ú A = A0e –0.1× 0 = 1 × A0 = A0
è T ø ë Ay û
A0
also at t = t, if A =
1 éæ TM ù A
2
ö 2
\ =ê ÷ - 1ú Mg
y êçè T ø úû 1
ë Þ = e –0.1t
2
80
21. (0.0314) Slope of F - x curve = – k = - Þ t = 10 ln 2 ;7s
0.2 bt
k = 400 N/m, -
2m
27. (0.729) Q A = A 0e
m
Time period, T = 2p = 0.0314 sec. (where, A0 = maximum amplitude)
k
According to the questions, after 5 second,
22. (1.9) Washer contact with piston Þ N = 0
b(5)
Given Amplitude A = 7 cm = 0.07 m. -
… (i)
amax = g = w2A 0.9A 0 = A 0 e 2m

The frequency of piston After 10 more second,


ω g 1 1000 1 b(15)
f< < < < 1.9 Hz. -
…(ii)
2π A 2π 7 2p A = A0 e 2m

23. (9) At resonance, amplitude of oscillation is From eqns (i) and (ii)
maximum. A = 0.729 A0
Þ 2 w2 – 36 w + 9 is minimum Þ derivative \ a = 0.729
must be zero

@ebooks_for_freee on Telegram
Oscillations 149
28. (0.37) 29. (0.1) As we know, Time-period of simple
pendulum, T µ l
L/2 L/2
DT 1 Dl
differentiating both side, =
m m T 2 l
M Q change in length Dl = r1 – r2
1 r1 - r2
L– X–L 5 ´ 10-4 =
2 1
1 C
f1 = ...(i) r1 – r2 = 10 × 10–4
2p 1
10–3 m = 10–1 cm = 0.1 cm
1 3C
= 30. ( 4 2 ) Displacement, x = 4(cos pt + sin pt)
2 ML2
1 C æ sin pt cos pt ö
f2 = = 2 ´ 4ç + ÷
2p 2æ M Mö ...(ii) è 2 2 ø
L ç + ÷
è 3 2 ø
= 4 2(sin p t cos 45° + cos p t sin 45°)
As frequency reduces by 80%
f2 x = 4 2 sin(p t + 45°)
\ f2 = 0.8 f1 Þ f = 0.8 ...(iii)
1 On comparing it with standard equation
Solving equations (i), (ii) & (iii)
x = A sin(wt + f)
m
Ratio = 0.37 we get A = 4 2
M

@ebooks_for_freee on Telegram
150 PHYSICS

CHAPTER

Waves
14
1. (a) y(x, t) = 0.005 cos (ax - bt) (Given) Hence, 10V = 1000u + 10u = 1010u
Comparing it with the standard equation of wave Putting u = 4 m/s,
y(x, t) = a cos (kx - wt) we get 1
we have V = [4040] = 404 m/s ; 400 m/s
k = a and w = b 10
4. (c) Using n Last = nFirst + (N – 1) x
2p 2p 2p
But k = and w = Þ = a and Þ 3n = n + (26 – 1) × 4
l T l
Þ n = 50 Hz
2p
=b
T æ v + vo ö
5. (d) n' = nç ÷
Given that l = 0.08 m and T = 2.0s
è v – vS ø
2p 2p Here, n = 600 Hz, n o = 15 m/s
\ a= = 25p and b= =p
0.08 2 vs = 20 m/s, v = 340 m/s
2. (a)
3. (c) The frequency that the observer receives æ 355 ö
\ v ' = 600 ç ÷ » 666 Hz
directly from the source has frequency n1 = è 320 ø
500 Hz. As the observer and source both move 1 1
towards the fixed wall with velocity u, the 6. (d) T= = s;
f 500
apparent frequency of the reflected wave Since compression alternates with rarefaction so
coming from the wall to the observer will have again compression appears after
frequency T 1
t= = s
æ V ö 2 1000
n2 = ç ÷ 500 Hz 7. (a) Equation of the harmonic progressive wave
èV –u ø given by :
where V is the velocity of sound wave in air. y = a sin 2p (bt – cx).
The apparent frequency of this reflected wave Here u = b
as heard by the observer will then be 2p 1
k= = 2pc take, = c
æV +u ö æ V + u öæ V ö æV +u ö l l
n3 = ç ÷ n2 = ç ÷ç ÷ 500 = ç ÷ 500
è V ø è V øè V – u ø èV –u ø 1 b
It is given, that the number of beat per second \ Velocity of the wave = ul = b =
c c
is n3 – n1 = 10
dy
= a 2pb cos 2p (bt – cx) = aw cos (wt – kx)
æ V +u ö dt
\ (n3 – n1) = 10 = ç ÷ 500 – 500 Maximum particle velocity = aw= a2pb = 2p ab
èV – u ø
b 2 1
éV + u ù given this is 2 ´ i.e. 2pa = or c =
Þ 10 = 500 ê –1ú c c pa
ëV – u û 8. (d) In case of closed organ pipe frequency,
v
2 ´ u ´ 500 fn = (2n + 1)
Þ 10 = 4l
V –u

@ebooks_for_freee on Telegram
Waves 151
for n = 0, f0 = 100 Hz; n = 1, f1 = 300 Hz
\ vm = 2 s
n = 2, f2 = 500 Hz; n = 3, f3 = 700 Hz
n = 4, f4 = 900 Hz; n = 5, f5 = 1100 Hz é v - vm ù
n = 6, f6 = 1300 Hz According to Doppler’s effect v ' = v ê ú
ë v û
Hence possible natural oscillation whose
frequencies < 1250 Hz = 6(n = 0, 1, 2, 3, 4, 5) é 330 - 2 s ù
9. (b) Frequency of 2nd harmonic of string = 0.94v = v ê ú Þ s = 98.01 m
Fundamental frequency produced in the pipe ë 330 û
14. (a) Using, b = 10
æ I ö
0.5 m or 120 = 10 log10 çè -12 ÷ø ...(i)
10

P 2
Also I = 2
= ...(ii)
0.8 m 4pr 4pr 2
é 1 Tù v On solving above equations, we get
\ 2´ê ú= r = 40 cm.
ë 2l1 m û 4l2
15. (d) y = A sin (wt – kx) + A sin (wt + kx)
1 50 320 y = 2A sin wt cos kx
\ = This is an equation of standing wave. For position
0.5 m 4 ´ 0.8
of nodes
\ m = 0.02 kg m–1 cos kx = 0
The mass of the string = m l1 = 0.02 × 0.5 kg = 10g 2p p
10. (d) Þ .x = (2n + 1)
l 2
11. (b) n1 = n2, T ® Same, r ® Same, l ® Same ( 2 n + 1) l
Frequency of vibration Þ x= , n = 0,1, 2,3,...........
4
p T
n= 16. (a) Fundamental frequency, f = 70 Hz.
pr 2 r
2l
The fundamental frequency of wire vibrating
As T, r, and l are same for both the wires; n1 = n2 under tension T is given by
p1 p p1 1
= 2 Þ = Q r2 = 4 r1 1 T
r1 r2 p2 2 f =
2L m
12. (d) We know that the apperent frequency
æ v - v0 ö
Here, µ = mass per unit length of the wire
f'=ç ÷f from Doppler's effect L = length of wire
è v - vs ø 1 540
where v0 = vs = 30 m/s, velocity of observer 70 =
and source 2 L 6 ´ 10-3
Speed of sound v = 330 m/s Þ L » 2.14 m
17. (b) Given : Frequency of tuning fork, n = 264
330 ∗ 30
\ f '< ´540 = 648 Hz. Hz
330 , 30 Length of column L = ?
(Q Frequency of whistle (f) = 540 Hz.) For closed organ pipe
13. (a) v2m - u 2 = 2as v
n=
4l
\ v 2m = 2 ´ 2 ´ s v 330
u=0
2
a = 2m/s vm Þl= = = 0.3125
4n 4 ´ 264
Electric s Motor or, l = 0.3125 × 100 = 31.25 cm
siren cycle In case of closed organ pipe only odd
harmonics are possible.

@ebooks_for_freee on Telegram
152 PHYSICS
Therefore value of l will be (2n – 1) l We know at t = 2 sec,
Hence option (b) i.e. 3 × 31.25 = 93.75 cm is 1
correct. y= ...(ii)
18. (b) Standard equation 1 + ( x - 1) 2
æw ö On comparing (i) and (ii) we get vt = 1;
y(x, t) = A cos ç x - wt ÷ 1
èV ø
v=
From any of the displacement equation t
Say y1
1
w As t = 2 sec \v= =0.5 m/s.
= 0.50 p and w = 100 p 2
V
v v l c 3v 2 3
100 p 23. (0.375) 3 ´ l = 4 ´ l or l = ´ =
\ = 0.5p 4c 20 0 4 4v 8
V
100 p 24. (0.1) Velocity of wave on string = T / µ
\ V= = 200 m/s
0.5p = 8 m/s
19. (c) According to Doppler’s effect, The pulse gets inverted after reflection from the
æ V +V ö
0 fixed end, so for constructive interference to take
Apparent, frequency f = ç V – V ÷ f 0 place between successive pulses, the first pulse
è S ø
æ f ö V f has to undergo two reflections from the fixed
0 0
Now, f = ç V – V ÷ V0 + V – V end.
è S ø s
f0 2L 2 ´ 0.4
Dt = = = 0.1s
So, slope = v 8
V – VS
Hence, option (c) is the correct answer. 25. (7.7) When the source S is between the wall
20. (a) It is given that 315 Hz and 420 Hz are two (W) and the observer (O) For direct sound the
resonant frequencies, let these be n th and (n + source is moving away from the observer,
therefore the apparent frequency
1)th harmonies, then we have nv = 315
2l v 330
v n¢¢ = n= × 256 = 252.2
v + vs 330 + 5
and (n + 1) = 420
2l and frequency of reflected sound
n + 1 420
Þ = v 330
n 315 n¢ = n= × 256 = 259.9
Þn=3 v - vs 330 -5
v v Number of beats/sec = n¢ – n¢¢ = 259 – 252.2 = 7.7
Hence 3 ´ = 315 Þ = 105 Hz
2l 2 l 26. (8516) Reflected frequency of sound reaching
The lowest resonant frequency is when bat
n=1
Therefore lowest resonant frequency é V - (-V0 ) ù é V + V0 ù V + 10
= ê úf= ê úf= f
= 105 Hz. ë V - Vs û ëV - Vs û V - 10
21. (0.85) x1 and x2 are in successive loops of
stationary waves. æ 320 + 10 ö
f1 = p and f2 = K ( Dx )
= ç ÷ ´ 8000 ; 8516 Hz
So è 320 - 10 ø
æ 3p p ö 7p = f1 = 6 27. (4) For observer, tone of B will not change due
= kç - =
è 2k 3k ÷ø 6 f2 7
to zero relative motion.
Observed frequency of sound produced by A
22. (0.5) The equation of wave at any time is
obtained by putting X = x – vt (330 - 30)
= 660 = 600 Hz
1 1 330
y= 2 = 1 + ( x - vt ) 2 ...(i) \ No. of beats = 600 – 596 = 4
1+ X

@ebooks_for_freee on Telegram
Waves 153
28. (35.00) Given,
B
Denisty of wire, s = 9 ´ 10 -3 kg cm–3
Vgas 2rair
Young's modulus of wire, Y = 9 × 1010 Nm–2 = Vair 300
Vair B Þ Vgas = = = 150 2m/s
Strain = 4.9 × 10–4 2 2
rair
Stress T / A
Y= =
Strain Strain nv
And fnth harmonic = (in open organ pipe)
2L
T (L = 1 metre given)
\ = Y ´ Strain = 9 × 109 × 4.9 × 10–4
A 2v v v
\ f2nd harmonic – ffundamental = – =
Also, mass of wire, m = Al s 2 ´1 2 ´ 1 2
\ f2n harmonic – ffundamental
m
Mass per unit length, m = = As 150 2 150
J = = » 106 Hz
2 2
Fundamental frequency in the string
1 T
30. (2.25) Using f = ,
1 T 1 T 2l m
f = =
2l m 2l sA
mass
where, T = tension and m =
length
1 9 ´ 109 ´ 4.9 ´ 10 -4
=
2 ´1 9 ´ 103 1 Tx 1 Tz
fx = and f z =
2l m 2l m
1 1
= 49 ´ 109 - 4 -3 = ´ 70 = 35 Hz
2 2 f x 450 T
= = x
29. (106) Given : Vair = 300 m/s, rgas = 2 r air f z 300 Tz

B Tx 9
Using, V = \ = = 2.25.
r Tz 4

@ebooks_for_freee on Telegram
154 PHYSICS

CHAPTER
Electric Charges
15 and Fields

Now if f is the force due to the sixth charge and


Charge
1. (a) Surface charge density (s) = F due to the remaining five charge then,
Surface area r r r r
– Q + 2Q = Q
F + f = 0 i.e., F = -f
-2Q r r 1 q´q 1 æ qö
2
So sinner = –2Q F =f = = ç ÷
4pb2 a b 4pe 0 L2 4pe 0 è L ø
Q c 5. (d) In equilibrium, Fe = T sin q
and sOuter = +2Q
4pc 2 mg = T cos q
2. (d)
3. (a) Initial force between the two spheres
carrying charge (say q) is, q
2 l T
1 q
F= (r is the distance between them) Tcos q
4pe 0 r 2 q
Further when an uncharged sphere is kept in Tsin q
q Fe
touch with the sphere of charge q, the net charge q
x
q+0 q
on both become = . mg
2 2
Force on the 3rd charge, when placed in center of F
tan q = e
the 1st two, mg
r/2 r/2
q2
=
B C A 4p Î0 x 2 ´ mg
q x/2
q/2 q/2 also tan q » sin q =
2 l
æqö æqö
qç ÷ ç ÷ x q2
F3 =
1 è2ø - 1 è2ø Hence, 2l =
4p Î0 x 2 ´ mg
4pe 0 æ r ö 2 4pe 0 æ r ö 2
1/3
ç ÷ ç ÷ 2q 2 l æ q2l ö
è2ø è2ø Þ x3 = \ x = çç ÷÷
1 q2 4p Î0 mg è 2p Î0 mg ø
= [2 - 1] = F
4pe 0 r 2 Therefore, x µ l1/3
4. (d) If there had been a sixth charge +q at the 6. (c) Force on Q2 is zero (q should be negative)
remaining vertex of haxagon, force due to all the R
six charges on – q at O will be zero. R–x x
E D

Q1 q Q2
F C kQ1Q2 kqQ2 x q q
O = or = =
2 2 R Q1 Q1
R x
A B

@ebooks_for_freee on Telegram
Electric Charges and Fields 155
Force on q is zero, 9. (b) Electric field at a point inside a charged
kQ1q kqQ 2 R-x Q1 conducting spherical shell is zero.
= or = 10. (b) Due to induction net charge on outer
2 2
(R - x) x x Q2 surfaces of shpere
R Q1 + Q 2 Q1 Q1 + Q 2 Q1 Q1 + Q2 Q1 + Q 2 + Q3
or = or = s= = =
2 2
x Q2 q Q2 4pR 4 p(2R) 4p(3R) 2
Q1Q 2 Q1 + Q 2 Q1 + Q 2 + Q3
or q= Þ Q1 = =
Q1 + Q 2( ) 2
4 9
Þ Q2= 3Q1 and Q3= 5Q1
7. (a) Three point charges +q, –2q and +q are
Þ Q1: Q2: Q3 = 1 : 3 : 5
placed at points B (x = 0, y = a, z = 0), O (x = 0, y 11. (c)
= 0, z = 0)
and A(x = a, y = 0, z = 0) 12. (c) E1 = E2 or
The system consists of two dipole moment 1 8q 1 2q
=
vectors due to (+q and –q) and again due to (+q 4 p Î 0 ( L + x) 2 4 p Î 0 x 2
and –q) charges having equal magnitudes qa \ x=L
uuur uuur
units i one along OA and other along OB . Thus distance from origin is = L + L = 2L
Hence, net dipole moment, 8q –2q
L x
2
uuur 2
pnet = (qa) + (qa) = 2qa along OP at an 13. (a) Electric field due to complete disc (R = 2a)
at a distance x and on its axis
angle 45° with positive X-axis.
y P
(a, a, 0) s é x ù
+q(0, a, 0) E1 = ê1 – ú
B B 2e 0 ê R 2 + x 2 úû
ë
(–2q) A
x
45° s é h ù
(0, 0, 0) O +q(a, 0, 0) O A E1 = ê1 – ú
(0, 0, 0) 2e 0 ê 4a + h2
2 úû
ë
z
2a
8. (a) Fe = F2 + F3 a
o
kq 2 kq 2
= +
( L sin q ) 2 ( 2L sin q ) 2
s é hù é here x =h ù
q = 1–
2e0 ëê 2a ûú êë and, R = 2a úû
L
T
q
2
Similarly, electric field due to disc (R = a)
F2 +q 1 +q +q 3
F3
s æ hö
mg E2 = ç 1– ÷
2e0 è a ø
5 kq 2 Electric field due to given disc
Fe = ...(i)
4 L2 sin 2 q E = E1 – E2
T sin q = Fe ... (ii)
s é hù s é hù sh
T cos q = mg ... (iii) ê1– 2a ú – 2e ê1– a ú =
From (i), (ii), and (iii) 2e 0 ë û 0 ë û 4e0 a
16 s
q= pe 0 mgL2 sin 2 q tan q Hence, c =
5 4ae 0

@ebooks_for_freee on Telegram
156 PHYSICS
uur ur T sin q = qE .... (i)
14. (b) Flux = E . A.
uur T cos q = mg .... (ii)
E is electric field vector & A is area vector.. Dividing (i) by (ii),
uur ur
Here, angle between E & A is 90º. qE q æ s ö sq
uur ur tan q = =
So, E . A = 0 ; Flux = 0 mg mg çè e0K ÷ø e0 K . mg
r
15. (c) Given E = Eo xˆ \ s µ tan q
This shows that the electric field acts along + x 18. (c) Force of interaction
direction and is a constant. The area vector 1 6p1p2 +q
makes an angle of 45° with the electric field. F= . +q
4p Î0 r 4
Therefore the electric flux through the shaded
portion whose area is
r r p1 p2
a ´ 2 a = 2 a 2 is f = E . A = EA cos q = E 0 ( 2a 2 )
2 1
cos 45° = E0 ( 2 a ) ´ = E0a2 –q –q
z 2
r
19. (a) The electric flux f1 entering an enclosed
surface is taken as negative and the electric flux
(a,0,a) (a,a,a) q = 45° f2 leaving the surface is taken as positive, by
2a convention. Therefore the net flux leaving the
E
q A
enclosed surface, f = f2 – f1
(0,0,0) (0,a,0)
y According to Gauss theorem
q
f= Þ q = Î0f = Î0(f2 – f1)
Î0
x
16. (d) Time period of the pendulum (T) is given 20. (c)
by +q
F1
L
T = 2p E1
geff
F2
(mg )2 + (qE )2 E2
geff = –q
m
2 As the dipole is placed in non-uniform field, so
æ gE ö
Þ geff = g 2 + ç ÷ the force acting on the dipole will not cancel each
è m ø other. This will result in a force as well as torque.
L 21. (12 × 10–3) Charges (q) = 2 × 10–6 C, Distance
Þ T = 2p
2 (d) = 3 cm = 3 × 10–2 m and electric field (E) = 2
æ qE ö
g2 + ç ÷ × 105 N/C. Torque (t) = q.d.E
è m ø t = (2 × 10–6) × (3 × 10–2) × (2 × 105)
17. (c) P = 12 × 10–3 N–m.
T cos q 22. (5 × 10–4) At equilibrium
q T
s
F = Eq = q q E
e0K
T
q
T sin q qE

mg mg

@ebooks_for_freee on Telegram
Electric Charges and Fields 157
T cos q = mg and T sin q = qE
KqQ KQ 2 Q
mg = 30.7 × 10–6 × 9.8 = 3 × 10–4 N \ 2 =- Þ = -2 2
2 2 q
a 2a
qE = 2 × 10–8 × 20,000 = 4 × 10–4 N
26. (2) r = Cr2
-4 2 -4 2 r r
\ T = (3 ´ 10 ) + ( 4 ´ 10 ) = 5 × 10–4 N
q = ò 4pr 2 drr = ò 4 pCr 4dr = 4 pCr5
23. (7.8 × 10–7) F = qE = mg (q = 6e = 6 × 1.6 × 10–19) 0 0 5
mass m m
Density (d) = = or r 3 = kq ( r = R ) k ( 4 / 5) pCR 5
volume 4 3 4 E r = 2R = =
pr pd ( 2R )2 4R 2
3 3 5
æ qE ö R kq ( r = R ) k ( 4 / 5 ) pC ( R / 2 )
Putting the value of d and m ç = and solv- Er = = =
è g ÷ø 2 ( R / 2 )2 ( R/2 )2
ing we get r = 7.8 × 10–7m E r = 2R
Now solve to get E =2
® r= R / 2
24. (0.125) E = E 0 iˆ + 2E 0 ˆj 27. (70.7%) Electric field intensity at the centre of
the disc.
Given, E 0 = 100N / c
® s
So, E = 100iˆ + 200ˆj E= (given)
2 Î0
Radius of circular surface = 0.02 m Electric field along the axis at any distance x
2 22 from the centre of the disc
Area = pr = ´ 0.02 ´ 0.02
7
s æ x ö
-3 ˆ 2
= 1.25 ´10 i m [Loop is parallel to Y-Z plane] E' = ç1 - ÷
2 Î0 çè x2 + R 2 ø
÷
Now, flux (f) = EA cosq
From question, x = R (radius of disc)
( ) -3
= 100iˆ + 200ˆj .1.25 ´ 10 ˆi cos q° [q = 0°]
\ E ' = s ç1 -
æ R ö
÷
= 125 × 10–3 Nm2/C = 0.125 Nm2/C 2 Î0 çè R2 + R2 ø
÷
r r r
25. ( –2 2 ) It is given that FA + FB + FD = 0 s æ 2R - R ö
= ç ÷
r r r 2 Î0 çè 2R ÷ø
Where FA , FB and FD are the forces applied
4
by charges placed at A, B and D on the charge ; E
placed at C. 14
r r r \ % reduction in the value of electric field
Þ FB + FD = -FA æ 4 ö
çè E - E÷ø ´ 100
a B 14 1000
A = = % ; 70.7%
E 14
28. (2) Let us consider a spherical shell of radius x
and thickness dx. The volume of this shell is
a 2 4px2(dx). The charge enclosed in this spherical
shell is
D C

r r KqQ
| FB + FD |= 2 dx
a2 x

r KQ2
Þ | FA |=
2a 2

@ebooks_for_freee on Telegram
158 PHYSICS

dq = (4px2)dx × kxa 3
4 æRö
\ dq = 4pkx2+a dx. r´ p ç ÷
r 3 è2ø
For r = R : Þ EA =
2
The total charge enclosed in the sphere of radius æRö
e 0 × 4p ç ÷
R is è2ø
R
R 3+ a
ò0
Q = 4pk x 2+ a dx = 4pk
(3 + a)
.
B A
\ The electric field at r = R is
3R
1 4 pkR3+ a 1 4pk 1+ a
E1 = = R 2 R/2
4 pe 0 (3 + a ) R 2 4pe 0 (3 + a)
For r = R/2 : r s ( R / 2 ) æ sR ö
The total charge enclosed in the sphere of radius Þ EA = =ç ÷
3e0 è 6e 0 ø
R/2 is
Electric fields at ‘B’
R /2
4pk ( R / 2)3+ a
ò0 4pk x 2 + a dx =
3
Q' = 4 4 æRö
(3 + a) r k ´ r´ pR 3 k ´ r´ p ç ÷
3 3 è2ø
EB = -
\ The electric field at r = R/2 is 2 2
R æ 3R ö
1+ a ç ÷
1 4pk ( R / 2)3+ a 1 4 pk æ R ö è 2 ø
E2 = = ç ÷
4 pe 0 3 + a ( R / 2) 2 4pe 0 (3 + a) è 2 ø r sR æ 1 ö ( s ) 4p æ R ö
3
Þ EB = -ç ÷ ç ÷
1 3e 0 è 4pe0 ø æ 3R ö2 3 è 2 ø
Given, E2 = E1 ç ÷
8 è 2 ø
\
1+ a
1 4 pk æ R ö 1 1 4 pk 1+ a r sR sR
çè ÷ø = 3´ R Þ EB = -
4pe0 (3 + a) 2 2 4pe 0 (3 + a) 3e0 54e 0
Þ 1+a=3 Þ a=2
29. (–670) 17 æ sR ö
Þ EB = ç ÷
æ Rö 54 è e0 ø
30. (0.53) Electric field at A ç R ' = ÷
è 2ø E A 1´ 54 æ 9 ö 9 2 18
= =ç ÷= ´ =
q EB 6 ´17 è 17 ø 17 2 34
E A .ds =
e0

@ebooks_for_freee on Telegram
Electrostatic Potential and Capacitance 159

CHAPTER
Electrostatic Potential and
16 Capacitance
1. (b) Potential at the centre of the triangle, 6. (d) Current will flow in connecting wire so
åq 2q - q - q that energy decreases in the form of heat
V= = =0 through the connecting wire.
4 p e0 r 4 p e0r
7. (c)
Obviously, E ¹ 0
e0 A
2. (c) Volume of big drop = 1000 × volume of each 8. (d) For a parallel plate capacitor C =
d
small drop
4 3 4
Cd 1 ´ 10 -3
pR = 1000 ´ pr 3 Þ R = 10r \ A= e = = 1.13 × 108 m2
3 3 0 8.85 ´ 10 -12
This corresponds to area of square of side 10.6
Q V = kq and V ' = kq ´ 1000 km which shows that one farad is very large unit
r R
Total charge on one small droplet is q and on the of capacitance.
9. (d) As system is isolated so charge remains
big drop is 1000q. constant,
V ' 1000r 1000 e A
Þ = = = 100 C= 0
V R 10 d
\ V ' = 100V so, if d increases then C decreases.
3. (b) Net electrostatic energy for the system Now, for keeping the charge constant V
increases.
é q2 Qq Qq ù
U = Kê + + =0 10. (c) C2 and C3 are parallel so V2 = V3
êë a a a 2 úú C1 and combination of C2 & C3 is in series.
û
Q So, V = V2 + V1 or V = V3 + V1
é 1 ù
Þ q = -Q ê1 + ú and also Q1 = Q2 + Q3
2 a a 2
ë û qæ 1 1 ö
-q 2 11. (b) w = w f - vi = 2 çç C - C ÷÷
ÞQ= a è f i ø
2+1 +q +q
(5 ´ 10)2 æ 1 1 ö 6
dV = çè - ÷ø ´ 10
4. (d) V = 5 + 4x 2 \ = 8x ...(1) 2 2 5
dx = 3.75 × 10–6J
Force on a charge is
(O,2 ) Q Q(4, + 2)
æ dV ö
F = qE = q ç - )
è dx ÷ø = q ( -8x ; from (1)
39. (b)
= -2 ´10-6 ´ (–8 ´ 0.5) = 8 ´ 10-6 N
5. (a) The electric potential V(x, y, z) = 4 x2 volt (O, – 2 ) Q
Q(4, – 2)

r æ ¶V ¶V ˆ ¶V ö
Now E = ç iˆ + ˆj +k ÷ Potential at origin
è ¶x ¶y ¶z ø
KQ KQ KQ KQ
¶V ¶V ¶V v= + + +
Now = 8 x, = 0 and =0 2 2 20 20
¶x ¶y ¶z
r æ 1 ö
Hence E = -8 xiˆ , so at point (1m, 0, 2m) and potential at ¥ = 0 = KQ ç 1+ ÷
r è 5ø
E = -8iˆ volt/metre or 8 along negative X-axis.

@ebooks_for_freee on Telegram
160 PHYSICS
\ Work required to put a fifth charge Q at origin C1C2
Heat loss, H = (V1 - V2 )2
Q2 æ 1 ö 2(C1 + C2 )
1+
4pe0 çè ÷
W = VQ =
5ø In the equation, put V2 = 0, V1 = V0
13. (b) C
C1 = C, C2 =
d 2
y k
C
xa dx C´
2 (V - 0) 2 = C V 2
a Loss of heat =
æ Cö 0 6
0
y d d 2çC + ÷
From figure, = Þy= x è 2ø
x a a 1
d 1 y (d - y) H = CV02
dy = (dx) Þ = + 6
a dc Ke 0adx e0 adx 16. (a) We have given two metallic hollow spheres
1 y æy ö of radii R and 4R having charges Q1 and Q2
= ç + d - y÷ø
dc e 0 abx è k respectively.
e 0 adx Potential on the surface of inner sphere (at A)
ò dc = ò y kQ kQ
+d-y VA = 1 + 2
k R 4R
d Potential on the surface of outer sphere (at B)
a dy
or, c = e 0 a. ò æ
d æ1 ö kQ kQ 1 ö
0 d + y ç - 1÷ VB = 1 + 2 ç Here, k = ÷
èk ø 4R 4R è 4pe 0ø
d
e0a 2 é æ æ 1 ööù
= êl n ç d + y çè - 1÷ø ÷ ú Q1 Q2
æ1 ö ë è k ø û0
çè - 1÷ø d
k
R A
æ æ1 öö
k Î0 a 2 ç d + d ç k - 1÷ ÷ 4R
B
= lnç è ø÷
( )
1 - k d ç d ÷
ç ÷ Potential difference,
è ø
3 kQ 3 Q
k Î0 a 2 2 DV = V A - VB = × 1 = × 1
æ 1 ö k Î0 a lnk 4 R 16p Î0 R
= lnç ÷ =
(1 - k ) d è k ø ( k - 1) d 17. (d) When charge Q is on inner solid
14. (b) W = – Du conducting sphere

= ( -1)
( ce ) 2 - ( ce ) 2
2kc 2c +Q
–Q
2
+Q
e c k -1
= E
2 k
= 508 J
Electric field between spherical surface
15. (d) When two capacitors with capacitance C1 r KQ r r
and C2 at potential V1 and V2 connected to each E = 2 Soò E.dr = V given
other by wire, charge begins to flow from higher r
to lower potential till they acquire common Now when a charge – 4Q is given to hollow
potential. Here, some loss of energy takes place shell
which is given by.

@ebooks_for_freee on Telegram
Electrostatic Potential and Capacitance 161

or V1 – V2 = 180 V
+Q
22. (200) At steady state, there is no current in
–Q –3Q capacitor.
2W and 10W are in series. There equivalent
resistance is 12W. This 12W is in parallel with
4W and there combined resistance is 12 × 4/
Electric field between surface remain unchanged.
(12 + 4). This resistance is in series with 6W.
r KQ
E= 2 Therefore, current drawn from battery
r æ ö
as, field inside the hollow spherical shell = 0 V ç 72 ÷
\ Potential difference between them remain i= =ç ÷ = 8A
R ç 6 + 12 ´ 4 ÷
unchanged
r r è 12 + 4 ø
i.e. ò E.dr = V
llel
6mF para
ies
30mC ser
18. (18) - + 2
- + - + - +
10mF 2mF i 6
4mF 72 V
4 10
As given in the figure, 6µF and 4µF are in parallel. 10 F
Now using charge conservation
Current in 10W resistor
6
Charge on 6µF capacitor = ´ 30 = 18µC æ 4 ö
6+4 i' = ç ÷8 = 2 A
Since charge is asked on right plate therefore è 4 + 12 ø
is +18µC Pd across capacitor, V = i’ R = 2 × 10 = 20V
19. (8.5) Capacitance of a capacitor with a \ Charge on the capacitor, q = CV
dielectric of dielectric constant k is given by = 10 × 20 = 200 mC.
23. (100) Volume of big drop = 1000 × volume of
k Î0 A each small drop
C=
d 4 3 4
V k Î0 AE pR = 1000 ´ pr 3 Þ R = 10r
Q E= \C= 3 3
d V Q V = kq and V ' = kq ´ 1000
-12
k ´ 8.86 ´ 10 ´ 10-4 ´ 106 r R
15 ´ 10-12 = Total charge on one small droplet is q and on the
500
big drop is 1000q.
k = 8.5
Q1 Q2 V ' 1000r 1000
Q Þ = = = 100
20. (1) V = V R 10
C
\ V ' = 100V
æ Q1 - Q2 ö
=ç ÷ 24. (10) Potential at any point inside the sphere
è 2C ø = potential at the surface of the sphere = 10V.
æ 4- 2 ö Q1 - Q2 (Q1 - Q2 ) 25. (4) The inner sphere is grounded, hence its
=ç ÷ =1V -
è 2 ´1 ø uur 2 2 potential is zero. The net charge on isolated outer
21. (180) Given, E = ( Ax + B ) i
ˆ sphere is zero.
or E = 20x + 10 Let the charge on inner sphere be q'.
Using V = ò Edx , we have \ Potential at centre of inner sphere is
1
1 q¢ 1 q q
V2 – V1 = ò ( 20 x + 10) dx = – 180 V =
4pe 0 a
+0+
4pe 0 4a
= 0 \ q¢ = -
4
-5

@ebooks_for_freee on Telegram
162 PHYSICS

CHAPTER

Current Electricity
17
1. (b) Rt = R0(1 + at) at t°C Rt = 3R0 8. (a) Let the resistance of single copper wire be
a = 4 × 10–3 /°C R1. If r is the specific resistance of copper wire,
3R0 = R0 (1 + 4 × 10–3 × t) then
r´ l r´l
2 R1 = =
\ 3 – 1 = 4 × 10–3t \ t= = 500°C ...(1)
A1 p r12
4 ´ 10-3
2. (a) The slope of V – I graph gives the When the wire is replaced by six wires, let the
resistance of a conductor at a given resistance of each wire be R2. Then
temperature. r´l r´l
R2 = = ...(2)
From the graph, it follows V A2 p r22
that resistance of a T1 From eqs. (1) and (2), we get
conductor at temperature T2
R1 r2 2 5 (3 ´10-3 )2
T1 is greater than at = or = ; R 2 = 45 W
R 2 r12 R 2 (9 ´10-3 )2
temperature T2 As the I
O These six wires are in parallel. Hence the required
resistance of a conductor
resistance would be R2 = 7.5 W
is more at higher temperature and less at lower
9. (a) Consider an element part of solid at a
temperature, hence T1 > T2
distance x from left end of width dx.
E E dx
3. (b) I= Þ V= R. x
R+r R+r

[Q V = IR ]
(E - V)
Þ r= R.
V V
4. (b) Since, the voltage is same for the two
1 Resistance of this elemental part is,
combinations, therefore H µ . Hence, the rdx r0 xdx
R dR = =
combination of 39 bulbs will glow more. pa 2 pa 2
P R SS L
5. (b) = where S = 1 2 r xdx r0 L2
Q S S1 + S 2 R = ò dR = ò 0 =
2
0 pa 2 pa 2
6. (c) The resistance of the square Y is given by V V ´ 2pa 2
2L r Current through cylinder is, I = =
RY = r = R r0 L2
( 2L ) t t
Potential drop across element is, dV = I dR =
same as before. Hence, Rx/RY = 1
2V
7. (d) R1 = R 0 (1 + a1t1 ) x dx
1
R 2 = R 0 (1 + a 2t2 ) L2
2
a1 t2 dV 2V
As R1 = R2 and R 0 = R 0 , Þ = E(x) = = x
1 2 a 2 t1 dx L2

@ebooks_for_freee on Telegram
Current Electricity 163
10. (d) The equivalent circuit is as shown in figure. V

i
i
r
E'

V = emf of battery,
E = emf of standard cell.
L = length of potentiometer wire
The resistance of arm AOD (= R + R) is in parallel El 30E
to the resistance R of arm AD. V= =
L 100
Their effective resistance 13. (a)
2R ´ R 2 14. (c) Balancing length l will give emf of cell
R1 = = R
2R + R 3 \ E = Kl
The resistance of arms AB, BC and CD is Here K is potential gradient.
2 8 If the cell is short circuited by resistance 'R'
R2 = R + R + R = R
3 3 Let balancing length obtained be l¢ then
The resistance R1 and R2 are in parallel. The
effective resistance between A and D is V = kl¢
2 8 æ E -V ö
R´ R
R1 ´ R 2 3 = 8 R. r= ç R
R3 = = 3 è V ÷ø
R1 + R 2 2 8 15
R+ R Þ V = E – V Þ 2V = E [Q r = R given]
3 3
11. (d) Let internal resistance of source = R l
or, 2Kl¢ = Kl \ l¢ =
V 2
Current in coil of resistance R1 = I1 =
R + R1
m
15. (c) r=
V ne 2 t
Current in coil of resistance R 2 = I 2 =
R + R2
9.1 ´ 10-31
Further, as heat generated is same, so =
8.5 ´1028 ´ (1.6 ´10 -19 ) 2 ´ 25 ´ 10 -15
I12 R1t 2
= I2 R 2t = 10–8 W-m
2 2 16. (a) Using, I = neAvd
æ V ö æ V ö
or çç ÷÷ R1 = çç ÷÷ R 2 1
è R + R1 ø è R + R2 ø \Drift speed v d =
neA
Þ R1 (R + R 2 ) 2 = R 2 (R + R1 ) 2 1.5
= 0.02 mms–1
9 ´ 10 ´ 1.6 ´10-19 ´ 5 ´ 10-6
28
Þ R 2 ( R 1 - R 2 ) = R 1R 2 ( R 1 - R 2 )
æ e ö
17. (d) i=ç ÷
Þ R= R 1R 2 èR+rø
12. (d) From the principle of potentiometer, V µ l Power delivered to R.
2
V l æ e ö
Þ = ; where P = i2 R = ç ÷ R
E L è R+r ø

@ebooks_for_freee on Telegram
164 PHYSICS

1
R 2 = k ò l -1 2dl = k.(2 - 2 l )
l
Putting R1 = R2
k2 l = k(2 - 2 l )

\ 2 l =1
1
dP
=0 l=
P to be maximum, 2
dR
1
d éæ e ö ù
2 i.e., l = m Þ 0.25 m
4
or dR êç R + r ÷ R ú = 0
êëè ø úû 20V
i1 xV i2 10 V
or R = r 21. (5)
18. (c) Current passing through resistance R1, A 2W C 4W B
v 10
i1 = = = 0.5A
R1 20 2W
and, i2 = 0
i
19. (d) Net Power, P
= 15 × 45 + 15 × 100 + 15 × 10 + 2 × 1000 O
= 15 × 155 + 2000 W Let voltage at C = xV
Power, P = VI From kirchhoff's current law,
P KCL : i1 + i2 = i
Þ I=
V 20 - x 10 - x x - 0
+ = Þ x = 10
15 ´155 + 2000 2 4 2
\ I main = = 19.66 A » 20 A
220
20. (c) We have given V X 10
\ i= = = =5A
R R 2
dR 1 dR 1
µ Þ = k´ (where k is 22. (1) P = V2/R
dl l dl l
constant) Þ R = V2/P × 104 / 100 = 400 W
dl S = 104/200 = 0.5 × 102 = 50 W
dR = k
l R/S=8
Let R1 and R2 be the resistance of AP and PB
respectively. rl
23. (1) Resistance, R =
Using wheatstone bridge principle A
R ' R1 l l rl 2
\ = or R1 = R 2 R= r ´ = [QVolume (V) = Al.]
R ' R2 A l V
dl Since resistivity and volume remains constant
Now, ò dR = k ò therefore % change in resistance
l
l DR 2Dl
\ R1 = k ò l -1 2dl = k.2. l = = 2 ´ (0.5) = 1%
R l
0

@ebooks_for_freee on Telegram
Current Electricity 165
24. (300) R3= 100W R4= 500W 26. (20) Colour code for carbon resistor
Bl, Br, R, O, Y, G, Blue, V, Gr, W
i i1
0 1 2 3 4 5 6 7 8 9
R1= 400W
Resistance, R = AB × C ± D
i2 R2 \ Resistance, R = 50 × 102 W
18V Now using formula, Power, P = i 2R
Across R4 reading of voltmeter, V4 = 5V P 2
\ i= = = 20mA
V R 50 ´ 102
Current, i4= 4 = 0.01A
R4 27. (11 × 10–5) Power, P = I2R
V3 = i1 R3 = 1V 4.4 = 4 × 10–6 × R
V3 + V4 = 6V = V2 Þ R = 1.1 × 106W
V1 + V3 + V4 = 18V When supply of 11 v is connected
Þ V1 =12 V v 2 112 112
Power, P’= = ´ ´10 –6 = 11 × 10–5 W
R 1.1 1.1
V1 dQ
i= = 0.03A 28. (53) I= = 10t + 3 = 10 × 5 + 3 = 53 A
R1 dt
Charge
i = i1 + i2 Þ i 2 = i – i, = 0.03 – 0.01A = 0.02 A 29. (6.25 × 1015) Current, I = ;
Time
V2 6 as charge q = n × 1.6 × 10–19
\R 2 = = = 300W
i 2 0.02 n×1.6×10-19
10- 3 amp = Þ n = 6.25 × 1015
1 sec
25. (4) 30. (3.6) The potential difference across 4W
resistance is given by V = 4 × i1 = 4 × 1.2 = 4.8
6W 6W volt
So, the potential across 8W resistance is also 4.8
volt.
A B V 4.8
6W
Current i 2 = = = 0.6 amp
8 8
Resistance, R µ l so resistance of each side of
the equilateral triangle = 6 W Current in 2W resistance i = i1 + i2
Resistance Req between any two vertices \ i = 1.2 + 0.6 = 1.8 amp

1 1 1 Potential difference across 2W resistance


= + Þ R eq. = 4 W
R eq 12 6 VBC = 1.8 × 2 = 3.6 volts

@ebooks_for_freee on Telegram
166 PHYSICS

CHAPTER
Moving Charges and
18 Magnetism
1. (c) As electron move with constant velocity 10–2 × 0.10 = 3.6 × 10–6 N-m.
without deflection. Hence, force due to magnetic 10. (a) Rg = 50W, Ig = 25 × 4 × 10–4W = 10–2 A
field is equal and opposite to force due to electric Range of V = 25 volts
field. R
V = Ig(HR + Rg)
A B
E 20
qvB = qE Þ v = = = 40 m / s
B 0.5 Ig HR Rg
2. (b) M = iA = i ´ pR 2 V
\ HR = - R g = 2450W
é Qù Ig
Qw 1
also i = Þ M = Qw R 2 êQ i = t ú 11. (a) In mass spectrometer, when ions are
2p 2 ë û
accelerated through potential V
F
F = qvB sin q Þ B = 1
3. (d) qv sin q mv 2 = qV ..........(i)
2
As the magnetic field curves the path of the ions
F
Bmin = (when q = 90°) in a semicircular orbit
qv
mv 2 BqR
Bqv = Þv= .......... (ii)
10-10 -3 R m
= = 10 Tesla Substituting (ii) in (i)
10-12 ´105 2
4. (a) 1 é BqR ù q 2V
m = qV or =
m 0 2pni1 m 0 2pni 2 2 êë m úû m B2 R 2
5. (d) B= . - . q 1
4p r1 4p r2 Since V and B are constants, \ µ 2
m R
m é ni ni ù 12. (d) From figure,
= 0 ê 1- 2ú
2 ë r1 r2 û sin a = d/R
a R
m 0 Ic m0 Ie I R mv 2
6. (a) = ÞH= e we know, = qvB
a
2R 2 pH pI c R d

7. (d) B = m 0 nI mv dqB
8. (a) Force between two long conductor carrying Þ R= \ sin a =
qB mv
current,
qé 1 2ù
m 2I I µ 2(2 I1 ) I 2
F'=- 0 l
sin a = Bd êQ qV = 2 mv ú
F = 0 1 2 ´l ; ë
2mV û
4p d 4p 3d 2
mv mv
13. (b) = qvB Þ r =
F ' -2 r qB
\ =
F 3 mpv p md vd ma va
9. (a) tmax = MB = niAB = ni (l × b) B Þ rp = ; rd = ; ra =
qpB qd B qa B
tmax = 600 × 10–5 × 5 × 10–2 × 12 ×
ma = 4m p , md = 2m p

@ebooks_for_freee on Telegram
Moving Charges and Magnetism 167
qa = 2q p , q d = q p (0, d)
From the problem
1 1 1
E p = Ed = Ea = m p v p 2 = md vd 2 = ma va2 30° V
2 2 2 (0, 0) Fm
r/2
Þ v 2p = 2vd2 = 4va2 r

Thus we have, ra = rp < rd C


qVB é - 3i - j ù
14. (c) The particles will not collide if a= ê ú
× × × × × m ë 2 û
d > (2r1 + 2r2) – v1
this option is not given in the all above four
d > 2 (r1 + r2) × 2r1 × × × choices.
r r r
æ mv mv 2 ö 19. (b) | t |= | m ´ B | [m = NIA]
d>2ç 1 + ×d × × × ×
è qB qB ÷ø =NIA × B sin 90o [A = pr2]
× 2 × 2r
× ×
Þ t =NIpr B 2
2m
or d > (v1 + v2 ) Å v2
qB × × × × ×
m i m i m
15. (d) B1 = – 0 1 + 0 2 = 0 (i2 – i1 )
2 pr / 2 2 pr / 2 pr
= 6 ´ 10 –6 T ...(i)
When the current is reversed in I2,
m (i + i ) m (i + i )
B2 = – 0 1 2 = – 0 1 2 = 3´10–5 T ...(ii) 20. (a) I1 I2 = Positive
p
2 r/2 pr (attract) F = Negative
Dividing (ii) by (i) we get I1 I2= Negative
–(i1 + i2 ) 30 (repell) F = Positive
= =5 Hence, option (a) is the correct answer.
i2 – i1 6 21. (6) At a distance x consider small element of
– (i1 + i2) = 5i2 – 5i1 Þ 6i2 = 4i1 width dx.
i1 3 Magnetic moment of the small element is
=
i2 2 æq ö
çè dx÷ø w
l
16. (a) dm = .px 2
m 0 NS iS 2p
dx
17. (a) Bsolenoid = m0 ns is = ,
LS
m 0 NS iS iNp r 2 x
t = BS iNA = l/2
LS qw 2
x dx ; M = qwl = qpfl
2 2
M= ò
4p ´ 10-7 ´ 500 ´ 3 ´ 0.4 ´ 10 ´ p ´ (0.01) 2 -l / 2
2l 24 12
t=
0.4 22. (5) The centre will be at C as shown :
y
= 6p2 × 10–7 N-m.
B=1T
18. (Bonus) Assuming particle enters from (0, d)
x
mv r
r= , d= (– 3–1)
60° 30°
C
qB 2

@ebooks_for_freee on Telegram
168 PHYSICS
Coordinates of the centre are (r cos 60°, – r sin E
60°) E = vB Þ v = ....(ii)
B
where r = radius of circle From equation (i) and (ii)
mv 1 ´ 1 æ1 3ö qB2 R 1.6 ´ 10
–19
´ ( 0.5) ´ 0.5 ´10 –2
2
= = = 1 i.e., ç , - ÷ m= =
Bq 1 ´ 1 è2 2 ø E 100
23. (27) Given : Radius = R; Distance x = 2 2R \ m = 2.0 × 10–24 kg
2pm
3/2 3/2 28. (4) Pitch = (v cos q)T and T =
Bcentre æ x2 ö æ (2 2R)2 ö qB
= çç1 + 2 ÷÷ = çç1 + ÷
Baxis è R ø è R 2 ÷ø 2pm
\ Pitch = (V cos q)
= (9)3/2 = 27 qB
24. (5 × 10–5) The magnetic field due to circular coil, 2 p æ 1.67 ´10-27 ö
= (4 ´ 105 cos 60°) ç ÷÷ = 4 cm
µ i m 0 i1 m ´ 3 ´ 102 0.3 çè 1.69 ´10-19
B1 = 0 1 = = 0 ø
2r 2 (2p ´ 10 -2 ) 4p
m0 i2 (1) 29. ( 500 3 )
B2 =
2(2p ´ 10 -2 )
30° a
m ´ 4 ´ 10 2 30°
= 0 (2)
4p
m0 I
B= B12 + B22 = . 5 × 102
4p 3a
2
Þ B = 10 - 7 ´ 5 ´ 10 2 Þ B = 5 × 10–5 Wb / m2
Magnetic field due to one side of hexagon
m0I m0 I
25. (7) B = (sin 90° + sin135°) B= (sin 30° + sin 30°)
R
4p 3a
2 4p
2
m0I
= ( 2 - 1) m0 I æ 1 1 ö m0 I
4 pR ÞB= çè + ÷ø =
26. (7) Magnetic field strength at P due to I1 2 3a 2 2 2 3ap
r Now, magnetic field due to one hexagon coil
m 0 I1 ˆ 4p´ 10-7 ´ 2 ˆ m I
B1 = k= k B = 6´ 0
2p(AP) 2p´1´10-2 2 3ap
Again magnetic field at the centre of hexagonal
= (4 ´10-5 T) kˆ
shape coil of 50 turns,
Magnetic field strength at P due to I2
m I é 10 ù
r B = 50 ´ 6 ´ 0 Qa = = 0.1 mú
m I 4p´10-7 ´ 3 ˆ 2 3ap ë ê 100 û
B2 = 0 2 ˆj = j
2p(BP) 2p´ 2 ´10-2 150m 0 I m0 I
or, B = = 500 3
= (3 ´ 10-5 T) ˆj 3 ´ 0.1 ´ p p
r 30. (20) Given,
Hence, B = (3 ´ 10-5 T) ˆj + (4 ´ 10-5 T) kˆ Area of galvanometer coil, A = 3 × 10–4 m2
27. (2 × 10–24) As particle is moving along a circular Number of turns in the coil, N = 500
path Current in the coil, I = 0.5 A
mv r r
\R = ...(i) Torque t =| M ´ B |= NiAB sin(90°) = NiAB
qB
Path is straight line, then t 1.5
ÞB= = = 20 T
qE = qvB NiA 500 ´ 0.5 ´ 3 ´10 -4

@ebooks_for_freee on Telegram
Magnetism and Matter 169

CHAPTER

Magnetism and Matter


19
1. (c) Here, q = 30°, t = 0.018 N-m, B = 0.06 T
I
Torque on a bar magnet : 5. (Bonus) We have, T = 2p MB
x
t = MB sin q
0.018 = M ´ 0.06 ´ sin 30° T12 Bx 2
\ =
1 T22 Bx1
Þ 0.018 = M ´ 0.06 ´ Þ M = 0.6 A-m 2
2
2
Position of stable equilibrium (q = 0°) æ 2ö B2 cos 45° B2 ´ 2
Position of unstable equilibrium (q = 180°) or çè ÷ø = =
1.5 B1 cos 30° 2 ´ B1 ´ 3
Minimum work required to rotate bar magnet from
stable to unstable equilibrium 2
æ 4ö B2 2
DU = U f - U i = -MB cos180° - (- MB cos0°) çè ÷ø = ´
3 B1 6
W = 2MB = 2 ´ 0.6 ´ 0.06
B1 9
\ W = 7.2 ´ 10-2 J \ = = 0.46
B2 8 6
2. (a) According to Curie law for paramagnetic 6. (b) Electromagnet should be amenable to
substance, magnetisation & demagnetization.
TC \ Materials suitable for making electromagnets
1 c 2 should have low retentivity and low coercivity
c µ T Þ 1 = T
C c2 C1 should be low.
7. (d)
2.8 ´ 10 –4 300
=
c2 350

2.8 ´ 350 ´10 –4


c2 = = 3.266 × 10–4
300
I
3. (d) Magnetic susceptibility, c=
H
Magnetic moment 20 ´10 –6
where, I = =
Volume 10 –6
8. (b) Given,
= 20 N/m2 Volume of iron rod, V = 10-3 m3
20 1 Relative permeability, m r = 1000
Now, c = = ´10 –3 = 3.3 ´10–4
60´103 3 Number of turns per unit length, n = 10
B Magnetic moment of an iron core solenoid,
4. (b) Corecivity, H = M = (m r - 1) ´ NiA
m0
æ Nö V N
and B = m 0 ni ç n = ÷ Þ M = (mr - 1) ´ Ni Þ M = (mr -1) ´ iV
è lø l l
N 100 10
or, H = i = × 5.2 = 2600 A/m Þ M = 999 ´ ´ 0.5 ´ 10 -3 = 499.5 » 500.
l 0.2 10 -2

@ebooks_for_freee on Telegram
170 PHYSICS
9. (d) For paramagnetic material. According to 16. (a) W = MB [1– cosq]
curies law W90o = MB
1 æ1ö MB

T W60o = MB ç ÷ Þ
è 2ø 2
For two temperatures T1 and T2 Þ MB =nMB/2 Þ n = 2
c1T1 = c2T2 17. (c) The potential energy of a magnetic dipole
m placed in an external magnetic dipole is
I r r
But c = U = - m.B .
B
Therefore, work done in rotating the dipole is-
I1 I W = DU = 2mB = 2 × 5.4 × 10-6 × 0.8
\ T1 = 2 T2
B1 B2 = 8.6 × 10-6 Joule.
18. (c) Along the equatorial line, magnetic field
6 I 0.3 strength
Þ ´ 4 = 2 ´ 24 Þ I 2 = = 0.75 A/m
0.4 0.3 0.4 m M
10. (b) When magnetic field is applied to a B= 0
4p 2
( )
3/2
diamagnetic substance, it produces magnetic r + l2
field in opposite direction so net magnetic field Given: M = 4JT–1 r = 200 cm = 2 m
inside the cavity of sphere will be zero. So, field
6cm
inside the paramagnetic substance kept inside l= = 3 cm = 3 × 10–2 m
the cavity is zero. 2
11. (b) Work done 4p´ 10-7 4
= MB (cos q1 – cos q2) \B= ´
4p 3/2
= MB (cos 0° – cos 60°)
4
æ 1 ö 2 ´ 10 ´ 6 ´ 10
–4
é 2
ë
(
ê 2 + 3 ´10
-2 2 ù
ú
û
)
= MB ç1 – ÷ = =6J –8
è 2ø 2 Solving we get, B = 5 × 10 tesla
12. (b) 19. (a) Magnetic field due to a bar magnet in the
broad-side on position is given by
13. (a)
µ0 M
V 3 éQ tan 37º = 3 ù B= ; M = ml .
14. (d) tan q = = ê
H 4 ë 4 úû 4p é 2 l 2 ù 3/2
êr + ú
3 ë 4û
\ V= H
4 20. (a) Couple acting on a bar magnet of dipole
V = 6 × 10–5 T moment M when placed in a magnetic field, is
4 –5 –5 given by t = MB sin q
H = ´ 6 ´ 10 T = 8 ´ 10 T where q is the angle made by the axis of magnet
3
with the direction of field.
\ Btotal = V2 + H2 = (36 + 64) ´ 10 –5 Given the m = 5Am, 2l = 0.2m, q = 30º
= 10 × 10–5 = 10–4T. and B = 15 Wbm–2
15. (d) Given, I = 9 × 10–5 kg m2, B = 16p2 × 10–5 T \ t =MB sinq = (m × 2l) B sin q
15 3 1
T= = s = 5 × 0.2 × 15 ×
= 7.5 Nm.
20 4 2
In a vibration magnetometer
B
I 4p 2 I 21. (2600) Corecivity, H = m
Time period, T = 2p or M = 0
MB BT 2
æ Nö
4p2 ´ 9 ´ 10-5 and B = m 0ni çè n = ÷ø
M = = 4 A m2 l
2
æ3ö N 100
16p2 ´ 10-5 ´ ç ÷ or, H = i = × 5.2 = 2600 A/m
è4ø l 0.2

@ebooks_for_freee on Telegram
Magnetism and Matter 171
22. (6.5 × 10–5) Using, MB sinq = F l Sinq (t) 25. (25)
26. (0.1)
27. (1023) Given, B = 4 × 10–5 T
F RE = 6.4 × 106 m
B Dipole moment of the earth M = ?
45° m M
B= 0 3
4p d
m 4p´10-7 ´ M
4 ´10-5 =
( )
3
l 4p´ 6.4 ´ 106
MB sin 45° = F sin 45°
2
F = 2MB = 2 × 1.8 × 18 × 10–6 = 6.5 × 10–5N \ M @ 1023 Am2
I 28. ( 3 ) In the first galvanometer
23. (3.3 × 10–4) Magnetic susceptibility, c =
H i1 = K1 tanq1 = K1 tan60o = K1 3
In the second galvanometes,
Magnetic moment 20 ´10 –6
where, I = = i2 = K2 tanq2 = K2 tan45o = K2
Volume 10 –6
In series i1 = i 2 Þ K1 3 = K 2
= 20 N/m2 K1 1
Þ =
20
1 K2 3
Now, c = = ´10 –3 = 3.3 ´10–4
3 3
60´10
1 K n n 3
24. (3.266 × 10–4 ) According to Curie law for But, Ka Þ 1 = 2 \ 1 =
n K 2 n1 n2 1
paramagnetic substance,
29. (0.51) For null deflection,
1
ca T M1 æ d1 ö
3
æ 40 ö
3
64
C =ç ÷ =ç ÷ =
TC M 2 è d2 ø è 50 ø 125
c1 2
= T 30. (15) In tangent galvanometer, I a tan q
c2 C1
I1 tan q1 I1 tan 45
2.8 ´ 10 –4 300
= Þ =
= I2 tan q2 I2 / 3 tan q2
c2 350
q2 = 30o
–4
2.8 ´ 350 ´10 so, deflection will decrease by 45o–30o = 15o
c2 = = 3.266 × 10–4
300

@ebooks_for_freee on Telegram
172 PHYSICS

CHAPTER

Electromagnetic Induction
20
1. (c) Total number of turns in the solenoid, 6. (d) According to Lenz's law, when switch is
N = 500; I = 2A. closed, the flux in the loop increases out of plane
Magnetic flux linked with each turn of paper, so induced current will be clockwise
and simillarly anticlock wise when switch will be
= 4 × 10–3 Wb open.
Nf 7. (a) The flux linked with the coil when the plane
As, N f = LI Þ L = = 1 H.
1 of the coil is perpendicular to the magnetic field
2. (b) ftotal = Blarge Asmall is
f = nAB cos q = nAB.
é m0 i 2ù m0 ´ 8 2il2
= ê 4p L / 2 (2sin 45°) ´ l ú ´ 4 = 4p´ ( L) the change in flux on rotating the coil by 180° is
ë û df = nAB – (–nAB) = 2nAB
On comparing with ftotal = Mi , we get df
m 8 2l 2 \ induced charge =
M = 0. R
4p L 2nAB 2 ´ 100 ´ 0.001 ´ 1
3. (a) Emf induced |e| = Blv = =
R 10
Bl v \ Induced charge = 0.02 C.
I=
r Df f2 - f1 BA - 0 BA
Force act on the rod due to magnetic field in 8. (b) Q= = = =
R R R R
opposite direction of velocity 9. (a) The direction of current in the loop will be
such as to oppose the increase in the magnetic
B2 l 2 v
F = BIl = field.
r o
10. (d) | e | = BlV sin q = 0.1 × 2 × 20 sin 30 = 2 volt
Therefore, an equal force must be provided to 11. (b) According to faraday’s law of electromag-
move the rod with velocity v.
4. (a) Take a very small element dx at a distance x -df
netic induction, e =
from one end then dt
l di 15 5
1 L´ = 25 Þ L ´ = 25 or L = H
e = - Bw ò xdx = - Bwl 2
2 dt 1 3
0
Change in the energy of the inductance,
1 1 5
5. (b) Rate of work =
W
t
= P = Fv;
2
( )
DE = L i12 – i 22 = ´ ´ (252 –10 2 )
2 3
5
= ´ 525 = 437.5 J
æ Bvl ö 6
also F = Bil = B ç ÷l
è R ø 12. (a) According to Faraday's law of
df
electromagnetic induction, e =
B 2v 2l 2 (0.5) 2 ´ (2)2 ´ (1) 2 1 dt
ÞP = = = W Also, e = iR
R 6 6
df
[Here W = Watts] \ iR = Þ ò d f = R ò idt
dt

@ebooks_for_freee on Telegram
Electromagnetic Induction 173
Magnitude of change in flux (df) = R × area under 19. (b) In the given question,
current vs time graph Current flowing through the wire, I = 1A
1 1 Speed of the frame, v = 10 ms–1
or, df = 100 ´ ´ ´ 10 = 250 Wb
2 2 Side of square loop, l = 10 cm
13. (d) According to Faraday’s law of Distance of square frame from current carrying
electromagnetic induction, wires x = 10 cm.
Ldi We have to find, e.m.f induced e = ?
Induced emf, e =
dt
According to Biot-Savart’s law
æ 5–2 ö
50 = L ç ÷ m 0 Idlsin q
è 0.1sec ø B=
50 ´ 0.1 5 4p x 2
Þ L= = = 1.67 H
3 3 4p´10-7 1´ 10-1
14. (a) Potential difference between two faces = ´
4p
( )
2
perpendicular to x-axis 10-1
= l V.B = 2 ´10 (6 ´ 0.1) =12mV
–2
= 10–6
15. (a) Induced, emF, e = Bvl Induced e.m.f. e = Blv
= 0.3 × 10–4 × 5 × 20 = 10–6 × 10–1 × 10 = 1 mv
= 3 × 10–3 V = 3 mV. 20. (a) Induced emf in a coil,
16. (d) The rate of mutual inductance is given by df
e=- = NBAsin wt
M = m0n1n2 pr12 ...(i) dt
The rate of self inductance is given by Also, e = e0 sin wt
L = m 0 n12 pr12 ...(ii) \ Maximum emf induced, e0 = nBAw
M n2 21. (1.25) The resistance of the loops,
Dividing (i) by (ii) Þ =
L n1 R1 = 2pr1 × 10 = 2p × 0.1 × 10
17. (a) Qcoil = ( NQ) µ i = 6.28 W.
Q1 i1 3 and R2 = 2pr2 = 2p × 1 × 10
So, Q = i =
2 2 2 = 62.8 W.
2 2 -3 Flux in the smaller loop, f = B2A1
or Q2 = Q1 = ´ 10 = 6.67 × 10–4 Wb
3 3 mi 2
18. (c) = 0 2 pr1
2r2
éV ù
m0 ê ú pr12
ë R2 û
Magnetic moment, M = NIA =
2r2
dQ = r dx
[4 + 2.5t ] 2
dQ m0 pr1
dI = .w R2
2p =
2r2
dM = dI × A
l e [d f / dt]
r0 The induced current, i1 = =
w r0 n p ò x 3 dx R1 R1
= . . x p x 2 dx Þ M =
2p l l
0 After substituting the value and simplifying we
p 3 get
= . nrl
4 i = 1.25 A.

@ebooks_for_freee on Telegram
174 PHYSICS
22. (4) For 0 = 2 sin t2 Þ t = 0
e 2 ´10-4
2 2 p The induced current, i = =
and 2 = 2sin t Þ t = , R 2.16 ´10-3
2
= 9.3 × 10–2 A.
\ t= p
2 Estored V2/ R
27. (0.36) The required ratio, = 2
1 2 ESupplied V0 / R
The energy spent, E = Li
2 2 -t / t
1 V 2 V0 (1 - e )
= L(2sin t )
2 2 = =
2 V02 V02
= 2L sin2 t –t/t
= (1 – e ) 2
= 2 × 2 sin2 p/2 = 4J. 2
23. (437.5) According to faraday’s law of é -æ
0.1´10 ö ù
= êë1 - e çè 1 ÷ø ûú
-df = 0.36
electromagnetic induction, e =
dt 28. (0.048) B = m 0 n i
di 15 = (4 p ´ 10 -7 ) (200 ´10 -2 ) ´1.5
L´ = 25 Þ L ´ = 25
dt 1 = 3.8 × 10–2 Wb / m2
5
or, L = H Magnetic flux through each turn of the coil
3
Change in the energy of the inductance, f = BA = (3.8 × 10–2) (3.14 × 10–4)
1 1 5 = 1.2 × 10–5 weber
2
( )
DE = L i12 – i 22 = ´ ´ (252 –10 2 )
2 3 When the current in the solenoid is reversed,
5 the change in magnetic flux
= ´ 525 = 437.5 J
6
= 2 ´ (1.2 ´ 10-5 ) = 2.4 ´10 -5 weber
24. (1.5 × 10–3) Induced emf, e = Bvl
= 0.3 × 10–4 × 5 × 10 Induced e.m.f.
= 1.5 × 10–3 V df 2.4 ´ 10 -5
=N = 100 ´ = 0.048 V.
25. (7.85 × 10–2) The magnetic flux, dt 0.05
fB = BA = B × pr2
Df B B ´ pr 2 BV
The induced emf, |e| = = 29. (0.173) We have, tan q =
BH
Dt Dt
0.50 ´ p(0.05) 2 \ BV = BH tan q
=
0.50 = 2 × 10–5 × tan 60°
= 7.85 × 10–2 V
= 2 3 ´10-5 T
26. (9.3 × 10–2) The resistance of the wire
The induced emf, e = Bvvl
rl (1.7 ´ 10-8) ´ (0.40)
R= 2 = = (2 3 ´ 10-5 ) ´ 250 ´ 20
pr p(10 -3)2
= 2.16 × 10–3 W = 0.173 V
The area of the loop = 0.10 × 0.10 = 10–2 m2. B2 v 2 l 2
30. (9 × 10–3) Power =
R
æ dB ö
The induced emf, |e| = A ç ÷ 0.5 ´ 0.5 ´ 12 ´ 12 ´ 15 ´ 15 ´ 10-8
è dt ø =
9 ´ 10-3
= 10–2 × (0.02) = 2 × 10–4 V = 9 × 10–3 watt

@ebooks_for_freee on Telegram
Alternating Current 175

CHAPTER

Alternating Current
21
1. (d) I = 2 sin wt 5. (b) R = XL = 2XC
p
V = 5cos wt = 5sin æç - wt ö÷ Z = R 2 + (X L - X C ) 2
è2 ø
p = (2X C )2 + (2X C - X C ) 2
Since, there is a phase difference of between
2 XL–XC
the current and voltage = 4X 2C 2
+ XC
Z
\ Average power over a complete cycle is zero.
2. (d) 5R
= 5X C =
3. (d) Pure resistor 2 f
R

R X - XC 2XC - XC
tan f = L =
R 2XC
V -1 æ 1 ö
Vs Þ f = tan çè ÷ø
2
L-R series circuit
1
R L 6. (c) P = Vr.m.s ´ I r.m.s ´ cos f = V0 I 0 cos f
2
1
V
= ´100 ´ (100 ´ 10-3 ) cos p / 3
V 2
For pure resistor circuit, power = 2.5 W
V2 7. (c) Frequency f =
1
P= Þ V 2 = PR
R 2p LC
Phasor diagram 1
R = ; 23Hz
q 2 ´ 3.14 24 ´ 2 ´ 10 -6
R 8. (d) The transformer converts A.C. high voltage
Z XL cos q =
Z into A.C. low voltage, but it does not cause any
change in frequency. The formula for voltage is
Z = impedance
For L-R series circuit, power Ns 5000
Es = × Ep = ´ 20 = 200V
2 Np 500
V2 V 2 R PR æ Rö
P1 = cos q = . = .R =Pç ÷ Thus, output has voltage 200 V and frequency
Z Z Z Z2 è Zø
50 Hz.
4. (c) Impedance at resonant frequency is 9. (d) In case of series RLC circuit,
minimum in series LCR circuit. Equation of voltage is given by
V 2 = VR2 + (VL - VC )
2 2
æ 1 ö
So, Z = R 2 + ç 2pfL - ÷
è 2pfC ø Here, V = 220 V; VL = VC = 300 V

@ebooks_for_freee on Telegram
176 PHYSICS
2
\ VR = V 2 = 220V 2
L
2 +w
R XL= w L
V 220
Current i = = = 2.2A f
R 100
R
Vs Ns
10. (a) Since = Net impedance of circuit Z = X L2 + R 2
Vp N p
Where R
Power factor, cos f =
Np = 50 Ns = 1500 Z
R
df d Þ cos f =
and Vp = = (f0 + 4t) = 4
dt dt R2 + w 2 L2
1500 15. (d) As V(t) = 220 sin 100 pt
Þ Vs = ´ 4 = 120 V
50 220
11. (a) Quality factor, so, I(t) = sin 100 pt
50
1 L 1 80 ´ 10 -3 i.e., I = Im = sin (100 pt)
Q= = For I = Im
R C 100 2 ´ 10-6
1 200 p 1 1
= 40 ´ 103 = =2 t1 = ´ = sec.
100 100 2 100 p 200
12. (a) Given, Inductance, L= 40 mH Im
and for I =
Capacitance, C = 100 mF 2
Impedance, Z = XC – XL Im p
Þ = I m sin(100 pt 2 ) Þ = 100 pt 2
1 æ 1 ö 2 6
Þ Z= – wL çQ X c = and X L = wL ÷
wC è wC ø 1
Þ t2 = s
1 600
= – 314 ´ 40 ´ 10 –3
314 ´ 100 ´ 10 –6 1 1 2 1
\ treq = - = = s = 3.3 ms
= 19.28W 200 600 600 300
V0 16. (a) The current (I) in LR series circuit is given
Current, i = sin(wt + p / 2) by
Z
Væ – ö
tR
10
Þ i= cos wt = 0.52 cos (314 t) I= ç1 – e L ÷
19.28 R çè ÷ø
P VI At t = ¥,
13. (b) Efficiency, h= out = s s
Pin VpIp æ –¥ ö
20
I¥ = ç I – e L/ R ÷ = 4 ...(i)
230 ´ Is 5 çè ÷ø
Þ 0.9 =
2300 ´ 5
At t = 40s,
Þ Is = 0.9 ´ 50 = 45A æ –40 ´ 5 ö
çè1 – e = 4(1 – e –20,000 ) ...(ii)
Output current = 45A –3 ÷ø
10 ´ 10
14. (b) Resistance of the inductor, XL = wL Dividing (i) by (ii) we get
The impedance triangle for resistance (R) and I¥ 1
inductor (L) connected in series is shown in the Þ = ,
I 40 1– e –20,000
figure.

@ebooks_for_freee on Telegram
Alternating Current 177
w0 w L C = 120 mf = 120 × 10–6f
17. (a) Quality factor Q = = 0
2Dw R
18. (a) Current in inductor circuit is given by, 1 1
xC = = = 26.52W
( -
i = i0 1 - e L
Rt
) wC 100p´120 ´10-6
xL = wL = 100 p × 20 × 10–3 = 2 pW
i0 -
Rt
-
Rt xC – xL= 20.24 » 20
1
= i0 (1 - e L ) Þ e L = R = 60W
2 2 f
Taking log on both the sides,
Rt
- = log1 - log 2
L
Z
L 300 ´ 10 -3
Þ t= log 2 = ´ 0.69 z = R + ( xC – xL )
2 2
R 2
Þ t = 0.1 sec.
z =20 10W
19. (c) Given: R = 60W, f = 50 Hz, w = 2 pf = 100 p
and v = 24v R 60 3
cos f= = =
C = 120 mf = 120 × 10–6f z 20 10 10
1 1 v v2
xC = = = 26.52W Pavg = VI cos f, I = = cos f = 8.64 watt
wC 100p ´ 120 ´ 10 -6 z z
xL = wL = 100 p × 20 × 10–3 = 2 pW Energy dissipated (Q) in time t = 60s is
xC – xL= 20.24 » 20 Q = P.t = 8.64 × 60 = 5.17 × 102J
R = 60W Pout Vs I s
f 22. (45) Efficiency, h= =
Pin V p I p
230 ´ I s
Þ 0.9 =
2300 ´ 5
Z
Þ I s = 0.9´ 50 = 45 A
z = R 2 + ( xC – x L )
2

Output current = 45A


z =20 10W 23. (3.3) As V(t) = 220 sin 100 pt
R 60 3 220
cos f= = = so, I(t) = sin 100 pt
z 20 10 10 50
v v2 i.e., I = Im = sin (100 pt)
Pavg = VI cos f, I = = cos f = 8.64 watt
z z For I = Im
Energy dissipated (Q) in time t = 60s is p 1 1
Q = P.t = 8.64 × 60 = 5.17 × 102J t1 = ´ = sec.
2 100p 200
V 100
20. (c) Across resistor, I = = = 0.1 A and for I = m
I
R 1000
At resonance, 2
1 1 Im
Þ = I m sin(100 pt2)
X L = XC = = = 2500 2
wC 200 ´ 2 ´ 10-6
Voltage across L is p 1
Þ = 100 pt2 Þ t 2 = s
I X L = 0.1 ´ 2500 = 250 V 6 600
1 1 2 1
21. (5.17 × 102) Given: R = 60W, f = 50 Hz, w = 2 pf \ t req = - = = s = 3.3 ms
200 600 600 300
= 100 p and v = 24v

@ebooks_for_freee on Telegram
178 PHYSICS

24. (63 × 10–9) For the maximum current, 6.52 ´10


\ P= = 0.625W
XC = XL 262
1 28. (5) From Kirchoff’s current law,
or = wL
wC
1 1 i3 = i1 + i2 = 3sin wt + 4sin (wt + 90°)
\ C = w2 = p 2
L (2 f ) L Þ i3 = i0 sin (wt + f)
1
=
(2p ´ 2 ´ 103) 2 ´ 100 ´10 -3 where i0 = 32 + 42 + 2(3)(4)cos90°
= 63 × 10–9 F 4sin 90° 4
and tan f = =
V 160 3 + 4 cos90° 3
25. (125) The impedance, Z = rms = = 80 W
i 2 \ i3 = 5 sin (wt + 53°)
We know that,Power,
1 2 25
29. (25) f = = = Hz
V2 R 2p LC 2p 5 ´ 80 ´10-6 p
P = rms2
Z
X L wL
30. (2) If first case,tan 60° = =
1602 ´ R R R
or 200 =
802
300L
\ R = 50 W or 3 = 100
We know that, Z = R 2 + X L2 \ L = 0.58 H

or 80 = 502 + X L 2 XC 1
In second case, tan 60° = =
R wCR
\ XL = 62.5 W
1
The back emf, VL = iXL = 2 × 62.5 = 125 V. or 3 = 300C ´ 100
26. (100) XL = wL = (2p × 500) × 8.1 = 25.4 W
\ C = 19.2 µF
1 1
and X C = = The impedance of the circuit is
wC (2p´ 500) ´ (12.5 ´10 -6)
= 25.4 W 2
R 2 + æç wL -
1 ö
As X L = X C , so resonance will occur and Z= ÷
è wC ø
VR = 100V.
27. (0.625) Average power consumed is given by
2
V2 R æ 1 ö
= 100 2 + ç 300 ´ 0.58 - ÷
P = rms2 , è 300 ´19.2 ´10 -6 ø
Z
where Z = R 2 + (wL)2 = 100 W

2 30 V 200
= 10 + (2p´ ´ 0.4) 2 = 26 W Current, i = = = 2A
p Z 100

@ebooks_for_freee on Telegram
Electromagnetic Waves 179

CHAPTER

Electromagnetic Waves
22
1. (d) Direction of propagation of electro- r
\ E should be along y-direction.
magnetic waves is perpendicular to Electric field r r
and Magnetic field. Hence, direction is given by So, electric field E = E0 sin E × ( x, t )
ur ur
ur ur uur E ´ B V
Poynting vector S = E ´ H = . = [ -36 sin (0.5 ´ 103 x + 1.5 ´ 1011 t ) ˆj ]
m0 m
12. (d) Average energy density of magnetic field,
E E 24
2. (c) =c Þ B= = = 8 ×10 –8 T B02
B c 3 ´ 108 uB =
4m 0
.
3. (c)
4. (d) Ultravoilet radiations are used in the Average energy density of electric field,
detection of invisible writing, forged
documents, finger prints in forensic lab. While e0 E 20
uE =
microwaves are used in microwave oven. 4
5. (d) Intensity of EM wave is given by
P 1 P 1
I= = Uav.c = e0E02 ´ c Þ E0 = Now, E0 = CB0 and C2 =
m0 Î0
4pR 2 2 2pR2e0c
800 e0 2
= uE = ´ C 2 B 20 = e0 ´ 1 ´ B20 = B0 = u B
2 -12 8 4
2 ´ 3.14 ´ (4) ´ 8.85 ´ 10 ´ 3 ´ 10 4 m 0 e0 4m 0
\ uE = u B
V Since energy density of electric and magnetic
= 54.77
m field is same, so energy associated with equal
6. (a) Both magnetic and electric fields have zero volume will be equal i.e., uE = uB
average value in a plane e.m. wave. 13. (c) Given: Amplitude of electric field,
7. (a) E0 = 4 v/m
8. (d) The power per unit area carried by an E.M. Absolute permitivity, e0 = 8.8 × 10–12 c2/N-m2
wave i.e., energy transported per unit time across Average energy density uE = ?
a unit crossection area is perpendicular to the Applying formula,
direction of both electric and magnetic field i.e. 1 2
in the direction of which the wave is travelling. Average energy density uE = e0 E
4
9. (a) 1 -12 2
10. (d) G > X > U > V > I > M > R [frequency order] Þ uE = ´ 8.8 ´ 10 ´ (4) = 35.2 × 10–12 J/m3
reverse is true for wavelength 4
14. (b)
11. (a) Relation between electric field E 0 and
15. (b) Q The E.M. wave are transverse in nature
magnetic field B0 of an electromagnetic wave is
i.e.,
given by r r
E k ´E r
c= 0 = =H …(i)
B0 (Here, c = Speed of light) m
r
r B
Þ E0 = B0 ´ c = 1.2 ´ 10-7 ´ 3 ´ 108 = 36 where H =
As the wave is propagating along x-direction, m
magnetic field is along z-direction r r
k ´H r
and = -E …(ii)
and ( Eˆ ´ Bˆ ) || Cˆ we

@ebooks_for_freee on Telegram
180 PHYSICS
r r r r 23. (1.4) EM wave intensity
k is ^ H and k is also ^ to E
The
r direction
r of wave propagation is parallel to Power 1
E ´ B. Þ I= = e 0 E 02 c
Area 2
The direction of polarization is parallel to electric [where E0= maximum electric field]
field.
16. (b) The orderly arrangement of different parts 27 ´ 10 –3 1
Þ = ´ 9 ´ 10 –12 ´ E 20 ´ 3 ´ 108
of EM wave in decreasing order of wavelength 10 ´10 –6 2
is as follows:
l radiowaves > l microwaves > l visible > l X-rays Þ E0 = 2 ´103 kV / m =1.4kV / m

hc hc 6.6´10-34 ´3´108 B20 B2 Iµ


17. (a) E = Þl = Þl = 24. (6 × 10–4) I = ·C Þ 0 = 0
l E 11´1000´1.6 ´10-19 2µ0 2 C
= 12.4 Å
Increasing order of frequency Þ Brms =
Iµ0 108 ´ 4 p ´ 10 -7
=
C 3 ´ 108
x-rays u-v rays visible Infrared
wavelength range of visible region is 4000Å to ; 6 × 10–4 T
7800Å.
25. (0.64) B0 = B02 + B12 = 302 + 2 2 ´ 10-6
B20 B20 Iµ0
18. (d) I= ·C Þ = » 30 × 10–6T \ E0 = cB = 3 ´108 ´ 30 ´10 -6
2µ0 2 C
= 9 × 103 V/m
8 -7
Iµ0 10 ´ 4 p ´ 10 E0 9
Þ Brms = = Erms= = ´103V / m
C 3 ´ 108 2 2
; 6 × 10–4 T Force on the charge,
Which is closest to 10–4.
9
E, Decreases F = ErmsQ = ´ 103 ´10-4 ; 0.64N
19. (c) 2
g-rays X-rays uv-rays Visible rays IR rays Radio
VIBGYOR Microwaves waves
Radio wave < yellow light < blue light < X-rays 26. (20 × 10–8) F = (1 + r) IA
C
(Increasing order of energy)
20. (a) Frequency range of g-ray, (1 + 0.25) ´ 50 ´1
= 8 ; 20 ´ 10 -8 N
b = 1018 – 1023 Hz 3 ´10
Frequency range of X-ray, a = 1016 – 1020 Hz 27. (4.8 × 10–7) E0 = 300 V/m,
Frequency range of ultraviolet ray,
\ B0= E0 = 300 = 1´ 10-6 N/A-m
c = 1015 – 1017 Hz \ a < b; b > c C 3 ´108
21. (2.1) As we know, The maximum electric force,
r
r |E| 6.3 F0 = E0q = 300 × 1.6 × 10–19 = 4.8 × 10–7 N.
| B|= = = 2.1 ´10-8 T and Eˆ ´ B
ˆ =C
ˆ
28. (1.33 × 10–10)
C 3 ´ 108
29. (6) E0 = B0C = 20 × 10–9 × 3 × 108 = 6 v/m
ˆ = ˆi [Q EM wave travels along +(ve)
Jˆ ´ B I
x-direction.] 30. (5 × 10–3) Pressure, P =
r C
\ B̂ = kˆ or B = 2.1´10–8 kT
ˆ
F I IA Dp I
Þ = Þ F= = Þ Dp = ADt
22. (3 × 104 N/C) Using, formula E0 = B0 × C A C C Dt C
= 100 × 10–6 × 3 × 108 = 3 × 104 N/C (25 ´ 25) ´104 ´10 –4 ´ 40 ´ 60
= N-s
Here we assumed that 3 ´ 108
B0 = 100 × 10–6 is in tesla (T) units = 5 × 10–3 N-s

@ebooks_for_freee on Telegram
Ray Optics and Optical Instruments 181

CHAPTER
Ray Optics and Optical
23 Instruments
1. (d) Let d be the depth of two liquids. 1
sin ic =
Then apparant depth n
1 1 1
( d / 2) ( d / 2) d \ sin 45° > or > Þn> 2
+ = or 1 + 2 = 1 n 2 n
m 1. 5 m 2 m 3m 5. (b) Magnifying power of telescope is
Solving we get m = 1.671
f0 225
| P1 | 2 M= = Þ M = 45 cm.
f
2. (a) = Þ 2 =2 ...(i) fe 5
| P2 | 3 f1 3
6. (b) From the fig.
Focal length of their combination Angle of deviation,
1 1 1 1 1 1´ 3 d = i+e- A A
= - Þ = -
f f1 f 2 30 f1 2 f1 d
Here, e = i i e
from (i)
3
1 1 é 3ù 1 æ 1ö and e = A
Þ = ê1 - ú = ´ ç - ÷ 4
30 f1 ë 2 û f1 è 2 ø
3 3 A
\ f1 = – 15 cm and \d= A+ A– A =
4 4 2
2 2
\ f2 = ´ f1 = ´ 15 = 10 cm For equilateral prism, A = 60°
3 3
3. (c) The focal length(F) of the final mirror is 60°
\d= = 30°
2
1 2 1
= +
F fl f m
//////////////

////

7. (b)
/////// //////////

24cm.
1 æ 1 1ö
= ( µ - 1) ç - ÷
///

Here
fl è R1 R2 ø Object Image
8cm.
40cm.
é1 1 ù 1 1 1 1 1 1 1 1 4 +1
= (1.5 - 1) ê - =
ë ¥ -30 úû 60 = + Þ
f u v
=
10 -40 v
+ Þ
v
=
40
1 1 1 1 Þ v = 8 cm.
\ = 2´ + = \ F = 10 cm
F 60 30 / 2 10 Hence, plane mirror must be at 24cm.
The combination acts as a converging mirror. V 2.2 ´ 108 m / sec 11
For the object to be of the same size of mirror, 8. (b) Sin C = P = =
VQ 2.4 ´ 108 m / sec 12
u = 2F = 20 cm
4. (b) Incident angle > critical angle, i > ic æ 11 ö
Þ C = sin -1 ç ÷
è 12 ø
\ sin i > sin ic or sin 45 > sin ic

@ebooks_for_freee on Telegram
182 PHYSICS

Dividing (i) by (ii)


9. (b)
fl (1.5 –1)1.42 1.42 142
Þ = = = »9
Combination f 0.08 0.16 16
Þ m2 v
m1 12. (b) Using, M <
u
Plano-convex Plano-concave v1
or ,2 < Þ v1 < ,2 x1
x1
1 1 1
= + 1 1 1
f f1 f2
We have v , u < f
æ1 1 ö æ 1 1ö 1 1 1
= (m1 – 1) ç - + (m2 – 1) ç - ÷ , <
è ¥ -R ÷ø è –R ¥ ø or ,2 x1 x1 20
R x1 = 30 cm
Solving we get, f = 1 1 1
m1 - m 2 , <
And
2 x2 x2 20
10. (d) The deviation produced as light passes
through a thin prism of angle A and refractive or x2 = – 10 cm
index m is d = A(m – 1). We want deviation x1 30
produced by both prism to be zero. So, x < 10 < 3
2
d = d' 13. (c) When two thin lenses are in contact
Þ A(m - 1) = A '(m '- 1) coaxially, power of combination is given by
P = P1 + P2
4 ´ (1.54 - 1) = (– 15 + 5) D
Þ A' = = 4 ´ 0.75 = 3°
(1.72 - 1) = – 10 D.
11. (b) Using lens maker’s formula 1
Also, P =
f
1 æ mg öé 1 1 ù 1 1
=ç –1÷÷ ê – ú Þ f = = metre
f çè ma R
øë 1 R2û P -10
æ1 ö
Here, mg and ma are the refractive index of glass \ f = - ç ´ 100÷ cm = -10 cm.
è 10 ø
and air respectively
14. (d) From the given figure
1 æ 1 1 ö As sin 60o = m sin 30o
Þ = (1.5 –1) ç – ÷ ...(i)
f è R1 R2 ø
sin 60o
When immersed in liquid Þm= = 3
sin 30o
1 æ mg öæ 1 1 ö a
= cos 60o Þ AO = 2a
= çç –1÷÷ ç – ÷
fl è m l AO
ø è R1 R2 ø
b 2b
[Here, ml = refractive index of liquid] = cos 30o Þ BO =
BO 3
1 æ 1.5 öæ 1 1 ö Optical path length = AO + mBO
Þ =ç –1÷ ç – ÷ ...(ii)
fl è 1.42 ø è R1 R2 ø
2b
= 2a + ( 3) = 2a + 2b
3

@ebooks_for_freee on Telegram
Ray Optics and Optical Instruments 183
15. (a) According question, M = 375
1 1 1
L = 150 mm, f0 = 5 mm and fe = ? Þ = -
v0 1.2 1.25
L æ Dö Þ v0 = 30 cm
Using, magnification, M ; f ç 1 + f ÷
0è eø Magnification at infinity,

150 æ 250 ö v0 D
Þ 375 = M¥ = - ´
ç1 + ÷
5 è f e ø (Q D = 25 cm = 250 mm) u0 f e

250 30 25
Þ 12.5 = 1 + = ´
fe 1.25 3
(Q D = 25 cm least distance of distinct vision)
250 = 200
Þ fe = = 21.7 » 22 mm
11.5 Hence the magnifying power of the compound
16. (d) Given, using lens maker's formula microscope is 200
æ 1 18. (b) Velocity of light in medium
1 1ö
= (k - 1) ç - ÷
f è 1
R R2ø 3cm 3 ´ 10-2 m
Vmed = = = 1.5 m/s
Here, R1 = R2 = R (For double convex lens) 0.2ns 0.2 ´ 10-9 s
Refractive index of the medium
1 æ1 1 ö
\ = (m - 1) ç -
f è R - R ÷ø Vair 3 ´108
m= = = 2 m/s
Vmed 1.5
1 2
ÞP= = (m - 1) ...(i)
f R 1
As µ =
For plano convex lens, sin C
R1 = R ', R2 = ¥ 1 1
Using lens maker's formula again, we have \ sin C = = = 30°
m 2
æ 1 1ö Condition of TIR is angle of incidence i must be
1.5 P = ( m - 1) ç - ÷ ...(ii)
è R ' ¥ø greater than critical angle. Hence ray will suffer
TIR in case of (B) (i = 40° > 30°) only.
3 m -1
Þ P= 19. (b) The number of images formed is given by
2 R'
From (i) and (ii), 360
n= -1
q
3 R' R
= Þ R' =
2 2R 3 360
Þ -1 = 3
17. (a) Given : f0 = 1.2 cm; fe = 3.0 cm q
u0 = 1.25 cm; M¥ = ? 360°
Þ q= = 90°
1 1 1 4
From = - 20. (a) When angle of prism is small,
f 0 v0 u0
Angle of deviation, D = (m – 1) A
1 1 1 Since lb < lr
Þ = -
1.2 v0 ( -1.25) Þ mr < mb Þ D1 < D2

@ebooks_for_freee on Telegram
184 PHYSICS

21. (10) For lens 24. (8.8) If v is the distance of image formed by
mirror, then
1 1 1
+ =
v u f
O I
1 1 1
or + =
v -5 -20
30 cm 80 cm
20
1 1 1 \ v= cm
- = 3
v u f
Distance of this image from water surface
1 1 1
or - = 20 35
v -30 20 = + 5 = cm
3 3
\ v = + 60 cm
RD
According to the condition, image formed by Using, =µ
AD
lens should be the centre of curvature of the
mirror, and so RD (35 / 3)
\ AD = d = m = 1.33 = 8.8 cm
2f’ = 20 or f’ = 10 cm
25. (57) If i and r are the angle of incident and angle
22. (57000) Using Snell’s law of refraction,
of refraction respectively, then
1 × sin 40° = 1.31 sin q i + r = 90°
0.64 \ r = 90° – i
Þ sin q = = 0.49 » 0.5
1.31
Þ q = 30°
i i 90°
20 mm
r
q m = 1.5
40° x
x = 20 µm × cot q
2 sin i
\ Number of reflections = By Snell's law, =µ
sin r
20 ´10-6 ´ cot q
sin i
2 ´106 or sin(90° – i) = µ
= = 57735 » 57000
20 ´ 3
sin i
v or =µ
23. (0.32) +5 = - Þ v = -5u cos i
u
or tan i = µ
1 1 1 1 1 -1 \ i = tan–1 (m) = tan–1 (1.5)
Using v + u = f Þ + =
-5u u 0.4 ; 57°
\ u = – 0.32 m. 26. (2) Here ÐMPQ + ÐMQP = 60°. If ÐMPQ = r
then ÐMQP = 60 – r

@ebooks_for_freee on Telegram
Ray Optics and Optical Instruments 185
Applying Snell’s law at P dq 1
=
sin60° = n sin r ...(i) dn cos 60° [sin 30° + cos 30° tan 30°] = 2
Differentiating w.r.t ‘n’ we get
Læ Dö
dr 27. (2.5) M = – f ç 1 + f ÷
O = sin r + n cos r × ...(ii) 0è eø
dn
20 æ 25 ö
or – 40 = - ç1 + ÷
5 è fe ø
60° or fe = 2.5 cm
28. (2.5) For the object O,
P Q u = – (PO)
60° r q
60°–rr = – 3 cm
M By refraction formula,
m2 m1 m2 - m1
- = , we have
v u R
Applying Snell’s law at Q
µ1 = 1.5, µ2 = 1, and R = – 5cm
sin q = n sin (60° – r) ...(iii)
Differentiating the above equation w.r.t ‘n’ we 1 1.5 1 - 1.5
\ - =
get v –3 –5
Þ v = –2.5 cm
dq é dr ù
cos q = sin (60° – r) + n cos (60° – r) ê – dn ú 29. (30) For concave lens, u = + 10 cm
dn ë û
(virtual object)
dq and v = + 15 cm
\cos q = sin (60° – r) – n cos (60° – r)
dn
1 1 1
We have - = f
é tan r ù +15 +10
ê– n ú [from (ii)]
\ f = –30 cm.
ë û
30. ( 2 )
dq 1 30°
\ = [sin (60° – r) + cos (60° – r) tan r]
dn cos q
...(iv)
45° 30°
From eq. (i), substituting n = 3 we get r = 30°

From eq (iii), substituting n = 3 , r = 30°


sin 45° 1/ 2
we get q = 60° µ= = = 2.
sin 30° 1/ 2
On substituting the values of r and q in eq (iv)
we get

@ebooks_for_freee on Telegram
186 PHYSICS

CHAPTER

Wave Optics
24
p amplitude = A = 2acos(f/2).
1. (b) f= , a = 4, a2 = 3
3 1 [Since, a 2 = a 12 + a 22 + 2a1a 2 cos f , here a1 = a2]
So, A = a12 + a 22 + 2a1a 2 cos f » 6 Intensity I,
2. (b) Here, D = 1.25m, mw = 4/3, qw = 0.2º æ px ö
Ι = kA 2 = k(4a 2 ) cos 2 ( f / 2) = Ι 0 cos 2 ç D
l 4 è b ø÷
mw = a = ...(i) æ p xd ö æ px ö
lw 3 = Ι 0 cos 2 ç . ÷ = Ι0 cos 2 ç ÷
èl Dø è bø
b (l a D / d) l a
Angular width qa = = = 5. (c) m = tan i
D D d
As d remains the same Þ i = tan -1 (m) = tan -1 ( 3) = 60°.
q l l 4 6. (c) Q ip = f, therefore, angle between reflected
\ a = a or qa = qw ´ a = 0.2º ´ = 0.27º
qw lw lw 3 and refracted rays is 90º.
3. (c) Let µ1 be refractive index of denser 4
7. (c) Given, refractive index, m =
medium and that of rarer medium be µ2 for 3
total internal reflection, According to Brewster’s law when unpolarised
m light strikes at polarising angle ip on an interface
m1 sin qiC = m 2 Þ sin qiC = 2
m1 then reflected and refracted rays are normal to
Brewsters angle each other and is given by :
æm ö m æ 4ö
qiB = tan -1 ç 2 ÷ Þ tan qiB = 2 tan ip = m \ i p = tan -1 ç ÷
è m1 ø m1 è 3ø
Þ sin qiC = tan qiB 8. (d) Initially, S2 L = 2 m
sin qiC tan qiB 2
= 1.28 Þ = 1.28 æ3ö 5
sin qiB sin qiB S1 L = 22 + ç ÷ = = 2.5 m
è2ø 2
1
Þ cos qiB = l
1.28 Path difference, Dx = S1 L - S 2 L = 0.5 m =
Relative refractive index 2
1 L'
1- /c
m
= 2 = tan qiB =
sin qiB
= 1.282 2N
m1 cos qiB 1 S2
2 N/c L
1.28
= 0.624 × 1.28 = 0.8 d
d
4. (c) D = x , where D is path difference
D S1
between two waves.
2p When the listner move from L, first maxima will
\ phase difference = f = D.
appear if path difference is integral multiple of
l
Let a = amplitude at the screen due to each slit. wavelength.
\ I0 = k (2a)2 = 4ka2, where k is a constant. For example
For phase difference f,
Dx = nl = 1l (n = 1 for first maxima)

@ebooks_for_freee on Telegram
Wave Optics 187

\ Dx = l = S1 L '- S 2 L Intensity of light from the 4th polaroid is


2
Þ1= d - 2 Þ d = 3 m I0 æ 1 ö I0
I4 = I3 cos 2 60° = ç ÷ =
9. (d) The resolving power of an optical 32 è 2 ø 128
instrument is inversely proportional to the Intensity of light from 5th polaroid is
wavelength of light used. I0 æ 1 ö
2
I0
I5 = I4 cos 2 60° = ç ÷ =
( R.P )1 l 2 5 128 è 2 ø 512
= =
( R.P )2 l1 4 13. (d) If unpolarised light is passed through a
10. (b) According to Brewster’s law, refractive in- polaroid P1, its intensity will become half.
dex of material (m) is equal to tangent of polarising I
angle So I1 = 0
2
1.5 Now this light will pass through the second
Qtan i b = μ =
μ polaroid P2 whose axis is inclined at an angle of
1 1.5 30° to the axis of P1 and hence, vibrations of I1.
< (Qsin ic < sin ib ) So in accordance with Malus law, the intensity
μ μ + (1.5)
2 2
of light emerging from P2 will be
1.5 2
\sin i b = æ 1 ö æ 3ö 3
2
μ + (1.5) I2 = I1 cos 30° = ç I0 ÷ ç ÷ = I0
2 2

è2 ø è 2 ø 8
or, μ 2 + (1.5 ) <1.5´ μ
2
So the fractional transmitted light
I2 3
Þ μ 2 + (1.5) < ( μ×1.5)
2 2
´ 100 = ´ 100 = 37.5%
I0 8
3 3
Þμ < i.e. minimum value of m should be 14. (c) Dx = ( SS1 + S10 ) - ( SS2 + S20 )
5 5
l
11. (a) kxDx + p = (2n + 1)p Þ kDx = 2np or = 2 D2 + d 2 - 2 D
2
2np 2np lD
Þ Dx = = ´ l = nl \ d=
k 2p 2
x 15. (d) Conditions for diffraction minima are
Dx = nl but Dx = BC + CD = = 3x Path diff. Dx = nl and Phase diff. df = 2np
3
Path diff. = nl = 2l
3x Phase diff. = 2np = 4p (Q n = 2)
So, 3x = nl Þ l =
n 16. (c)
for n = 1 17. (b) a = 0.1 mm = 10–4 cm,
l = 3x l = 6000 × 10–10 cm = 6 × 10–7 cm, D = 0.5 m
12. (d) Let I0 be the intensity of incident light. Then for 3rd dark band, a sin q = 3 l
the intensity of light from the 1st polaroid is 3l x
or sin q = =
I0 a D
I1 = The distance of the third dark band from the
2
central bright band
Intensity of light from the 2nd polaroid is
I0 æ 1 ö I0
2 3 lD 3 ´ 6 ´ 10-7 ´ 0.5
x= = = 9 mm
I2 = I 1cos2 60° = çè ÷ø = a 10-4
2 2 8
Intensity of light from the third polaroid is : 18. (a)
2 19. (b)
I0 æ 1 ö I0
I3 = I2 cos2 60° = ç ÷ = 20. (c) For first minimum, a sin q = nl = 1l
8 è2ø 32

@ebooks_for_freee on Telegram
188 PHYSICS

l 5000 ´ 10 –10 27. (3) For maxima


sin q = = = 0.5
a 0.001 ´ 10 –3 Path defference = ml
q = 30° \ S2A – S1A = ml
21. (641) For 'n' number of maximas S1
d sin q = nl
0.32 ´ 10–3 sin 30° = n ´ 500 ´ 10–9
d x 2 + d2
0.32 ´ 10 -3 1
\ n= -9
´ = 320
500 ´ 10 2 x A
Hence total no. of maximas observed in angular
range – 30° £ q £ 30°
= 320 + 1 + 320 = 641
l S2
22. (0.85) Given, path difference, Dx =
8 é 2 2 2 2ù 2 2
Phase difference (Df) is given by \ ê (n - 1) d + x + d + x ú – d - x = ml
ë û
2p \ (n – 1) (d 2 + x 2 ) = ml
Df = ( Dx)
l
æ4 ö 2 2
\ ç 3 –1÷ d + x = ml
Df =
( 2p ) l = p è ø
l 8 4 \ d 2 + x 2 = 3ml
For two sources in different phases, \ d2 + x2 = 9m2l2
\ x2 = 9m2l2 – d2
æ pö
I = I0 cos 2 ç ÷ \ p2 = 9 Þ p = 3
è 8ø
28. ( 28 ) The resultant amplitude is given by
I æ pö
= cos2 ç ÷ R = ( a12 + a22 + 2a1a2 cos f)1/ 2
I0 è 8ø
p 1+ 1 = (22 + 4 2 + 2 ´ 2 ´ 4cos60°)1/ 2
1 + cos
= 4 = 2 = 0.85
2 2 = 28 .
23. (5)
1.22l 1.22 ´ 600 ´10-9
1.22l 1.22 ´ 500 ´ 10 -9 29. (3 × 10–7) q= =
24. (305 × 10–9) q = = d 250 ´10-2
d 2
–9 = 3.0 × 10–7 rad
= 305 × 10 rad.
30. (20) The distance of nth maxima from central
1.22l maxima is given by
25. (0.24) x=
2µsinq Dl
yn = n ,
= 0.24 µm d

26. (3) Dxmax = 0 and Dxmax = 2 l For yn to be constant, nl = constant. Thus


n1l1 = n2l2
Theortical maximas are = 2n + 1 = 2 × 2 + 1 = 5
But on the screen there will be three maximas. n1l1 16 ´ 6000
\ n2 = = = 20
l2 4800

@ebooks_for_freee on Telegram
Dual Nature of Radiation and Matter 189

CHAPTER
Dual Nature of Radiation
25 and Matter
h 1 h
1. (a) According to De-broglie p = or P µ Initial wavelength, l 0 =
l l mv0 2
1
Pµ represents rectangular hyperbola. Final wavelength,
l
2. (a) Ek = E – f0 = 6.2 – 4.2 = 2.0 eV, h
l=
Ek = 2 × 1.6 × 10–19 = 3.2 × 10–19 J æ eE t ö
2
m 2v 2 +ç 0 ÷
0
1 è m ø
3. (d) Iµ 2
d l 1
=
4. (b) Photoelectrons are emitted in A alone. l0 2
æ eE0t ö
Energy of electron needed if emitted from 1+ ç
ç 2mv ÷÷
hu è 0ø
A= eV
e l0

\ EA =
( )(
6.6 ´ 10-34 ´ 1.8 ´ 1014 ) = 0.74 eV Þl=
1+
e 2 E02t 2
1.6 ´10-19 2m 2 v02

EB =
( 6.6 ´10-34 ) ´ ( 2.2 ´1014 ) = 0.91 eV 7. (b) P1 – P2 = (P1 + P2) = P
1
1.6 ´10-19 As P µ
Incident energy 0.825 eV is greater than EA l
(0.74 eV) but less than EB (0.91 eV). 1 1 1
or - =
5. (b) The momentum of the photon is energy/ lx l y l
speed of light. In black holes the gravity pull is l y - lx 1
so high that even photon cannot escapes. or =
lxl y l
6. (c) Given, Initial velocity, u = v0iˆ + v0 ˆj
8. (b) f0 = 4 × 1014 Hz
qE0 eE0 W0 = hf0 = 6.63 × 10–34 × (4 × 1014) J
Acceleration, a = =
m m
Using v = u + at (6.63 ´ 10-34 ) ´ (4 ´1014 )
=
eE0 ˆ 1.6 ´ 10 -19
v = v0 iˆ + v0 ˆj + tk
m = 1.66 eV
2 nE
r æ eE t ö 9. (c) using, intensity I =
\ | v | = 2v02 + ç 0 ÷ At
è m ø
n = no. of photoelectrons
h
de-Broglie wavelength, l = æ n ö 10 ´1.6 ´10
–19
n
p Þ16 ´10 –3 = ç ÷ ´ –4 or, = 10
12

t
è ø 10 t
h So, effective number of photoelectrons ejected
Þl= (Q p = mv)
mv per unit time = 1012 × 10/100 = 1011

@ebooks_for_freee on Telegram
190 PHYSICS

10. (a) From the de-Broglie relation, hc


12. (d) - f = eV0
h l
p1 =
l1 hc f
v0 = -
el e
h For metal A For metal B
p2 =
l2 fA 1 fB 1
= =
Momentum of the final particle (pf) is given by hc l hc l

\ pf = p12 + p22 1
As the value of (increasing and decreasing)
l
h h2 h 2 is not specified hence we cannot say that which
Þ = + metal has comparatively greater or lesser work
l l12 l 22 function (f).
13. (a) Work function, f = 6.2 eV = 6.2 ×1.6 × 10–19 J
1 1 1
Þ = + Stopping potential, V = 5 volt
l2 l12 l 22 From the Einstein’s photoelectric equation
11. (d) According to question, there are two EM hc
waves with different frequency, - f = eV0
l
B1 = B0 sin (p × 107c)t
hc
and B2 = B0 sin (2p × 107c)t Þl=
f + eV0
To get maximum kinetic energy we take the
6.6 ´ 10 -
photon with higher frequency 34 8
´ 3 ´ 10 -7
= -19
m » 10
w 1.6 ´ 10 (6.2 + 5)
using, B = B0 sin wt and w = 2 pv Þv =
2p This range lies in ultra violet range.
14. (d) From the Einstein photoelectric equation K.E.
10 7
B1 = B0sin (p × 107c)t Þ v1 = ´c = hn – f
2
Here, f = work function of metal
B2 = B0sin (2p × 107c)t Þ v2 = 107c h = Plank's constant
where c is speed of light c = 3 × 108 m/s slope of graph of K.E. & n is h (Plank’s constant)
Clearly, v2 > v1 which is same for all metals.
so KE of photoelectron will be maximum for 15. (c) According to photo-electric equation :
photon of higher energy.
K.Emax = hv – hv0 (Work function)
v2= 107c Hz
Some sort of energy is used in ejecting the
hv = f + KEmax
photoelectrons.
energy of photon 16. (c) Applying Einstein's formula for photo-
Eph = hv = 6.6 × 10–34 × 107 × 3 × 109 electricity
Eph = 6.6 × 3× 10–19J
1 2
hn = f + mv ; hn = f + K ; f = hn – K
6.6 ´ 3 ´10 –19
2
= eV =12.375eV
1.6 ´10 –19 If we use 2n frequency then let the kinetic energy
becomes K'
KEmax = Eph–f
So, h × 2n = f + K' Þ 2hn = hn – K + K'
= 12.375 – 4.7 = 7.675 eV » 7.7 eV
Þ K' = hn + K

@ebooks_for_freee on Telegram
Dual Nature of Radiation and Matter 191
17. (a) Given, l = 660 nm, Power = 0.5 kW, t = 60 ms 4hc hc 1 æ 4 1 ö
Þ - = 3f Þ f = hc ç - ÷
nhc plt l 2 l1 3 è l 2 l1 ø
Power P = Þn= = 1020
lt hc 1 æ 4 ´ 350 - 540 ö
18. (a) As we know, hu = hu0 + K.E.max = ´ 1240 ç ÷ = 1.8 eV
3 è 350 ´ 540 ø
hc hc 22. (7.7) According to question, there are two EM
or = + eVs waves with different frequency,
lincident l0
B1 = B0 sin (p × 107c)t
when lincident= l, Vs = 3V and for lincident = 2l, and B2 = B0 sin (2p × 107c)t
V5 = 1V.
To get maximum kinetic energy we take the
hc hc photon with higher frequency
hc hc = + 1eV ...(ii)
\ = + 3eV ...(i) and w
l l0 2l l 0
using, B = B0 sin wt and w = 2 pv Þv =
2p
On simplifying (i) and (ii)
10 7
2hc 1 hc B1 = B0sin (p × 107c)t Þ v1 = ´c
= Þ l 0 = 4l. 2
l0 2 l B2 = B0sin (2p × 107c)t Þ v2 = 107c
19. (b) By using hv – hv0 = Kmax where c is speed of light c = 3 × 108 m/s
Þ h (v1 – v0) = K1 ..... (i) Clearly, v2 > v1
And h(v2 – v0) = K2 ..... (ii) so KE of photoelectron will be maximum for
photon of higher energy.
v1 - v0 K1 1 kv - v v2= 107c Hz
Þ = = , Hence v0 = 1 2 .
v 2 - v0 K 2 K K -1 hv = f + KEmax
energy of photon
20. (b) E = W0 + Kmax Þ hf = WA + KA ...(i)
Eph = hv = 6.6 × 10–34 × 107 × 3 × 109
and 2hf = WB + KB = 2WA + KB ...(ii)
Eph = 6.6 × 3× 10–19J
æ WA 1 ö 6.6 ´ 3 ´10 –19
çèQ W = 2 ÷ø = eV =12.375eV
B 1.6 ´10 –19
KEmax = Eph–f
KA 1
Dividing equation (i) by (ii) = = 12.375 – 4.7 = 7.675 eV » 7.7 eV
KB 2
nE
21. (1.8) From Einstein's photoelectric equation, 23. (1011) Using, intensity I =
At
hc 1 n = no. of photoelectrons
f + m ( 2v )
2
l1 = 2
....(i)
æ n ö 10 ´1.6 ´10
–19
n
hc 1 2 Þ16 ´10 –3 = ç ÷ ´ –4 or, = 10
12

and = f + mv ....(ii) t
è ø 10 t
l2 2 So, effective number of photoelectrons ejected
As per question, maximum speed of per unit time = 1012 × 10/100 = 1011
photoelectrons in two cases differ by a factor 2 24. (1.45 × 106) de-Broglie wavelength,
From eqn. (i) & (ii)
h æ 3 ´ 108 ö é cù
l= = 10-3 ç
hc mv ÷
è 6 ´ 1014 ø êëQ l = v úû
-f
l1 hc 4hc
Þ =4Þ -f = - 4f 6.63 ´ 10-34 ´ 6 ´ 1014
hc l1 l2 v=
-f 9.1 ´ 10-31 ´ 3 ´ 105
l2
v = 1.45 × 106 m/s

@ebooks_for_freee on Telegram
192 PHYSICS

25. (1) Let f = work function of the metal, 28. (5 × 1015) Energy of photon (E) is given by
hc hc
=f+ eV1 ......(i) E=
l1 l
hc Number of photons of wavelength l emitted in
=f+ eV2 ......(ii)
l2 t second from laser of power P is given by
Sutracting (ii) from (i) we get
Pt l
æ1 1ö n=
hc ç – ÷ = e(V1 – V2 ) hc
è l1 l 2 ø
n=
2´ l 2 ´10-3 ´ 5 ´10-7
é ù Þ = (Q t = 1S)
ê ll = 300nm ú hc 2 ´10-25
ê ú Þ n = 5 × 1015
hc æ l – l1 ö ê l 2 = 400nm ú
Þ V1 – V2 = ç 2
e è l1·l 2 ÷ø ê hc ú 29. (0.149) If vmax is the speed of the fastest
ê =1240 nm – V ú electron emitted from the metal surface, then
ëe û
hc 1 2
æ 100nm ö = W0 + mvmax
= (1240 nm – v) ç l 2
è 300 nm ´ 400nm ÷ø
= 1.03 V » 1V (6.63 ´ 10-34) ´ (3 ´108)
26. (14.14) de-Broglie wavelength (l) is given by (180 ´10-9)
K = qV -19 1 -31 2
= 2 ´ (1.6 ´ 10 ) + (9.1 ´ 10 )vmax
2
l=
h
p
=
h
2mK
=
h
2mqV
(
Q p = 2mK ) \ v = 1.31 × 106 m/s
Substituting the values we get The radius of the electron is given by

\
lA
=
2m B q B VB
=
4m.q.2500 mv (9.1´10-31) ´ (1.31´106)
r= =
lB 2m A q A VA m.q.50 qB (1.6 ´ 10-19) ´ (5 ´10-9)
= 2 50 = 2 ´ 7.07 = 14.14 = 0.149 m
27. (400) If E is the energy of each photon, then 30. (1.5) KEmax= E – f0
nE = P
(where E = energy of incident light f0 = work
P 200
\ E= = = 50 ´ 10-20 J hc hc
n 4 ´1020 function) = -
l l0
If l is the wavelength of light, then
hc
= 1237 é
1 1 ù
E= -
l ëê 260 380 ûú
hc (6.63 ´ 10-34) ´ (3 ´108) 1237 ´ 120
\ l= = = = 1.5eV
E 500 ´ 10-20 380 ´ 260
; 400 nm

@ebooks_for_freee on Telegram
Atoms 193

CHAPTER

Atoms
26
1. (a) The significant result deduced from the n 2 (n 2 - 1)
Rutherford's scatter ing is that whole of the Again, 6 = , in second case.
2
positive charge is concentrated at the centre of
atom i.e. nucleus. Or n 22 - n 2 - 12 = 0 or (n 2 - 4)(n 2 + 3) = 0.
2. (a) Distance of closest appr oach
Take positive root, or n 2 = 4
Ze(2e)
r0= 2pKZe2
æ 1 2ö Now velocity of electron, v =
4pe 0 ç mv ÷ nh
è2 ø
v n 4
\ 1 = 2= .
Energy, E = 5 ´ 106 ´ 1.6 ´ 10-19 J v2 n1 3
9 ´ 109 ´ (92 ´ 1.6 ´ 10-19 ) (2 ´ 1.6 ´ 10 -19 ) 8. (a) Energy of ground state 13.6 eV
\ r0 =
6 -19
5 ´ 10 ´ 1.6 ´ 10 13.6
Energy of first excited state = - = -3.4 eV
4
Þ r = 5.2 ´ 10 -14 m = 5.3 × 10–12 cm. Energy of second excited state
2
n 13.6
3. (a) l min = [Q n =1for Lymen series] = - = -1.5 eV
R 9
1 Difference between ground state and 2nd excited
l min = ; 910 Å
R state = 13.6 – 1.5 = 12.1 eV
4. (d) Circumference, 2prn = nl So, electron can be excited upto 3rd orbit
1 N 2 sin 4 (q1 / 2) No. of possible transition
5. (c) Nµ ; = 3 ® 2, 3 ® 1, 2 ® 1
sin 4 q / 2 N1 sin 4 (q2 / 2)
So, three lines are possible.
N2 sin 4 (60° / 2)
or = 9. (d) [E = – 13.6/n2]
5 ´ 106 sin 4 (120° / 2) 13.6 8
4
DE =13.6 – =13.6 ´ eV
N2 sin 30° 9 9
or = = 12.08 eV
6
5 ´ 10 sin 4 60°
= 12.08 × 1.6 × 10–19
4 4
æ 1ö æ 2 ö 5 =19.34 × 10–19 J
or N 2 = 5 ´ 106 ´ ç ÷ ç = ´ 10 6
è 2 ø è 3 ÷ø 9 10. (c) Transition A (n = ¥ to 1) : Series lime of
6. (c) As a-particles are doubly ionised helium Lyman series
He++ i.e. Positively charged and nucleus is also Transition B (n = 5 to n = 2) : Third spectral line
positively charged and we know that like charges of Balmer series
repel each other. Transition C (n = 5 to n = 3) : Second spectral
7. (a) Number of emission spectral lines line of Paschen series.
n(n - 1) n (n - 1) 11. (b) Work done to stop the a particle is equal to
N= \3 = 1 1 , in first case.
2 2 K.E.
2
Or n1 - n1 - 6 = 0 or (n1 - 3)(n1 + 2) = 0 1 2 K (Ze) 1 2
Take positive root. \ n1 = 3 \ qV = mv Þ q ´ = mv
2 r 2

@ebooks_for_freee on Telegram
194 PHYSICS

2 14. (b) Energy required to remove e– from singly


2(2e) K ( Ze) 4 KZe
Þr= =
mv
2
mv
2 (13.6)Z 2
ionized helium atom = = 54.4 eV
1 1 12
Þrµ and r µ .
v 2 m (Q Z = 2)
12. (b) According to Bohr's Theory the Energy required to remove e– from helium atom
wavelength of the radiation emitted from = x eV
hydrogen atom is given by Accordin g to question, 54.4 eV = 2.2x
Þ x = 24.73 eV
1 é1 1ù
= RZ 2 ê 2 – 2 ú Therefore, energy required to ionize helium atom
l êë n1 n2 ûú = (54.4 + 24.73) eV = 79.12 eV
Q Z= 3 15. (b) For first excited state, n = 2 and for Li + +
1 æ 1ö Z= 3
\ = 9 R ç1 – ÷
l è 9ø 13.6
13.6
1 1 En = ´ Z2 = ´ 9 = 30.6 eV
Þ l= = n 2 4
8R 8 ´ 10973731.6
(R = 10973731.6 m–1) 16. (b) If n1 = n and n2 = n + 1
Þ l = 11.39 nm
n 2 ( n + 1)
2
13. (b) Total energy of electron in nth orbit of
Maximum wavelength lmax =
hydrogen atom ( 2n + 1) R
Rhc
En = -
n2 Therefore, for large n, l max µ n3
Total energy of electron in (n + 1)th level of
17. (c) It is given that transition from the state
hydrogen atom
n = 4 to n = 3 in a hydrogen like atom result in
Rhc ultraviolet radiation. For infrared radiation
En +1 = -
( n + 1)2
æ 1 1 ö
When electron makes a transition from (n + 1)th ç 2 – 2 ÷ should be less. The only option is
level to nth level è n1 n2 ø
Change in energy, 5 ® 4.
Increasing
DE = En +1 - En Energy n=5
é1 1 ù n=4
hn = Rhc × ê 2 - ú (Q E = hn)
ën (n + 1)2 û n=3
é (n + 1) 2 - n 2 ù n=2
n = R×c ê 2 2 ú n=1
ë n (n + 1) û 18. (d) We have to find the frequency of emitted
photons. For emission of photons electron
é 1 + 2n ù should makes a transition from higher energy
n = R×c ê 2 2ú
ë n (n + 1) û level to lower energy level. so, option (a) and (b)
are incorrect.
For n > > 1
Frequency of emitted photon is given by
é 2n ù 2 RC
Þ n = R×c ê 2 = 3 æ 1 1 ö
ë n ´ n 2 úû n h n = -13.6 ç 2 - 2 ÷
çn ÷
è 2 n1 ø
1
Þnµ For transition from n = 6 to n = 2,
n3

@ebooks_for_freee on Telegram
Atoms 195

-13.6 æ 1 1 ö 2 æ 13.6 ö 1 æ 1 1ö
n1 = - = ´ç ÷ \ = Rç 2 – 2÷
h çè 62 22 ÷ø 9 è h ø l è n1 n2 ø
For transition from n = 2 to n = 1, 1
Þ
-13.6 æ 1 1 ö 3 æ 13.6 ö . 122 ´ 10–9 m
n2 = - = ´ç ÷
h çè 22 12 ÷ø 4 è h ø é1 1ù é 1 ù 3R
= R ê – ú = R ê1 – ú =
\ n1 < n2 ë12 2
2 û ë 4û 4
19. (d) When electron jumps from M ® L shell 4
Þ R= m –1
1 æ 1 1 ö K´5 3 ´ 122 ´ 10–9
= Kç 2 - 2 ÷ = ..... (i)
l è2 3 ø 36 The highest frequency and smallest wavelength
for infrared region will be for transition of electron
When eletron jumps from N ® L shell
from ¥ to 3rd orbit.
1 æ 1 1 ö K´3
=Kç 2 - 2 ÷ = æ 1
l' è2 4 ø 16
....(ii) 1 1ö
\ = Rç 2 – 2÷
l è n1 n2 ø
solving equation (i) and (ii) we get
20
l' = l 1 4 æ 1 1ö
27 Þ = –
l 3 ´ 122 ´ 10 çè 32 ¥ ÷ø
–9
c c
20. (d) v = =
137 n 137 ´ 3
3 ´ 122 ´ 9 ´ 10 –9
h h h h \ l= = 823.5nm
l = p = mv = æ m ´ c ö = mc ´ (3 ´ 137) 4
çè ÷ 24. (25) If l is the de-Broglie wavelength, then for
3 ´ 137 ø
= 9.7 Å nth orbit
2prn = nl
1 æ 1 1 ö 5R
21. (488.9) = Rç 2 - 2 ÷ =
l1 è 2 3 ø 36 Î0 h2n 2
where rn =
pme2Z
1 æ 1 1 ö 3R
= Rç 2 - 2 ÷ = me2Z
l2 è 2 4 ø 16 1
\ = 2 … (i)
l 2 Î0 h n
l 2 80
\ = For Lyman series
l1 108
80 80 1 2 æ1 1ö
l2 = l1 = ´ 660 = 488.9nm. = Z Rç 2 - 2 ÷ … (ii)
108 108 l è1 n ø
22. (5) E = E1 + E2 From equations (i) and (ii), we have

z 2 1240 1240 æ 1ö 2
13.6 2 = + Z 2R ç1 - 2 ÷ = me Z … (iii)
n l1 l2 è n ø 2 Î0 h2 n

13.6(2)2 me4
= 1240 æç
1 1 ö 1 where R= … (iv)
or + ÷´
n 2 è 108.5 30.4 ø 10-9 8 Î02 ch3
On solving, n = 5 After substituting the values in (iii) & (iv), we
23. (823.5) The smallest frequency and largest get
wavelength in ultraviolet region will be for n = 25
transition of electron from orbit 2 to orbit 1.

@ebooks_for_freee on Telegram
196 PHYSICS

25. (0.34) From energy level diagram, using DE = -13.6


28. (0.66) E3 = = -1.51eV
hc 32
l -13.6
and E4 = = -0.85 eV
hc 42
For wavelength l1 DE = – E – (–2E) =
l1
\ E4 – E3 = 0.66 eV
hc
\ l1 =
E z2
29. (30.6) For lithium, E2 = -13.6 2
æ 4 E ö hc n
For wavelength l2 DE = – E – ç - ÷ =
è 3 ø l2
13.6 ´ 32
hc l 1 = -
\ l2 = \ r= 1 = 22
æ ö
E l2 3 = –30.6 eV
çè ÷ø
3
So energy needed to remove the electron
26. (0.18) Wavelength of first line of Lyman series
= 30.6 eV.
1 æ1 1 ö
= Rç 2 - 2 ÷ æ 1ö
l è1 2 ø 30. (6.8 × 10–27) DE = 13.6 ç1 - ÷
1 3R è 16 ø
or =
l1 4
13.6 ´ 15 51
4 = =
or l1 = . 16 4
3R
1 æ 1 1 ö 5R 12.75 ´1.6 ´10-19
and = Rç 2 - 2 ÷ = = 12.75 eV =
l2 è 2 3 ø 36 3 ´ 108
36
or l2 = Photon will take away almost all of the energy
5R
l1 5 hc
\ = = 12.75 eV
l2 27 l
27. (122.4) E = 13.6 Z2 eV h æ 12.75 ö
Þ =ç ÷ = Pphoton = Prevoled atom
= 13.6 × (3)2 l è c ø
= 122.4 eV = 6.8 × 10–27 kg m/s

@ebooks_for_freee on Telegram
Nuclei 197

CHAPTER

Nuclei
27
1. (a) As we know, R = R0 (A)1/3 R = R0A1/3,
where A = mass number where R0 = 1.2 × 10–15m and A = mass number
RAl = R0 (27)1/3 = 3R0 In case of 27
13 Al, let nuclear radius be R1
5
RTe = R0 (125)1/3 = 5R0 = RAl and for 125
3 32Te, nuclear radius be R2
2. (c) Here, conservation of linear momentum can 27
For 13 Al , R1 = R0 (27)1/ 3 = 3 R0
be applied
238 × 0 = 4 u + 234 v For 125
32Te, R2 = R0 (125)
1/ 3
= 5R0
4 R2 5R0 5 5
\ v=- u = Þ R2 = R1 = ´ 3.6 = 6fm.
234 R1 3R0 3 3
r 4 8. (a) The binding energy per nucleon is lowest
\ speed =| v |= u
234 for very light nuclei such as 42 He and is greatest
3. (a) For substance A : around A = 56, and then decreases with
48/12 increasing A.
æ 1ö N0 N0 9. (b) Effective half life is calculated as
2N0 ® N A = 2N 0 ç ÷ = =
è 2ø 23 8 1 1 1
For substance B : = +
T T1 T2
48/16
æ 1ö N N 1 1 1
N0 ® N B = N 0 ç ÷ = 0 = 0 = + Þ T = 12 years
è 2ø 2 3 8 T 16 48
NA : NB = 1 : 1 3
4. (d) p1 = p2 Þ m1v1 = m2v2 Time in which will decay is 2 half lives
4
2m1 = m2 = 24 years
4 4 R3 10. (c) A = A 0 e -lt ;
2r. pR13 = r. pR 32 ; 13 = 1: 2 ;
3 3 R2 2100 = 16000e-12l ® e12l = 7.6
R1 : R2 = 1 : 21/3 2 1
5. (c) Radius R of a nucleus changes with the Þ 12l = log e 7.6 = 2 Þ l = =
12 6
nucleon number A of the nucleus as 0.6931 ´ 6
\ T= =4
R = 1.3 × 10–15 × A1/3 m 1
Hence, Mass mA
11. (d) Density of nucleus, r = =
1/ 3 1/ 3 Volume 4 3
R 2 æ A2 ö æ 128 ö 1/ 3 pR
= =ç = (8) = 2
R1 çè A1 ÷ø è 16 ÷ø mA
3
Þr= (Q R = R0 A1/ 3 )
\ R2 = 2R1 = 2 (3 × 10–15)m = 6 × 10–15 m 4
6. (c) Binding energy p( R0 A1/ 3 )3
3
= [ZMP + (A – Z)MN – M]c2 Here m = mass of a nucleon
= [8MP + (17 – 8)MN – M]c2 3 ´1.67 ´ 10-27
= [8MP + 9MN – M]c2 \r =
= [8MP + 9MN – Mo]c2 4 ´ 3.14 ´ (1.3 ´10-15 )3
7. (d) It has been known that a nucleus of mass (Given, R0 = 1.3 × 10–15)
number A has radius Þ r = 2.38 ´ 10 kg/m3
17

@ebooks_for_freee on Telegram
198 PHYSICS

12. (d) Mass defect, æ In2 ö


Dm = (50m p + 70mn ) - (msn ) A = A0 e – tIn 2/T1/ 2 çèQ l = T ÷ø
1/2
= (50×1.00783 + 70 × 1.008) – (119.902199)
Þ 500 = 700 e -tIn 2/T1/ 2
= 1.096
Binding energy 7 30 In 2
Þ In = (Q t = 30 minute)
= (Dm)C 2 = ( Dm) ´ 931 = 1020.56 5 T1/2
Binding energy 1020.5631 In 2
= = 8.5 MeV Þ T1/2 = 30 = 61.8 minute
Nucleon 120 In 1.4
13. (b) Power output of the reactor, (Q ln 2 = 0.693 and ln.1.4 = 0.336)
energy Þ T1/2 » 62 minute
P=
time 17. (c) The range of energy of b-particles is from
2 6.023 ´ 10 26 ´ 200 ´1.6 ´ 10-19 zero to some maximum value.
= ´
235 30 ´ 24 ´ 60 ´ 60 TaTb
18. (c) Tav = T + T
; 60 MW a b
If a and B are emitted simultaneously.
14. (b) Given, for 14C
A0 = 16 dis min–1 g–1 19. (d) ( T1/2 )A = ( t mean )B
A = 12 dis min–1 g–1 0.693 1
t1/2 = 5760 years Þ = Þ l A = 0.693 l B
lA lB
0.693
Now, l = or lA < lB
t1/ 2
Also rate of decay = lN
0.693
l= per year Initially number of atoms (N) of both are equal but
5760
2.303 A since l B > l A , therefore B will decay at a faster
Then, from, t = log10 0
l A rate than A
2.303 ´ 5760 16 20. (c) When one a- particle emitted then danghter
= log10 nuclei has 4 unit less mass number (A) and 2
0.693 12
unit less atomic (z) number (z).
2.303 ´ 5760
= log10 1.333 232
Th ® 78
208
Y + 6 42 He
0.693 90

2.303 ´ 5760 ´ 0.1249 208


Y ® 82
208
X + 4 b praticle
= 78
0.693
dN
= 2390.81 » 2391 years. 21. (200) According to question, at t = 0, A0 =
dt
15. (a) The chemical reaction of process is
= 1600 C/s
212 H ® 42He
Binding energy of two deuterons, and at t = 8s, A = 100 C/s
4 × 1.1 = 4.4 MeV
\ A = æç 1 ö÷
Binding energy of helium nucleus = 4 × 7 A0 è 16 ø
= 28 MeV Therefore half life period, t1/2 = 2s
Energy released = 28 – 4.4 = 23.6 MeV 3
16. (b) We know that æ 1ö
\ Activity at t = 6s = 1600 ç ÷ = 200C/s
è 2ø
Activity, A = A0 e – l t

@ebooks_for_freee on Telegram
Nuclei 199

22. (1) Nuclear density is independent of atomic ln 4 / 3 ln 4 / 3


or t= =
number. l (0.693 / t1/2)
dN 1
23. (1) We know that, dt = lN = T N ln 4 / 3
mean =
(0.693 / 4.5 ´109)
\ 1010 =
1
´N = 1.868 × 109 year.
109
27. (3.8 × 104) If m kg is the required mass of the
\ N = 1019
i.e. 1019 radioactive atoms are present in the uranium, then number of nuclei
freshly prepared sample. (m ´ 1000) ´ 6.02 ´ 10 23
The mass of the sample = 1019 × 10–25 kg = 10–6kg =
235
= 1 mg. Each U235 nucleus releases energy 200 MeV,
24. (16.45) Disintegration constant \ total energy released in 10 years
0.693 0.693
l = = = 0.182 per day
t1/2 3.8 m ´ 6.02 ´ 1026
Ein = ´ 200
The number of particles left after time t 235
N = N0e–lt Energy required in 10 years, Eour= Pt
N0 = (1000 ×106) × (10 ×365× 24 ×3600)
or = N0e–lt Eout
20
Efficiency h = E
or elt = 20 in
ln 20 Substituting the values, we get
or t= m = 3.8 × 104 kg.
l
ln 20 dN
= = 16.45 days 28. (6.1) We know that the rate of integration -
0.182 dt
25. (3.91 × 103) If A0 is the initial activity of radio- =A
active sample, then activity at any time \ A = A0e–lt
A = A0e–lt
or 1 × 106 = 4 × 106 e–l × 20 or 2700 = 4750e–l×5
1 or l = 0.1131 per minute
or e–20l =
4 0.693
Half life t1/2 =
The count rate after 100 hour is given by l
A¢ = A0e–l×100 = A0e–100l 0.693
= A0[e–20l]5 = = 6.1 minute
0.1131
5 Binding energy 0.0303 ´ 931
6 é1 ù = »7
= 4 ´10 ê ú 29. (7)
Nucleon 4
ë4û
= 3.91 × 103 per second 30. (2) R = R0(A)1/3
26. (1.868 × 109) Suppose x is the number of Pb206
1/3 1/3
nulei. The number of U238 nuclei will be 3x, Thus R æA ö
= æç
256 ö
\ 1 =ç 1÷ ÷ =4
3x + x = N0 R2 è A2 ø è 4 ø
We know that N = N0e–lt
or 3x = 4xe–lt R1
R2 = = 2 fermi
4 4
\ elt =
3

@ebooks_for_freee on Telegram
200 PHYSICS

CHAPTER
Semiconductor Electronics: Mate-
28 rials, Devices and Simple Circuits
hc I c 0.96 I e
1. (b) l= \ Current gain, b = = = 24
E I b 0.04I e
6.62 ´10 -34 ´ 3 ´108
= = 2.07 ´ 10 -5 m
-3 -19
(60 ´ 10 ´ 1.6 ´10 ) Vo R 5 ´103 ´ 62
10. (b) = o ´b =
2. (b) In half wave rectifier only half of the wave Vin R in 500
is rectified.
3. (a) Positive terminal is at higher potential (– = 10 ´ 62 = 620
5V) and negative terminal is at lower potential – Vo = 620 × Vin= 620 × 0.01 = 6.2 V
10 V. \ Vo = 6.2 volt.
4. (b) The power gain in case of CE amplifier,
11. (d) I-V characteristic of a photodiode is as
Power gain = b2 × Resistance gain
follows :
R
= b2 ´ o = (10)2 × 5 = 500.
Ri mA
5. (d) Output of upper AND gate = AB
Output of lower AND gate = AB
V
\ Output of OR gate, Y = A B + BA Reverse bias
This is boolean expression for XOR gate.
6. (b) Given : µe = 2.3 m2 V–1 s–1
µh = 0.01 m2 V–1 s–1, ne = 5 × 1012 / cm3
= 5 × 1018/m3, nh = 8 × 1013/cm3 = 8 × 1019/m3. mA
Conductivity s = e[neµe + nhµh] On increasin g the biasin g voltage of a
= 1.6 × 10–19 [5 × 1018 × 2.3 + 8 × 1019 × 0.01] photodiode, the magnitude of photocurrent first
= 1.6 × 10–1 [11.5 + 0.8] increases and then attains a saturation.
= 1.6 × 10–1 × 12.3 = 1.968 W–1 m–1. 12. (d) Given,
7. (d) For a p-type semiconductor, the acceptor
Wavelength of photon, l = 400 nm
energy level, as shown in the diagram, is slightly
above the top Ev of the valence band. With very A photodiode can detect a wavelength
small supply of energy an electron from the corresponding to the energy of band gap. If the
valence band can jump to the level EA and ionise signal is having wavelength greater than this
acceptor negatively value, photodiode cannot detect it.

V 0.50 hc 1237.5
8. (a) E= =
6
V / m = 1.0 × 10 V/m \ Band gap E g = = = 3.09 eV
d 5 ´ 10 –7 l 400

Ic 1
= 0.96 Þ 13. (d) Using Uav = e 0 E02
9. (c) I c = 0.96 I e 2
Ie
P
But I e = I c + I b = 0.96 I e + I b But U av =
4p r 2 ´ c
Þ I b = 0.04 I e

@ebooks_for_freee on Telegram
Semiconductor Electronics: Materials, Devices and Simple Circuits 201
19. (d) A logic gate is reversible if we can recover
P 1
\ = e0 E02 ´ c input data from the output. Hence NOT gate.
4p r 2 2
20. (c) According to question, when diode is
forward biased,
2P 2 ´ 0.1 ´ 9 ´ 109
E 20 = = Vdiode = 0.5 V
4 pr 2 e 0 c 1 ´ 3 ´ 108
Safe limit of current, I = 10 mA = 10–2 A
\ E0 = 6 = 2.45V/m Rmin = ?

14. (a) P n
–2 R
10 A
For forward bias, p-side must be at higher
0.5 volt
potential than n-side. DV = (+ )Ve
15. (c) Relation between drift velocity and current is
I = nAeVd 1.5 V
Voltage through resistance
Ie ne eAve
= VR = 1.5 - 0.5 = 1 volt
Ih nh eAvh
iR = 1 (=VR)
7 7 ve
Þ = ´ 1 1
4 5 vh \ Rmin = = -2 = 100 W
i 10
v 5
Þ e = 21. (0.4) As we know, current density,
vh 4
16. (b) Forward bias resistance j = sE = nevd
DV 0.1 vd
= = = 10 W s = ne = nem
DI 10 ´ 10-3 E
1 1
10
= 107 W
=r =
Reverse bias resistance =
-6 s n e eme =Resistivity
10
Ratio of resistances 1
=
Forward bias resistance 10 ´ 1.6 ´ 1019 - 19 ´ 1.6
19
= = 10–6
Reverse bias resistance or P = 0.4 Wm
17. (a) In reverse biasing the width of depletion 22. (0.4) Initially Ge and Si are both forward
region increases, and current flowing through biased so current will effectivily pass through
diode is zero. Thus, electric field is zero at middle Ge diode
of depletion region.
\ V° = 12 – 0.3 =11.7 V
18. (d) Band gap = energy of photon of wavelength
And if "Ge" is revesed then current will flow
2480 nm. So,
through "Si" diode
hc \ V° = 12 – 0.7 =11.3 V
Band gap, E g =
l Clearly, V° changes by 11.7 – 11.3 = 0.4 V
æ 6.63 ´ 10 - 34 ´ 3 ´ 108 ö 1 23. (8.49 × 1026) If M is the molar mass and r is the
= çç ÷´ eV density then volume of one mole
2480 ´ 10 - 9 ÷ 1.6 ´ 10 -19
è ø
M
= 0.5 eV V= .
r

@ebooks_for_freee on Telegram
202 PHYSICS

The number of atoms per unit volume \ izenerdiode= 14 × 10–3 – 5 × 10–3 = 9 × 10–3 A
NA NA Nr = 9 mA
= = = A
V M /r M 26. (50) In half wave rectification, output frequency
23
(6.02 ´ 10 ) ´ (8.96) remains same as input i.e., 50Hz.
= 27. (2.0) The band gap
63.54
= 8.49 ´ 1022 cm–3 hc (6.63 ´ 10 -34 ) ´ (3 ´ 108 )
= 8.49 × 1028 m–3 Eg = =
l 620 ´ 10 -9
As each copper (monovalent) atom has one
= 3.2 × 10–19 J
electron, so number of electrons per unit volume
= 2.0 eV.
= 8.49 × 1026 m–3
24. (650) If l is the wavelength of emitted light, Dic
28. (50) We know thatb = Di
then B
hc
Eg =
l (3.5 –1.0) ´ 10-3
hc = = 50 .
(80 – 30) ´10 -6
or l= E
g
Dic 200 - 100
(6.63 ´ 10 -34) ´ (3 ´108) 29. (20 × 103) b = = = 20
= Dib 10 - 5
(1.9) ´ (1.60 ´ 10-19)
= 6.5 × 10–7 m = 650 nm R2 20 ´100 ´103
25. (9) The voltage across zener diode is constant Voltage gain = b = = 20 × 103
R1 100
i i –i1 30. (0.02) As D2 is reversed biased, so no current
120V 5kW(R1) R2 10kW through 75W resistor.
now Req= 150 + 50 + 100 = 300 W
V 50 BatteryVoltage
i( R 2 ) = = = 5´10 –3 A So, required current I =
R 10 ´103 300
V 120 – 50 70 6
i ( R1 ) = = = 14 ´ 10–3 A I= = 0.02
R 5 ´10 3
5´103 300

@ebooks_for_freee on Telegram
Communication Systems 203

CHAPTER

Communication Systems
29
1. (b) Amax - Amin
actual frequency deviation = Am = 3 ...(ii)
m= ´ 100% 2
max.allowed frequency deviation From equation (i) + (ii),
(Df )actual Maximum amplitude, Amax = 8.
= ´100% From equation (i) – (ii),
( Df )max
Minimum amplitude Amin = 2.
if (Df) actual = (Df ) max
Am 2
m = 100 % 10. (d) Modulation index, µ = A = 4 = 0.5
2. (c) Modulation index c
E A 20000 p
ma = m = = 1 Given, fe = = 10000 Hz.
Ec A 2p
Equation of modulated signal [Cm(t)] 2000p
and fm = = 1000 Hz.
= E(C) + maE(C) sin wmt 2p
= A (1+ sin wCt) sin wmt (As E(C) = A sinwCt) \ LSB = fe – fm = 10000 – 1000 = 9000 Hz.
3. (d) 11. (d)
4. (b) The frequencies present in amplitude 12. (a) Here, fc = 1.5 MHz = 1500 kHz, fm = 10 kHz
modulated wave are : \ Low side band frequency
Carrier frequency = wc = fc – fm = 1500 kHz – 10 kHz = 1490 kHz
Upper side band frequency = wc + wm Upper side band frequency
Lower side band frequency = wc – wm. = fc + fm = 1500 kHz + 10 kHz = 1510 kHz
5. (c) 13. (b) The frequency of AM channel is 1020 kHz
B whereas for the FM it is 89.5 MHz (given). For
6. (b) Modulation index = higher frequencies (MHz), space wave
A
B = 25, A = 60 communication is needed. Very tall towers are
25 used as antennas.
Þ M.I. = = 0.416 Þ m% = 41.6% 14. (a) Comparing (x AM)t = 100 [1 + 0.5 t] coswct for
60
7. (d) 0<t<1
8. (a) The critical frequency of a sky wave for with standard AM signal x AM
reflection from a layer of atmosphere is given by = Ec [1 + ma cos wmt] cos wct
fc = 9(Nmax)1/2 We have modulating signal t and ma = 0.5.
Þ 10 × 106 = 9(Nmax)1/2 15. (c) For x(t), BW = 2(Dw + w)
6 ö2
æ 10 ´ 10 Dw is deviation and w is the band width of
12 –3
Þ N max = ç ÷ ; 1.2 ´10 m modulating signal.
è 9 ø
\ BW = 2(90 + 5) = 190
9. (b) From the given expression, For x2 (t), BW = 2 × 190 = 380
Vm = 5 (1 + 0.6 cos 6280t) sin(211 × 104t)
16. (a) Modulating signal frequency ® 10kHz
Modulation index, m = 0.6
Carrier signal frequency® 10 MHz
Q Am = mAc
\ Side band frequency are
Amax + Amin
= Ac = 5 ...(i) USB = 10 MHz + 10 kHz = 10010 kHz
2
LSB = 10MHz – 10 kHz = 9990 kHz

@ebooks_for_freee on Telegram
204 PHYSICS

17. (b) Comparing the given equation with


(0.3) 2
standard modulated signal wave equation, m = Ac Þ Vrms = 100 1 +
2
mA c = 104.5 volts.
sin wc t + cos (wc – ws) t
2
mA c 24. (2.7) f c = 9 N m = 9 ´ 9 ´ 1010
– cos (wc + ws) t
2 = 2.7 ´ 106 Hz = 2.7 MHz
Ac 2
m = 10 Þ m = (modulation index) ma 2
2 3 25. (6.25) Average side-band power Pav = Pc
4
Ac = 30 Here ma = 0.5
wc – ws = 200p Pc = 10
wc + ws = 400p
Þ fc = 150, fs = 50 Hz. 0.5 ´ 10 ´ 10
\ Pav = = 6.25
18. (b) Frequency of radio waves for sky wave 4
propagation is 2 MHz to 30 MHz. 26. (6) Ratio of AM signal Bandwidths
19. (c) E max = ( 1+ ma) Ec = ( 1+ 0.6) × 10 = 16 V
Emin = ( 1– ma) Ec = ( 1– 0.6) × 10 = 4V. 15200 - 200 15000
= = = 6.
20. (a) 2700 - 200 2500
21. (2.7) f c = 9 N m = 9 ´ 9 ´ 1010 27. (0.43)

= 2.7 ´ 106 Hz = 2.7 MHz Vmax - Vmin 10 - 4 6


ma = = = = 0.43
22. (10) Comparing given expression with Vmax + Vmin 10 + 4 14
(e)AM = Ec(1 + ma cos wmt) cos wct
Pt 12 12
peak value of carrier wave, Ec= 10V. 28. (9.6) Pc = = =
é m 2ù 1+
m 2a
1+
( 0.5 ) 2 1.25
23. (104.5) Pt = Pc ê1 + a ú 2 2
ë 2 û
= 9.6 kW
V 2rms V 2c é m a 2 ù
Þ = ê1 + ú 29. (1.28p × 103) Area covered by T.V. signals
2 2 ë 2 û A = 2phR = 2 × 3.14 × 100 × 6.4 × 106 = 128p × 108
é m 2ù Þ A = 1.28p × 103 km2
V 2 rms = V 2 c ê1 + a ú
ë 2 û 30. (71) Total signal B.W= 12 ´ 5 = 60 kHz
11 guard band are required between 12 signal
ma 2
Þ Vrms = Vc 1 + \ guard bandwidth = 11 × 1 kHz = 11 kHz
2 \ total bandwidth = 6 + 11 = 71 kHz

@ebooks_for_freee on Telegram
@unacademyplusdiscounts_link

https://telegram.me/Unacademyplusdiscounts_link

https://telegram.me/Unacademyplusdiscounts_link

https://telegram.me/Unacademyplusdiscounts_link

Join Us Now For all study Materials for free

@ebooks_for_freee on Telegram

You might also like